4,, Peds Exam 1 Weeks 2/3 CH 4, 5, 6, 7, 8, Peds test 1, Exam 1 Week 1 CH 10, 12, 13, 22, NR602 Pediatric Midterm Study Set, Burns Ch. 7: Development Management of School-Age Children, PNP Exam Questions, Exam 2 Pediatrics 21,22,24,25,26, Exam 3 Pedi...

Pataasin ang iyong marka sa homework at exams ngayon gamit ang Quizwiz!

A child has several circular, scaly lesions on the arms and abdomen, some of which have central clearing. The NP notes a smaller, scaly lesion on the child's scalp. How will the NP treat this child?

Prescribe oral griseofulvin for 2 to 4 weeks.

A preschoolage child who attends day care has a 2day history of matted eyelids in the morning and burning and itching of the eyes. The primary care pediatric nurse practitioner notes yellowgreen purulent discharge from both eyes, conjunctival erythema, and mild URI symptoms. Which action is correct?

Prescribe topical antibiotic drops.

All newborns get the first dose of HepB before discharge except: Newborns who get 2 doses before discharge Preemies who are less than 2000g Unless they wait til age 4 Asian newborns

Preemies who are less than 2000g

12. Fever accounts for 30% of office and emergency room visits? T/F

10 %- to 20% answer is false

What percentage of children achieve glycemic control with diet/exercise alone? 5% 25% 50% 10%

10%

The parent of a 6 year old expresses concern that a child may have ADHD. Which screening tool will the NP use to evaluate this possibility?

Conner's 3 Parent and Teacher Rating Scale

When providing well child care for an infant in the first year of life, the primary care pediatric nurse practitioner is adhering to the most recent American Academy of Pediatrics Recommendations for Preventative Pediatric Health Care guidelines by:

scheduling well baby visits to coinside with key developmental milestones

Which region globally has the highest infant mortality rate

sub-saharan Africa

a 14 year old female has not had her menses and is concerned. She states she has never been sexually active. Which lab test would a prudent nurse practitioner perform first with a physical assessment?

a pregnancy test

4. Systemic medications like dicloxacillin should be used in the treatment of superficial folliculitis in which circumstances (check all that apply)? a. Sever3 cases -* b. Immunocompromised patients- * c. Patient's with atopic dermatitis d. Patient's diagnosed with diabetes - *

a, b, d

7. An infant with polydipsia, polyuria, irritability and failure to thrive should be evaluated for: a-Diabetes Insipidus b-Homocystinuria c-Growth Hormone Deficiency d-hyperglycemia

a-Diabetes Insipidus

6. The most common cause of hyperthyroidism in children and adolescents is a-Graves Disease b-thyroid cancer c-thyroid nodules d-pituitary tumor

a-Graves Disease

25. A child who has psoriasis, who has been using a moderatepotency topical steroid on thick plaques on the extremities and a highpotency topical steroid on more severe plaques on the elbows and knees, continues to have worsening of plaques. In consultation with a dermatologist, which treatment will be added? a. Anthralin ointment in high strength applied for 10 to 30 minutes daily b. Calcipotriol cream applied liberally each day to the entire body c. Oral steroids and methotrexate therapy until plaques resolve d. Wideband ultraviolet therapy for 15 minutes twice daily

b

The primary care pediatric nurse practitioner notes a small, round object in a child's external auditory canal, near the tympanic membrane. The child's parent thinks it is probably a dried pea. What will the nurse practitioner do to remove this object?

Refer the child to an otolaryngologist for removal.

What is the correct sequence of motor development in average infants? a. Reach & miss, sit alone, creep, climb stairs, stand alone b. creep, sit alone, climb stairs, stand alone, reach & miss c. creep, sit alone, climb stairs, reach and miss, stand alone d. sit alone, creep, climb stairs, reach and miss, stand alone

a

When assessing the heart of a healthy 4-year-old child, the PNP would expect to find: A) A visible apical pulse or point of maximal impulse B) Sinus tachycardia C) S2 heart sounds louder than S1 heart sounds at the apex D) Long, low-pitched heart sounds

a

The most appropriate analgesia for a 4yo with post operative tonsillectomy pain would include a. ibuprofen b. acetaminophen and oxycodone (Roxicet) c. Ketorolac d. aspirin and oxycodone (Percodan).

b

The most common cause of myocarditis is: a. bacterial b. viral c. drug reaction d. radiation therapy

b

The most common congenital heart defect in children is: a. tricuspid atresia b. ventricular septal defect c. coarctation of the aorta d. pulmonary atresia with intact ventricular septum

b

An adolescent has acne with lesions on the cheeks and under the chin. What distribution is this?

Hormonal

5'8" 6'2" 6'0" 5'8 3/4

Child's predicted growth potential Boys: (mother's height + 5″) + (father's height)/2 Girls: (father's height - 5″) + (mother's height)/2

2. The common practice of using "time-outs" with young children is a direct application of: a. Operant conditioning b. Classical conditioning c. Separation-individuation d. Maturational reinforcement

2. A: Time-out is a practice of behavioral modification as promulgated by B. F. Skinner's model of operant con-ditioning with a negative consequence for an unac-ceptable behavior.

16. Respirations increase how many per minute with each elevation of degree of degree centigrade? 2 3-5 4-6 None

3-5

Grayson was just diagnosed with Type 1 diabetes, when should he have his first ophthalmologic exam? 3-5 years 2 years When he turns 15 When he turns 21

3-5 years

3. Good communication among families, schools, and primary care providers is an example of which eco-logical concept? a. Microsystem b. Mesosystem c. Exosystem d. Macrosystem

3. B: A mesosystem is the link or relationship between the various settings (microsystems) within which a child exists (i.e., home, school, day care, etc.).

30. Which of the following diagnoses is not more com-mon among males? a. ADHD b. Conduct disorders c. Suicide d. FTT

30. D: The incidence of failure to thrive has no predomi-nance in males or females.

49. Adolescents who engage in risky behavior, such as driving without a seat belt, are displaying: a. A type of egocentrism b. A need for independence c. Role experimentation d. Low self-esteem

49. A: The belief that one is immune to poor or bad out-comes (e.g., death, disease) is a form of egocentrism known as personal fable in which adolescents believe that the laws of nature do not apply to them.

7. One of the major psychosocial tasks of infancy is: a. Development of secure attachment b. Separation-individuation c. Symbiosis d. Regulation

7. A: The development of an infant-caregiver bond is key to the prevention of long-term psychological effects associated with deprivation and/or failure to develop secure, stable bonds.

8. Which developmental theory best explains the multi-factorial etiology of failure to thrive? a. Organismic-maturational theory b. Social learning theory c. Transactional theory d. Psychoanalytic theory

8. C: Transactional theory explains risks and protective factors associated with resilience and vulnerability. It may explain some of the environmental factors asso-ciated with FTT.

Who would you be most concerned with An 8 year old with thelarche 9 year old boy with pubarche

9 year old boy with pubarche

You are evaluating recurrent stomach pain in a school-age child. The child's exam is normal. You learn that the child reports pain most evenings after school and refuses to participate in sports but doesn't have n/v. The child's grandmother recently had gallbladder surgery. Which action is correct? a. Encourage the child to keep a log of pain, stool patterns, and dietary intake. b. Order radiologic studies and lab tests to rule out systemic causes c. Reassure the child and encourage resuming sports when symptoms subside d. Refer the child to a counselor to discuss anxiety about health problems.

A

6. The mother of a 15yearold adolescent female tells the primary care pediatric nurse practitioner that her daughter has extreme mood swings prior to her periods, which the adolescent vehemently denies. When asked if she notices anything different just before her periods, the adolescent points to her mother and says, "She gets really hard to live with." This demonstrates which characteristic of adolescent thinking? A. Apparent hypocrisy B. Imaginary audience C. Overthinking D. Personal fable

A. Apparent hypocrisy

TC who is three years of age presents with a history of fever for the past several days pain in his left leg and refusal to bear weight on the left leg. 10 days ago he fell from a slide and bruised his leg his WBC count is slightly elevated. You suspect either toxic synovitis or osteomyelitis. Which finding supports a diagnosis of osteomyelitis more so than toxic synovitis? A: Recent injury B: leg pain C: Non-weight bearing D: elevated white blood count

A (Osteomyelitis is associated with local trauma wild toxic synovitis is associated with recebt upper respiratory illness)

A 2 and a half year-old child presents with macrocephaly, developmental delay, course facial features, large tongue, kyphosis, hip dislocation, tonsillar and adenoidal hypertrophy and hepatomegaly. She is receiving early intervention services without improvement. What is the next best step? A. Referral to genetics for further evaluation B. Follow up in three months C. Reassure the parents D. Follow up in six months

A (This child has many of the signs of Hurler's syndrome. Given the lack of progress in early intervention and the physical assessment findings a referral to genetics will be the best approach in this child)

Payment by capitation means that: A. Financial risk is shifted from payers to providers B. Care is provided on a fee-for-service basis C. Care resources are rationed D. Target populations have unlimited access to healthcare

A (and capitation providers are responsible for target population which they receive an age-and gender-adjusted budget)

Tonya, at five years of age, has sickle cell disease. To decrease the risk of vaso-occlusive crisis, it is important to stress which of the following to Tonya and her parents? A. The need for frequent handwashing B. The need for a diet high in iron C. Avoidance of the use of mothballs in the house D. Limitation of milk intake to one glass a day

A (vaso-occlusive crisis is usually associated with infection, dehydration, acidosis or exposure to cold)

The PNP has prescribed salmeterol (Serevent) by inhalation for an asthmatic. The family should be warned about which of the following side effects that occur with this medication?A) Tachycardia and headache B) Weight gain and acne C) Nausea and vomiting D) Dysrhythmia and dry mouth

A) Tachycardia and headache

A child has a fever and arthralgia. The primary care pediatric nurse practitioner learns that the child had a sore throat 3 weeks prior and auscultates a murmur in the clinic. Which test will the nurse practitioner order?

ASO titer

19. Temp peaks in the evening between 4 & 6 pm, and is lowest between 4 & 8 am. For fever management which dosing items are not correct? Pick 2 Acetaminophen 15 mg/kg/ dose every 4-6 hours Ibuprofen 5 mg/ kg/ dose every 6-8 hour Alternating Ibuprofen and acetaminophen 1 chewable St. Joseph aspirin every 4 hours until the fever has subsided

Acetaminophen 15 mg/kg/ dose every 4-6 hours Ibuprofen 5 mg/ kg/ dose every 6-8 hour

A child who has been diagnosed with asthma for several years has been using a shortacting B2agonist (SABA) to control symptoms. The primary care pediatric nurse practitioner learns that the child has recently begun using the SABA two or three times each week to treat wheezing and shortness of breath. The child currently has clear breath sounds and an FEV1 of 75% of personal best. What will the nurse practitioner do next?

Add a daily inhaled corticosteroid

The primary care pediatric nurse practitioner performs a well baby assessment of a 5dayold infant and notes mild conjunctivitis, corneal opacity, and serosanguinous discharge in the right eye. Which course of action is correct?

Admit the infant to the hospital immediately.

Aaliyah come in and is very tall for her age and is having early puberty, and her parents are concerned. What must you rule out: Marfan syndrome Overnutrition advancing bone age/puberty Excess adrenal androgens All of the above

All of the above

Riley, from the above question is referred to a pediatric endocrinologist. What would the management look like? Antithyroid drugs Subtotal thyroidectomy Radioiodine All of the above None of the above

All of the above

5. You are seeing a 17-year-old female patient. During the visit, you consider her risk factors for type 2 DM would likely include all of the following except: A. Obesity B. Native American ancestry C. Family history of type 1 DM D. Personal history of polycystic ovary syndrome

Answer C

13. Fever is the least common presenting complaint to a pediatric practice T/F

Answer is False- most common complaint

A child is diagnosed with tinea versicolor. What is the correct management of this disorder?

Application of selenium sulfide 2.5% lotion twice weekly for 2 to 4 weeks.

The parent of a 4monthold infant is concerned that the infant cannot hear. Which test will the primary care pediatric nurse practitioner order to evaluate potential hearing loss in this infant?

Auditory brainstem response (ABR)

Which of the following suggests internal tibial torsion rather than internal femoral torsion in a two-year-old child presenting with an in -toeing gate? A: sitting in "w" position B: knees face forward when walking C: Generalized ligament laxity D: Limited external rotation of hip

B (The patella will rotate in word if the problem is above the knee)

The differential diagnosis of dysfunctional uterine bleeding includes all but which of the following? A. Pregnancy-related disorders B. Anemia C. Foreign body D. Endometriosis

B (anemia is considered to be a complication of dysfunctional uterine bleeding rather than a part of the differential diagnosis)

An 8yearold boy has a recent history of an upper respiratory infection and comes to the clinic with a maculopapular rash on his lower extremities and swelling and tenderness in both ankles. The pediatric nurse practitioner performs a UA, which shows proteinuria and hematuria and diagnoses HSP. What ongoing evaluation will the nurse practitioner perform during the course of this disease?

Blood pressure measurement

The most important lab test to be performed when a UTI is suspected in a school-age child is a: A. CBC with differential B. Urine dipstick C. Clean catch urine for U/A and C/S D. Voiding cystourethrogram

C

The mother of a 14-year-old girl indicates to you that she is concerned because the girl has not yet started to menstruate. The history is noncontributory and the physical exam is normal. Breast development and pubic hair have been present for 12 months the most appropriate initial step would be to: A. Do a pregnancy test B. Obtain a Buccal smear for chromosomal analysis C. Reassure, educate the family, and follow up D. Draw LH and FSH levels

C

Coach Jones asks for advice on how to prevent little league elbow (epicondylitis) in his eight and nine-year-old players. Which of the following would be an incorrect advice? A: have each child pitch only three innings B: limit or eliminate curveballs C: Use ice massage before and after pitching D: Conduct slow warm-ups

C (LLE results from reptitive Forearm supination and pronation. Therefore the goal is to prevent injury by reducing the repetitive motion. Ice falsely reassures parents or coaches that the injury can be prevented by applying the before and after pitching)

During a wellbaby assessment on a 1weekold infant who had a normal exam when discharged from the newborn nursery 2 days prior, the primary care pediatric nurse practitioner notes moderate eyelid swelling, bulbar conjunctival injections, and moderate amounts of thick, purulent discharge. What is the likely diagnosis?

Chlamydia trachomatis conjunctivitis

There are several skin features seen with the tuberous sclerosis. Which of the following is more characteristic? A. Café au lait spots B. Blue or purple striae C. Papules in a "Christmas tree" pattern D. Hypopigmented macules in an "ash leaf" shape

D

To establish a diagnosis of sickle cell disease, which lab test is appropriate? A. CBC with RBC indices B. Sickle cell prep C. Sickledex D. Hemoglobin electrophoresis

D

To prevent complications of sickle cell disease during the ages three months to five years, daily doses of which medication should be prescribed prophylactically? A. Baby aspirin B. Acetaminophen C. Diphenhydramine D. Penicillin

D

Tonya, at five years of age, has sickle cell disease. Tonya's mother tells you that she is upset because Tonya sometimes wets the bed. A urinalysis reveals no significant findings. After review of the pathophysiology of sickle cell and vaso-occlusive crisis with her, which of the following should you tell the mother? A. To limit Tonya's fluids after dinner and especially before bedtime B. To keep reminding Tonya than most girls her age to not with the bed C. To wait Tonya in the middle of the night and take her to the bathroom D. To encourage Tonya to drink fluids and put a waterproof covering on her bed

D

Upon physical examination of a 4 yo boy, you note seven café Au last spots > than 5 mm in diameter. This finding maybe indicative of: A. Tuberous sclerosis B. Sturge-Weber dz C. Duchenne's dystrophy D. Neurofibromatosis

D

What congenital infection can present with snuffles, lymphadenopathy, pseudoparalysis of Parrot , CNS abnormalities, and low birth weight? A. Cytomegalovirus B. Rubella C. Toxoplasmosis D. Syphillis

D

What hormone needs replacement in adolescents with Klinefelter's syndrome? A. Thyroid hormone B. Growth hormone C. Estrogen D. Testosterone

D

What physical exam finding distinguishes a new born with congenital toxoplasmosis from a new born with congenital cytomegalovirus? A. Jaundice B microcephaly C. Cerebral calcification D. Petechial rash

D

The primary care pediatric nurse practitioner understands that a major child health outcome associated with worldwide climate change is

nutrition

When planning screening protocols, it is important for the practitioner to know that iron deficiency anemia is most common in which life period(s)? A. The first month of life B. The period when the child is most sedentary C. The preschool years D. Periods of rapid growth

D

Which of the following historic points would not alert the practitioner to the possibility of a brain tumor? A. Headache in the morning with associated w/ vomiting B. Failure to thrive and developmental delay in young children C. School failure and personality changes in older children D. Sleep apnea

D

Which of the following is a risk factor for adolescent pregnancy? A. Good grades in school B. Asks for contraceptive at the school health clinic C. Plans to study nursing in college D. Unsure if monogamous partner uses condoms

D

Which of the following is the most helpful in confirming a diagnosis of chlamydia pneumoniae in a one-month-old child with a cough? A. Chest chest x-ray showing lobar consolidation B. Decrease serum immunoglobulins C. Lymphocytosis D. Mother with no prenatal care

D

Which of the following signs is not scored in the Glasgow coma scale? A. Eye opening B. Verbal response C. Upper limb response D. Fine motor responses

D

Which of the following statements is not true with regard to primary nocturnal enuresis? A. There is often a positive family history of enuresis B. It appears to be related to maturation delay C. Some nighttime wetters stop wetting without any form of treatment D. The incidence is higher in girls than in boys

D

Which of the following would not be an appropriate indicator for developmental dysplasia of the hip in a six month old child? A: Allis sign B: skinfold symmetry C: galeazzi sign D: ortolani maneuver

D

You are evaluating a 13-year-old girl for graves disease. Which of the following signs would not support this diagnosis? A. An enlarged thyroid B. Exophthalmos C. A positive family history D. An elevated TSH level

D

You have ordered red blood cell indices for a 10-year-old girl. Results reveal a decrease in both the mean corpuscular hemoglobin MCH and the mean corpuscular volume MCV. Differential diagnosis should include: A. Sickle cell anemia B. Vitamin B 12 deficiency anemia C. Pernicious anemia D. Iron deficiency anemia

D

Patients with acute nephrotic syndrome may present with all but which of the following signs. A. Edema B. HTN C. Dark amber colored urine D. History of weight loss

D

Scope of practice statements: A. Are prepared by national professional organizations B. Provide the basis for reimbursement policies C. Are written by employers D. May vary from state to state

D

The most common type of brain tumor in children is: A. Ependymoma B. Brain stem tumor C. Craniopharyngioma D. Medulloblastoma

D

1 year old John is diagnosed with a constitutional growth delay. Appropriate management includes: Starting him on low dose testosterone therapy now Counseling regarding delayed onset of puberty Thyroxine replacement Nutritional counseling

Counseling regarding delayed onset of puberty

The mother of an 11-year-old boy is concerned that her son is developing secondary sexual characteristics too early. Your counseling for the family is based on the knowledge that puberty is considered precocious in boys if secondary sexual characteristics appear prior to age: A. 12 B. 11 C. 10 D. 9

D

The parent of an adolescent reports noting cutting marks on the teen's arms and asks the primary care pediatric nurse practitioner what it means. What will the NP tell this parent?

Cutting is a way of dealing with emotional distress.

Achondroplasia refers to a growth delay that is: A. Due to malabsorption B. Associated with Noonan syndrome C. Associated with endocrine disorders D. Manifested by disproportionately short stature

D

9. The primary care pediatric nurse practitioner performs a physical examination on a 9monthold infant and notes two central incisors on the lower gums. The parent states that the infant nurses, takes solid foods three times daily, and occasionally takes water from a cup. What will the pediatric nurse practitioner counsel the parent to promote optimum dental health? A. To begin brushing the infant's teeth with toothpaste B. To consider weaning the infant from breastfeeding C. To discontinue giving fluoride supplements D. To make an appointment for an initial dental examination

D. To make an appointment for an initial dental examination

Layla's mother comes in and is concerned because Layla is going through so much insulin and her weight has dropped. What do you think is going on? Worsening of Type 1 diabetes Diabulemia Help syndrome Layla has been ditching her insulin

Diabulemia

A child is brought to the clinic with a generalized, annular rash characterized by raised wheals with pale centers. On physical examination, the child's lungs are clear and there is no peripheral edema. A history reveals ingestion of strawberries earlier in the day. What is the initial treatment?

Diphenhydramine 0.5 to 1 mg/kg/dose every 4 to 6 hours.

The parent of a newborn has quit smoking cigarettes within the past month and reports feeling fidgety. Using a "reframing" technique, how will the primary care pediatric nurse practitioner respond?

Explore ways that the parent can use this extra energy to do things for the baby.

The NP is performing a well child exam on a 12 year old female who has achieved early sexual maturation. The mother reports that she spends ore time with her older sister's friends instead of her own classmates. What will the NP tell this parent?

Girls who join an older group of peers may become sexually active at an earlier age.

In secondary adrenal insufficiency, there is a deficiency of what? Mineralocorticoid Glucocorticoid Mineralocorticoid and glucocorticoid Testosterone

Glucocorticoid

A child is brought to clinic with several bright red lesions on the buttocks. The NP examines the lesions and notes sharp margins and an "orange peel" look and feel. The child is afebrile and does not appear toxic. What is the course of treatment for these lesions?

Initiate empiric antibiotic therapy and follow up in 24 hours to assess response.

A child who has otitis externa has severe swelling of the external auditory canal that persists after 2 days of therapy with ototopical antibiotic/corticosteroid drops. What is the next step in treatment for this child?

Insert a wick into the external auditory canal

A school-age child is brought to clinic after a pediculosis capitis infestation is reported at the child's school. If the child is positive, what will the NP expect to find on physical examination, along with live lice near the scalp?

Itching of the scalp with skin excoration on the back of the head.

The parent of a newborn infant asks the NP when to intervene to help the infant's future intellectual growth. What will the NP tell the parent?

Language and literacy skills begin at birth

A child has an inborn error of metabolism, which characteristic would you find? Hyperactivity Weight gain Musty odor Fruity breath

Musty odor

The primary care pediatric nurse practitioner obtains a tympanogram on a child that reveals a sharp peak of 180 mm H2O. What does this value indicate?

Negative ear pressure

During a well child examination of an infant, the primary care pediatric nurse practitioner notes 10 cafe au lait spots on the infant's trunk. What is the potential concern associated with this finding?

Neurofibromatosis

Which is true about the health status of children in the United States?

Obesity rates among 2- to 5- year olds have shown a recent significant decrease.

The parents of a 3 year old child are concerned that the child has begun refusing usual foods and wants to eat mashed potatoes and chicken strips at every meal and snack. The child's rate of weight and slowed, but the child remains at the same percentile for weight on a growth chart. What will the primary care pediatric nurse practitioner tell the parents to do?

Place a variety of nutritious foods on the child's plate at each meal.

10. When counseling an adolescent with a family history of hyperinsulinemia and type 2 diabetes, the primary care pediatric nurse practitioner will recommend avoiding Processed breads Canned vegetables Baked potato chips High-fiber cereals

Processed breads

Paxton, age 8 years comes in with the what you suspect is Type 1 diabetes, what would not see on physical examination: Tachycardia fruity -smelling breath Rusty urine Vaginal yeast infection

Rusty urine

The primary care pediatric nurse practitioner is assessing the health literacy of the parent of a toddler. Which tool will the nurse practitioner use to estimate reading level?

SMOG

The primary care pediatric nurse practitioner is examining a schoolage child who has had several hospitalizations for bronchitis and wheezing. The parent reports that the child has

Spirometry testing

The parent of a 5 month old is worried because the infant becomes fussy but doesn't always seem interested in nursing. What will the NP tell this parent?

The infant may be expressing the desire to play or rest.

An adolescent female has grouped vesicles on her oral mucosa. To determine whether these are called by HSV-1 or HSV-2 the primary care pediatric nurse practitioner will order which test?

Viral culture

2. A child will need an occlusive dressing to treat lichen simplex chronicus. What will the primary care pediatric nurse practitioner tell the parents about applying this treatment? a. Apply ointment before the dressing. b. Plastic wrap should not be used. c. The dressing should be applied to dry skin. d. Change the dressing twice daily.

a

8. A child is brought to the clinic immediately after being stung by a wasp while playing in the yard. The physical examination reveals localized redness and edema at the site, along with abdominal tenderness, watery eyes, and generalized hives. What is the initial treatment? a. Administer intramuscular epinephrine b. Apply a topical glucocorticoid cream c. Give oral diphenhydramine d. Order a bronchodilator treatment

a

A 17yo girl is referred to your clinic by the school nurse to be assessed for an eating disorder. Which of the following dynamics is not characteristic of anorexia nervosa? a. excessive eating followed by purging b. a pervasive sense of helplessness and ineffectiveness c. wt loss that gives the patient a sense of mastery and control d. low body temp, pulse, and bp

a

A parent reports that her 18mo cries during the entire aerosol asthma treatment with albuterol and budesonide. What is an alternative managment? a. prescribing oral albuterol b. using MDI and Spacer c. using nebulizer when child is sleeping. d. adding montelukast

a

Assessment of the red reflex may be used to rule out which of the following: a. opacities b. myopia or hyperopia c. decreased visual acuity d. blindness

a

Following an episode of meningitis, it is most important to assess the child for: a: hearing loss b. changes in taste c. cervical lymphadenopathy d. tinnitus

a

Premature infants should receive routine immunizations based on: a. chronological age b. gestational age c. birth wt d. current wt

a

The most useful test for evaluation of suspected acute rheumatic fever is: a. antistreptolysin-O (ASO) titer b. electrocardiogram c. hemoglobin electophoresis d. urinalysis

a

Tracy, who is 9 yo, complains that she does not like to wear shorts bc her knees look funny. Upon exam you note a genu valgum angle of > than 15 degees. You should: a. reevaluate in one year if still presnt b. consult with an orthopedic specialist c. instruct her to avoid the "W" sitting position d. encourage exercise to strengthen quadriceps

a

Which of the following does not place an infant at increased risk for sudden infant death syndrome? a. documented episodes of periodic breathing b. prematurity c. severe bronchopulmonary dysplasia d. apnea of prematurity

a

Which of the following would not be included in the management of pinworms? a. nutritional support and iron supplementation b. simultaneous treatment of all family members c. washing bed linen in hot water d. keeping fingernails short and clean

a

When counseling an adolescent with a family history of hyperinsulinemia and type 2 diabetes, the primary care pediatric nurse practitioner will recommend

avoiding processed breads.

BW, at 6 years of age, c/o sharp epigastric pain radiating to his back. Which labatory data would be consistent with these physical signs? a. decreased albumin b. elevated serum amylase c. elevated serum gastrin d. decreased serum protein

b

Justin, 11 yo, presents with c/o chronic diarrhea and abd pain. You note a 5 kg wt loss from last year's annual exam. Today, his exam reveals RLQ pain and perianal skin tags. The most likely dx is: a. encopresis b. crohn's dz c. irritable bowel dz d. ulcerative coliits

b

The appropriate mngmt of Osgood-shlatters dz includes: a. local injection of soluble corticosteroids b. decreasing activity applying ice and taking NSAIDs c. program strengthening and stretching for quadriceps d. casting in adduction for 6 weeks

b

The mother of 4 yo TW states that the infant has been irritable and has not been sleeping well. During the physical exam, you note papular lesions on his feet and erythematous papules over his back. To confirm your suspicion of scabies you would order a: a. wood's lamp examination b. microscopic skin scraping c. KOH prep of skin scraping d. skin culture

b

To promote normal growth in the child with cystic fibrosis, dietary management should include: a. limited fats and 50% more calories than usual daily allowances b. liberal fats and 50% more calories than usual daily allowances c. the usual # of calories as indicated by ht and wt d. limited fat and sodium in moderation

b

Which of the following is recommended as the standard immunizing agent for healthy preschool children? a. DTP b. DTaP c. DT d. Td

b

A 2-month-old infant with spina bifida and shunted hydrocephalus has a history of increased gagging with feedings, spitting up, intermittent stridor, and failure to thrive. The most likely explanation is: A) Tethered cord B) Shunt malfunction C) Arnold-Chiari deformity type II D) Esophageal structure

c

JP, 5 yo, presents with a hx of stool staining his underwear, evidence of bright red blood after wiping, and abd discomfort. The physical exam reveals moderate abd distention with midline abd mass. Rectal exam is positive for an impacted rectum and two small anal fissures. The priority in your management plan would be: a. increasing H2O and fiber in the diet, and limiting milk intake b. regular toilet sitting for 10 mins, 3 x a day c. 2 fleets enemas d. mineral oil after breakfast and before bed

c

LR, at 6 years of age, presents at clinic with a solitary nonprruritic lesion around his upper lip. Closer inspection reveals some vesicles and honey colored crusts. The treatment of choice for LR would be: a. acyclovir b. topical steroids c. topical antibiotics d. petrolatum/lanolin ointment

c

Mrs. J brings her 6 yo son in bc of "hives" that she describes as a red raised rash. Which finding below would support a dx of erythema multiforme rather than uriticaria? a. lesions that blanch with pressure b. eyelid edema c. lesions that are present for more than 24 hrs d. intense pruritus

c

When providing anticipatory guidance about infant development, you might teach parents that a normal infant could first transfer an object from hand to hand at which age? a. 2 mos b. 4 mos c. 7 mos d. 9 mos

c

When reviewing immunization protocols at your clinic, you are aware that the varicella vaccine can be administered to susceptible children beginning at what age a. 4 mos b. 6 mos c. 12 mos d. 15 mos

c

Which of the following best describes behavior associated with Piaget's concrete operations? a. learning primarily by trial and error b. interpreting events in relationship to themselves c. categorizing information into lower to higher classes d. drawing logical conclusions from observations

c

Which of the following physical findings in a 2 month old child warrants an immediate referral to a neurologist/neurosurgeon? a. head circumference growing faster than height and weight b. unresolved cephalhematoma c. rigid and immobile sagittal suture d. snapping sensation when pressure is applied to parietal bone

c

Which of the following vaccines is contraindicated for routine use in a 3 yo child for maintenance treatment of acute lymphocytic leukemia? a. DPT b. injectable influenza c. MMR d. hepatitis b

c

While completing the hip examination on newborn infant you were able to dislocate the infants right hip. The appropriate management plan will be to: A: triple diaper and reevaluate in two weeks B: recommend positioning prone while awake C: Referred to orthopedic specialist D: Order tight swaddling of the infant

c

JD, at 7 mos of age, is seen with a 2 day hx of diarrhea. He has had 3 to 4 wet diapers in the past 24 hrs. The anterior fontanel is slightly depressed. Capillary refill is normal. Which degree of dehydration is most consistent with these findings? a. 1 to 2% b. 3 to 5% c. 6-10% d. greater than 10%

c ( depressed fontanel & decreased UOP are indicators of moderate 8% dehydration)

Mrs. Chancellor contracted rubella while pregnant with 2 mos old Andrew. Andrew should be considered contagious for what time perioud? a. between 5 and 7 days b. until he is afebrils c. at least the first year of life d. he is not contagious

c (infants with congenital rubella may excrete the virus from the nasopharynx and in the urine for a year)

7.5yo Jimmy presents to your clinic with inspiratory stridor, drooling, and a temp of 105F. He insists on sitting up during the exam. What is your diagnosis? a. foreign body aspiration b. reactive airway disease c. croup d. epiglottitis

d

9yo Tom is in the clinic for a "cold" that lasted for 10 days. He has no complaints of breathing problems. A physical exam reveals temp of 100F, edematous cervical lymph nodes, mucopurulent drainage from the middle meatus, erythematous pharynx & malodorous breath. Tom's management should include: a. culture of nasal drainage b. radiograph of the sinuses c. measurement of erythrocyte sedimentation rate d. use of an antibiotic

d

A 15-year-old adolescent has pallor and fatigue. The CBC results are consistent with iron-deficiency anemia. What would be appropriate information to give this adolescent? A) Antacids increase the absorption of iron B) Dairy foods are good sources of dietary iron C) Juices fortified with vitamin C inhibit the absorption of nonheme iron D) Tannin-containing products, such as tea, inhibit the absorption of nonheme iron

d

A 2 wk old infant is in the clinic for a scheduled weight check. The best indication that a 2 wk old infant is receiving adequate breastmilk is that the baby: a. passes a least 4 stools a day b. feeds every 3 hrs c. voids 4 times a day d. has regained birth wt

d

A 7yo child in your caseload has recently been placed on methylphenidate for behavioral concerns associated with ADHD. Which of the following side effects are not associated with this drugs? a. decreased appetite b. weight loss c. irritability d. decreased heart rate

d

A well-nourished 10 yo girl presents to the clinic with low-grade fever, sore throat, fatigue and malaise, and left upper abd pain. Based on clinical presentation and lab results, a dx of infectious mononuclesis is made. Which of the presenting signs and symptoms requires further investigation immediately? a. low-grade fever b. sore throat with lymphadenopathy c. fatigue and malaise d. left upper abd pain

d

The PNP is examining a neonate with a heart murmur. The S2 sound is loudest at the apex. The respiratory rate is 65 breaths per minute, and the heart rate is 180 beats per minute. Which of the following would be an appropriate action by the PNP? A) Reevaluate the neonate in 24 hours B) Increase the number of oral feedings C) Order cardiac catheterization D) Refer the neonate to a cardiologist

d

The least likely physical finding in a 2 mos old infant with CHF is: a. tachypnea b. tachycardia c. hepatomegaly d. pedal edema

d

The mother of 4 mos old ND reports episodes of vomiting and diarrhea beginning 2 days ago. He has also had several episodes of screaming and drawing up his legs. Prior to this he has been healthy with a normal wt gain. The likely dx is. a. incarcerated hernia b. gastroenteritis c. intussusception d. pyloric stenosis

d

The parents of a son with hemophilia are considering another pregnancy. The mother has been identified as a carrier of the hemophilia gene. What chance does each female offspring have of having hemophilia. a. 100% b. 50% c. 25% d. none

d

What is the appropriate treatment for genu varum in a 15 mos old child? a. passive exercise w/ each diaper change b. Denis Browne splint at night c. Blount brace at night d. no treatment is warrented

d

When examining 7 mos old RV, you note red scaly plaques in his diaper area, particularly in the inguinal folds, with satellite lesions on his abdomen. The appropriate treatment would be: a. petrolatum/lanolin ointment b. petroleum jelly c. zinc oxide d. nystatitn

d

Which of the following is an expected finding after treatement of acute suppurative otitis media? a. otitis externa b. central auditory dysfunction c. functional hearing loss d. middle ear effusion

d

Which of the following is not a sign of readiness to toilet train? a. can sit for extended periods b. can follow directions c. occasional waking from naps with dry diapers d. regularity of bowel movements

d

Which of the following is not an expected finding in a child with myocarditis? a. persistent tachycardia b. hx of antecedent flulike illness c. a gallop rhythm d. a significant heart murmur

d

Which of the following is true regarding innocent murmurs? a. the murmur is often holosystolic b. prompt referral to a cardiologist is indicated c. a precordial thrill is present d. the murmur is low in intensity, grade 1 through 3

d

Which of the following scenarios is suggestive of a child who may not be ready to enter first grade? An inability to: a. recognize six colors and remember one's phone number b. accurately use pronouns c. empathize with others d. count to five and draw a person with three parts

d

The primary care pediatric nurse practitioner is evaluating an 11monthold infant who has had three viral respiratory illnesses causing bronchiolitis. The child's parents both have seasonal allergies and ask whether the infant may have asthma. What will the nurse practitioner tell the parents?

"Although it is likely, based on family history, it is too soon to tell."

When do you see fever and rash after an MMR vaccine? 1-2 weeks Less than 7 days At the month mark You never see a rash or fever with the MMR

1-2 weeks

1. Most stage-based theories of development focus pri-marily on: a. The continuity of development b. The discontinuity of development c. Persistence of inherent personality characteristics d. The influence of context on development

1. B: These theories address deviations from develop-mental progress norms.

15. Heart Rate increases how many BPM with each elevation of degree of centigrade? 5-7 10-15 20-30 0-5

10-15

10. The pincer grasp is a fine motor skill that involves the ability to pick up a small object such as a raisin or piece of cereal with the thumb and forefinger and that usually is mastered around: a. 4 months b. 6 months c. 9 months d. 16 months

10. C: This is a developmental norm for a child of 9 months.

17. Fever phobia by parents is unnecessary because fever does not produce cellular level damage until what temperature? 104 102 for two hours 105.3 108.5 for 30 minutes

105.3

11. You would be concerned about the language develop-ment of a child who: a. Repeats simple phrases at 32 months b. Stutters when excited or tired at the age of 7 years c. Has a vocabulary of 10 words at 12 months d. Pronounces words that are not understandable at 24 months

11. B: Stuttering associated with fatigue or excitement is not unusual in a preschooler but may indicate a more pervasive problem in a 7-year-old.

12. The most common temperamental profile is: a. Easy b. Difficult c. Slow-to-warm-up d. Intermediate

12. A: Approximately 40% of children are described as having an easy (rhythmic, approachable, adaptive) temperament.

When do you call puberty delayed? 13 15 11 12

13

13. The underlying emotion of an insecurely attached (avoidant) relationship is: a. Ambivalence b. Deprivation c. Anger d. Conditional love

13. C: Attachment is the bond that develops throughout the first year of life; underlying anger characterizes avoidance or an insecure bond.

14. The stage of cognitive development that Piaget described as characteristic of the way preschoolers think is the: a. Preoperational stage b. Mental combinations stage c. Tertiary circular function stage d. Sensorimotor stage

14. A: Piaget characterizes preschoolers as preoperational thinkers.

15. A preschool boy whose parents have separated and are beginning divorce procedures: a. May think that he caused the divorce by misbehaving b. Should not be told of the impending divorce until the parents are sure of their decision c. Is likely to experience gender identity confusion d. Should be able to make a decision about which parent he prefers living with

15. A: Preschoolers are characterized by egocentrism; they think the world revolves around them and that everything that happens is because they did or did not do something.

16. Which behavior would you expect to decrease during the preschool years? a. Rough-and-tumble play b. Instrumental aggression c. Hostile aggression d. Cooperative play

16. B: Preschoolers strongly defend what is "theirs," be it a toy or a space or a special privilege; this aggres-sion wanes as they begin to understand sharing and appropriate impulse control.

17. A preschool child who says that the sky is blue because it is his favorite color is illustrating the concept of: a. Symbolic thinking b. Egocentrism c. Centration d. Imaginary audience

17. B: Egocentrism is the hallmark of preschoolers; there is little they think they do not control, from the weather to the color of the sky.

18. Which of the following strategies would not be appro-priate to include as part of your management of a 9-year-old boy who is obese? a. Referral to nutritionist for weight reduction plan b. Increase physical exercise c. Behavior modification strategies to deal with stress and/or reinforce treatment plan d. Involve family in management program

18. A: Treatment of obesity in a 9-year-old requires a mul-tifactorial approach, including "eating healthy" and exercise, and needs not necessarily focus on weight reduction. The goal is to prevent weight gain and maintain weight until linear growth catches up.1

20. The first physical sign indicating the onset of female puberty is: a. Sparsely distributed, fine, pale pubic hairs b. Breast buds c. Menarche d. Peak height velocity

20. B: The first sign of female puberty is the development of breast buds, closely followed by the development of pubic hair. Peak height velocity and menarche gener-ally occur at age 12½ years.

21. Which of the following findings would be helpful in distinguishing obesity from large body frame in an adolescent who is concerned with her weight? a. Tricep skinfold measurement b. Weight-for-height ratio c. Body mass index d. Percent of ideal body weight

21. C: Body mass index is generally considered the best index for evaluation of weight; it correlates weight with height.22. C: Soft tissue injuries, such as bruises, abrasions, and lacerations, are the most common form of abuse, occurring in all age groups.23. D: Prader-Willi is associated with uncontrolled appe-tite and obesity; it has no outward sign that could be confused with signs of child abuse.24. D: Anger is not usually a manifestation of ADHD, whereas high distractibility and inability to sit quietly or follow directions are red flags that a child may have ADHD.25. C: Eating disorders with a purging component are characterized by constipation, rather than diarrhea, resulting from chronic laxative use.26. B: This child may be at risk for a mental health issue, but he appears to be coping well and immediate refer-ral is not indicated.27. B: There is a very high incidence of anorexia nervosa in ballet dancers and gymnasts.28. C: Opiates cause constricted pupils. Amphetamines and LSD cause dilated pupils, and nicotine generally does not have any effect on the pupils

22. The most common form of child abuse seen in pedi-atric primary care is: a. Burns b. Fractures c. Soft tissue injuries d. Shaken baby syndrome

22. C: Soft tissue injuries, such as bruises, abrasions, and lacerations, are the most common form of abuse, occurring in all age groups.

23. A differential diagnosis for child abuse would include all of the following except: a. Birth marks b. Unintentional injury c. Inadequate parenting d. Prader-Willi syndrome

23. D: Prader-Willi is associated with uncontrolled appe-tite and obesity; it has no outward sign that could be confused with signs of child abuse.

24. Which of the following symptoms is not typical of a child with ADHD? a. Easily distracted b. Difficulty playing quietly c. Doesn't follow directions d. Frequently angry and resentful

24. D: Anger is not usually a manifestation of ADHD, whereas high distractibility and inability to sit quietly or follow directions are red flags that a child may have ADHD.

25. Which of the following clinical findings would not suggest an eating disorder with a purging component? a. Sore throat b. Brittle nails c. Diarrhea d. Finger calluses

25. C: Eating disorders with a purging component are characterized by constipation, rather than diarrhea, resulting from chronic laxative use.

The primary care pediatric nurse practitioner is prescribing ibuprofen for a 25 kg child with JIA who has oligoarthitis. If the child will take 4 doses per day, what is the maximum amount the child will receive per dose?

250 mg

26. Which of the following situations does not necessarily warrant immediate mental health assessment and/or referral? a. 13-year-old girl who has been "down" for the last month with varied somatic complaints b. 9-year-old boy whose parents recently separated and filed for a divorce and who seems to be doing well c. 16-year-old girl who has a history of long-stand-ing depression and has started to have "slipping grades" at school d. 15-year-old boy who expresses suicidal thoughts

26. B: This child may be at risk for a mental health issue, but he appears to be coping well and immediate refer-ral is not indicated.

27. Which adolescent would be at greatest risk for devel-oping anorexia nervosa? a. 12-year-old female who just had her first period b. 14-year-old female gymnast c. 16-year-old male runner d. 18-year-old female college student

27. B: There is a very high incidence of anorexia nervosa in ballet dancers and gymnasts.

28. Which of the following substances is associated with pupillary constriction? a. Amphetamines b. LSD c. Heroin d. Nicotine

28. C: Opiates cause constricted pupils. Amphetamines and LSD cause dilated pupils, and nicotine generally does not have any effect on the pupils

29. A risk factor that is common to many psychosocial pediatric problems, including failure to thrive, conduct or oppositional disorders, and childhood depression, is: a. Maternal depression or other psychiatric disorder b. Substance abuse c. Prematurity d. History of sexual abuse

29. A: All of these problems have multifactorial etiolo-gies; maternal psychiatric disorders that could affect parenting or development of a secure and stable bond are important factors to consider.

31. The diagnostic criteria for autism spectrum disorder include: a. A noted lack of back-and-forth conversation b. Tolerance of flexibility with routines c. Fascination with light or movement d. Abnormal eye movements or body language

31. B: Children with ASD do not tolerate alterations to prescribed routines and may benefit from predictable schedules.

32. In addition to specific academic skill deficits, learning disabilities are commonly associated with which of the following characteristics? a. Perceptual-motor impairments, normal motor function b. Perceptual-motor impairments, impulsiveness c. Perceptual-motor impairments, Down syndrome d. Lack of impulsiveness, perceptual-motor impairment

32. B: Children with LD may demonstrate impulsive behaviors as attention-getting behaviors

33. H. O. is a 5-year-old Vietnamese child who has fallen off of his growth curve. The best intervention would be to: a. Suggest high-calorie breakfast drinks as supplements b. Incorporate traditional foods into a manage-ment plan that will provide increased calories and nutrients c. Educate the family on the need for increased calo-ries and nutrients d. Refer family to a growth clinic for evaluation

33. B: It is important for healthcare professionals to understand the cultural norms and perspectives of others. This often helps in compliance with sug-gestions for improved health. Asian families, out of respect, often do not ask questions or challenge advice. By understanding their food patterns and incorporating that into a diet plan, the healthcare pro-fessional may increase compliance.

34. While taking the history of 6-month-old E. M., you learn that she is not sleeping through the night and will not fall back to sleep without the parents rocking or feeding her. This is an example of: a. Somnambulism b. Pavor nocturnus c. Learned behavior d. Delayed sleep phase

34. C: Sleepwalking (somnambulism) and pavor noctur-nus (night terrors) are sleep disturbances that occur in school-age and preschool-age children, respec-tively. Learned behavior is a result of parents interfer-ing with the child's attempts to return to sleep without stimulation from the parents.

35. Which of the following scenarios is suggestive of a child who may not be ready to enter first grade? An inability to: a. Recognize six colors and remember one's phone number b. Accurately use pronouns c. Empathize with others d. Count to five and draw a person with three parts

35. D: Children entering first grade should have the requi-site skills to master the tasks they will encounter. This includes language, fine and gross motor skills, and personal and social skills. At this age the child should be able to draw a person with at least six parts and count to 10 or more.

36. While examining 10-year-old R. M.'s teeth, you note that the upper incisors slightly overlap the lower inci-sors. The second and lower first molars are absent. Your assessment is: a. Malocclusion b. Delayed mandibular dentition c. Normal dentition d. Hyperdontia

36. B: The mandibular (lower) molars usually erupt between ages 6 and 7. Even allowing for individual variation, this is a considerable delay. Hyperdontiarefers to supernumerary teeth.

37. The mother of 5-year-old D. W. is concerned that her son often cheats when playing board games with his older sister. What is the most appropriate response to D. W.'s behavior? a. Encourage the parent to use 5-minute time-outs when cheating occurs. b. Explain that D. W. is developmentally unable to comprehend rigid rules. c. Make sure that D. W. understands the rules before starting to play the game. d. Explain to D. W. that cheating is like lying and is not acceptable behavior.

37. B: Developmentally, the concept of cheating is not well understood until 7 years of age. The idea of play-ing fairly to ensure everyone an equal chance occurs with maturity and the ability to differentiate among moral choices.

38. Which of the following physical findings in a 2-month-old child warrants an immediate referral to a neurologist/neurosurgeon? a. Head circumference growing faster than height and weight b. Unresolved cephalhematoma c. Rigid and immobile sagittal suture d. Snapping sensation when pressure is applied to parietal bone

38. C: Ridged and immobile sutures indicate premature fusing resulting in craniosynostosis. For proper brain growth, sutures need to approximate each other yet remain mobile.

39. While listening to 2½-year-old K. L. talk, you note that she frequently omits final consonants and her sen-tences are two to three words in length. The appropri-ate plan of care would be: a. Routine follow-up at the next well-child visit b. Referring for hearing screen c. Assessing for developmental delays d. Referring to a speech pathologist

39. A: Children aged 2 to 3 years have several articula-tion dysfluencies, among them is the dropping of final consonants. Two- to three-word sentences are normal for the 24- to 30-month-old child.

4. Which of the following findings would most likely be associated with asymmetric intrauterine growth retardation? a. Weight, length, and head circumference ranging from 3rd to 5th percentile b. Heavy maternal smoking throughout pregnancy c. Weight at 3rd percentile and length at 25th d. Gestational diabetes

4. C: The weight and length are at significantly differ-ent percentiles; if the IUGR were symmetrical, these would be at the same percentile.

Boys should attain Tanner stage 5 within how many years? 3 2 4.5 6

4.5

40. The mother of 3-year-old G. W. reports that he has begun to stutter. Further probing reveals that the stut-tering occurs frequently and lasts 1 to 2 seconds. G. W. does not seem bothered by the stuttering. The appro-priate management would be: a. Referral to a speech pathologist b. Referral for an evaluation for an anxiety disorder c. Reassuring the mother that this is a mild problem d. Demonstrating to G. W. slow, deep breathing before talking

40. C: This represents a mild stuttering problem but does not warrant immediate referral unless the child or parent is increasingly concerned or if it continues indefinitely.

41. You would expect a school-age child to: a. Grow 1.5 inches per year b. Grow 0.5 inch per year c. Gain about 6 pounds per year d. Gain about 3 pounds per year

41. C: The recognized standard of physical growth of school-age children is to gain 5 to 7 pounds per year and grow about 2.5 inches per year.

42. During 8-month-old L. B.'s physical examination, the father boasts that L. B. is going to be a left-handed batter since he prefers doing everything with his left hand. The appropriate response would be to: a. Ask if others in the family are left handed b. Suggest play activities that require using both hands c. Present toys more often to the right hand d. Perform a careful neurologic examination

42. D: Handedness before a year is cause for concern and may indicate cerebral palsy. A neurologic examination is indicated. The examiner should carefully assess for increase in deep tendon reflexes and tone.

43. Which of the following best describes behavior associ-ated with Piaget's concrete operational phase?a. Learning primarily by trial and error b. Interpreting events in relationship to themselves c. Categorizing information into lower or higher classes d. Drawing logical conclusions from observations

43. C: Concrete operations occur during the school-age years as children begin to understand the character-istics of things and objects. Classification is a thought process that develops during this time.

44. Jeffrey, at 8 years of age, has been diagnosed with ADHD and is receiving stimulant medication. Which of the following interventions would be least helpful? a. Monthly height and weight checks b. Small frequent meals and snacks c. High-calorie supplemental drinks d. Elimination of refined sugar from diet

44. D: Stimulant medication may decrease the appetite, so careful monitoring of growth and a nutritional plan that encourages adequate calories are important. There is no sound evidence that sugar or artificial additives play a role in ADHD.

45. The principle that growth and development become increasingly integrated is best demonstrated by: a. Gaining head control before raising the chest b. Bringing cup to mouth, tipping, and swallowing c. Rolling over before sitting d. Grasping with fist before using fingers

45. B: Infants must first develop hand-mouth coordina-tion before incorporating tipping and swallowing, which is a more integrated function. Head control before raising the chest demonstrates the principle of cephalocaudal progression. Options C and D suggest proximal-distal progression.

46. In males, Tanner stage III can be distinguished from Tanner stage II by: a. Fine, downy pubic hair at the base of penis b. Adult-like pubic hair not extending to thighs c. Penile growth in width d. Penile growth in length

46. D: Most penile growth in Tanner stage III is in length rather than width because of underdevelopment of the corpora cavernosa. Fine, downy pubic hair appears in stage II, and adult-like appearance occurs in stage IV.

47. T. J., 13 years old, reluctantly shares with you that his "chest hurts." On physical examination, you note uni-lateral breast enlargement, which is tender to palpa-tion. You suspect physiologic gynecomastia. Which Tanner stage would support that diagnosis? a. Tanner stage I b. Tanner stage III. c.Tanner stage IV d. Tanner stage V

47. B: Physiologic gynecomastia is a common clinical finding in young adolescent males. It is usually pres-ent during Tanner stage III.

48. During a physical examination of 10½-year-old Melissa, you note the appearance of breast buds. You tell her that she can expect which of the following in approximately 2 years? a. Growth of pubic hair b. Peak height velocity c. Onset of menses d. Axillary hair

48. C: Understanding the sequencing of pubertal devel-opment is important, but it must be remembered that individual timing may differ. In the female patient, pubic hair, axillary hair, and the peak height velocity generally occur before menarche.

Girls should have menarche how many years after thelarche? 4 5 2 1

5

5. Early reflexive responses that are not related to sur-vival include all but: a. Babinski b. Moro c. Swimming d. Rooting

5. D: Rooting (i.e., moving the head to locate the nipple) is a key survival reflex.6. A: An infant's weight should approximately triple by the age of 1 year.7. A: The development of an infant-caregiver bond is key to the prevention of long-term psychological effects associated with deprivation and/or failure to develop secure, stable bonds.8. C: Transactional theory explains risks and protective factors associated with resilience and vulnerability. It may explain some of the environmental factors asso-ciated with FTT.9. B: These are gross developmental norms associated with the 6-month-old child.10. C: This is a developmental norm for a child of 9 months.11. B: Stuttering associated with fatigue or excitement is not unusual in a preschooler but may indicate a more pervasive problem in a 7-year-old.

50. An increase in which of the following behaviors is seen more frequently in late rather than in early adolescence? a. Value conflict with parents b. Focus on physical appearance c. Peer group involvement d. Understanding inner motivations of others

50. D: Late adolescence is characterized by increased autonomy and beginning to appreciate the complexi-ties and motivations of other people's behaviors.

6. The most likely weight of a 1-year-old child whose weight at birth was 6 pounds would be: a. 19-20 pounds b. 13-14 pounds c. 25-26 pounds d. Impossible to estimate

6. A: An infant's weight should approximately triple by the age of 1 year.

9. Most healthy infants are able to reach, grasp, and hold on to a rattle or other small toy by about: a. 2 months b. 6 months c. 8 months d. 10 months

9. B: These are gross developmental norms associated with the 6-month-old child.

19. Which of the following issues or concepts is relevant to the school-aged child? a. Operational thinking b. Initiative c. Concrete operations d. Separation-individuation

9. C: Concrete operational thinking is key to success-ful adaptation to school. It involves the concepts of reversibility, conservation, classification, and seriation.

3. The parents of a 12yearold child are concerned that some of the child's older classmates may be a bad influence on their child, who, they say, has been raised to believe in right and wrong. What will the primary care pediatric nurse practitioner tell the parent? A. Allowing the child to make poor choices and accept consequences is important for learning values B. Children at this age have a high regard for authority and social norms, so this is not likely to happen C. Moral values instilled in the early schoolage period will persist throughout childhood D. The pressures from outside influences may supersede parental teachings and should be confronted

? D. The pressures from outside influences may supersede parental teachings and should be confronted

7. A 17yearold sexually active female who began having periods at age 14 reports having moderate to severe dull lower abdominal pain associated predominantly with periods but that occurs at other times as well. The history reveals a recent onset of these symptoms. A pregnancy test is negative. Which course of action is most important? a. Perform a full diagnostic workup to evaluate potential causes. b. Prescribe a prostaglandin synthetase inhibitor. c. Start a 3 to 6month trial of oral contraceptive pills. d. Suggest using transcutaneous electrical nerve stimulation.

A

A five month old boy, a former 28 week Premature infant, is being evaluated in your practice because of a concern about delayed development. In formulating the differential diagnosis, you keep in mind that spastic cerebral palsy is characterized by: A: Increased deep tendon reflexes and sustained clonus B: Dystonic posturing C: Abnormal involuntary movements D: nystagmus

A

A five-year-old girl presents for a school physical with a complaint of hyperactivity, A mild developmental delay, aversion of gaze, hand mannerism, a long thin face with a slightly dysmorphic ear. What is the most likely diagnosis? A. Fragile X B. Turner's syndrome C. Fetal alcohol syndrome D. Williams syndrome

A

A macular, salmon to red colored rash with irregular borders and central clearing is typical of which of the following? A: Systemic juvenile arthritis B: Lyme disease C: Systemic lupus erythematosus D: rheumatic fever

A

A newborn presents with microcephaly, low-set ears, prominent occiput, micrognathia, heart murmur, and clenched hands with overriding fingers and crossed thumb. What is the best diagnostic test? A. Karyotype B. Amino acid urine screen C. Newborn screening D. Methylation rest

A

A two month old child presents after an episode of sepsis for a follow-up visit. He has gained 2 pounds since birth and has a decrease in head circumference from the 50th to the 25th percentile. In addition, he has and inguinal and axillary adenopathy of 1 cm, decrease in head, and has hepatomegaly. Which of the following is included in the infectious disease differential? A. Herpes simplex type 2 infection B. Human immunodeficiency virus infection C. Chlamydia infection D. Congenital gonorrhea

A

And 11-year-old girl presents at a well child visit with symptoms of polyuria and polydipsia. Which of the following diagnoses must be ruled out A. Diabetes mellitus B. Hyperthyroidism C. Adrenalcortical insufficiency D. Nephrotic syndrome

A

During a well child exam of school-age child, you learn that the child has been having angry episodes at school. You observe the child to appear withdrawn and sad. Which action is appropriate? a. Ask the child and parent about stressors at home. b. Make a referral to a child behavioral specialist. c. Provide info about anger management. d. Suggest consideration of a different classroom

A

In a child with suspected meningitis, Lumbar puncture should be delayed and a CT scan obtained first in which of the following circumstances? A. There are signs of increased ICP B. The child has tachycardia C. The child has a negative brudzinski D. The WBC is > 10,000/mm3

A

In working with a child with Tourette syndrome and his family, the nurse practitioner should be aware that: A. Symptoms become more unpredictable during adolescence B. Boys are more likely than girls to exhibit behavioral problems such as obsessive compulsive disorder C. Sleep disturbance from tics increases with age D. There are several medications now available to control the tics without interfering with daily functioning

A

Mrs. C has brought her two-year-old daughter April to the clinic with complaints of anorexia and irritability for the past several weeks. You know that she is afebrile And appears pale. Based on the signs and symptoms, which initial action is appropriate? A. Ask Mrs. C to describe April's diet, including the specific foods she's eating, prior to the onset of the symptoms B. Prescribe supplemental iron therapy to be given three times a day C. Order laboratory work to assess red blood cell count and indices D. Referrer April to a pediatrician for further evaluation

A

Primary dysmenorrhea is due to: A. Elevated prostaglandin levels B. Pelvic inflammatory disease C. Endometriosis D. Fibroids

A

Primary immunization is of paramount importance for preventing meningitis especially in young children that is caused by: A. Hib B. Neisseria meningitidis C. E. Coli D. Klebsiella pneumonia

A

Routine lab studies have revealed that a two-year-old child has a decreased level of serum ferritin. Red cell count and indices are within normal limits for age. Based on this information, you may assume that the child: A. May have stage I iron deficiency anemia B. Likely has stage II iron deficiency anemia C. Likely has stage III iron deficiency anemia D. Does not have any stage of anemia

A

Systematically develop statements to assist practitioner and patient about appropriate care for specific clinical outcomes are called: A. Clinical practice guideline B. Benchmarking C. Protocol requirements D. Written collaborative agreement

A

The mobilization , monitoring, and control of resources used by a patient over the course of an illness is called: A. Case management B. Quality improvement C. Quality assurance D. Risk management

A

The mother of a well-developed, full-term three week old boy brings him to the clinic because he has been fussy and not eating well for the past week. A CBC reveals that he is anemic. When exploring the etiology of the anemia, is important to know that which of the following is not a common cause of anemia in the newborn? A. Dietary iron deficiency B. Blood loss C. Hemolysis D. Decreased RBC production

A

The pathophysiology of type one diabetes is: A. Autoimmune destruction of the pancreatic beta cells B. Primary insulin receptor resistance C. Increased hepatic glucose production D. Reduce glucose uptake by target tissue

A

The routine screening of a newborn in your practice indicates that the baby has congenital hypothyroidism and is in need of a referral to a pediatric endocrinologist. The treatment of choice for congenital acquired hypothyroidism is: A. Levothyroxine B. Propylthiouracil C. Potassium iodide D. Radiation therapy

A

Upon examination of a two-month-old boy, you noticed a swelling in the right inguinal canal. Your differential diagnosis would not include: A. Diastasis recti B. Hydrocele of the spermatic cord C. Inguinal hernia D. Lymphadenopathy

A

When lab results reveal hypochromic, microcytic anemia in a two-year-old child, differential diagnosis must include: A. Lead poisoning B. Pernicious anemia C. Hemophilia D. Folic acid deficiency

A

Which of the following is a measure of childhood intelligence? A. Wechsler scales B. Denver II C. Bayley scales D. Vineland scakes

A

Which of the following is a nontreponemal test for syphilis? A. Venereal disease research laboratory microscopic slide test B. Fluorescent treponemal antibody absorbed C. Treponema pallium particle agglutination D. TORCH titer

A

Which of the following is an absolute contraindication for use of combined hormonal contraceptives and adolescent women: A. Hx of thromboembolism B. Diabetes C. Smokes 5 cigarettes per day D. Sickle cell dz

A

Which of the following is not true with regard to hypospadias? A. The meatus is formed along the dorsum of the penis B. It is one of the most common penile abnormalities C. Circumcision should be deferred D. A referral for an endocrine evaluation may be indicated

A

Which of the following is the primary diagnostic tool used in the evaluation of seizure disorder? A: EEG B: Cerebral blood flow studies C: CT scan D: MRI

A

Which of the following is true with regard to advance practice licensure? A. It is granted by some states based on specialty certification B. It will guarantee reimbursement C. It may be obtained on a national basis D. It is a federal process verifying that a PNP has met standards for specialty practice

A

Which of the following signs is not characteristic of generalized seizures? A: Unilateral motor manifestations B: disturbance of consciousness C: tonic stiffening of the trunk D: Simultaneous and symmetric cerebral hemisphere discharge

A

While evaluating the CBC results of a three-year-old child, the practitioner notes that in addition to hypochromia and microcytosis of the red cell, there are many poikilocytes and target cells. Based on this finding, differential diagnoses must include: A. Thalassemia major B. Iron deficiency anemia C. Pernicious anemia D. Vitamin B 12 deficiency

A

You are examining a school-age child who complains of frequent stomach pain and HAs. The parent reports that the child misses several days of school each month. The child has a normal exam. Before proceeding w/further diagnostic tests, what will you initially ask the parent? a. About the timing of the symptoms each day and during the week. b. How well the child performs in school and in extracurricular activities. c. If the parent feels a strong need to protect the child from problems. d. Whether there are any unusual stressors or circumstances at home.

A

You are preparing to conduct a well child assessment of an 8yo. How will you begin the exam? a. Ask the child about school, friends, home activities, and sports. b. Discuss the purpose of the visit and explain the procedures that will be performed. c. Offer age-appropriate info about usual developmental tasks. d. Provide info about healthy nutrition and physical activities.

A

You have prescribed iron supplements for a three-year-old child. When instructing the parents about how to give the iron preparation, it is important to tell them that iron: A. Is best absorbed on an empty stomach B. Is best absorbed when given with meals C. Is the best absorbed when given with milk D. Should not be given near bedtime

A

You perform a physical exam on a 12yo and note poor hygiene and inappropriate clothes for the weather. The child's mom appears clean and well-dressed. The child reports getting 6-7hrs of sleep each night because of texting w/friends late each evening. What action will help promote healthy practices? a. Discuss setting clear expectations about self-care w/mom b. Give the child info about sleep and self-care c. Reassure the mom that this "non-compliance" is temporary d. Tell the mom that experimenting with self-care behaviors is normal

A

A child has been diagnosed with diabetic ketoacidosis (DKA) and is in the pediatric intensive care unit. Which nursing diagnosis does the nurse direct interventions toward as the priority? A. Fluid volume deficit B. Ineffective breathing patterns C. Knowledge deficit D. Risk for infection

A Because the child with DKA can be severely dehydrated, priority interventions are directed toward the goal of rehydration. Breathing patterns for the child in DKA may consist of Kussmaul respirations, which are actually the body's way of trying to compensate for the acidosis. As the glucose decreases, this will self-correct. Knowledge deficit can be addressed when the child's condition is stable. Risk for infection is always a potential diagnosis, and the nurse ensures proper technique to prevent this from occurring. But because the dehydration is so severe, this takes priority.

Which of the following signs or symptoms is not associated with congenital adrenal hyperplasia: A. Hypernatremia B. Progressive weight loss C. Dehydration D. Hyperkalemia

A ( Hyponatremia is a sign of CAH as there is excess of sodium loss through the kidneys and an inability to maintain serum electrolyte balance)

A foul-smelling vaginal discharge that emits a fishy odor when combined with 10% potassium hydroxide is most likely due to: A. Gardnerella vaginalis B. Candida albicans C. Chlamydia trachomatis D. N. Gonorrhoeae

A (bacterial vaginosis, often caused by an overgrowth of GV, is suspected on the basis of malodorous vaginal discharge and positive "whiff" test when mixing the discharge w/ potassium hydroxide.)

Jay is a nine-year-old boy who has had no significant health problems by history, but his mother is very concerned because he's wetting himself. As you begin your history and physical exam, you keep in mind that the most common type of in enuresis in school age children is: A. Primary nocturnal enuresis B. Occasional daytime enuresis C. Secondary nocturnal enuresis D. Primary diurnal enuresis

A [Primary nocturnal enuresis occurs and 90% of enuretic children (For example they are wet only at night during sleep and have never had a sustained period of dryness)]

Congenital adrenal hyperplasia- females will have Ambiguous genitalia Enlarged breast tissue Deficiency of enzymes in cortisol pathway A and C

A and C

Hypothyroidism has just been diagnosed in a 5-year-old child, and thyroid replacement therapy has been initiated. Anticipatory guidance for the child and family should include: A) Emphasizing the importance of compliance and periodic monitoring of the child's response to therapy B) Initiating appropriate referrals for the child regarding possible mental retardation caused by hypothyroidism C) Helping the family accept the child's short stature, which cannot be prevented but can be coped with in a healthy manner D) Referring the child to a dermatologist for treatment of mixed edematous skin changes

A) Emphasizing the importance of compliance and periodic monitoring of the child's response to therapy

Constitutional growth delay is a common variation and what clinical findings would you see? Circle all that apply: Normal length/ weight at birth Slowed linear growth at 1-3 years Height at or just below 3rd percentile Delayed puberty History of similar pattern in family A, B, C, D, E A, D, E

A, B, C, D, E 111 000

8. The mother of a 6monthold infant is distressed because the infant can say "dada" but not "mama" and asks the primary care pediatric nurse practitioner why this is when she is the one who spends more time with the infant. How will the nurse practitioner respond? A. "At this age, your baby does not understand the meaning of sounds." B. "Babies at this age cannot make the 'ma' sound." C. "Most sounds made by babies at this age are accidental." D. "This may mean that your baby doesn't hear well."

A. "At this age, your baby does not understand the meaning of sounds."

11. The primary care pediatric nurse practitioner is examining a schoolage child who complains of frequent stomach pain and headaches. The parent reports that the child misses several days of school each month. The child has a normal exam. Before proceeding with further diagnostic tests, what will the nurse practitioner initially ask the parent A. About the timing of the symptoms each day and during the week B. How well the child performs in school and in extracurricular activities C. If the parent feels a strong need to protect the child from problems D. Whether there are any unusual stressors or circumstances at home

A. About the timing of the symptoms each day and during the week

5. The primary care pediatric nurse practitioner is counseling the parents of a toddler about appropriate discipline. The parents report that the child is very active and curious, and they are worried about the potential for injury. What will the pediatric nurse practitioner recommend? A. Allow the child to explore and experiment while providing appropriate limits. B. Be present while the child plays to continually teach the child what is appropriate. C. Let the child experiment at will and to make mistakes in order to learn. D. Say "no" whenever the child does something that is not acceptable.

A. Allow the child to explore and experiment while providing appropriate limits

5. The primary care pediatric nurse practitioner is counseling an adolescent who was recently hospitalized for an asthma exacerbation and learns that the child usually forgets to use twice daily inhaled corticosteroid medications that are supposed to be given at 0800 and 2000 each day. Which strategy may be useful in this case to improve adherence? A. Ask the adolescent to identify two times each day that may work better. B. Consider having the school nurse supervise medication administration. C. Prescribing a daily oral corticosteroid medication instead. D. Suggest that the parent enforce the medication regimen each day.

A. Ask the adolescent to identify two times each day that may work better.

5. The primary care pediatric nurse practitioner is preparing to conduct a well child assessment of an 8yearold child. How will the nurse practitioner begin the exam? A. Ask the child about school, friends, home activities, and sports B. Discuss the purpose of the visit and explain the procedures that will be performed C. Offer age appropriate information about usual developmental tasks D. Provide information about healthy nutrition and physical activities

A. Ask the child about school, friends, home activities, and sports

4. During a well child exam of a schoolage child, the primary care pediatric nurse practitioner learns that the child has been having angry episodes at school. The nurse practitioner observes the child to appear withdrawn and sad. Which action is appropriate? A. Ask the child and the parent about stressors at home B. Make a referral to a child behavioral specialist C. Provide information about anger management D. Suggest consideration of a different classroom

A. Ask the child and the parent about stressors at home

1. The primary care pediatric nurse practitioner is evaluating a 2yearold with a documented speech delay. Screenings to assess motor skills and cognition are normal, and the child passed a recent hearing test. What will the pediatric nurse practitioner do next? A. Ask the child's parents whether they read to the child. B. Give parents educational materials to encourage speech. C. Refer the child to an early intervention program. D. Suggest that they purchase ageappropriate music videos.

A. Ask the child's parents whether they read to the child

1. The primary care pediatric nurse practitioner is performing a well child exam on a 4monthold infant who is nursing exclusively. The mother reports that the infant has had a marked decrease in the number of stools each day, from 3 to 5 stools each day to only one stool every other day. How will the nurse practitioner respond? A. Ask the mother to describe the color and consistency of the stools. B. Explain to the mother that breastfed infants should have daily stools. C. Recommend using a glycerin suppository as needed. D. Suggest to the mother that she increase her intake of fluids.

A. Ask the mother to describe the color and consistency of the stools.

4. The primary care pediatric nurse practitioner is discussing toileting issues with the parent of a 3yearold toddler who reports that the child has been toilet trained for several months but has recently been refusing to have bowel movements and is becoming constipated. What will the nurse practitioner do? A. Ask the parent about bathroom facilities in the child's day care. B. Refer the child to a gastroenterologist for evaluation of pathology. C. Suggest putting the child in diapers and resuming toilet training in a few weeks. D. Tell the parent that this represents a developmental delay.

A. Ask the parent about bathroom facilities in the child's day care.

7. The primary care pediatric nurse practitioner sees a developmentally delayed toddler for an initial visit. The family has just moved to the area and asks the nurse practitioner about community services and resources for their child. What should the nurse practitioner do initially? A. Ask the parents if they have an individualized family service plan (IFSP). B. Consult with a physician to ensure the child gets appropriate care. C. Inform the family that services are provided when the child begins school. D. Refer the family to a social worker for assistance with referrals and services.

A. Ask the parents if they have an individualized family service plan (IFSP).

4. The primary care pediatric nurse practitioner sees a 3yearold child whose parents report is a picky eater in spite of their continued efforts to provide nutritious meals. The parents ask whether a multivitamin is necessary. How will the nurse practitioner respond? A. Ask the parents to provide a 3day food diary. B. Prescribe a daily multivitamin with iron. C. Reinforce the need to meet DRIs each day. D. Tell them that supplements are unnecessary

A. Ask the parents to provide a 3day food diary.

5. The primary care pediatric nurse practitioner is discussing fitness and exercise with the parents of a 5yearold child who ask what kinds of activities are developmentally appropriate for their child. What will the nurse practitioner recommend? A. Bike riding B. Interactive play C. Martial arts D. Organized sports

A. Bike riding

8. The primary care pediatric nurse practitioner has a cohort of patients who have special health care needs. Which is an important role of the nurse practitioner when caring for these children? A. Care coordination and collaboration B. Developing protocols for parents to follow C. Monitoring individual education plans (IEPs) D. Providing lists of resources for families

A. Care coordination and collaboration

1. A pharmaceutical company has developed a new drug that was tested only on adults. The FDA has declared this drug to have potential benefits for ill children. According to the Pediatric Research Equity Act (PREA), what may the pharmaceutical company be required to do? A. Conduct pediatric drug studies to determine whether the drug is safe and effective in children. B. Provide labeling stating that the safety and efficacy of the drug is not established for children. C. Receive a patent extension for conducting pediatric studies to determine use in children. D. Survey existing data about the drug to determine potential use in the pediatric population.

A. Conduct pediatric drug studies to determine whether the drug is safe and effective in children.

7. The parents of a toddler tell the primary care pediatric nurse practitioner that they get frustrated trying to get the child to eat any vegetables other than squash and carrots. What will the nurse practitioner recommend? A. Continue to offer a variety of foods without forcing the child to eat them. B. Offer snacks to make up for calories the child misses by not eating the vegetables. C. Prepare dishes the child likes to ensure that a vegetable is eaten at each meal. D. Require the child to take 1 to 2 bites of each food at each meal.

A. Continue to offer a variety of foods without forcing the child to eat them.

3. During an assessment of a 4weekold infant, the primary care pediatric nurse practitioner learns that a breastfed infant nurses every 2 hours during the day but is able to sleep for a 4hour period during the night. The infant has gained 20 grams per day in the interval since last seen in the clinic. What will the nurse practitioner recommend? A. Continuing to nurse the infant using the current pattern B. Nursing the infant for longer periods every 4 hours C. Supplementing with formula at the last nighttime feeding D. Waking the infant every 2 hours to nurse during the night

A. Continuing to nurse the infant using the current pattern A healthy newborn is expected to lose 7% to 10% of the birth weight, but should regain that weight within the first 2 weeks or so after birth. During their first month, most newborns gain weight at a rate of about 1 ounce (30 grams) per day.

12. The parent of an adolescent reports noting cutting marks on the teen's arms and asks the primary care pediatric nurse practitioner what it means. What will the nurse practitioner tell this parent? A. Cutting is a way of dealing with emotional distress. B. It is a method of fitting in with other adolescents. C. The behavior is common and will usually stop. D. This type of behavior is a type of suicide attempt.

A. Cutting is a way of dealing with emotional distress.

4. The primary care pediatric nurse practitioner is considering using a drug for an "offlabel" use in a child. The nurse practitioner has used the drug in a similar situation previously, has consulted a pharmacology resource and the FDA website, and has determined that there are no significant contraindications and warnings for this child. What else must the nurse practitioner do when prescribing this drug? A. Discuss recommendations with the parents and document their consent. B. Document anecdotal reports of previous use of the drug by other providers. C. Follow up daily with the parents to determine safe administration of the drug. D. Report this use to the FDA Medwatch website for tracking purposes.

A. Discuss recommendations with the parents and document their consent.

9. The primary care pediatric nurse practitioner performs a physical examination on a 12yearold child and notes poor hygiene and inappropriate clothes for the weather. The child's mother appears clean and well dressed. The child reports getting 6 to 7 hours of sleep each night because of texting with friends late each evening. What action by the nurse practitioner will help promote healthy practices? A. Discuss setting clear expectations about selfcare with the mother B. Give the child information about sleep and selfcare C. Reassure the mother that this "noncompliance" is temporary D. Tell the mother that experimenting with selfcare behaviors is normal

A. Discuss setting clear expectations about selfcare with the mother

11. The parent of a 16yearold tells the primary care pediatric nurse practitioner that the teen was recently caught smoking an electronic cigarette (ecigarette). What will the nurse practitioner tell this parent? A. Ecigarette use may be a risk factor for later substance abuse. B. Experimentation with ecigarettes does not lead to future tobacco use. C. Most teens who experiment with tobacco usually do not become addicted. D. This form of nicotine ingestion is safer than regular cigarettes.

A. Ecigarette use may be a risk factor for later substance abuse.

1. The primary care pediatric nurse practitioner counseling the parent of an overweight schoolage child about improving overall fitness. What will the nurse practitioner include? A. Encourage the child to begin by engaging in swimming or cycling. B. Exercise will help lower total cholesterol and lowdensity lipoproteins. C. Schoolage children need 60 minutes of moderate exercise daily. D. Strength training exercises are not safe for schoolage children.

A. Encourage the child to begin by engaging in swimming or cycling.

12. The primary care pediatric nurse practitioner is evaluating recurrent stomach pain in a schoolage child. The child's exam is normal. The nurse practitioner learns that the child reports pain most evenings after school and refuses to participate in sports but does not have nausea or vomiting. The child's grandmother recently had gallbladder surgery. Which action is correct? A. Encourage the child to keep a log of pain, stool patterns, and dietary intake B. Order radiologic studies and laboratory tests to rule out systemic causes C. Reassure the child and encourage resuming sports when symptoms subside D. Refer the child to a counselor to discuss anxiety about health problems

A. Encourage the child to keep a log of pain, stool patterns, and dietary intake

5. The primary care pediatric nurse practitioner is examining a toddler who is below the 3rd percentile for weight even though the parents claim that the child eats "constantly." What will the nurse practitioner do initially? A. Evaluate the child's feeding and elimination behaviors and ask the family to describe mealtime routines. B. Recommend giving a multivitamin and offering high calorie foods, such as ice cream. C. Refer the child to a feeding evaluation clinic for a swallow study and evaluation of possible GERD. D. Suggest that the parents supplement the child's food intake with a high calorie formula.

A. Evaluate the child's feeding and elimination behaviors and ask the family to describe mealtime routines.

8. The primary care pediatric nurse practitioner evaluates a 4yearold girl whose parent reports frequent urination in the evenings on weekdays, incontinence after voiding. The parent reports that the child has soft formed stools 5 or 6 times weekly. Which assessment will the nurse practitioner make initially? A. Examination for labial adhesions B. Palpation for abdominal masses C. Screening for potential child abuse D. Urine culture and sensitivity

A. Examination for labial adhesions

10. The primary care pediatric nurse practitioner is evaluating a 4yearold female child for enuresis. The parents reports that the child has never been dry at night and has recently begun having daytime incontinence, usually when at preschool. The nurse practitioner learns that the child does not appear to have an abnormal urine stream. What will the nurse practitioner do next? A. Examine the urethral meatus and labia and obtain a dipstick clean catch urinalysis. B. Reassure the parent that the child probably gets distracted and puts off voiding until it is urgent. C. Refer the child to a pediatric urologist for evaluation of possible vesicoureteral reflux. D. Suggest a bladder retraining program and use of a nighttime bedwetting alarm.

A. Examine the urethral meatus and labia and obtain a dipstick clean catch urinalysis.

A schoolage child who uses a SABA and an inhaled corticosteroid medication is seen in the clinic for an acute asthma exacerbation. After 4 puffs of an inhaled shortacting B2 agonist (SABA) every 20 minutes for three treatments, spirometry testing shows an FEV1 of 60% of the child's personal best. What will the primary care pediatric nurse practitioner do next?

A. Order an oral corticosteroid, continue the SABA every 3 to 4 hours, and follow closely.

10. The primary care pediatric nurse practitioner is examining a 12monthold infant who was 6 weeks premature and observes that the infant uses a raking motion to pick up small objects. The PEDS questionnaire completed by the parent did not show significant developmental delays. What will the nurse practitioner do first? A. Perform an indepth developmental assessment. B. Reassure the parent that this is normal for a premature infant. C. Refer the infant to a developmental specialist. D. Suggest activities to improve fine motor skills.

A. Perform an indepth developmental assessment.

2. The primary care pediatric nurse practitioner is examining a 15yearold female who reports having her first period at age 13. She states that she has had five periods in the last year, with the last one 2 months prior. She participates in basketball at school. Which action is correct? A. Perform biometric screening to determine lean body mass. B. Prescribe oral contraceptives pills to regulate her periods. C. Reassure her that this is perfectly normal at her age. D. Refer her to an endocrinologist for hormonal evaluation.

A. Perform biometric screening to determine lean body mass.

7. The parent of a 14yearold child asks the primary care pediatric nurse practitioner how to help the child prevent injuries when basketball tryouts begin later in the school year. Which recommendation will be of most benefit? A. Preseason conditioning B. Proper footwear C. Protective knee braces D. Stretching before practices

A. Preseason conditioning

9. The primary care pediatric nurse practitioner is performing a well child examination on a high school age adolescent who plays football who has hypercalciuria. Which dietary supplement will the nurse practitioner question the adolescent about? A. Protein supplements B. Salt tablets C. Sports drinks D. Vitamin C

A. Protein supplements

3. The primary care pediatric nurse practitioner is discussing lifestyle changes with an adolescent who has hypertension. What will the nurse practitioner recommend about exercise for this client? A. Regular to vigorous activity initially with a combination of resistance and aerobic exercise to maintain lower blood pressure B. Moderate daily exercise such as walking for 20 minutes daily with increasing intensity as blood pressure drops C. Vigorous aerobic exercise combined with maximal strength training to lower blood pressure D. Vigorous aerobic exercise only to reduce blood pressure and then to maintain lowered blood pressure

A. Regular to vigorous activity initially with a combination of resistance and aerobic exercise to maintain lower blood pressure

10. The primary care pediatric nurse practitioner is performing a preparticipation sports physical examination on a 14yearold male who will be on the wrestling team at school. What will the nurse practitioner include when discussing healthy practices with this adolescent? A. Risks associated with repeatedly losing and gaining weight B. The need for an electrocardiogram or echocardiogram prior to participation C. The need to consume 20 to 30 grams of protein after exercise D. To consume water with CHO prior to activity lasting up to an hour

A. Risks associated with repeatedly losing and gaining weight

15. A 12yearold child who plays soccer is diagnosed with vocal cord dysfunction. What will the primary care nurse practitioner say when the child's parents ask about continued sports participation? A. The child may continue to participate in soccer. B. The child should limit activity to nonaerobic sports. C. This condition is a contraindication for all sports. D. This condition predisposes the child to sudden cardiac death.

A. The child may continue to participate in soccer.

7. The parent of a 5monthold is worried because the infant becomes fussy but doesn't always seem interested in nursing. What will the nurse practitioner tell this parent? A. The infant may be expressing a desire to play or to rest. B. The parent should give ibuprofen for teething pain before nursing. C. This is an indication that the infant is ready for solid foods. D. This may indicate gastrointestinal discomfort such as constipation.

A. The infant may be expressing a desire to play or to rest.

1. A single mother of an infant worries that living in a household with only one parent will cause her child to be maladjusted. To help address the mother's concerns, the primary care pediatric nurse practitioner will suggest A. developing consistent daily routines for the child. B. exposing her child to extended family members when possible. C.not working outside the home during the first few years. D.taking her child to regular play date activities with other children.

A. developing consistent daily routines for the child.

3. The primary care pediatric nurse practitioner is considering use of a relatively new drug for a 15monthold child. The drug is metabolized by the liver, so the nurse practitioner will consult a pharmacologist to discuss giving the drug: A. less often or at a lower dose. B. more often or at a higher dose. C. via a parenteral route. D. via the oral route.

A. less often or at a lower dose.

A 15-year-old female reports fainting at school in class on two occasions. The adolescent's orthostatic blood pressures are normal. The primary care pediatric nurse practitioner suspects a cardiac cause for these episodes and will order which tests before referring her to a pediatric cardiologist? 12-lead electrocardiogram Echocardiogram Tilt table testing Treadmill exercise testing

ANS: A A 12-lead ECG is useful for initial evaluation. Echocardiogram may be performed if the ECG shows potential cardiomyopathy. Tilt table testing is not recommended for use in primary care due to poor reliability. Treadmill exercise testing may be used in cases of exercise-related syncope.

A toddler is brought to the clinic after grabbing the hot end of his mother's curling iron. An examination reveals a pale, yellow burned area to the palm of one hand. What is true about this burn? a. It may take up to 3 weeks to heal with scarring likely. b. Scarring is unlikely, with healing expected in 3 to 7 days. c. Surgical intervention and skin grafting are usually required. d. This type of burn usually heals without scarring in 7 to 14 days.

ANS: A A deep partial-thickness burn appears pale and yellow and scarring is more likely to occur, with complete healing taking up to 3 weeks. A superficial burn is erythematous without blisters and heals in 3 to 7 days without scarring. A full-thickness burn involves extensive destruction of underlying tissues and requires surgical intervention and skin grafts. A superficial partial-thickness burn is red, mottled, moist, and painful and may scar, with healing in 7 to 14 days.

An initial key part of management of a child suspected of having an inborn error of metabolism is consulting a metabolic specialist. obtaining a complete family history. ordering metabolic screening tests. referring the family to a dietician.

ANS: A A metabolic specialist should be consulted as soon as an IEM is suspected and common etiologies for symptoms have been ruled out. A complete family history and metabolic screening tests will be part of the ongoing diagnostic process, but these should not delay referral since many of these disorders can have life-threatening or devastating outcomes. Once the diagnosis is known, a dietician may be consulted.

A 3-year-old child with pressure-equalizing tubes (PET) in both ears has otalgia in one ear. The primary care pediatric nurse practitioner is able to visualize the tube and does not see exudate in the ear canal and obtains a type A tympanogram. What will the nurse practitioner do? Order ototopical antibiotic/corticosteroid drops. Prescribe a prophylactic antibiotic medication. Reassure the parent that this is a normal exam. Refer the child to an otolaryngologist for follow-up

ANS: A A normal, or type A, tympanogram in a child with PET may indicate a clogged tube. Ototopical antibiotic/corticosteroid drops can occasionally clear a clogged PET. Prophylactic antibiotics are not recommended to prevent otitis media. It is not necessary to refer unless the pain continues in spite of standard measures.

The primary care pediatric nurse practitioner is examining a 2-week-old infant and auscultates a wide splitting of S2 during expiration. What condition may this finding represent? Atrial septal defect Coarctation of the aorta Patent ductus arteriosis Ventricular septal defect

ANS: A A wide splitting of S2 without becoming a single sound on expiration may indicate increased pulmonary flow, typical of atrial septal defect. Coarctation of the aorta may cause a systolic ejection murmur. A patent ductus arteriosus has a characteristic machinery-like murmur. A ventricular septal defect has a harsh, high-pitched, grade II to IV/VI holosystolic murmur.

The parent of a child diagnosed with ADHD tells the primary care pediatric nurse practitioner that the child gets overwhelmed by homework assignments, doesn't seem to know which ones to do first, and then doesn't do any assignments. The nurse practitioner tells the parent that this represents impairment in which executive function? Activation Effort Emotion Focus

ANS: A Activation is an executive function that helps individuals organize, prioritize, and begin activities. This child cannot prioritize a group of assignments and winds up not doing any of them, showing an inability to prioritize and begin activities. Effort is the function associated with sustaining effort and regulating awareness. Emotion is the function of managing frustration. Focus is associated with sustaining and shifting attention to a task.

The primary care pediatric nurse practitioner is performing a well child exam on a 17-year-old female whose mother is present during the history. The mother expresses concern that her daughter wishes to have an eyebrow piercing and states that she is opposed to the idea. What will the nurse practitioner do? Provide information about piercings and encourage continued discussion. Remind the adolescent that her mother is responsible for her health. State that piercings are relatively harmless and are an expression of individuality. Suggest that she wait until she is 18 years old and can make her own decisions.

ANS: A Adolescents who pierce their noses or have strange haircuts may be irritating to parents, but these are ways of expressing individuality and help them to achieve psychosocial milestones. The fact that the teen and her mother are discussing this is a good sign that the adolescent isn't in complete rebellion. The PNP should provide accurate health information and encourage continued dialogue. Although it is true that piercings are relatively harmless, the PNP shouldn't "side" with the teen during an open discussion or tell the teen that the mother is "in charge."

The primary care pediatric nurse practitioner is examining a 15-year-old female who reports having her first period at age 13. She states that she has had five periods in the last year, with the last one 2 months prior. She participates in basketball at school. Which action is correct? Perform biometric screening to determine lean body mass. Prescribe oral contraceptives pills to regulate her periods. Reassure her that this is perfectly normal at her age. Refer her to an endocrinologist for hormonal evaluation.

ANS: A Although it can take 18 to 24 months for adolescents to establish regulatory cycles, periods can also be affected by athletic activity that decreases body fat. The PNP should assess the percentage of lean body mass, which should be 75% or less to maintain regular ovulatory cycles. OCPs are useful for regulating periods if this persists and other causes are ruled out. It is not necessary to refer her to an endocrinologist unless problems persist in spite of standard management.

The parent of a 16-year-old tells the primary care pediatric nurse practitioner that the teen was recently caught smoking an electronic cigarette (e-cigarette). What will the nurse practitioner tell this parent? E-cigarette use may be a risk factor for later substance abuse. Experimentation with e-cigarettes does not lead to future tobacco use. Most teens who experiment with tobacco usually do not become addicted. This form of nicotine ingestion is safer than regular cigarettes.

ANS: A Although many adolescents consider e-cigarettes to be a safe form of tobacco use, increasing evidence indicates that their use may be a significant risk factor for later marijuana and substance abuse. The risk of dependence and addiction is the same for e-cigarettes and other cigarettes, since both use nicotine. Only 41% of teens try tobacco; 80% of older adolescents do not smoke.

A child is diagnosed with community-acquired pneumonia and will be treated as an outpatient. Which antibiotic will the primary care pediatric nurse practitioner prescribe? Amoxicillin Azithromycin Ceftriaxone Oseltamivir

ANS: A Amoxicillin is given to children with community-acquired pneumonia. Azithromycin is used to treat atypical pneumonia. Ceftriaxone is used for inpatient treatment. Oseltamivir is used for viral pneumonia.

A child who has psoriasis, who has been using a moderate-potency topical steroid on thick plaques on the extremities and a high-potency topical steroid on more severe plaques on the elbows and knees, continues to have worsening of plaques. In consultation with a dermatologist, which treatment will be added? Anthralin ointment in high strength applied for 10 to 30 minutes daily Calcipotriol cream applied liberally each day to the entire body Oral steroids and methotrexate therapy until plaques resolve Wideband ultraviolet therapy for 15 minutes twice daily

ANS: A Anthralin ointment is useful for plaques that are resistant to steroids. Calcipotriol cream is effective for mild to moderate plaques, but when applied in excessive quantities over large areas can cause hypercalcemia. Oral steroids are not indicated and may worsen symptoms by causing pustular flare. Methotrexate is used for severe disease, and these symptoms indicate that this is moderate disease. If UV light is used, narrowband UVB light therapy is preferred in children for safety and efficacy.

The mother of a 15-year-old adolescent female tells the primary care pediatric nurse practitioner that her daughter has extreme mood swings prior to her periods, which the adolescent vehemently denies. When asked if she notices anything different just before her periods, the adolescent points to her mother and says, "She gets really hard to live with." This demonstrates which characteristic of adolescent thinking? Apparent hypocrisy Imaginary audience Overthinking Personal fable

ANS: A Apparent hypocrisy is the notion that rules apply differently to adolescents than to others. The adolescent who chalks up the conflict with her mother related to her premenstrual mood swings does not see her own role in the conflict. Imaginary audience is the perception that everyone is thinking about them. Personal fable is the idea that they are special. Overthinking involves making things more complicated than they need to be.

6. The mother of a 6-month-old infant is distressed because the infant can say "dada" but not "mama" and asks the primary care pediatric nurse practitioner why this is when she is the one who spends more time with the infant. How will the nurse practitioner respond? a. "At this age, your baby does not understand the meaning of sounds." b. "Babies at this age cannot make the 'ma' sound." c. "Most sounds made by babies at this age are accidental." d. "This may mean that your baby doesn't hear well."

ANS: A At 6 months, infants delight in vocalizing sounds that they learn by imitation but do not ascribe meaning to the sounds they make. Infants can say "mama" but without meaning. Babies make sounds on purpose by imitating what they hear. A preference for one sound early in speech does not indicate a hearing deficit.

The mother of a 6-month-old infant is distressed because the infant can say "dada" but not "mama" and asks the primary care pediatric nurse practitioner why this is when she is the one who spends more time with the infant. How will the nurse practitioner respond? "At this age, your baby does not understand the meaning of sounds." "Babies at this age cannot make the 'ma' sound." "Most sounds made by babies at this age are accidental." "This may mean that your baby doesn't hear well."

ANS: A At 6 months, infants delight in vocalizing sounds that they learn by imitation but do not ascribe meaning to the sounds they make. Infants can say "mama" but without meaning. Babies make sounds on purpose by imitating what they hear. A preference for one sound early in speech does not indicate a hearing deficit.

9. The parent of a 5-month-old is worried because the infant becomes fussy but doesn't always seem interested in nursing. What will the nurse practitioner tell this parent? a. The infant may be expressing a desire to play or to rest. b. The parent should give ibuprofen for teething pain before nursing. c. This is an indication that the infant is ready for solid foods. d. This may indicate gastrointestinal discomfort such as constipation

ANS: A At this age, infants may cry when they are tired or need social interaction and not just when they are hungry. The PNP should teach parents about this change in social development so they can be responsive to their infant's needs. Solid foods are not added until age 6 months. Teething usually does not begin until at least 6 months. GI discomfort usually occurs after eating.

The parent of a 5-month-old is worried because the infant becomes fussy but doesn't always seem interested in nursing. What will the nurse practitioner tell this parent? The infant may be expressing a desire to play or to rest. The parent should give ibuprofen for teething pain before nursing. This is an indication that the infant is ready for solid foods. This may indicate gastrointestinal discomfort such as constipation.

ANS: A At this age, infants may cry when they are tired or need social interaction and not just when they are hungry. The PNP should teach parents about this change in social development so they can be responsive to their infant's needs. Solid foods are not added until age 6 months. Teething usually does not begin until at least 6 months. GI discomfort usually occurs after eating.

The primary care pediatric nurse practitioner is discussing fitness and exercise with the parents of a 5-year-old child who ask what kinds of activities are developmentally appropriate for their child. What will the nurse practitioner recommend? Bike riding Interactive play Martial arts Organized sports

ANS: A Bike riding away from traffic or with parents is a good activity for the preschool to early school-age child. Interactive play is recommended for toddlers. Martial arts and organized sports are recommended for school-age children.

A school-age child has had abdominal pain for 3 months that occurs once or twice weekly and is associated with a headache and occasional difficulty sleeping, often causing the child to stay home from school. The child does not have vomiting or diarrhea and is gaining weight normally. The physical exam is normal. According to Bishop, what is included in the initial diagnostic work-up for this child? CBC, ESR, amylase, lipase, UA, and abdominal ultrasound CBC, ESR, CRP, and fecal calprotectin CBC, ESR, CRP, UA, stool for ova, parasites, and culture Stool for H. pylori antigen and serum IgA, IgG, tTg

ANS: A Bishop suggests these labs as an initial approach in children suspected of having functional abdominal pain, along with a 3-day trial of a lactose-free diet. Fecal calprotectin is added if the child has changes in stool habits suggestive of inflammatory changes in the intestinal tract. Answer C is correct according to Rasquin's recommendations as is stool for H. pylori antigen.

What will the primary care pediatric nurse practitioner teach the parents of a child who has new pressure-equalizing tubes (PET) in both ears? Parents should notice improved hearing in their child. PET will help by reducing the number of ear infections the child has. The child should use earplugs when showering or bathing. The tubes will most likely remain in place for 3 to 4 years.

ANS: A By reducing middle ear fluid, the child with hearing loss from this condition should show improvement in hearing. Children may still have infections but without persistent effusion. Earplugs are not necessary unless the child's head is submerged. PETs usually fall out on their own; if they are still in place 2 to 3 years after placement, they should be removed by the otolaryngology surgeon.

9. The primary care pediatric nurse practitioner has a cohort of patients who have special health care needs. Which is an important role of the nurse practitioner when caring for these children? a. Care coordination and collaboration b. Developing protocols for parents to follow c. Monitoring individual education plans (IEPs) d. Providing lists of resources for families

ANS: A Care coordination is one of the key elements for children with special health care needs. PNPs are especially suited for this role and have the unique skills to function as care coordinators. Care for these children should involve shared decision making and individualized care and not "cookbook" approaches. The PNP may advocate for children's health care needs for the IEP but does not monitor these. The PNP should not just give parents lists of phone numbers but should assist them to make appointments.

The primary care pediatric nurse practitioner has a cohort of patients who have special health care needs. Which is an important role of the nurse practitioner when caring for these children? Care coordination and collaboration Developing protocols for parents to follow Monitoring individual education plans (IEPs) Providing lists of resources for families

ANS: A Care coordination is one of the key elements for children with special health care needs. PNPs are especially suited for this role and have the unique skills to function as care coordinators. Care for these children should involve shared decision making and individualized care and not "cookbook" approaches. The PNP may advocate for children's health care needs for the IEP but does not monitor these. The PNP should not just give parents lists of phone numbers but should assist them to make appointments.

A child is in the clinic because of symptoms of purulent, foul-smelling nasal discharge from the right nostril. Nasal visualization reveals something shiny in a mass of mucous in the nasal cavity. What will the primary care pediatric nurse practitioner do? Attempt to remove the mass gently using alligator forceps. Perform a saline nasal rinse using a water jet device. Refer the child to a pediatric otolaryngologist. Suction the mucoid mass using a bulb syringe.

ANS: A Children often insert foreign bodies into their nasal cavities and, if undetected for any period of time, will develop foul-smelling, unilateral, purulent nasal discharge. The foreign body may become embedded in granulation tissue or mucosa. If possible, the PNP should attempt removal if the FB is visible and can be easily removed without causing trauma. Saline nasal rinses with pressure may push the FB farther into the cavity. Referral to ENT may be necessary if attempts to remove the FB are not successful. Suction is not indicated.

A 7-year-old child who has a history of a repaired congenital heart defect has many dental caries along with gingival erythema and irritation and a temperature of 102.5°F. What will the primary care pediatric nurse practitioner do next? Admit to the hospital with a pediatric cardiology consult. Obtain blood cultures and a CBC and consult a pediatric cardiologist. Refer the child to a pediatric dental surgeon immediately. Start prophylactic antibiotics such as penicillin twice daily for 2 weeks.

ANS: A Children who are suspected of having SBE should be admitted to the hospital and referred to pediatric cardiology. The child should begin treatment as soon as SBE is suspected, so getting labs and then consulting a cardiologist is not correct. The SBE is the priority treatment, not the gingivitis or caries. Treatment should include IV antibiotics in the hospital.

2. The primary care pediatric nurse practitioner performs a well child examination on a 9-month-old infant who has a history of prematurity at 28 weeks' gestation. The infant was treated for retinopathy of prematurity (ROP) and all symptoms have resolved. When will the infant need an ophthalmologic exam? a. At 12 months of age b. At 24 months of age c. At 48 months of age d. At 60 months of age

ANS: A Children who have a history of ROP requiring treatment, even if ROP has completely resolved, will need yearly ophthalmologic follow-up. Less frequent follow-up is required for children with ROP who did not require treatment.

The primary care pediatric nurse practitioner performs a well child examination on a 9-month-old infant who has a history of prematurity at 28 weeks' gestation. The infant was treated for retinopathy of prematurity (ROP) and all symptoms have resolved. When will the infant need an ophthalmologic exam? At 12 months of age At 24 months of age At 48 months of age At 60 months of age

ANS: A Children who have a history of ROP requiring treatment, even if ROP has completely resolved, will need yearly ophthalmologic follow-up. Less frequent follow-up is required for children with ROP who did not require treatment.

The primary care pediatric nurse practitioner is performing a well child examination on a school-age child who had complete repair of a tetralogy of Fallot defect in infancy. What is important in this child's health maintenance regime? Cardiology clearance for sports participation Restriction of physical activity to avoid pulmonary complications Sub-acute bacterial endocarditis prophylaxis precautions Teaching about management of hypercyanotic episodes

ANS: A Children who have had TOF repair must be cleared by cardiology before participation in sports, but there is no need to restrict all physical activity. SBE prophylaxis is given prior to surgery and for 6 months afterward. Hypercyanotic episodes occur before repair.

The primary care pediatric nurse practitioner performs a well child examination on a 12-month-old child who had repair of a congenital heart defect at 8 months of age. The child has a normal exam. The parent reports that the child is not taking any medications. The nurse practitioner will contact the child's cardiologist to discuss whether the child needs which medication? Amoxicillin Capoten Digoxin Furosemide

ANS: A Children who have had complete repair of CHD should have SBE prophylaxis with amoxicillin for 6 months after the procedure. Capoten, an antihypertensive, digoxin, an inotropic medication, and furosemide, a diuretic, are given for specific symptoms as indicated.

The primary care pediatric nurse practitioner is evaluating a child who has short stature. Although bone age studies reveal a delay in bone age, the child's growth is consistent with bone age. Which diagnosis is most likely? Constitutional growth delay Growth hormone deficiency Idiopathic short stature Klinefelter syndrome

ANS: A Children with constitutional growth delay have a delay in bone age but growth patterns consistent with bone age. Children with growth hormone deficiency have a delay in both bone age and growth. Children with idiopathic short stature will have a bone age consistent with chronological age. Klinefelter syndrome is an overgrowth syndrome.

A previously healthy school-age child develops herpes zoster on the lower back. What will the primary care pediatric nurse practitioner do to manage this condition? Order Burow solution and warm soothing baths as comfort measures. Prescribe oral acyclovir 30 mg/kg/day in 4 doses/day for 5 days. Recommend topical antihistamines to control itching. Stress the need to remain home from school until the lesions are gone.

ANS: A Children with herpes zoster should be treated with comfort measures (symptomatic treatment). Oral acyclovir is not recommended for all children but may be useful in children who are immunosuppressed or have more severe forms. Topical antihistamines are used with caution in children because of the risk of toxicity. If the lesions can be covered, children do not need to be kept home from school.

The primary care pediatric nurse practitioner cares for a preschool-age child who was exposed to drugs prenatally. The child bites other children and has tantrums when asked to stop but is able to state later why this behavior is wrong. This child most likely has a disorder of executive function. information processing. sensory processing. social cognition.

ANS: A Children with prenatal drug or alcohol exposure often have executive function disorders, characterized by an inability to stop or delay a response or interrupt an inappropriate behavior and an inability to modify emotional expression appropriately. Information processing refers to thinking and problem-solving ability. Sensory processing has to do with the ability to take in information through senses and to process it appropriately. Social cognition refers to the ability to interpret behavior and emotions of the self and others.

A 3-year-old child has just completed a 7-day course of amoxicillin for a second febrile urinary tract infection and currently has a negative urine culture. What is the next course of action? Obtain a renal and bladder ultrasound. Prescribe prophylactic antibiotics to prevent recurrence. Refer the child for a voiding cystourethrogram. Screen urine regularly for leukocyte esterase and nitrites.

ANS: A Children with recurrent UTI should have a renal and bladder US to assess for hydronephrosis, scarring, or other atypical findings. If the US is concerning, VCUG and/or DMSA screen may be performed. Screening regularly is not indicated.

The primary care pediatric nurse practitioner is examining a school-age child who complains of frequent stomach pain and headaches. The parent reports that the child misses several days of school each month. The child has a normal exam. Before proceeding with further diagnostic tests, what will the nurse practitioner initially ask the parent? About the timing of the symptoms each day and during the week How well the child performs in school and in extracurricular activities If the parent feels a strong need to protect the child from problems Whether there are any unusual stressors or circumstances at home

ANS: A Children with school refusal or school phobia often have symptoms that gradually improve as the day progresses and often disappear on weekends. The PNP should ask about the frequency and duration of the symptoms to evaluate this pattern. The other options are important questions when management of school phobia has begun as a way of understanding underlying causes for the reluctance to go to school.

The parent of a 14-year-old child asks the primary care pediatric nurse practitioner how to help the child prevent injuries when basketball tryouts begin later in the school year. Which recommendation will be of most benefit? Preseason conditioning Proper footwear Protective knee braces Stretching before practices

ANS: A Conditioning in the preseason is one of the most important things children can do to build muscle strength, to prevent sports injuries, and to learn how to make twisting, jumping, and landing movements safely. Proper footwear is also recommended but is not the most important. Protective knee braces may be worn but do not prevent injury. Stretching should be done after warming up to maintain flexibility.

3. The primary care pediatric nurse practitioner sees a developmentally delayed toddler for an initial visit. The family has just moved to the area and asks the nurse practitioner about community services and resources for their child. What should the nurse practitioner do initially? a. Ask the parents if they have an individualized family service plan (IFSP). b. Consult with a physician to ensure the child gets appropriate care. c. Inform the family that services are provided when the child begins school. d. Refer the family to a social worker for assistance with referrals and services.

ANS: A Families with children who have developmental delays are eligible for early intervention services and should have IFSPs in place. This family may have one from their previous community, and it can be used as a starting point to determine needs. It is not necessary to consult with a physician to coordinate community resources. Early intervention is provided from birth, according to federal law. Until the specific referrals are known, the social worker is not consulted.

The primary care pediatric nurse practitioner sees a developmentally delayed toddler for an initial visit. The family has just moved to the area and asks the nurse practitioner about community services and resources for their child. What should the nurse practitioner do initially? Ask the parents if they have an individualized family service plan (IFSP). Consult with a physician to ensure the child gets appropriate care. Inform the family that services are provided when the child begins school. Refer the family to a social worker for assistance with referrals and services.

ANS: A Families with children who have developmental delays are eligible for early intervention services and should have IFSPs in place. This family may have one from their previous community, and it can be used as a starting point to determine needs. It is not necessary to consult with a physician to coordinate community resources. Early intervention is provided from birth, according to federal law. Until the specific referrals are known, the social worker is not consulted.

An adolescent who had cradle cap as an infant is in the clinic with thick crusts of yellow, greasy scales on the forehead and behind the ears. What will the primary care pediatric nurse practitioner recommend? Daily application of ketoconazole 2% topical cream High-potency topical corticosteroids applied daily Mineral oil and shampoo on the affected areas Selenium sulfide shampoo twice weekly to the face

ANS: A For facial dermatitis, daily ketoconazole 2% topical cream may be used. If steroids are prescribed, only low-dose steroids should be used on the face. Mineral oil and shampoo are recommended for cradle cap in infants. Selenium sulfide shampoo is used for scalp dermatitis.

The parent of a 3-month-old reports that the infant arches and gags while feeding and spits up undigested formula frequently. The infant's weight gain has dropped to the 5th percentile from the 12th percentile. What is the best course of treatment for this infant? Begin a trial of extensively hydrolyzed protein formula for 2 to 4 weeks. Institute an empiric trial of acid suppression with a proton pump inhibitor (PPI). Perform esophageal pH monitoring to determine the degree of reflux. Reassure the parent that these symptoms will likely resolve by 12 to 24 months.

ANS: A Formula-fed infants may be given a trial of a hydrolyzed protein formula to see if improvement occurs. An empiric trial of a PPI may be used in children and adolescents but is not recommended in infants. Esophageal pH monitoring may be performed in consultation with a specialist but not as first-line evaluation. The infant has warning signs of GERD that require further investigation and not just reassurance.

A child has gross hematuria, abdominal pain, and arthralgia as well as a rash. What diagnosis is most likely? Henoch-Schönlein purpura Rhabdomyosarcoma Sickle cell disease Systemic lupus erythematosus

ANS: A HSP may presents with gross hematuria in the presence of abdominal pain with or without bloody stools, arthralgias, and a purpuric rash. Rhabdomyosarcoma is characterized by gross hematuria and voiding dysfunction. Sickle cell disease can cause gross hematuria but not always.

A school-age female has had vulvovaginitis for 2 months. All cultures and tests are negative, but the symptoms persist after treatment with both topical antibiotics and oral amoxicillin. What is the next course of action to treat this condition? Estrogen cream at bedtime for 2 to 3 weeks Referral to a pediatric gynecologist for further evaluation Trimethoprim-sulfamethoxazole daily for 1 to 2 months Workup for possible sexual abuse

ANS: A If antibiotics fail to treat prepubertal, nonspecific vulvovaginitis, the PNP should order estrogen cream to thicken the vulvar epithelium. If treatment fails, referral to a pediatric gynecologist should be made. TMP-SMX is used for recurrent vulvovaginitis. Sexual abuse is considered if signs of trauma are present or if STI testing is positive.

A child who has otitis externa has severe swelling of the external auditory canal that persists after 2 days of therapy with ototopical antibiotic/corticosteroid drops. What is the next step in treatment for this child? Insert a wick into the external auditory canal. Irrigate the external auditory canal with saline. Order systemic corticosteroids. Prescribe an oral antibiotic medication.

ANS: A If significant swelling is present, inserting a wick into the EAC is helpful and should be impregnated with antibiotics as long as it is in place. Irrigation is contraindicated during an acute infection. Systemic steroids and antibiotics are not indicated.

An adolescent female has heavy periods that are also irregular. The physical exam is normal. A complete blood count reveals a hemoglobin of 8.9 g/dL. What test will the primary care pediatric nurse practitioner order next? Coagulation studies C-reactive protein Thyroid function Ultrasound of pelvis

ANS: A If the patient's hemoglobin is low, coagulation studies should be ordered. CRP is ordered if infection is suspected. Thyroid function is indicated if systemic disease is suspected. A pelvic ultrasound is ordered if a mass is palpated, anomaly is suspected, bimanual exam cannot be completed, or if the condition is unresponsive to treatment.

3. During an assessment of a 4-week-old infant, the primary care pediatric nurse practitioner learns that a breastfed infant nurses every 2 hours during the day but is able to sleep for a 4-hour period during the night. The infant has gained 20 grams per day in the interval since last seen in the clinic. What will the nurse practitioner recommend? a. Continuing to nurse the infant using the current pattern b. Nursing the infant for longer periods every 4 hours c. Supplementing with formula at the last nighttime feeding d. Waking the infant every 2 hours to nurse during the night

ANS: A Infants who are encouraged to breastfeed every 2 to 3 hours may have one longer stretch of 4 hours at night. This infant is gaining between 0.5 and 1 gram per day, which is appropriate. It is not necessary to alter the pattern of nursing or to supplement with formula.

During an assessment of a 4-week-old infant, the primary care pediatric nurse practitioner learns that a breastfed infant nurses every 2 hours during the day but is able to sleep for a 4-hour period during the night. The infant has gained 20 grams per day in the interval since last seen in the clinic. What will the nurse practitioner recommend? Continuing to nurse the infant using the current pattern Nursing the infant for longer periods every 4 hours Supplementing with formula at the last nighttime feeding Waking the infant every 2 hours to nurse during the night

ANS: A Infants who are encouraged to breastfeed every 2 to 3 hours may have one longer stretch of 4 hours at night. This infant is gaining between 0.5 and 1 gram per day, which is appropriate. It is not necessary to alter the pattern of nursing or to supplement with formula.

The primary care pediatric nurse practitioner is evaluating a 2-year-old with a documented speech delay. Screenings to assess motor skills and cognition are normal, and the child passed a recent hearing test. What will the pediatric nurse practitioner do next? Ask the child's parents whether they read to the child. Give parents educational materials to encourage speech. Refer the child to an early intervention program. Suggest that they purchase age-appropriate music videos.

ANS: A Language development requires oral-motor ability, auditory perception, and cognitive ability, which this child has been shown to have, as well as the psychosocial-cultural environment to motivate the child to engage in language use. The PCPNP's initial step should be to determine whether the parents provide such an environment. Educational materials may be used after it is determined that these are useful. Early intervention may be used if the speech delay persists. Music videos do not necessarily engage the child in expression of speech.

During a well child examination, a 15-year-old female tells the primary care pediatric nurse practitioner that some of her friends have begun having sex. She has a boyfriend but denies engaging in sex with him. What will the nurse practitioner do initially? Ask her for her definitions of "sex." Discuss the risks of sexually transmitted diseases. Find out if she is considering sexual relations. Give her information about contraception.

ANS: A Many adolescents do not equate oral or anal intercourse with sex, so it is important to find out how this patient defines sex. The other options also may be considered depending on the situation, however, clarity about the words used in the discussion are most important initially for the nurse practitioner to focus the subsequent conversation appropriately.

A child will need an occlusive dressing to treat lichen simplex chronicus. What will the primary care pediatric nurse practitioner tell the parents about applying this treatment? Apply ointment before the dressing. Plastic wrap should not be used. The dressing should be applied to dry skin. Change the dressing twice daily.

ANS: A Occlusive dressings are placed over creams and ointments to enhance hydration and absorption of topical medications. Plastic wrap is often used. The medications and dressings should be applied to damp skin. The dressing should not be left on more than 8 hours.

An adolescent has 2+ proteinuria in a random dipstick urinalysis. A subsequent first-morning voided specimen is negative. What will the primary care pediatric nurse practitioner do to manage this condition? Monitor for proteinuria at each annual well child examination. Order a 24-hour timed urine collection for creatinine and protein excretion. Reassure the parents that this is a benign condition with no follow-up needed. Refer the child to a pediatric nephrologist for further evaluation.

ANS: A Orthostatic proteinuria, demonstrated by proteinuria of greater than 1+ with activity and low-protein to normal urine on a first-morning void, is common in adolescents. If the first-morning void is negative, the adolescent should be monitored annually. A 24-hour urine collection is not indicated unless the first-morning void is elevated. Although the orthostatic proteinuria is mostly benign, annual monitoring is recommended and patient education should stress the importance of follow-up to evaluate the cause of proteinuria. Children with mild asymptomatic proteinuria who have a normal first-morning specimen do not require extensive testing for kidney disease but should be monitored annually. Unless proteinuria is severe or persistent, referral to a nephrologist is not indicated.

The primary care pediatric nurse practitioner performs a physical examination on a 12-year-old child and notes poor hygiene and inappropriate clothes for the weather. The child's mother appears clean and well dressed. The child reports getting 6 to 7 hours of sleep each night because of texting with friends late each evening. What action by the nurse practitioner will help promote healthy practices? Discuss setting clear expectations about self-care with the mother Give the child information about sleep and self-care Reassure the mother that this "non-compliance" is temporary Tell the mother that experimenting with self-care behaviors is normal

ANS: A Parents of school-age children should be advised to set clear limits for their children for cleanliness, healthy exercise, hours of sleep, and other health promotion behaviors to encourage the development of responsibility for these things. Giving the child information can be done along with setting expectations, but, at this age, the parent should still be supervising. While "non-compliance" is a part of this process, and is a means of asserting independence, parents need to discuss this with children to resolve the issue.

The primary care pediatric nurse practitioner is counseling the parents of a 13-year-old female who has Down syndrome about sexual maturation. What will the nurse practitioner tell these parents? It is important to discuss and support healthy sexuality. Providing too much information about sexuality may be confusing given the child's cognitive level of understanding. Suppressing periods with contraceptives will lessen their daughter's distress. They should give her information about periods but not about sexuality.

ANS: A Persons with disabilities have the same desires to make decisions and foster fulfilling relationships with others as other people have. Unless healthy sexuality is taught and supported, unhealthy and abusive sexuality is more likely to occur. Parents should give information when it is desired and delivered in a manner appropriate to the child's level of understanding. Suppressing periods only ignores the issue but does not change the increased feelings that accompany puberty.

The primary care pediatric nurse practitioner is performing a well child examination on a high school age adolescent who plays football who has hypercalciuria. Which dietary supplement will the nurse practitioner question the adolescent about? Protein supplements Salt tablets Sports drinks Vitamin C

ANS: A Protein supplements can cause hypercalciuria with calcium loss and dehydration if protein intake is too high. Salt tablets can cause hypernatremia and delayed gastric emptying. Sports drinks are high in sugar and electrolytes, which will not affect the calcium content of the urine. It is not necessary to take vitamin C.

10. A single mother of an infant worries that living in a household with only one parent will cause her child to be maladjusted. To help address the mother's concerns, the primary care pediatric nurse practitioner will suggest : a. developing consistent daily routines for the child. b. exposing her child to extended family members when possible. c. not working outside the home during the first few years. d. taking her child to regular play date activities with other children

ANS: A Providers can teach parents that providing predictable, consistent, and loving care helps an infant to learn trust and help influence positive brain development. Involving extended family members and going to play dates are good ways to socialize children but are not essential to learning trust. It may not be possible for her to be a stay-at-home mother.

A single mother of an infant worries that living in a household with only one parent will cause her child to be maladjusted. To help address the mother's concerns, the primary care pediatric nurse practitioner will suggest : developing consistent daily routines for the child. exposing her child to extended family members when possible. not working outside the home during the first few years. taking her child to regular play date activities with other children.

ANS: A Providers can teach parents that providing predictable, consistent, and loving care helps an infant to learn trust and help influence positive brain development. Involving extended family members and going to play dates are good ways to socialize children but are not essential to learning trust. It may not be possible for her to be a stay-at-home mother.

The primary care pediatric nurse practitioner is discussing lifestyle changes with an adolescent who has hypertension. What will the nurse practitioner recommend about exercise for this client? Regular to vigorous activity initially with a combination of resistance and aerobic exercise to maintain lower blood pressure Moderate daily exercise such as walking for 20 minutes daily with increasing intensity as blood pressure drops Vigorous aerobic exercise combined with maximal strength training to lower blood pressure Vigorous aerobic exercise only to reduce blood pressure and then to maintain lowered blood pressure

ANS: A Regular to vigorous physical activity for 30 minute 3 days per week helps to lower blood pressure. Resistance training may be combined with aerobic exercise after blood pressure is lowered to help maintain lowered blood pressure. Strength training is contraindicated in children with hypertension.

During a well child exam of a school-age child, the primary care pediatric nurse practitioner learns that the child has been having angry episodes at school. The nurse practitioner observes the child to appear withdrawn and sad. Which action is appropriate? Ask the child and the parent about stressors at home Make a referral to a child behavioral specialist Provide information about anger management Suggest consideration of a different classroom

ANS: A School-age children are learning to manage emotions and need help to manage their feelings in acceptable ways. A variety of stressors, including parental divorce, substance abuse, bullying in school, and early responsibilities, can cause anxiety in the child, who may not manage these feelings well. Until the underlying cause is better understood, management options cannot be determined, so referrals to specialists, information about anger management, or moving to a different classroom may not be indicated.

A child is diagnosed with tinea versicolor. What is the correct management of this disorder? Application of selenium sulfide 2.5% lotion twice weekly for 2 to 4 weeks Oral antifungal treatment with fluconazole once weekly for 2 to 3 weeks Sun exposure for up to an hour every day for 2 to 4 weeks Using ketoconazole 2% shampoo on lesions twice daily for 2 to 4 weeks

ANS: A Selenium sulfide lotion or 1% shampoo is first-line treatment for children and younger adolescents. Oral antifungal medications are used in resistant cases in older adolescents. Sun exposure only intensifies lesions. Ketoconazole shampoo is used on older adolescents.

The parent of an adolescent reports noting cutting marks on the teen's arms and asks the primary care pediatric nurse practitioner what it means. What will the nurse practitioner tell this parent? Cutting is a way of dealing with emotional distress. It is a method of fitting in with other adolescents. The behavior is common and will usually stop. This type of behavior is a type of suicide attempt.

ANS: A Self-injurious behavior (SIB) is used as a coping strategy to relieve distress, anger, and stress. It is not commonly done among adolescents and is not a way of fitting in with a peer group. Because it indicates underlying distress, adolescents must get help identifying these causes. Many have a history of physical, sexual, or emotional abuse. Although individuals who engage in SIB are more likely to attempt suicide in the future, the act itself is not a suicide attempt.

The primary care pediatric nurse practitioner uses the Neurodevelopmental Learning Framework to assess cognition and learning in an adolescent. When evaluating social cognition, the nurse practitioner will ask the adolescent about friends and activities at school. if balancing sports and homework is difficult. to interpret material from a pie chart. to restate the content of something just read.

ANS: A Social cognition is one construct of the Neurodevelopmental Learning Framework and is the ability to know what to discuss when, with whom, and for how long as well as the ability to work and play with others in a cooperative manner. Asking about friends will tell the PNP something about this ability. Determining the ability to manage a schedule assesses temporal-sequential ordering ability. Evaluating interpretation of material presented in a chart format assesses spatial ordering skills or visual thinking. Asking the adolescent to restate something in his or her own words assesses language and verbal skills.

The primary care pediatric nurse practitioner is evaluating health literacy in the mother of a new preschool-age child. How will the nurse practitioner assess this? Ask the child how many books he has at home. Ask the mother about her highest grade in school. Ask the mother to determine the correct dose of a drug from a label. Ask the mother to read a health information handout aloud.

ANS: A The "newest vital sign," or health literacy, can be determined quickly by asking the parent how many children's books are in the home. Greater than 10 books in the home is an independent positive predictor of adequate parent health literacy. The other questions may determine a specific level of literacy in general but are not as efficient.

The primary care pediatric nurse practitioner counseling the parent of an overweight school-age child about improving overall fitness. What will the nurse practitioner include? Encourage the child to begin by engaging in swimming or cycling. Exercise will help lower total cholesterol and low-density lipoproteins. School-age children need 60 minutes of moderate exercise daily. Strength training exercises are not safe for school-age children.

ANS: A The AAP suggests that overweight children initially participate in activities that place less stress on weight-bearing joints, such as swimming or cycling. Exercise helps raise HDL levels but does not reduce total cholesterol or LDL levels. School-age children need 60 minutes of physical activity but not necessarily exercise each day. Strength training exercises are safe, but powerlifting and maximal weight training are not, because of effects on developing bones.

What will the primary care pediatric nurse practitioner elicit when obtaining a positive Barlow maneuver when screening for developmental dysplasia of the hip? Dislocation of an unstable hip Dropping of the iliac crest with a raised leg Reduction of a dislocated hip Unequal knee heights in a supine child

ANS: A The Barlow maneuver dislocates an unstable or dislocatable hip posteriorly. Having the child raise a leg and watching for dropping of the iliac crest on that side is the Trendelenberg sign. The Ortolani maneuver reduces a posteriorly dislocated hip. Unequal knee heights in a supine child occur with a positive Galeazzi maneuver.

The primary care pediatric nurse practitioner is performing a well child assessment on an adolescent and is concerned about possible alcohol and tobacco use. Which assessment tool will the nurse practitioner use? CRAFFT HEEADSSS PHQ-2 RAAPS

ANS: A The CRAFFT tool is a six-question tool used to screen for adolescent substance abuse. The HEEADSSS is used as a psychosocial screening tool. The PHQ-2 is a rapid screen for depression. The RAAPS is used to assess risk behaviors that contribute to most morbidity, mortality, and social problems in teens.

When formulating developmental diagnoses for pediatric patients, the primary care pediatric nurse practitioner may use which resource? DC: 0-3R ICD-10-CM ICSD-3 NANDA International

ANS: A The DC: 0-3R refers to the Diagnostic Classification of Mental Health and Developmental Disorders of Infancy and Early Childhood and is useful for developmental problem diagnosis. The ICD-10-CM is the International Classification of Diseases-Tenth Revision, Clinical Modification and is useful for identifying physiologic diseases. The ICSD-3 is the International Classification of Sleep Disorders - 3rd edition. NANDA International is used to label problems in the functional health domain.

A 15-year-old female has a positive pregnancy test and asks the primary care pediatric nurse practitioner not to tell her parents. She is tearful and says she isn't sure she wants to keep the baby. What will the nurse practitioner do first? Determine the state-mandated reporting laws. Encourage the adolescent to talk to her parents. Obtain a social work consult to discuss adoption options. Refer her to a prenatal care specialist for follow-up.

ANS: A The PNP should first determine what the state's reporting laws are in case there are mandatory provisions for reporting statutory rape. The other options may be correct, depending on the laws and on the decisions of the adolescent.

14. A school-age child is seen in the clinic after a fragment from a glass bottle flew into the eye. What will the primary care pediatric nurse practitioner do? a. Refer immediately to an ophthalmologist. b. Attempt to visualize the glass fragment. c. Irrigate the eye with sterile saline. d. Instill a topical anesthetic.

ANS: A The PNP should never attempt to remove an intraocular foreign body or any projectile object but should refer immediately to an ophthalmologist. Visualizing the object, irrigating the eye, or instilling drops may further injure the eye.

A school-age child is seen in the clinic after a fragment from a glass bottle flew into the eye. What will the primary care pediatric nurse practitioner do? Refer immediately to an ophthalmologist. Attempt to visualize the glass fragment. Irrigate the eye with sterile saline. Instill a topical anesthetic.

ANS: A The PNP should never attempt to remove an intraocular foreign body or any projectile object but should refer immediately to an ophthalmologist. Visualizing the object, irrigating the eye, or instilling drops may further injure the eye.

The primary care pediatric nurse practitioner is evaluating recurrent stomach pain in a school-age child. The child's exam is normal. The nurse practitioner learns that the child reports pain most evenings after school and refuses to participate in sports but does not have nausea or vomiting. The child's grandmother recently had gallbladder surgery. Which action is correct? Encourage the child to keep a log of pain, stool patterns, and dietary intake Order radiologic studies and laboratory tests to rule out systemic causes Reassure the child and encourage resuming sports when symptoms subside Refer the child to a counselor to discuss anxiety about health problems

ANS: A The PNP suspects a somatic disorder after a normal exam and should encourage the child to keep a food or pain diary to help manage symptoms. The PNP should not "medicalize" the problem with tests. The child should be encouraged to resume sports and participate in normal activities. If the symptoms persist, referral for counseling is warranted.

The primary care pediatric nurse practitioner is evaluating a school-age child who has been diagnosed with ADHD. Which plan will the nurse practitioner recommend asking the child's school about to help with academic performance? 504 FAPE IDEA IEP

ANS: A The Section 504 plan specifies "reasonable accommodations" to help children with disabilities, such as physical or mental conditions, to benefit from their education. Many children with ADHD with learning disabilities but not cognitive deficits are eligible for this plan. FAPE, or free and appropriate public education, is a part of the special education system and lays out regulations for providing special education. IDEA is the Individuals with Disabilities Education Act, which provides mandates for providing education for children with disabilities. An IEP, or Individualized Education Plan, is a written plan defining disabilities, educational needs, and specific annual goals for meeting these needs. It is not generally used for children who do not have cognitive deficits, such as those with ADHD.

A 10-year-old child has had abdominal pain for 2 days, which began in the periumbilical area and then localized to the right lower quadrant. The child vomited once today and then experienced relief from pain followed by an increased fever. What is the likely diagnosis? Appendicitis with perforation Gastroenteritis Pelvic inflammatory disease (PID) Urinary tract infection (UTI)

ANS: A The child has the progression of symptoms typical of appendicitis with perforation - pain before vomiting that localizes to the RLQ and then relief of pain with onset of fever upon perforation. With gastroenteritis, vomiting precedes pain. PID symptomology includes increasing pain over time. The symptoms of an UTI include fever, chills, and urinary symptoms.

The primary care pediatric nurse practitioner is counseling the parents of a toddler about appropriate discipline. The parents report that the child is very active and curious, and they are worried about the potential for injury. What will the pediatric nurse practitioner recommend? Allow the child to explore and experiment while providing appropriate limits. Be present while the child plays to continually teach the child what is appropriate. Let the child experiment at will and to make mistakes in order to learn. Say "no" whenever the child does something that is not acceptable.

ANS: A The child who is securely attached uses the parents as a base from which to safely explore the world. Toddlers learn by doing and need to experiment to gain mastery over the environment. It is important that parents are present for safety, but parents should not be ever-present and controlling. Parents should be close by and should intervene if the child is at risk for injury. Continual criticism and the use of the word "no" can make the toddler feel powerless.

During a well baby examination of a newborn, the primary care pediatric nurse practitioner notes adduction of the right forefoot, with normal position of the mid- and hind-foot, along with a convex-shaped lateral border of the foot. What will the nurse practitioner do to evaluate this deformity? Grasp the heel with one hand and abduct the forefoot with the other hand. Observe both legs for medial and lateral rotations. Order anterior-posterior and lateral radiographs of both feet. Refer the infant immediately to a pediatric orthopedic specialist.

ANS: A The foot position is characteristic of metatarsus adductus. The PNP should evaluate whether the deformity is rigid, requiring treatment by an orthopedist, or not. If the forefoot can be brought past the midline with the heel held in place, it is not considered to be a rigid deformity. Assessing legs for medial and lateral rotations is part of the exam for in-toeing. Radiographs are not routinely performed. Referral to an orthopedic specialist, if this is found to be rigid deformity, is not urgent.

A child who plays soccer is in the clinic reporting pain and swelling in both knees. A physical examination reveals swelling and focal tenderness at the tibial tuberosities, with pain worsening when asked to extend the knees against resistance. What is the treatment for this condition? Apply ice packs to both knees and avoid activities that cause pain. Begin quadriceps-stretching exercises now to prevent further injury. Obtain radiographic studies to rule out fractures or ligament tears. Refer to a pediatric orthopedic specialist to evaluate the need for surgery.

ANS: A The history of sports involving kicking a ball and the location and type of pain and swelling are consistent with Osgood-Schlatter disease (OSD). Management involves rest and ice and sometimes NSAIDs. Quadriceps-stretching exercises are not encouraged until the acute symptoms pass. Radiographic studies are not necessary. Referral is not indicated for OSD.

The primary care pediatric nurse practitioner performs a physical examination on a 9-month-old infant with congenital hypothyroidism who takes daily levothyroxine sodium and notes a recent slowing of the infant's growth rate. What will the nurse practitioner order? Free serum T4 and TSH levels Serum levothyroxine level Total T4 and free T4 levels TSH and total T4 levels

ANS: A The infant is showing signs of hypothyroidism. Free serum T4 will be low and TSH will be high in central hypothyroidism; these are monitored to determine whether doses of levothyroxine are adequate. Serum levothyroxine levels are not used. Total T4 is used to diagnose TBG deficiency.

An adolescent female tells the primary care pediatric nurse practitioner that she had unprotected sexual intercourse 4 days prior and is worried she might become pregnant. What will the nurse practitioner do? Prescribe ulipristal acetate (Ella). Recommend levonorgestrel (Plan B One Step). Start a combination OCP at regular doses. Suggest using the less expensive After Pill preparation.

ANS: A The prescription medication ulipristal can be used up to 5 days after unprotected intercourse. Levonorgestrel, which is the active ingredient in Plan B and the After Pill, should be taken within 72 hours of unprotected intercourse. The After Pill must be purchased online. A combination OCP may be used, but it is an alternative approach.

The mother of a newborn tells the primary care pediatric nurse practitioner that she is worried that her child will develop allergies and asthma. Which tool will the nurse practitioner use to evaluate this risk? Three-generation pedigree Review of systems Genogram Ecomap

ANS: A The three-generation pedigree is used to map out risks for genetic diseases in families, as well as conditions with modifiable risk factors. The review of systems is used to evaluate the history of the child's body systems. The genogram is an approach to developing a family database to provide a graphic representation of family structure, roles, and problems of recurring significance in a family. The ecomap is used to identify relationships in the family and community that are supportive or harmful.

The primary care pediatric nurse practitioner is prescribing contraception for an adolescent who has not used birth control previously. The adolescent has a normal exam and has no family history of cardiovascular and peripheral vascular disease or diabetes. Which preparation is used initially? A combination oral contraceptive pills (OCP) with 30 to 35 mcg of estrogen and low progestin A combination OCP with low androgenic potency, such as Ortho-Cyclen A progestin-only mini-pill oral contraceptive A subdermal implant contraception, such as Implanon or Nexplanon

ANS: A The usual initial OCP is a combination with 30 to 35 mcg of estrogen and low progestin potency. The combination OCP with low androgenic potency is used for adolescents with hirsutism or PCOS. The progestin-only mini-pill is used in patients for whom progestin is contraindicated, such as for lactating women, and is not generally used in adolescents because of irregular bleeding and higher failure rates. The subdermal implants are used in older adolescents who are committed to long-term contraception.

A 17-year-old sexually active female who began having periods at age 14 reports having moderate to severe dull lower abdominal pain associated predominantly with periods but that occurs at other times as well. The history reveals a recent onset of these symptoms. A pregnancy test is negative. Which course of action is most important? Perform a full diagnostic workup to evaluate potential causes. Prescribe a prostaglandin synthetase inhibitor. Start a 3- to 6-month trial of oral contraceptive pills. Suggest using transcutaneous electrical nerve stimulation.

ANS: A This adolescent has symptoms consistent with secondary dysmenorrhea and should have a full diagnostic workup to evaluate potential causes. Prostaglandin synthetase inhibitors, OCPs, and TENS are useful to treat discomfort, but determining the cause is more important.

The primary care pediatric nurse practitioner is reviewing lab work and diabetes management with a school-age child whose HbA1C is 7.6% who reports usual blood sugars before meals as being 80 to 90 mg/dL. The nurse practitioner will consult with the child's endocrinologist to consider which therapy? Continuous glucose monitoring Continuous subcutaneous insulin infusion Self-monitoring of blood glucose Use of a long-acting insulin analogue

ANS: A This child has an elevated HbA1C and signs of hypoglycemia before meals. Continuous glucose monitoring reports both real time interstitial glucose levels and directional trending graphs every few minutes with alarms that warn of low and high blood glucose levels. Recent research has indicated that use of the CGM is effective in lowering HbA1C and decreasing hypoglycemic episodes. CSII allows flexibility with timing of meals and adjustment of insulin doses. Self-monitoring of blood glucose does not lower HbA1C or decrease hypoglycemic episodes.

11. A preschool-age child is seen in the clinic after waking up a temperature of 102.2°F, swelling and erythema of the upper lid of one eye, and moderate pain when looking from side to side. Which course of treatment is correct? a. Admit to the hospital for intravenous antibiotics. b. Obtain a lumbar puncture and blood culture. c. Order warm compresses 4 times daily for 5 days. d. Prescribe a 10- to 14-day course of oral antibiotics.

ANS: A This child has periorbital cellulitis and must be hospitalized because of having pain with movement of the eye, indicating orbital involvement. LP is performed on infants under 1 year of age. Warm compresses are used for mild cases. Oral antibiotics are not indicated.

A preschool-age child is seen in the clinic after waking up a temperature of 102.2°F, swelling and erythema of the upper lid of one eye, and moderate pain when looking from side to side. Which course of treatment is correct? Admit to the hospital for intravenous antibiotics. Obtain a lumbar puncture and blood culture. Order warm compresses 4 times daily for 5 days. Prescribe a 10- to 14-day course of oral antibiotics.

ANS: A This child has periorbital cellulitis and must be hospitalized because of having pain with movement of the eye, indicating orbital involvement. LP is performed on infants under 1 year of age. Warm compresses are used for mild cases. Oral antibiotics are not indicated.

A child is brought to the clinic immediately after being stung by a wasp while playing in the yard. The physical examination reveals localized redness and edema at the site, along with abdominal tenderness, watery eyes, and generalized hives. What is the initial treatment? a. Administer intramuscular epinephrine. b. Apply a topical glucocorticoid cream. c. Give oral diphenhydramine. d. Order a bronchodilator treatment.

ANS: A This child has signs of a severe reaction to the sting and should receive epinephrine first, followed by oral diphenhydramine and bronchodilators if wheezing. Topical glucocorticoids are used for mild, localized reactions.

A child who has been taking antibiotics is brought to the clinic with a rash. The parent reports that the child had a fever associated with what looked like sunburn and now has "blisters" all over. A physical examination shows coalescent target lesions and widespread bullae and areas of peeled skin revealing moist, red surfaces. What will the primary care pediatric nurse practitioner do? Consult with a pediatric intensivist for admission to a pediatric intensive care unit. Order oral acyclovir 20 mg/kg/day in two doses for 6 to 12 months. Prescribe systemic antihistamines and antimicrobial medications as prophylaxis. Recommend analgesics, cool compresses, and oral antihistamines for comfort.

ANS: A This child has symptoms consistent with toxic epidermal necrolysis, which is potentially life-threatening. Children with symptoms should be admitted to the PICU for management. The other options are treatments for erythema multiforme, a more benign, viral-induced rash. Oral acyclovir is given when herpes simplex infection is possible.

A school-age child has a history of chronic otitis media and is seen in the clinic with vertigo. The primary care pediatric nurse practitioner notes profuse purulent otorrhea from both pressure-equalizing tubes and a pearly-white lesion on one tympanic membrane. Which condition is most likely? Cholesteatoma Mastoiditis Otitis externa Otitis media with effusion

ANS: A This child has symptoms of cholesteatoma, especially with a pearly white lesion on the TM. Mastoiditis involves the mastoid bone behind the ear.

A school-age child has had nasal discharge and daytime cough but no fever for 12 days without improvement in symptoms. The child has not had antibiotics recently and there is no significant antibiotic resistance in the local community. What is the appropriate treatment for this child? Amoxicillin 45 mg/kg/day Amoxicillin 80-90 mg/kg/day Amoxicillin-clavulanate 80-90 mg/kg/day Saline irrigation for symptomatic relief

ANS: A This child meets criteria for treatment of acute rhinosinusitis (ARS) based on duration of symptoms without clinical improvement. The initial treatment is amoxicillin 45 mg/kg/day. The higher dose is used to treat ARS in communities with resistant S. pneumonia. If antibiotics have been used previously, amoxicillin-clavulanate is used. The use of buffered isotonic saline into the nasal cavity by squeeze bottle or neti pot (in late childhood and adolescence) may be helpful, but the clinical guidelines do not support or negate the use of saline.

The primary care pediatric nurse practitioner is performing a well child check-up on a 20-month-old child. The child was 4 weeks premature and, according to a parent-completed developmental questionnaire, has achieved milestones for a 15-month-old infant. Which action is correct? Perform an in-depth developmental assessment screen at this visit to evaluate this child. Reassure the parent that the child will catch up to normal development by age 2 years. Re-evaluate this child's development and milestone achievements at the 2-year visit. Refer the child to a specialty clinic for evaluation and treatment of developmental delay.

ANS: A This child should be at a 19-month adjusted age for prematurity so, according to the parent screen, is 4 months behind. The PNP should perform a more in-depth screen to evaluate this delay. Waiting to see if the child will "catch up" or assuring the parent that this will happen will cause the delays to become more severe. A referral to a specialty clinic should not be made solely on the basis of the parent-completed questionnaire but only after further evaluation of possible delays.

A 14-year-old boy who is overweight develops a unilateral limp with pain in the hip and knee on the affected side. An exam reveals external rotation of the hip when flexed and pain associated with attempts to internally rotate the hip. What is most important initially when managing this child's condition? Place the child on crutches or in a wheelchair to prevent weight-bearing. Provide information about weight loss to minimize further injury. Recommend seeing an orthopedic specialist as soon as possible. Refer the child to physical therapy to improve range of motion.

ANS: A This child's age, history, and symptoms are consistent with slipped capital femoral epiphysis. The child should be placed on crutches or in a wheelchair to prevent weight bearing. Obesity is often part of the history and should be managed, but the immediate need is to prevent further damage to the hip. Referral to orthopedics should immediately follow prevention of weight bearing. Physical therapy may be part of treatment after the epiphysis is stabilized.

A 14-year-old female has menometrorrhagia with moderate increase in menstrual flow and irregular periods. Her hemoglobin is 13.1 g/dL. How will this be managed? Iron supplementation and prostaglandin inhibitors One OCP twice daily for 3 to 4 days and then daily Progestin every day for 10 to 14 days Referral to a pediatric gynecologist for treatment

ANS: A This patient has mild AUB and may be managed by observation and reassurance along with iron to prevent anemia and prostaglandin inhibitors to reduce heavy bleeding. Patients with moderate AUB may be prescribed OCPs or progestins. Referral to a gynecologist is warranted with severe AUB.

The primary care pediatric nurse practitioner is preparing to conduct a well child assessment of an 8-year-old child. How will the nurse practitioner begin the exam? Ask the child about school, friends, home activities, and sports Discuss the purpose of the visit and explain the procedures that will be performed Offer age-appropriate information about usual developmental tasks Provide information about healthy nutrition and physical activities

ANS: A To build rapport with the child and parent, the PNP will begin by asking direct questions to the child, encouraging the child to share information about daily routines. The other answers list aspects of the well child visit that can be introduced after the initial conversation.

A young adolescent female is observed to have mild unequal scapula prominences on gross examination while standing. In the Adams forward bending position, this inequality disappears. What will the primary care pediatric nurse practitioner do? Discuss posture and exercise and ask about backpacks and books. Obtain radiographic studies of the entire spine and neck. Reassure the child's parent that functional scoliosis will self-resolve. Refer to an orthopedic specialist for evaluation and possible bracing.

ANS: A Unequal scapulas noted on standing can denote scoliosis and deserves a more thorough physical assessment. Functional scoliosis can be diagnosed by assessing curves in the spine in the Adams forward bending position. Although it is relatively benign, functional scoliosis can progress to structural scoliosis if not treated, which can be done with physical therapy or other means, such as exercises or removing external forces (carrying heavy loads, heavy one-sided backpacks) that place unequal pressures on the spine, to prevent progression. Radiographs may be necessary if this worsens. This will not self-resolve but must be managed to prevent progression. Orthopedic referral is not necessary at this early, modifiable stage.

A 12-year-old child who plays soccer is diagnosed with vocal cord dysfunction. What will the primary care nurse practitioner say when the child's parents ask about continued sports participation? The child may continue to participate in soccer. The child should limit activity to non-aerobic sports. This condition is a contraindication for all sports. This condition predisposes the child to sudden cardiac death.

ANS: A Vocal cord dysfunction causes shortness of breath and must be managed but does not prevent children from participation in sports. It does not indicate underlying cardiac problems and does not mean children should avoid any sport that may increase heart or respiratory rates.

A previously healthy school-age child develops a cough and a low-grade fever. The primary care pediatric nurse practitioner auscultates wheezes in all lung fields. Which diagnosis will the nurse practitioner suspect? Atypical pneumonia Bacterial pneumonia Bronchiolitis Bronchitis

ANS: A Wheezing in a child over 5 years of age without a history of wheezing may point to an atypical pneumonia. Bacterial pneumonia is characterized by diminished breath sounds or crackles along with high fever. Bronchiolitis causes coarse wheezing. Bronchitis is characterized by cough without adventitious lung sounds.

10. The primary care pediatric nurse practitioner is examining a 12-month-old infant who was 6 weeks premature and observes that the infant uses a raking motion to pick up small objects. The PEDS questionnaire completed by the parent did not show significant developmental delays. What will the nurse practitioner do first? a. Perform an in-depth developmental assessment. b. Reassure the parent that this is normal for a premature infant. c. Refer the infant to a developmental specialist. d. Suggest activities to improve fine motor skills.

ANS: A When developmental screening indicates an infant is not progressing at the expected rate, additional testing to determine the degree of delay is necessary. A referral may be needed if a delay is determined. This is not normal for this degree of prematurity; infants should develop a pincer grasp by 9 to 10 months of age.

The primary care pediatric nurse practitioner is examining a 12-month-old infant who was 6 weeks premature and observes that the infant uses a raking motion to pick up small objects. The PEDS questionnaire completed by the parent did not show significant developmental delays. What will the nurse practitioner do first? Perform an in-depth developmental assessment. Reassure the parent that this is normal for a premature infant. Refer the infant to a developmental specialist. Suggest activities to improve fine motor skills.

ANS: A When developmental screening indicates an infant is not progressing at the expected rate, additional testing to determine the degree of delay is necessary. A referral may be needed if a delay is determined. This is not normal for this degree of prematurity; infants should develop a pincer grasp by 9 to 10 months of age.

In a respiratory disorder causing a check-valve obstruction, which symptoms will be present? Air entry on inspiration with expiratory occlusion Complete obstruction on inspiration and expiration Narrowing of the lumen with increased air flow resistance Obstruction of air entry with unimpeded expiratory air flow

ANS: A With check-valve or ball-valve obstruction, air entry is possible, but the lumen is completely occluded during expiration so that air escape is impossible. A complete obstruction does not allow inspiratory or expiratory air flow. A bypass valve obstruction allows air flow but with increased resistance. There is no obstruction that impedes inspiration while allowing expiratory air flow.

The primary care pediatric nurse practitioner is performing a pre-participation sports physical examination on a 14-year-old male who will be on the wrestling team at school. What will the nurse practitioner include when discussing healthy practices with this adolescent? Risks associated with repeatedly losing and gaining weight The need for an electrocardiogram or echocardiogram prior to participation The need to consume 20 to 30 grams of protein after exercise To consume water with CHO prior to activity lasting up to an hour

ANS: A Wrestlers often try to lose weight rapidly prior to wrestling matches to put themselves into a lower weight category. It is important to teach young athletes about the risks associated with repeated weight loss and gain. ECG and echocardiograms are not recommended as a requirement for all pre-participation physical exams unless there is an indication for doing so, such as with syncope or murmurs. Athletes do not need to consume 10 to 20 grams of protein after exercise; complex carbohydrates are recommended to improve muscle glycogen resynthesis. Plain water is recommended before, during, and after all activity lasting up to an hour.

7. The primary care pediatric nurse practitioner performs a Hirschberg test to evaluate a. color vision. b. ocular alignment. c. peripheral vision. d. visual acuity.

ANS: B The Hirschberg test, or corneal light reflex, assesses ocular mobility and alignment by looking for symmetry of reflected light. Color vision testing is performed with Richmond pseudo-isochromatic plates. Peripheral vision is tested by watching the child's response to objects as they are moved in and out of the visual fields. Visual acuity is performed using eye charts or visual-evoked potential readings.

The primary care pediatric nurse practitioner performs a Hirschberg test to evaluate color vision. ocular alignment. peripheral vision. visual acuity.

ANS: B The Hirschberg test, or corneal light reflex, assesses ocular mobility and alignment by looking for symmetry of reflected light. Color vision testing is performed with Richmond pseudo-isochromatic plates. Peripheral vision is tested by watching the child's response to objects as they are moved in and out of the visual fields. Visual acuity is performed using eye charts or visual-evoked potential readings.

A 16-year-old female reports breast tenderness and a "lump." The primary care pediatric nurse practitioner palpates a small fluid-filled mass in her right breast. A pregnancy test is negative. Which action is correct? Obtain a CBC to rule out infection. Order an ultrasound of the mass. Prescribe NSAIDs to treat her discomfort. Reassure her that the findings are normal.

ANS: B A cyst is usually a fluid-filled mass and should be evaluated by ultrasound to confirm this. The mass is not warm or consistent with mastitis, so a CBC is not indicated. NSAIDs may be offered once the diagnosis is confirmed. If the US reveals a cyst, reassurance can be given.

A 2-month-old infant cries up to 4 hours each day and, according to the parents, is inconsolable during crying episodes with fists and legs noted to be tense and stiff. The infant is breastfeeding frequently but is often fussy during feedings. The physical exam is normal and the infant is gaining weight normally. What will the primary care pediatric nurse practitioner recommend? A complete work-up, including laboratory and radiologic tests Eliminating certain foods from the mother's diet Empiric treatment with a proton pump inhibitor medication Stopping breastfeeding and beginning a hydrolyzed formula

ANS: B A first step in a breastfeeding infant with symptoms of colic should be to try eliminating certain foods such as cow's milk products, nuts, eggs, and fish from the mother's diet to see if improvement occurs. It is not necessary to stop breastfeeding; hydrolyzed formulas may be tried in formula-fed infants. An infant with a normal exam and normal weight gain does not need further diagnostic studies. PPIs are not indicated as first-line therapy.

A 6-month-old infant has a retractile testis that was noted at the 2-month well baby exam. What will the primary care pediatric nurse practitioner do to manage this condition? Reassure the parent that the testis will most likely descend into place on its own. Refer the infant to a pediatric urologist or surgeon for possible orchiopexy. Teach the parent to manipulate the testis into the scrotum during diaper changes. Tell the parent that hormonal therapy may be needed to correct the condition.

ANS: B A retractile testis that does not retain scrotal residence should be referred to a pediatric urologist or surgeon by 6 months of age. By that age, the child should be seen by a specialist to rule out orchiopexy, in which the testis does not descend on its own (parental reassurance would not be appropriate). Teaching the parent to manipulate the testis is not indicated. Hormonal therapy has not demonstrated efficacy in stimulating testicular descent.

A 9-month-old infant is brought to the clinic with scrotal swelling and fussiness. The primary care pediatric nurse practitioner notes a tender mass in the affected scrotum that is difficult to reduce. What is the correct action? Obtain an abdominal radiograph. Refer immediately to a pediatric surgeon. Schedule an appointment with a pediatric urologist. Teach the parents signs of incarceration.

ANS: B A scrotal mass that is difficult to reduce or is painful is likely to be a hernia. Immediate referral is indicated to rule out incarceration, which is a medical emergency with potentially severe consequences if not promptly treated. The PNP may order radiographs to distinguish a hernia from a hydrocele, but not when these symptoms occur. The referral must be immediate, since surgery is required. A child with a non-tender, reducible hernia will require referral, but parents can be taught signs of incarceration until an appointment can be scheduled and the specialist seen.

A child is in the clinic after swallowing a metal bead. A radiograph of the GI tract shows a 6 mm cylindrical object in the child's stomach. The child is able to swallow without difficulty and is not experiencing pain. What is the correct course of treatment? Administer ipecac to induce vomiting. Have the parents watch for the object in the child's stool. Insert a nasogastric tube to flush out the object. Refer the child for endoscopic removal of the object.

ANS: B A small foreign body that is not corrosive or sharp and that has reached the stomach is most likely to continue to pass through the GI tract and no intervention is necessary. Inducing vomiting increases the risk of aspiration of the FB. NG tube removal and endoscopy are not indicated unless the object has the potential to damage the GI tract; most objects that are not sharp or corrosive that have reached the abdomen will pass through without causing damage.

The primary care pediatric nurse practitioner is examining a 3-year-old child who speaks loudly, in a monotone, does not make eye contact, and prefers to sit on the exam room floor moving a toy truck back and forth in a repetitive manner. Which disorder does the nurse practitioner suspect? Attention-deficit/hyperactivity disorder Autism spectrum disorder Executive function disorder Sensory processing disorder

ANS: B Autism spectrum disorder manifests in toddlers by alterations in socialization and speech as described above, along with repetitive behaviors. ADHD manifests with a lack of focus on activities and distractibility. Executive function disorders can manifest in a variety of ways but not with repetitive behaviors. Children with sensory processing disorders have altered responses to sensations.

A 10-year-old is hit in the head with a baseball during practice and is diagnosed with concussion, even though no loss of consciousness occurred. The primary care pediatric nurse practitioner is evaluating the child 2 weeks after the injury and learns that the child is still experiencing some sleepiness every day. The neurological exam is normal. The child and the parent are adamant that the child be allowed to return to play baseball. What will the nurse practitioner recommend? Continuation of cognitive rest only Continuation of physical and cognitive rest Continuation of physical rest only Returning to play

ANS: B Both physical and cognitive rest is indicated after diagnosis of concussion in youth, particularly if symptoms continue following injury. Cognitive recovery may lag behind physical recovery and is a key factor in return-to-play decisions. Only after all symptoms resolve may athletes progress through steps to gradually return to play.

5. The mother of a 3-month-old child tells the primary care pediatric nurse practitioner that it is "so much fun" now that her infant coos and smiles and wants to play. What is important for the nurse practitioner to teach this mother? a. Appropriate ways to stimulate and entertain the infant b. How to read the infant's cues for overstimulation c. The importance of scheduling "play dates" with other infants d. To provide musical toys to engage the infant

ANS: B By 3 months, infants demonstrate a social smile and will become more active, alert, and responsive. Parents may mistakenly assume that the infant can handle more activity and stimulation when this occurs, and the PNP should teach caregivers how to recognize infant cues for the need to rest or to have decreased stimulation.

The mother of a 3-month-old child tells the primary care pediatric nurse practitioner that it is "so much fun" now that her infant coos and smiles and wants to play. What is important for the nurse practitioner to teach this mother? Appropriate ways to stimulate and entertain the infant How to read the infant's cues for overstimulation The importance of scheduling "play dates" with other infants To provide musical toys to engage the infant

ANS: B By 3 months, infants demonstrate a social smile and will become more active, alert, and responsive. Parents may mistakenly assume that the infant can handle more activity and stimulation when this occurs, and the PNP should teach caregivers how to recognize infant cues for the need to rest or to have decreased stimulation.

4. During a well-baby assessment on a 1-week-old infant who had a normal exam when discharged from the newborn nursery 2 days prior, the primary care pediatric nurse practitioner notes moderate eyelid swelling, bulbar conjunctival injections, and moderate amounts of thick, purulent discharge. What is the likely diagnosis? a. Chemical-induced conjunctivitis b. Chlamydia trachomatis conjunctivitis c. Herpes simplex virus (HSV) conjunctivitis d. Neisseria gonorrhea conjunctivitis

ANS: B C. trachomatis conjunctivitis usually begins between 5 to 14 days of life and causes moderate eyelid swelling, palpebral or bulbar conjunctivitis, and moderate, thick, purulent discharge. Chemical-induced conjunctivitis manifests as nonpurulent discharge. HSV is characterized by serosanguinous discharge. N. gonorrhea causes acute conjunctival inflammation and excessive purulent discharge.

During a well-baby assessment on a 1-week-old infant who had a normal exam when discharged from the newborn nursery 2 days prior, the primary care pediatric nurse practitioner notes moderate eyelid swelling, bulbar conjunctival injections, and moderate amounts of thick, purulent discharge. What is the likely diagnosis? Chemical-induced conjunctivitis Chlamydia trachomatis conjunctivitis Herpes simplex virus (HSV) conjunctivitis Neisseria gonorrhea conjunctivitis

ANS: B C. trachomatis conjunctivitis usually begins between 5 to 14 days of life and causes moderate eyelid swelling, palpebral or bulbar conjunctivitis, and moderate, thick, purulent discharge. Chemical-induced conjunctivitis manifests as nonpurulent discharge. HSV is characterized by serosanguinous discharge. N. gonorrhea causes acute conjunctival inflammation and excessive purulent discharge.

The primary care pediatric nurse practitioner is performing a well child examination on a 5-year-old girl. The parents ask if the child s adult height can be predicted. The nurse practitioner learns that the mother is 5'8" tall and the father is 5'11" tall. The nurse practitioner will estimate which expected adult height for this child? 5'11" tall 5'7" tall 5'8" tall 6' tall

ANS: B Calculation of expected adult height for a girl is: [(Father s height - 5 in.) + (Mother s height)]/2, so [(71" - 5") + 68"]/2 = 134/2 = 67" or 5'7". A calculation of boy s expected height: [(Mother s height + 5 in.) + (Father s height)]/2 would be 6' tall.

A school-age child has begun refusing all cooked vegetables. What will the primary care pediatric nurse practitioner recommend to the parent? Allow the child to make food choices since this is usually a phase Ensure that the child has three nutritious meals and two nutritious snacks each day Prepare vegetables separately for the child to encourage adequate intake Teach the child how important it is to eat healthy fruits and vegetables

ANS: B Children have food jags that are generally self-limited. The parent's responsibility is to provide three nutritious meals and two nutritious snacks each day so that all available choices are acceptable. Allowing food choices may result in an overabundance of non-nutritious foods selected. It is not necessary to prepare separate dishes for a child who is going through a temporary phase. Teaching the child about nutrition is important but will not likely have much impact during this phase.

The parent of a 4-year-old child reports that the child gets upset when the hall light is left on at night and won't leave the house unless both shoes are tied equally tight. The primary care pediatric nurse practitioner recognizes that this child likely has which type of sensory processing disorder? Dyspraxia Over-responder Sensory seeker Under-responder

ANS: B Children who are over-responders have difficulties with clothing, physical contact, light, sounds, and food. Dyspraxia refers to difficulty recognizing and distinguishing shapes and textures. Sensory seekers are on perpetual overdrive and often in trouble. Under-responders have little or no reaction to stimulation, pain, and extreme hot or cold.

The parent of a toddler diagnosed with grade V vesicoureteral reflux asks the primary care pediatric nurse practitioner how the disease will be treated. What will the nurse practitioner tell this parent? That long-term antibiotic prophylaxis will prevent scarring That surgery to correct the condition is possible that the child will most likely require kidney transplant that the condition will probably resolve spontaneously

ANS: B Children with grade V VUR generally do not experience spontaneous resolution and will likely have to have surgery to correct the condition. Antibiotic prophylaxis will not necessarily prevent scarring. Unless scarring occurs and is severe, kidney transplantation is not likely.

The parent of a child who has asthma asks the primary care pediatric nurse practitioner about whether the child may engage in strenuous exercise. What will the nurse practitioner tell the parent? Children with asthma should be excluded from vigorous exercise and most strenuous sports. Children with asthma show improved aerobic and anaerobic fitness with moderate to vigorous/physical activity. Physical activity has been shown to improve overall pulmonary function in children with asthma. Vigorous exercise helps improve symptoms in children with poorly controlled asthma.

ANS: B Children with mild or well-controlled asthma may participate in moderate to vigorous sports and show benefits to aerobic and anaerobic fitness, which helps lung function and overall health outcomes. It is not necessary to exclude children with asthma from sports as long as symptoms are well controlled. Overall pulmonary function does not substantially improve with exercise. Children with poor control should not engage in sports until symptoms are under control.

The primary care pediatric nurse practitioner diagnoses an 8-year-old child with type 1 diabetes after a routine urine screen is positive for glucose and negative for ketones and plasma glucose is 350 mg/dL. The child's weight is normal and the parents report a mild increase in thirst and urine output in the past few days. Which course of action is correct? Admit the child to the hospital for initial insulin management. Begin insulin and refer the child to a children's diabetes center. Order a fasting serum glucose and a dipstick UA in the morning. Send the child to the emergency department for fluids and IV insulin.

ANS: B Children without diabetic ketoacidosis may be managed as outpatients, but newly diagnosed patients should be referred to a children's diabetes center for insulin therapy and diabetes education. Unless the child is ketoacidotic, hospitalization or ED referral is not indicated. It is not necessary to order further testing.

A 5-year-old child has an elevated blood pressure during a well child exam. The primary care pediatric nurse practitioner notes mottling and pallor of the child's feet and lower legs and auscultates a systolic ejection murmur in the left infraclavicular region radiating to the child's back. The nurse practitioner will suspect which condition? Aortic stenosis Coarctation of the aorta Patent ductus arteriosus Pulmonic stenosis

ANS: B Coarctation of the aorta may not have symptoms until later childhood and may present as high blood pressure in the upper extremities and poor perfusion in the lower extremities. A systolic ejection murmur at the left infraclavicular region with transmission to the back is characteristic. Aortic stenosis is characterized by a louder, harsh systolic crescendo-decrescendo murmur at the upper right sternal border with radiation to the neck, LLSB, and apex. PDA has a machinery-like murmur. Pulmonic stenosis may be asymptomatic or may have a harsh, late systolic ejection murmur at the upper left sternal border that transmits to both lung fields.

The primary care pediatric nurse practitioner is examining a 17-year-old male who is on his high school swim team. The adolescent is concerned about "lumps" on his chest. The nurse practitioner notes a marked increase in weight since the last visit along with worsening of the adolescent's acne. Given this set of symptoms, which performance-enhancing substance will the nurse practitioner be most concerned about and ask about? Creatine Dehydroepiandrosterone (DHEA) Ephedra Growth hormone

ANS: B DHEA is a prohormone that is converted to either testosterone or estrone and will cause adverse changes similar to anabolic steroids, such as increased weight, gynecomastia, and acne. Creatine is taken because athletes believe it enhances endurance. Side effects include weight gain but not androgenic effects such as gynecomastia or acne. Ephedra is similar to amphetamine, with most side effects related to the heart, such as tachycardia and arrhythmias. Growth hormone will cause increased weight and has side effects associated with diabetes, cardiomyopathy, hepatitis, and renal failure.

The primary care pediatric nurse practitioner performs a developmental assessment on a 3-year-old child and notes normal cognitive, fine-motor, and gross-motor abilities. The child responds appropriately to verbal commands during the assessment but refuses to speak when asked questions. The parent tells the nurse practitioner that the child talks at home and that most other adults can understand what the child says. The nurse practitioner will : ask the parent to consider a possible speech delay and report any concerns. continue to evaluate the child's speech at subsequent visits. refer the child for a speech and hearing evaluation. tell the parent to spend more time in interactive conversations with the child.

ANS: B Development should be monitored over time and within the context of the child's overall well-being, rather than at an isolated testing session. The child has normal development in observed measures and appears to hear and understand well. By parental report, the child is able to speak. The PNP should continue to evaluate speech over time, since this refusal to speak may be associated with shyness or intimidation in the clinic. It is not necessary to tell the parent that the child has a possible speech delay. Unless an actual speech delay is observed, a referral is not indicated, nor is it necessary to implement a home therapy.

5. Which recommendation will a primary care pediatric nurse practitioner make when parents ask about ways to discipline their 3-year-old child who draws on the walls with crayons? a. Give the child washable markers so the drawings can be removed easily. b. Provide a roll of paper for drawing and teach the child to use this. c. Put the child in "timeout" each time the child draws on the walls. d. Take the crayons away from the child to prevent the behavior.

ANS: B Discipline involves training or education that molds appropriate behavior and is used to teach the child what is permitted and encouraged. Providing an appropriate outlet for drawing helps to teach the child where to use the crayons. Using washable markers allows the parents to clean the walls but does not teach the child appropriate behaviors. Timeout and taking away the crayons are forms of punishment, or a loss of privileges, that are administered as a form of retribution.

Which recommendation will a primary care pediatric nurse practitioner make when parents ask about ways to discipline their 3-year-old child who draws on the walls with crayons? Give the child washable markers so the drawings can be removed easily. Provide a roll of paper for drawing and teach the child to use this. Put the child in "timeout" each time the child draws on the walls. Take the crayons away from the child to prevent the behavior.

ANS: B Discipline involves training or education that molds appropriate behavior and is used to teach the child what is permitted and encouraged. Providing an appropriate outlet for drawing helps to teach the child where to use the crayons. Using washable markers allows the parents to clean the walls but does not teach the child appropriate behaviors. Timeout and taking away the crayons are forms of punishment, or a loss of privileges, that are administered as a form of retribution.

An adolescent has suspected infectious mononucleosis after exposure to the virus in the past week. The primary care pediatric nurse practitioner examines the adolescent and notes exudate on the tonsils, soft palate petechiae, and diffuse adenopathy. Which test will the primary care pediatric nurse practitioner perform to confirm the diagnosis? Complete blood count EBV-specific antibody testing Heterophile antibody testing Throat culture

ANS: B EBV-specific IgG antibody testing is the specific serologic test for EBV infection. Heterophile antibody testing can be helpful in school-age children and adolescents after the first week of infection. A CBC can identify lymphocytosis with atypical lymphocytes but is non-specific. A throat culture is performed to identify bacterial causes; however, in this case of known exposure to EBV this would not be the appropriate confirmatory test.

The parents of a pre-pubertal female who is on the local swim team tell the primary care pediatric nurse practitioner that their daughter wants to begin a strength training program to help improve her swimming ability. What will the nurse practitioner recommend? Avoiding strength training programs until after puberty to minimize the risk for injury Enrolling their daughter in a program that uses fixed weight machines or resistance bands Having their daughter participate in weight training 4 or 5 times each week for maximum effect Making sure that their daughter begins with the greatest weight tolerable using lower repetitions

ANS: B Fixed weights or resistance bands are recommended for pre-pubertal youth to help prevent injury. Strength training prior to menarche helps to strengthen long bones and is considered beneficial. Weight training should be 2 to 3 times weekly with a day in between sessions. Initially, youth should begin with a low number of sets and low intensity.

A child who has attention-deficit/hyperactivity disorder (ADHD) has difficulty stopping activities to begin other activities at school. The primary care pediatric nurse practitioner understands that this is due to difficulty with the self-regulation component of emotional control. flexibility. inhibition. problem-solving.

ANS: B Flexibility is a component of self-regulation, which is under the control of executive functions in the cerebral cortex and is the ability to shift or transition between activities or thoughts. Emotional control is the ability to modify emotional expression to the most adaptive expression. Inhibition refers to the ability to stop or delay an initial response. Problem solving is a component of metacognition.

A sexually active adolescent female tests positive for N. gonorrhoeae and C. trachomatis. She tells the primary care pediatric nurse practitioner that she wants to be treated today since she is moving out of town the next day. What will the nurse practitioner order? Azithromycin 1 g PO in a single dose Ceftriaxone 250 mg IM and azithromycin 1 g PO one time each Doxycycline 100 mg PO bid for 7 days Erythromycin base 500 mg PO qid for 7 days

ANS: B Follow-up cultures for gonorrhea are required unless ceftriaxone is used. Azithromycin is used to treat both gonorrhea and chlamydia. The other options involve twice daily and four times daily dosing, which present difficulties with compliance.

A child is brought to the clinic after falling from a swing and scraping both knees and hands. An examination reveals abraded skin with oozing serous fluid and blood, along with dirt and grime from the playground surface. What will the primary care pediatric nurse practitioner do to minimize the risk of infection? a. Apply povidone-iodine to all areas. b. Irrigate gently with normal saline. c. Rinse with hydrogen peroxide. d. Scrub the abraded areas with alcohol.

ANS: B Gentle irrigation with water or normal saline is the preferred method for cleaning a wound. Povidone-iodine, alcohol, and hydrogen peroxide should not be used on open wounds.

A child is diagnosed with nephrotic syndrome, and the pediatric nurse practitioner provides primary care in consultation with a pediatric nephrologist. The child was treated with steroids and responded well to this treatment. What will the nurse practitioner tell the child's parents about this disease? "Future episodes are likely to have worse outcomes." "Steroids will be used when relapses occur." "This represents a cure from this disease." "Your child will need to take steroids indefinitely."

ANS: B In situations in which a child responds well to steroids, this shows promise of a good prognosis, indicating that the child may be treated successfully with steroids during future anticipated relapses. The fact that a child is a "steroid responder" indicates that future episodes of treatment will be successful and have positive outcomes. This disease is chronic and not curable. Steroid use with children who respond positively is intermittent during episodes of relapse. Steroids are not given continuously and are not seen as prophylactic.

The primary care pediatric nurse practitioner provides primary care for a 4-month-old infant who has a ventricular septal defect. The infant has been breastfeeding well but in the past month has dropped from the 20th percentile to the 5th for weight. What will the nurse practitioner recommend? Adding solid foods to the infant's diet to increase caloric intake Fortifying breast milk to increase the number of calories per ounce Stopping breastfeeding and giving 30 kcal/ounce formula Supplementing breastfeeding with 24 kcal/ounce formula

ANS: B Infants with heart defects who have CHF may need modification of formula or breast milk to increase calories. This infant is nursing well, so fortifying the breast milk to increase calories is the first and best option. Adding solids does not significantly increase caloric intake.

The primary care pediatric nurse practitioner manages care in conjunction with a pediatric pulmonologist for a child with cystic fibrosis. Which medication regimen is used to facilitate airway clearance for this child? Ibuprofen and azithromycin Inhaled dornase alfa Ivacaftor Prophylactic clindamycin

ANS: B Inhaled dornase alfa is given to promote airway clearance by reducing mucus viscosity. Ibuprofen and azithromycin is given to reduce chronic airway inflammation. Ivacaftor is given to patients with specific gene mutations. Antibiotic therapy is based on regular sputum cultures.

A child is diagnosed with Crohn disease. What are likely complications for this child? Cancer of the colon and possible colectomy Intestinal obstruction with scarring and strictures Intestinal perforation and hemorrhage Liver disease and sepsis

ANS: B Intestinal obstruction with scarring and strictures are the major complications of CD. The other answers describe complications of ulcerative colitis.

A 2-year-old child has an acute diarrheal illness. The child is afebrile and, with oral rehydration measures, has remained well hydrated. The parent asks what can be done to help shorten the course of this illness. What will the primary care pediatric nurse practitioner recommend? Clear liquids only Lactobacillus Loperamide Peppermint oil

ANS: B Lactobacillus, given early in a viral diarrheal illness, can decrease the duration of diarrhea by about 25 hours and is safe to use in children. Parents should begin refeeding early to stimulate enterocyte growth and help facilitate mucosal repair. Loperamide may be given to children over the age of 3 years. Peppermint oil may help reduce cramping, but its efficacy is not certain.

The primary care pediatric nurse practitioner evaluates children's growth to screen for endocrine and metabolic disorders. Which is a critical component of this screening? Measuring supine length in children over the age of 2 years Obtaining serial measurements to assess patterns over time Using the CDC growth chart for children under age 2 years Using the WHO growth chart for children over age 2 years

ANS: B Measuring stature is an important part of a physical examination to screen for endocrine and metabolic disorders. Serial measurements are critical to assess growth patterns over time. Supine length is measured in children under the age of 2 years. CDC growth charts should be used in children older than 2 years and WHO charts in children under age 2 years.

An adolescent female has periods every 30 days that are consistently heavy and last from 5 to 8 days. What is her diagnosis? Menometrorrhagia Menorrhagia Metrorrhagia Polymenorrhea

ANS: B Menorrhagia is characterized by normal period intervals with excessive flow or duration of menses. Menometrorrhagia involves both excessive bleeding and irregular cycles. Metrorrhagia is irregular frequency with bleeding between cycles. Polymenorrhea is when there are fewer than 21 days between cycles.

The primary care pediatric nurse practitioner is evaluating a heart murmur during a pre-participation examination of a high school athlete. Which finding would be a concern requiring referral to a cardiologist? A murmur that is louder when squatting and softer when standing A murmur that is quieter when squatting and louder with a Valsalva maneuver A murmur with narrow and variable splitting of S2 A systolic murmur that is grade 1 or 2

ANS: B Normally, squatting will increase venous return to the heart and cause murmurs to be louder, while standing or performing a Valsalva maneuver will cause murmurs to be quieter. If the reverse is true, then hypertrophic cardiomyopathy or mitral valve prolapse must be ruled out. A murmur with a wide or fixed splitting of S2 must be evaluated. A split S2 that is variable, particularly in synchrony with respirations, is common, and a narrow S2 split is of less concern but should be monitored over time. Systolic murmurs of grade 3 or greater must be evaluated by specialists; however, murmurs of grades 1 and 2 do not need to be evaluated by a cardiologist.

A child with a history of otitis externa asks about ways to prevent this condition. What will the primary care pediatric nurse practitioner recommend? Cleaning ear canals well after swimming Drying the ear canal with a hair dryer Swimming only in chlorinated pools Using cerumenolytic agents daily

ANS: B Otitis externa is most frequently caused by retained moisture in the ear canal after swimming and when the protective barriers on the skin break down. Drying the ear canals with a hair dryer on a low setting helps to remove the moisture. Cleaning the ear canals, swimming in chlorinated water, and using a cerumenolytic remove the wax that protects the ear canal from superficial infection.

A child complains of itching in both ears and is having trouble hearing. The primary care pediatric nurse practitioner notes periauricular edema and marked swelling of the external auditory canal and elicits severe pain when manipulating the external ear structures. Which is an appropriate intervention? Obtain a culture of the external auditory canal. Order ototopical antibiotic/corticosteroid drops. Prescribe oral amoxicillin-clavulanate. Refer the child to an otolaryngologist.

ANS: B Ototopical antibiotic/corticosteroid drops are the mainstay of therapy for OE. It is not necessary to obtain a culture unless the infection does not respond to treatment. Oral antibiotics are not indicated unless impetigo occurs and is severe. A referral to a specialist is not recommended.

A 9-month-old infant has vesiculopustular lesions on the palms and soles, on the face and neck, and in skin folds of the extremities. The primary care pediatric nurse practitioner notes linear and S-shaped burrow lesions on the parent's hands and wrists. What is the treatment for this rash for this infant? Ivermectin 200 mcg/kg for 7 to 14 days, along with symptomatic treatment for itching Permethrin 5% cream applied to face, neck, and body and rinsed off in 8 to 14 hours Treatment of all family members except the infant with permethrin 5% cream and ivermectin Treatment with permethrin 5% cream for 7 days in conjunction with ivermectin 200 mcg/kg

ANS: B Permethrin 5% cream is the drug of choice for treating scabies and is intended for use in infants as young as 2 months of age. Infants will get lesions on the face and neck, and permethrin may be applied to the face, avoiding the eyes. Ivermectin is not recommended for children under 5 years old. Treatment must include the infant as well as all family members whether symptomatic or not.

A 3-year-old child has head lice. What will the initial treatment recommendation be to treat this child? Lindane Permethrin Pyrethrin Spinosad

ANS: B Permethrin is the treatment of choice for head lice because of its safety and efficacy. Pyrethrin has more treatment failures and is not the first-line treatment. Lindane has neurotoxic side effects and is only recommended when treatment failure occurs. Spinosad is used in children 4 years and older.

A school-age child steps on a nail while wearing tennis shoes and develops cellulitis in that foot. The child's immunizations are up-to-date. What antibiotic will the pediatric nurse practitioner empirically prescribe? a. Amoxicillin-clavulanate b. Ciprofloxacin c. Clindamycin d. Trimethoprim-sulfamethoxazole

ANS: B Plantar puncture wounds, particularly those wounds that occur following puncture of sneakers/shoes, require ciprofloxacin to cover potential Pseudomonas infection and to protect against an osteomyelitis. Amoxicillin-clavulanate is used in other puncture wounds with signs of infection. Clindamycin is used for similarly wounded children allergic to penicillins. TMP-SMX is used if MRSA is cultured.

A dipstick urinalysis is positive for leukocyte esterase and nitrites in a school-age child with dysuria and foul-smelling urine but no fever who has not had previous urinary tract infections. A culture is pending. What will the pediatric nurse practitioner do to treat this child? Order ciprofloxacin ER once daily for 3 days if the culture is positive. Prescribe trimethoprim-sulfamethoxazole (TMP) twice daily for 3 to 5 days. Reassure the child's parents that this is likely an asymptomatic bacteriuria. Wait for urine culture results to determine the correct course of treatment.

ANS: B Short-term antibiotics of 3 to 5 days may be as effective for treating UTI in non-febrile bladder infections and TMP is generally a first-line drug in children without history of UTI. Ciprofloxacin is used in adolescents older than 18 years and this child is symptomatic with positive leukocyte esterase and nitrites and will need treatment. Asymptomatic bacteriuria occurs when bacteria are in the urine of a child who is asymptomatic (without symptoms).

The primary care pediatric nurse practitioner is performing a well child exam on an 8-year-old girl and notes the presence of breast buds. What will the nurse practitioner include when initiating anticipatory guidance for this patient? A discussion about the risks of pregnancy and sexually transmitted diseases Information about sexual maturity and menstrual periods Material about the human papillomavirus vaccine Sexual orientation and the nature of sexual relationships

ANS: B Since this child is 8 years old, it is early to discuss sexual behavior and reproduction given the level of the child's cognition and understanding. However, with these early changes in telearche marking the onset of puberty, it is wise to discuss menstruation in an age-appropriate manner before it occurs so that the child can be prepared. Since this child is showing signs of early puberty, this information can be included in anticipatory guidance.

The primary care pediatric nurse practitioner notes a small, round object in a child's external auditory canal, near the tympanic membrane. The child's parent thinks it is probably a dried pea. What will the nurse practitioner do to remove this object? Irrigate the external auditory canal to flush out the object. Refer the child to an otolaryngologist for removal. Remove the object with a wire loop curette. Use a bayonet forceps to grasp and remove the object.

ANS: B Spherical objects are the most difficult to remove and should be referred. Irrigation is not recommended for objects made of organic material and also increases the risk of pushing the object farther down.

A child who has nephrotic syndrome is on a steroids and a salt-restricted diet for a relapse of symptoms. A dipstick urinalysis shows 1+ protein, down from 3+ at the beginning of the episode. In consultation with the child's nephrologist, what is the correct course of treatment considering this finding? Begin a taper of the steroid medication while continuing salt restrictions. Continue with steroids and salt restrictions until the urine is negative for protein. Discontinue the steroids and salt restrictions now that improvement has occurred. Relax salt restrictions and continue administration of steroids until proteinuria is gone.

ANS: B Steroid medications and salt restrictions are continued until proteinuria resolves

The primary care pediatric nurse practitioner is considering medication options for a school-age child recently diagnosed with ADHD who has a primarily hyperactive presentation. Which medication will the nurse practitioner select initially? Low-dose stimulant Moderate-dose stimulant Low-dose non-stimulant Moderate-dose non-stimulant

ANS: B Stimulants are generally the first-line medication for ADHD, with non-stimulants recommended for non-responders. Moderate- to high-dose stimulants are recommended for children with primarily hyperactive presentations.

The primary care pediatric nurse practitioner is performing an examination on a 5-year-old child who exhibits ritualistic behaviors, avoids contact with other children, and has limited speech. The parent reports having had concerns more than 2 years ago about autism, but was told that it was too early to diagnose. What will the nurse practitioner do first? Administer an M-CHAT screen to screen the child for communication and socialization delays. Ask the parent to describe the child's earlier behaviors from infancy through preschool. Reassure the parent that if symptoms weren't present earlier, the likelihood of autism is low. Refer the child to a pediatric behavioral specialist to develop a plan of treatment and management.

ANS: B The DSM-5 criteria state that a patient must show symptoms from early childhood even if the symptoms are not recognized until later in life. The parent had noticed symptoms prior but was told not to worry; these symptoms should be evaluated in light of the current symptoms. The M-CHAT is used for infants and toddlers and not for school-age children. Autism symptoms are generally evident by age 3 years. The PNP should complete the assessment before making a referral.

The primary care pediatric nurse practitioner prescribes metformin for a 15-year-old adolescent newly diagnosed with type 2 diabetes. What will the nurse practitioner include when teaching the adolescent about this drug? That insulin therapy will be necessary in the future The importance of checking blood glucose 3 or 4 times daily To consume a diet with foods that are high in vitamin B12 To use a stool softener to prevent gastrointestinal side effects

ANS: B The PNP should counsel youth taking metformin to increase foods high in vitamin B12. Insulin therapy is necessary only for those with ketonuria or diabetic ketoacidosis or later, when the body becomes unable to produce sufficient insulin. Metformin does not cause hypoglycemia, so patients only need to check blood glucose twice daily: once before breakfast and again 2 hours after dinner. GI side effects are mild and self-limiting.

8. The primary care pediatric nurse practitioner is performing a well child examination on a 9-month-old infant whose hearing is normal but who responds to verbal cues with only single syllable vocalizations. What will the nurse practitioner recommend to the parents to improve speech and language skills in this infant? a. Provide educational videos that focus on language. b. Read simple board books to the infant at bedtime. c. Sing to the child and play lullabies in the baby's room. d. Turn the television to Sesame Street during the day.

ANS: B The best way to improve language skills is to read to children. As long as the reading includes positive interactions with the baby and the reader, the baby is learning language. Educational videos, music, and television are all passive media and do not involve this interaction.

The primary care pediatric nurse practitioner is performing a well child examination on a 9-month-old infant whose hearing is normal but who responds to verbal cues with only single syllable vocalizations. What will the nurse practitioner recommend to the parents to improve speech and language skills in this infant? Provide educational videos that focus on language. Read simple board books to the infant at bedtime. Sing to the child and play lullabies in the baby's room. Turn the television to Sesame Street during the day.

ANS: B The best way to improve language skills is to read to children. As long as the reading includes positive interactions with the baby and the reader, the baby is learning language. Educational videos, music, and television are all passive media and do not involve this interaction.

A child is brought to clinic with several bright red lesions on the buttocks. The primary care pediatric nurse practitioner examines the lesions and notes sharp margins and an "orange peel" look and feel. The child is afebrile and does not appear toxic. What is the course of treatment for these lesions? Hospitalize the child for intravenous antibiotics and possible I&D of the lesions. Initiate empiric antibiotic therapy and follow up in 24 hours to assess response. Obtain blood cultures prior to beginning antibiotic treatment. Perform gram stain and culture of the lesions before initiating antibiotics.

ANS: B The child has clinical signs of erysipelas, which is a superficial variant of cellulitis. Because the child is afebrile and doesn't appear toxic, outpatient antibiotics with 24-hour follow-up can be initiated. If the child does not respond or becomes toxic, hospitalization and IV antibiotics are indicated. Blood cultures rarely are positive. Gram stain and cultures are performed if unusual organisms are suspected or if pus is present.

An adolescent has right-sided flank pain without fever. A dipstick urinalysis reveals gross hematuria without signs of infection or bacteriuria, and the primary care pediatric nurse practitioner diagnoses possible nephrolithiasis. What is the initial treatment for this condition? Extracorporeal shockwave lithotripsy (ESWL) Increasing fluid intake up to 2 L daily Percutaneous removal of renal calculi Referral to a pediatric nephrologist

ANS: B The first line of therapy for all stone types is increasing fluids. ESWL may be indicated if symptoms worsen and stones are not passed. Percutaneous removal of renal calculi and referral to nephrology may be indicated with worsening symptoms.

8. The primary care pediatric nurse practitioner performs a well baby assessment of a 5-day-old infant and notes mild conjunctivitis, corneal opacity, and serosanguinous discharge in the right eye. Which course of action is correct? a. Administer intramuscular ceftriaxone 50 mg/kg. b. Admit the infant to the hospital immediately. c. Give oral erythromycin 30 to 50 mg/kg/day for 2 weeks. d. Teach the parent how to perform tear duct massage.

ANS: B The infant has symptoms consistent with HPV conjunctivitis and requires hospitalization for topical and systemic antiviral medications to prevent spread to the central nervous system, mouth, and skin. IM ceftriaxone is given for gonococcal conjunctivitis. Oral erythromycin is given for chlamydial conjunctivitis. Tear duct massage is performed for lacrimal duct obstruction.

The primary care pediatric nurse practitioner performs a well baby assessment of a 5-day-old infant and notes mild conjunctivitis, corneal opacity, and serosanguinous discharge in the right eye. Which course of action is correct? Administer intramuscular ceftriaxone 50 mg/kg. Admit the infant to the hospital immediately. Give oral erythromycin 30 to 50 mg/kg/day for 2 weeks. Teach the parent how to perform tear duct massage.

ANS: B The infant has symptoms consistent with HPV conjunctivitis and requires hospitalization for topical and systemic antiviral medications to prevent spread to the central nervous system, mouth, and skin. IM ceftriaxone is given for gonococcal conjunctivitis. Oral erythromycin is given for chlamydial conjunctivitis. Tear duct massage is performed for lacrimal duct obstruction.

The primary care pediatric nurse practitioner notes velvety, brown thickening of skin in the axillae, groin, and neck folds of an adolescent Hispanic female who is overweight. What is the initial step in managing this condition? Consultation with a pediatric dermatologist Performing metabolic laboratory tests Prescribing topical retinoic acid cream Referral to a pediatric endocrinologist

ANS: B The initial step is to determine whether metabolic syndrome is the underlying cause for these lesions, which, according to the other physical findings, is most likely. If hyperinsulinemia is present, referral to a pediatric endocrinologist is the next step. A dermatology referral is not indicated. Unless the lesions are thick or cause discomfort, prescribing retinoic acid is not necessary.

The primary care pediatric nurse practitioner needs to assess a potential hymenal tear in a prepubertal female who is apprehensive about the exam. Which approach will the nurse practitioner use? Have the child sit frog-legged on the parent's lap. Place the child in the knee-chest position on the exam table. Put the child supine on the exam table with her feet in the stirrups. Refer the child for a speculum exam under sedation.

ANS: B The knee-chest position is the best position for noninvasive, internal examination of the vulva and vagina. The frog-leg position does not allow for internal visualization. Putting the child on the exam table with her feet in the stirrups will increase her anxiety. If an exam cannot be performed in the office because of the child's anxiety, sedation may be indicated, but it is not the first choice.

The primary care pediatric nurse practitioner is performing a well child assessment on a 13-year-old female whose mother asks when her daughter's periods may start. Which information will the nurse practitioner use to help estimate the onset of periods? The age of the mother's menarche The patient's age at thelarche When adrenarche occurred Whether linear growth has stopped

ANS: B Thelarche, or the development of breast budding, generally precedes menarche by 2.5 years, so this should be determined when attempting to predict this milestone. The age of the mother at menarche is not a reliable indicator. Adrenarche is related to adrenal and not gonadal development and is less valid than other secondary sex characteristics in assessing sexual maturation. Rapid linear growth usually begins after thelarche and peaks about a year later but is not used to predict menarche.

A preschool-age child with no previous history has mild flank pain and fever but no abdominal pain or vomiting. A urinalysis is positive for leukocyte esterase and nitrites. A culture is pending. Which is the correct course of treatment for this child? Hospitalize for intravenous antibiotics. Order amoxicillin clavulanate. Prescribe trimethoprim-sulfamethoxazole. Refer for a voiding cystourethrogram.

ANS: B These symptoms suggest this young child may have pyelonephritis. Amoxicillin clavulanate may be given to young children with uncomplicated pyelonephritis who are well hydrated with no abdominal pain or vomiting. Hospitalization is not necessary for uncomplicated pyelonephritis in this age child. TMP is not a first-line drug for pyelonephritis. Voiding cystourethrogram is not indicated for a first febrile UTI.

An 18-month-old child with no previous history of otitis media awoke during the night with right ear pain. The primary care pediatric nurse practitioner notes an axillary temperature of 100.5°F and an erythematous, bulging tympanic membrane. A tympanogram reveals of peak of +150 mm H2O. What is the recommended treatment for this child? Amoxicillin 80 to 90 mg/kg/day in two divided doses An analgesic medication and watchful waiting Ceftriaxone 50 to 75 mg/kg/dose IM given once Ototopical antibiotic drops twice daily for 5 days

ANS: B This child has no previous history and only has a mild fever and can be managed by watchful waiting, with parents given instructions about when and why to notify the provider. Analgesia is essential so that the child can be comfortable. If antibiotics are indicated as a result of no improvement after 48 to 72 hours, amoxicillin is the first-line drug. Ceftriaxone is given if the child is vomiting. Topical antibiotics are given when there is a perforation in the tympanic membrane.

The primary care pediatric nurse practitioner diagnoses acute otitis media in a 2-year-old child who has a history of three ear infections in the first 6 months of life. The child's tympanic membrane is intact and the child has a temperature of 101.5°F. What will the nurse practitioner prescribe for this child? Amoxicillin twice daily for 10 days An analgesic medication and watchful waiting Antibiotic ear drops and ibuprofen Ceftriaxone given once intramuscularly

ANS: B This child has no recent history, is over 24 months, and has relatively mild symptoms, so can be treated by watchful waiting with adequate follow-up and analgesic medication. Antibiotics are not indicated unless the child worsens or does not improve in 48 to 72 hours.

A school-age child has recurrent diarrhea with foul-smelling stools, excessive flatus, abdominal distension, and failure-to-thrive. A 2-week lactose-free trial failed to reduce symptoms. What is the next step in diagnosing this condition? Lactose hydrogen breath test Serologic testing for celiac disease Stool for ova and parasites Sweat chloride test for cystic fibrosis

ANS: B This child has symptoms consistent with celiac disease, especially FTT and foul-smelling stools. Since the lactose-free trial did not reduce symptoms, the likelihood of lactose intolerance is less and thus testing is not likely to be helpful. The symptoms are recurrent, so giardiasis is less likely. CF is still possible, but most children with CF are diagnosed as infants and have accompanying respiratory symptoms of some type.

A 3-year-old child is brought to the clinic by a parent who reports that the child refuses to use the right arm after being swung by both arms while playing. The child is sitting with the right arm held slightly flexed and close to the body. There is no swelling or ecchymosis present. What will the primary care pediatric nurse practitioner do? Consider maltreatment as a possible cause of injury. Gently attempt a supination and flexion technique. Immobilize the arm with a sling and refer to orthopedics. Obtain a radiograph of the child's right arm and elbow.

ANS: B This is most likely an annular ligament displacement injury, or "nursemaid's elbow." The primary provider can attempt to reduce the elbow using either a supination/flexion technique or a pronation technique. Consider maltreatment if recurrent dislocations or other symptoms or signs are present. If this fails after three attempts, immobilization and referral are indicated. Radiologic studies are rarely necessary.

A 9-month-old infant has a grade III/VI, harsh, rumbling, continuous murmur in the left infraclavicular fossa and pulmonic area. A chest radiograph reveals cardiac enlargement. The primary care pediatric nurse practitioner will refer the infant to a pediatric cardiologist and prepare the parents for which intervention to repair this defect? Cardiopulmonary bypass surgery Coil insertion in the catheterization laboratory Indomethacin administration Observation for spontaneous closure

ANS: B This murmur is characteristic of a PDA and, because of cardiac enlargement, represents a larger shunt, requiring repair. Infants older than 8 months of age may have a coil or plug inserted into the shunt in the cardiac catheterization laboratory. Cardiopulmonary bypass surgery is not indicated, even with ligation of the shunt. Indomethacin is administered to premature infants in the early post-natal period and is not useful in term or older infants. Because this infant is symptomatic, observation for spontaneous closure is not recommended.

A 12-month-old infant exhibits poor weight gain after previously normal growth patterns. There is no history of vomiting, diarrhea, or irregular bowel movements, and the physical exam is normal. What is the next step in evaluating these findings? Complete blood count and electrolytes Feeding and stooling history and 3-day diet history Stool cultures for ova and parasites Swallow study with videofluoroscopy

ANS: B Vomiting, diarrhea, and bowel irregularities are more predictive of organic causes of FTT, which are not present in this infant. A careful history and physical examination and limited laboratory evaluation are the first steps unless there is reason to think that an organic cause is present. The fact that the infant was previously gaining weight appropriately makes a swallowing disorder less likely.

A pre-school age child has honey-crusted lesions on erythematous, eroded skin around the nose and mouth, with satellite lesions on the arms and legs. The child's parent has several similar lesions and reports that other children in the day care have a similar rash. How will this be treated? Amoxicillin 40 to 5 mg/kg/day for 7 to 10 days Amoxicillin-clavulanate 90 mg/kg/day for 10 days Bacitracin cream applied to lesions for 10 to 14 days Mupirocin ointment applied to lesions until clear

ANS: B When children have multiple impetigo lesions or non-bullous impetigo with infection in multiple family members or child care groups, oral antibiotics are indicated. Amoxicillin-clavulanate is a first-line drug for this indication. Amoxicillin is not used for skin infections. Bacitracin is bacteriostatic and may be used when only a few lesions are present and if bacterial resistance is not an issue. Mupirocin is used for mild impetigo when the case is isolated.

The primary care pediatric nurse practitioner is teaching a parent of a child with dry skin about hydrating the skin with bathing. What will the nurse practitioner include in teaching? Apply lubricating agents at least 1 hour after the bath. Have the child soak in a lukewarm water bath. Keep the child in the bath until the skin begins to "prune." Soaping should be done at the beginning of the bath.

ANS: B When using bathing to hydrate dry skin, lukewarm water should be used. Lubricating agents should be applied immediately after patting the skin dry. The bath should last long enough to allow the skin to become moisturized without becoming supersaturated or "pruned." Soaping and shampooing should be performed at the end of the bath followed by thorough rinsing.

The primary care pediatric nurse practitioner is performing a well child exam on a 12-year-old female who has achieved early sexual maturation. The mother reports that she spends more time with her older sister's friends instead of her own classmates. What will the nurse practitioner tell this parent? Early-maturing girls need to identify with older adolescents to feel a sense of belonging. Girls who join an older group of peers may become sexually active at an earlier age. Spending time with older adolescents indicates a healthy adjustment to her maturing body. The association with older adolescents will help her daughter to gain social maturity.

ANS: B While it is true that early maturing females may join an older group of peers to feel that they fit in, the ones who do put themselves at risk for risky behaviors, including sexual activity. Although many teens feel awkward when they mature at different rates than their peers, joining a group of older peers demonstrates a poor adjustment and does not promote social maturity.

The primary care pediatric nurse practitioner is counseling a parent about bicycle helmet use. The parent reports having a helmet used a year previously by an older child and wonders about using it for a younger child since they are so expensive. What will the nurse practitioner tell the parent? "As long as the helmet does not have cracks, you may use it." "If the helmet is free from marks, you may use it." "You may continue to use a helmet up to 10 years." "You should always purchase a new helmet for each child."

ANS: B While parents should be taught not to purchase a secondhand helmet, using a fairly new, undamaged helmet from an older child is acceptable. Any helmet that has marks should be discarded, even if not cracked. Helmets should be replaced every 5 years or sooner, depending on the manufacturer's recommendations. It is not necessary to purchase a new helmet for each child, especially if money is an issue.

A school-age child has a fractured wrist with a Salter-Harris Type II fracture, according to the radiologist. What is true about this type of fracture? Growth disturbance of the long bones of the arm is likely. There is a metaphyseal fragment on the compression side of fracture. There is usually a compression or crushing injury to the physis. This will require anatomic reduction using an open approach.

ANS: B With a Salter-Harris Type II fracture, a metaphyseal is present on the compression side of the fracture. Types I and II Salter-Harris fractures rarely show growth disturbances. Type V fractures have a compression or crushing injury to the physis. Anatomic reduction with an open approach is usually necessary for a Type III fracture.

The parents of a 3-year-old child are concerned that the child has begun refusing usual foods and wants to eat mashed potatoes and chicken strips at every meal and snack. The child's rate of weight has slowed, but the child remains at the same percentile for weight on a growth chart. What will the primary care pediatric nurse practitioner tell the parents to do? Allow the child to choose foods for meals to improve caloric intake. Place a variety of nutritious foods on the child's plate at each meal. Prepare mashed potatoes and chicken strips for the child at mealtimes. Suggest cutting out snacks to improve the child's appetite at mealtimes.

ANS: B Young children should have three meals and two nutritious snacks each day. The parents' responsibility is to provide nutritious foods and allow children to choose how much they will eat. Children who are allowed to choose foods will likely make selections that are not healthy. Parents should be discouraged from preparing separate meals for their children. Snacks are necessary to maintain adequate intake and energy.

The primary care pediatric nurse practitioner auscultates a new grade II vibratory, mid-systolic murmur at the mid sternal border in a 4-year-old child that is louder when the child is supine. What type of murmur is most likely? Pathologic murmur Pulmonary flow murmur Still's murmur Venous hum

ANS: C A Still's murmur is characterized by a vibratory or musical low-grade sound, along the sternal border, which is louder when the child is supine or during inspiration. It is usually heard in children between the ages of 2 and 6 years old. Pathologic murmurs are usually harsh, not vibratory. A pulmonary flow murmur has a soft, blowing sound and radiates to the lung fields. A venous hum has a soft, high-pitched swishing sound.

6. The primary care pediatric nurse practitioner enters an exam room and finds a 2-month-old infant in a car seat on the exam table. The infant's mother is playing a game on her smart phone. The nurse practitioner interprets this behavior as : a. a sign that the mother has postpartum depression. b. extremely concerning for potential parental neglect. c. of moderate concern for parenting problems. d. within the normal range of behavior in early parenthood.

ANS: C A parent who seems disinterested in a child raises moderate concerns for parenting problems. It does not necessarily signal postpartum depression. It is not a mark for extreme concern. It is not within the expected range of behaviors.

The primary care pediatric nurse practitioner enters an exam room and finds a 2-month-old infant in a car seat on the exam table. The infant's mother is playing a game on her smart phone. The nurse practitioner interprets this behavior as : a sign that the mother has postpartum depression. extremely concerning for potential parental neglect. of moderate concern for parenting problems. within the normal range of behavior in early parenthood.

ANS: C A parent who seems disinterested in a child raises moderate concerns for parenting problems. It does not necessarily signal postpartum depression. It is not a mark for extreme concern. It is not within the expected range of behaviors.

The primary care pediatric nurse practitioner evaluates a school-age child whose body mass index (BMI) is greater than the 97th percentile. The nurse practitioner is concerned about possible metabolic syndrome and orders laboratory tests to evaluate this. Which diagnosis will the nurse practitioner document for this visit? Metabolic syndrome Nutritional alteration: more than required Obesity Rule out type 2 diabetes mellitus

ANS: C A problem should never be included on the problem list that is not supported by subjective and objective data found and recorded in the database. This child has a BMI that suggests obesity, so this may be used as a diagnosis. Metabolic syndrome is a diagnosis that is determined by laboratory data, which has not been evaluated yet. Nutritional alteration is a NANDA diagnosis and not acceptable for reimbursement. "Rule out" should not be used as a diagnosis, but may be considered part of a plan.

The parent of a 4-month-old infant is concerned that the infant cannot hear. Which test will the primary care pediatric nurse practitioner order to evaluate potential hearing loss in this infant? Acoustic reflectometry Audiometry Auditory brainstem response (ABR) Evoked otooacoustic emission (EOAE) testing

ANS: C ABR is not a direct measure of hearing but allows for inferences to be made about hearing thresholds and is useful for identifying hearing loss in a young infant. Although sedation is occasionally required, this test is useful in infants and young children unable to cooperate with EOAE or audiometry. Acoustic reflectometry is used to detect middle ear effusion. Audiometry requires a cooperative child. EOAE is used for universal screening in newborns. The American Academy of Pediatrics (AAP) Bright Futures guidelines (AAP, 2014) recommends pure-tone audiometry at 3, 4, 5, 6, 8, 10, 12, 15, and 18 years of age.

A 3-year-old child has had one episode of acute otitis media 3 weeks prior with a normal tympanogram just after treatment with amoxicillin. In the clinic today, the child has a type B tympanogram, a temperature of 102.5°F, and a bulging tympanic membrane. What will the primary care pediatric nurse practitioner order? A referral for tympanocentesis Amoxicillin twice daily Amoxicillin-clavulanate twice daily Intramuscular ceftriaxone

ANS: C Amoxicillin-clavulanate should be given for failed therapy with amoxicillin or when the child has had AOM treated with amoxicillin within the past month.

A child is bitten on one arm by a neighbor's dog. The dog is immunized against rabies and the child's last tetanus immunization was 4 years prior. The wound edges are gaping and avulsed. What is an important initial intervention when treating this injury? a. Administration of rabies prophylaxis and a tetanus booster b. Debriding and suturing the wound to prevent infection c. Irrigation of the wounds with high-pressure normal saline d. Reporting the animal bite to the local animal control authority

ANS: C Animal and human bites need to be irrigated with normal saline using >5 psi of pressure. The animal has been vaccinated for rabies and the child's tetanus is current, so prophylaxis for both of these is not indicated. There is controversy about whether primary closure is appropriate. Reporting the animal is not a primary action.

The parent of a 10-year-old boy tells the primary care pediatric nurse practitioner that the child doesn't appear to have any interest in girls and spends most of his time with a couple of other boys. The parent is worried about the child's sexual identity. The nurse practitioner will tell the parent children at this age who prefer interactions with same-gender peers usually have a homosexual orientation. children experiment with sexuality at this age as a means of deciding later sexual orientation. this attachment to other same-gender children is how the child learns to interact with others. to encourage mixed-gender interactions in order to promote development of sexual values.

ANS: C At age 10, children usually develop an intense same-gender relationship with a peer. This is how the child learns to expand the self, shares feelings, and learns how others manage problems. It does not indicate later sexual orientation and is not a characteristic of experimentation with sexuality. It is not necessary to encourage mixed-gender interactions.

An African-American child has recurrent tinea capitis and has just developed a new area of alopecia after successful treatment several months prior. When prescribing treatment with griseofulvin and selenium shampoo, what else will the primary care pediatric nurse practitioner do? Monitor CBC, LFT, and renal function during therapy. Order oral prednisone daily for 5 to 14 days. Perform fungal cultures on family members and pets. Prescribe oral itraconazole or terbinafine.

ANS: C Because asymptomatic carriers may be present in the household, family members and pets should be cultured. It is not necessary to monitor lab work with griseofulvin unless there is a change in clinical status, due to the favorable safety profile of griseofulvin. Prednisone is used when severe inflammation is present. Oral itraconazole or terbinafine is used if resistance to griseofulvin occurs; this child has responded to griseofulvin.

An infant with trisomy 21 has a complete AV canal defect. Which finding, associated with having both of these conditions, will the primary care pediatric nurse practitioner expect? Crackles in both lungs Hepatomegaly Oxygen desaturation Peripheral edema

ANS: C Because infants with trisomy 21 maintain neonatal high pulmonary vascular resistance, they often do not show signs of CHF but instead will have signs of pulmonary hypertension with loud single S2 and desaturation with agitation or effort. Crackles, hepatomegaly, and edema are signs of CHF.

The primary care pediatric nurse practitioner is offering anticipatory guidance to the parents of a 6-year-old child who has Down syndrome. What will the nurse practitioner tell the parents about physical activity and sports in school? Children with Down syndrome get frustrated easily when engaging in sports. Children with Down syndrome should not participate in strenuous aerobic activity. Their child should have a cervical spine evaluation before participation in sports. Their child should only participate in sports sanctioned by the Special Olympics.

ANS: C Because up to 40% of children with Down syndrome have a hypermobility or instability between C1-C2 and up to 61% have occipito-atlantal hypermobility, they should undergo radiological evaluation of the cervical spine to be cleared for strenuous sports. Many children and adolescents with intellectual and developmental disabilities (including those with Down, fragile X, Turner, or Klinefelter syndromes or autism) are capable of performing exercise or strenuous activities. Special needs children should be encouraged to participate in sports to increase physical abilities and increase self-confidence. Children with Down syndrome may benefit from strenuous aerobic activity and may participate in any sports once cervical spine stability is evaluated, not just those sanctioned by the Special Olympics.

When prescribing topical glucocorticoids to treat inflammatory skin conditions, the primary care pediatric nurse practitioner will initiate therapy with a high-potency glucocorticoid. order lotions when higher potency is necessary. prescribe brand-name preparations for consistent effects. use fluorinated steroids to minimize adverse effects.

ANS: C Brand-name preparations often have a more consistent base and potency. PNPs should be familiar with a few high-, medium-, and low-potency products and use those consistently. Therapy should be initiated with the lowest possible potency. Lotions have a lower potency than ointments and creams. Fluorinated steroids have the highest potency and a higher risk of side effects

During a well child examination of an infant, the primary care pediatric nurse practitioner notes 10 café au lait spots on the infant's trunk. What is the potential concern associated with this finding? Endocrine disorders Malignancy Neurofibromatosis Sturge-Weber syndrome

ANS: C Café au lait spots are significant for neurofibromatosis and should be referred if more than 5 lesions are present. Atypical nevi are concerning for malignancy. Port-wine stains are concerning for Sturge-Weber syndrome. Acanthosis is a sign of diabetes.

The parent of a toddler and a 4-week-old infant tells the primary care pediatric nurse practitioner that the toddler has just been diagnosed with pertussis. What will the nurse practitioner do to prevent disease transmission to the infant? Administer the initial diphtheria, pertussis, and tetanus vaccine. Instruct the parent to limit contact between the toddler and the infant. Order azithromycin 10 mg/kg/day in a single dose daily for 5 days. Prescribe erythromycin 10 mg/kg/dose four times daily for 14 days.

ANS: C Chemoprophylaxis for pertussis exposure is recommended for all household and close contacts of infected persons regardless of immunization status. Azithromycin is the drug of choice for infants from 1 month to 6 months of age. Administering the vaccine is not indicated since there isn't sufficient time to develop immunity. Infants under 1 month of age should not receive erythromycin because of the increased risk for pyloric stenosis associated with this drug

The primary care pediatric nurse practitioner is performing a well child examination on a 12-year-old child who was diagnosed with type 1 diabetes at age 9. The child had a lipid screen at age 10 with an LDL cholesterol <100 mg/dL. What will the nurse practitioner recommend as part of ongoing management for this child? Annual lipid profile evaluation Annual screening for microalbuminuria Comprehensive ophthalmologic exam Hypothyroidism screening every 5 years

ANS: C Children who have been diagnosed for 3 to 5 years and who are over age 10 should be referred for annual comprehensive ophthalmologic exams. This child had a previous normal lipid profile and needs to be evaluated every 5 years and not annually. Annual screening for microalbuminuria begins at 5 years after onset. Hypothyroidism screening should be done annually.

A child who was treated with amoxicillin and then amoxicillin-clavulanate for acute otitis media is seen for follow-up. The primary care pediatric nurse practitioner notes dull-gray tympanic membranes with a visible air-fluid level. The child is afebrile and without pain. What is the next course of action? Administering ceftriaxone IM Giving clindamycin orally Monitoring ear fluid levels for 3 months Watchful waiting for 48 to 72 hours

ANS: C Children with AOM may have effusion up to 3 months after the acute infection. The child should be monitored to ensure that this resolves. Antibiotics are not indicated. There is no acute infection, so watchful waiting for worsening of symptoms is not indicated.

The parent of a 12-year-old child who has sickle cell trait (SCT) asks the primary care pediatric nurse practitioner whether the child may play football. What will the nurse practitioner tell this parent? Children with SCT should not play any contact sports. Children with SCT may not play for NCAA schools in college. Children with SCT should follow heat acclimatization guidelines. Children with SCT should not participate in organized sports.

ANS: C Children with SCT may play in sports as long as preventative measures, including heat acclimatization, are taken to prevent sickling crises. They may play contact sports and may play for NCAA teams as long as their sickle cell trait status is known.

The parent of a preschool-age child who is diagnosed with a sensory processing disorder (SPD) asks the primary care pediatric nurse practitioner how to help the child manage the symptoms. What will the nurse practitioner recommend? Establishing a reward system for acceptable behaviors Introducing the child to a variety of new experiences Maintaining predictable routines as much as possible Providing frequent contact, such as hugs and cuddling

ANS: C Children with SPD do best with an environment that is predictable and routine and the same from day to day. Discipline and/or a reward system is not effective. Children with SPD can become overwhelmed by new experiences or frequent touch.

The parent of a child newly diagnosed with epilepsy asks the primary care pediatric nurse practitioner if the child will ever be able to participate in gym or sports. What will the nurse practitioner recommend? Bicycle riding is not safe for children with seizures. Contact sports should be avoided. Direct supervision of some activities is necessary. Underwater sports are not recommended.

ANS: C Children with epilepsy may participate in most sports but may require direct supervision in some cases to reduce the risk of injury to self or others if a seizure should occur during sports. Bicycle riding, contact sports, and underwater sports may be engaged in, but certain precautions must be taken (e.g., supervision).

A 7-year-old female has recently developed pubic and axillary hair without breast development. Her bone age is consistent with her chronological age, and a pediatric endocrinologist has diagnosed idiopathic premature adrenarche. The primary care pediatric nurse practitioner will monitor this child for which condition? Adrenal tumor Congenital adrenal hyperplasia Polycystic ovary syndrome Type 1 diabetes mellitus

ANS: C Children with idiopathic premature adrenarche are at increased risk for PCOS and metabolic syndrome. Adrenal tumor and CAH are both possible causes of premature adrenarche, but these have been ruled out to make a diagnosis of idiopathic premature adrenarche. Type 2 diabetes is related to metabolic syndrome, not type 1.

The primary care pediatric nurse practitioner is performing a sports physical on an adolescent whose history reveals mild aortic stenosis. What will the nurse practitioner recommend? Avoidance of all sports to prevent sudden death Clearance for any sports since this is mild Evaluation by a cardiologist prior to participation Low-intensity sports, such as golf or bowling

ANS: C Children with mild AS may participate in any sport but must have annual cardiac evaluations. Children with severe AS should avoid sports to prevent sudden death. The PNP should not clear the child for sports without a cardiology evaluation. Low-intensity sports are recommended for children with moderate AS.

The primary care pediatric nurse practitioner notes a musty odor when examining a newborn at a 2-week checkup. What will the nurse practitioner suspect? Galactosemia Glucose-6-phosphatase deficiency Phenylketonuria Urea cycle disorder

ANS: C Children with phenylketonuria have a musty or mousy odor. Galactosemia causes poor weight gain, lethargy, and jaundice after milk feeding has begun. Glucose-6-phosphatase deficiency causes cardiomegaly and seizure. Urea cycle disorders are characterized by vomiting and lethargy.

A child is brought to the clinic with a generalized, annular rash characterized by raised wheals with pale centers. On physical examination, the child's lungs are clear and there is no peripheral edema. A history reveals ingestion of strawberries earlier in the day. What is the initial treatment? Aqueous epinephrine 1:1000 subcutaneously Cetirizine once in clinic and then once daily for 2 weeks Diphenhydramine 0.5 to 1 mg/kg/dose every 4 to 6 hours Prednisone 1 to 2 mg/kg/day for 1 week with rapid taper

ANS: C Diphenhydramine is given initially as long as anaphylaxis and angioedema are not present. Aqueous epinephrine is used for anaphylaxis and angioedema. Cetirizine is less effective than diphenhydramine. Prednisone is used for refractory episodes.

The parent of a 14-year-old child tells the primary care pediatric nurse practitioner that the adolescent has expressed a desire to be a vegetarian, is refusing all meat served at home, and wants the family to eat vegetarian meals. What will the nurse practitioner tell the parent? Do not allow a vegetarian diet in order to maintain appropriate limits for the adolescent. Provide vegetarian options for the adolescent that preserve adequate nutrition and protein intake. Suggest that the adolescent prepare appropriate vegetarian dishes to complement family meals. Tell the adolescent that a vegetarian diet may be considered in adulthood but not while living at home.

ANS: C Early adolescents begin to develop their own value system and may try value systems other than the one that they have learned from their family, which is a normal part of establishing personal identity. The parent may allow expression of other values, such as a vegetarian diet, as long as nutritional needs are met and the adolescent takes responsibility for preparing the food.

The primary care pediatric nurse practitioner sees a 3-year-old child who chronically withholds stools, in spite of the parents' attempts to stop the behavior, requiring frequent treatments with laxative medications. Which diagnosis will the nurse practitioner use to facilitate third-party reimbursement? Altered elimination pattern Elimination disorder Encopresis Parenting alteration

ANS: C Encopresis is a medical diagnosis, classified in the ICD-10-CM, and is recognized for reimbursement purposes. "Altered elimination pattern" and "Parenting alteration" are NANDA International diagnoses and are not recognized for reimbursement. "Elimination disorder" is a developmental diagnosis.

The parent of a 5-year-old child who has just begun kindergarten expresses concern that the child will have difficulty adjusting to the birth of a sibling. What will the primary care pediatric nurse practitioner recommend? Allowing the child opportunities to discuss feelings about the baby Giving the child specific baby care tasks to promote sibling bonding Having snack time with the child each day to discuss the school day Providing reassurance that the sibling will not replace the child

ANS: C Family routines provide support to children and help them self-regulate, especially during times of change, and serve as a buffer during times of change and transition. This child has two major changes, so setting aside regular time to spend with the child will help stabilize these changes. The other options may be useful as well, but routines and special activities are most important.

The parent of a 14-year-old child tells the primary care pediatric nurse practitioner that the child skips classes frequently in spite of various disciplinary measures, such as grounding and extra homework and is earning Cs and Ds in most classes. What will the nurse practitioner recommend? Counseling for emotional problems Development of an Individual Education Plan Evaluation for possible learning disorders Referral for a behavioral disorder

ANS: C Frequent school absenteeism, class skipping, and other types of school avoidance may indicate a problem with cognitive ability and should be assessed. When cognitive disorders are ruled out, other issues, such as behavioral and emotional problems may be considered. IEPs are used for children who have identified special physiological or cognitive needs and may be useful if a cognitive disorder is identified.

The primary care pediatric nurse practitioner diagnoses a high school basketball player with mononucleosis. The adolescent asks when she may resume play. What will the nurse practitioner tell her? After 3 weeks, she may begin lifting weights but not full sports. After 4 weeks, she may return to full play and practice. At 4 weeks, she must have an exam to determine fitness for play. She may engage in moderate exertion and practice after 3 weeks.

ANS: C Full return to play should be determined on a case-by-case basis and is generally considered safe at 4 weeks after symptom onset, assuming physical stamina has returned, all symptoms have resolved, and the sport does not increase intraabdominal pressure during play. Athletes should avoid any form of exertion, including all sports during the first 3 weeks at a minimum and should avoid anything with a risk of chest or abdominal contact or anything that involves increased intra-abdominal pressure. Splenic rupture can occur spontaneously (rare), but the risk of rupture increases when participating in a contact or collision sport or a sport in which there is an increase in intraabdominal pressure. The nurse practitioner should recommend an exam at 4 weeks to determine fitness for play.

2. The parent of a newborn infant asks the primary care pediatric nurse practitioner when to intervene to help the infant's future intellectual growth. What will the nurse practitioner tell the parent? a. Cognitive learning begins during the toddler years. b. Intellectual growth begin when speech develops. c. Language and literacy skills begin at birth. d. Preschool is an optimal time to begin general learning.

ANS: C General learning and acquisition of skills for later reading and writing begin at birth, not in kindergarten or first grade, and these skills grow with everyday loving interactions between infants and caregivers. Cognitive learning changes during toddler years but begins at birth. Intellectual growth is not tied to speech alone.

The parent of a newborn infant asks the primary care pediatric nurse practitioner when to intervene to help the infant's future intellectual growth. What will the nurse practitioner tell the parent? Cognitive learning begins during the toddler years. Intellectual growth begin when speech develops. Language and literacy skills begin at birth. Preschool is an optimal time to begin general learning.

ANS: C General learning and acquisition of skills for later reading and writing begin at birth, not in kindergarten or first grade, and these skills grow with everyday loving interactions between infants and caregivers. Cognitive learning changes during toddler years but begins at birth. Intellectual growth is not tied to speech alone.

A school-age child is brought to clinic after a pediculosis capitis infestation is reported at the child's school. If this child is positive, what will the primary care pediatric nurse practitioner expect to find on physical examination, along with live lice near the scalp? Excoriated macules along the child's collar and underwear lines Inflammation and pustules on the face and neck Itching of the scalp, with skin excoriation on the back of the head Linear or S-shaped lesions in webs of fingers and sides of hands

ANS: C Head lice commonly cause itching of the scalp, especially on the back of the head and neck, along with skin excoriation, and may be the only initial sign. Excoriated lesions along underwear lines are typical of body lice. Inflammation and pustules occur with acne. Linear or S-shaped lesions occur with scabies infestations.

An adolescent has acne with lesions on the cheeks and under the chin. Which distribution is this? Athletic Frictional Hormonal Pomadal

ANS: C Hormonal acne has a beard distribution. Athletic acne occurs on the forehead, chin, and shoulders, caused by helmets and pads. Frictional occurs where bras, tight clothes, and headbands rub. A pomadal distribution occurs along the temple and forehead, as a result of pomades or oil-based cosmetics.

A 12-year-old child has a recent history of increased thirst and frequent urination. The child's weight has been in the 95th percentile for several years. A dipstick UA is positive for glucose, and random plasma glucose is 350 mg/dL. Which test will the primary care pediatric nurse practitioner order to determine the type of diabetes in this child? Fasting plasma glucose Hemoglobin A1C levels Pancreatic antibodies Thyroid function tests

ANS: C If the type of diabetes is uncertain, screening for pancreatic antibodies should be considered to confirm the diagnosis of type 1 diabetes. Fasting plasma glucose may be elevated in both types. Hemoglobin A1C does not distinguish among types. Thyroid function tests are not indicated unless there is a suspicion of concomitant associated autoimmune conditions.

7. The primary care pediatric nurse practitioner is performing a well baby examination on a 2-month-old infant who has gained 25 grams per day in the last interval. The mother is nursing and tells the nurse practitioner that her infant seems fussy and wants to nurse more often. What will the nurse practitioner tell her? a. She may not be making as much breastmilk as before. b. She should keep a log of the frequency and duration of each feeding. c. The infant may be going through an expected growth spurt. d. The infant should stay on the previously established nursing schedule.

ANS: C Infants may have a growth spurt at 6 to 8 weeks, and mothers who are breastfeeding may be concerned that they are not making enough milk when they notice that the infant is fussy and wanting to nurse more often. The PNP should reassure the mother that this is expected. It is not necessary, since the infant is gaining weight appropriately, for the mother to keep a log. The mother should follow the infant's cues for feeding since the extra suckling will increase the milk supply to meet the growing infant's needs.

The primary care pediatric nurse practitioner is performing a well baby examination on a 2-month-old infant who has gained 25 grams per day in the last interval. The mother is nursing and tells the nurse practitioner that her infant seems fussy and wants to nurse more often. What will the nurse practitioner tell her? She may not be making as much breastmilk as before. She should keep a log of the frequency and duration of each feeding. The infant may be going through an expected growth spurt. The infant should stay on the previously established nursing schedule.

ANS: C Infants may have a growth spurt at 6 to 8 weeks, and mothers who are breastfeeding may be concerned that they are not making enough milk when they notice that the infant is fussy and wanting to nurse more often. The PNP should reassure the mother that this is expected. It is not necessary, since the infant is gaining weight appropriately, for the mother to keep a log. The mother should follow the infant's cues for feeding since the extra suckling will increase the milk supply to meet the growing infant's needs.

The primary care pediatric nurse practitioner performs a developmental assessment on a 32-month-old child. The child's parent reports that about 70% of the child's speech is intelligible. The pediatric nurse practitioner observes that the child has difficulty pronouncing "t," "d," "k," and "g" sounds. Which action is correct? Evaluate the child's cognitive abilities. Obtain a hearing evaluation. Reassure the parent that this is normal. Refer the child to a speech therapist.

ANS: C Intelligibility of speech reaches about 66% between the ages of 24 and 36 months. Tongue-contact sounds are more intelligible by age 5 years. This child exhibits normal speech for age. It is not necessary to perform a cognitive assessment based on these findings. Referrals for hearing and speech evaluations are not indicated, since these findings are within normal limits.

An 18-month-old child has a 1-day history of intermittent, cramping abdominal pain with non-bilious vomiting. The child is observed to scream and draw up his legs during pain episodes and becomes lethargic in between. The primary care pediatric nurse practitioner notes a small amount of bloody, mucous stool in the diaper. What is the most likely diagnosis? Appendicitis Gastroenteritis Intussusception Testicular torsion

ANS: C Intussusception is characterized by intermittent pain associated with drawing up the legs, "currant jelly" stools, and lethargy in between episodes. Appendicitis is characterized by pain localizing to the RLQ and is not intermittent. Gastroenteritis is likely when vomiting precedes symptoms of pain or discomfort. Testicular torsion involves the testicles and thus has different physical findings and would not be accompanied with bloody stools.

A 12-month-old infant who had cardiopulmonary bypass with RBC and plasma infusions during surgery at 8 months is seen for a well child examination. Which vaccine may be administered at this visit? MMR OPV PCV-13 Varivax

ANS: C Live vaccines should be delayed until 6 months after cardiopulmonary bypass and exposure to RBCs and plasma. The PCV-13 is not a live-virus vaccine and the others are.

1. During a well child exam, the primary care pediatric nurse practitioner learns that the parents of a young child fight frequently about finances. The parents state that they do not fight in front of the child and feel that the situation is temporary and related to the father's job layoff. What will the nurse practitioner do? a. Reassure them that the child is too young to understand. b. Recommend that they continue to not argue in front of the child. c. Suggest counseling to learn ways to handle stress. d. Tell them that the conflict will resolve when the situation changes.

ANS: C Marital problems can result in child behavior difficulties and anxieties, and conflict can be picked up by the child. The parents should try to learn to modify unhealthy behaviors, such as increased conflict during stressful situations. Even when children do not understand, they pick up on cues from the parents about anxiety and stress and can internalize these feelings. Avoiding arguments in front of the child does not alleviate the underlying conflict and stress. The behavior of fighting during this stressful situation may indicate a pattern of response to stress and will only recur with each subsequent stressful period.

During a well child exam, the primary care pediatric nurse practitioner learns that the parents of a young child fight frequently about finances. The parents state that they do not fight in front of the child and feel that the situation is temporary and related to the father's job layoff. What will the nurse practitioner do? Reassure them that the child is too young to understand. Recommend that they continue to not argue in front of the child. Suggest counseling to learn ways to handle stress. Tell them that the conflict will resolve when the situation changes.

ANS: C Marital problems can result in child behavior difficulties and anxieties, and conflict can be picked up by the child. The parents should try to learn to modify unhealthy behaviors, such as increased conflict during stressful situations. Even when children do not understand, they pick up on cues from the parents about anxiety and stress and can internalize these feelings. Avoiding arguments in front of the child does not alleviate the underlying conflict and stress. The behavior of fighting during this stressful situation may indicate a pattern of response to stress and will only recur with each subsequent stressful period.

A child has small, firm, flesh-colored papules in both axillae which are mildly pruritic. What is an acceptable initial approach to managing this condition? Application of trichloroacetic acid 25% to 50% using a dropper Applying liquid nitrogen for 2 to 3 seconds to each lesion Reassuring the parents that these are benign and may disappear spontaneously Referral to a dermatologist for manual removal of lesions with curettage

ANS: C Molluscum contagiosum is a benign viral skin infection; most lesions disappear within 6 months to 2 years. An initial "wait and see" approach is acceptable. If itching is severe, the risk is autoinoculation and spread of lesions, along with increased discomfort and then other treatment measures may be attempted, depending on the severity. Topical medications, such as trichloroacetic acid or liquid nitrogen may be used if the lesions become uncomfortable or persist and should be used with caution. More severe outbreaks may require removal with curettage.

A school-age child falls off a swing and suffers a closed fracture of the right clavicle. How will this be managed? Application of a figure-eight clavicle brace for 6 to 8 weeks Hospitalization for traction of the affected extremity and shoulder Immobilization with a sling to support the affected extremity Referral to an orthopedic specialist for possible surgical reduction

ANS: C Most children with fractured clavicle can be treated with sling immobilization for 3 to 4 weeks. Figure-eight clavicle braces are uncomfortable and have questionable effectiveness; they are used if displacement leads to decreased shaft length. Hospitalization for traction is not indicated. Surgical reduction is uncommon and used for open fractures, neurovascular compromise, multiple trauma, rib cage fractures, and severe displacement.

A 4-year-old child with an upper respiratory tract infection has cloudy nasal discharge and moderate nasal congestion interfering with sleep. The parent asks what product to use to help with symptoms. What will the primary care pediatric nurse practitioner recommend? Antihistamines Decongestant sprays Saline rinses Zinc supplements

ANS: C Normal saline nose drops, nasal rinses, or sprays are helpful for all ages of children to clear nasal passages. The use of decongestants, antihistamines, and cough medicine does not shorten the course of a disease. While their use may help with relieving nasal symptoms, their use is not recommended for children younger than 6 years old. Zinc is not recommended in children because of potential side effects and questionable efficacy.

The mother of a 3-year-old child takes the child to a play group once a week. She expresses concern that the child plays with toys but does not interact with the other toddlers. What will the primary care pediatric nurse practitioner counsel the mother? The child probably is very shy but will outgrow this tendency with repeated exposure to other children. The toddler may have a language delay that interferes with socialization with other children. Toddlers may be interested in other children but usually do not engage in interactive play. Toddlers need more structured play to encourage interaction and socialization with others.

ANS: C Parallel play is common among toddlers who, although they may be fascinated by other children, generally do not engage with peers in an interactive manner. This does not mean that the child is shy or has a language delay, although in preschool years, the development of symbolic language increases interactive play. Children need both structured and free play, but structured play will not increase interaction during this normally parallel period.

During a well child exam on a 5-year-old child, the primary care pediatric nurse practitioner assesses the child for school readiness. Which finding may be a factor in limiting school readiness for this child? Adherence to daily family routines and regular activities Having two older siblings who attend the same school Parental concerns about bullying in the school The child's ability to recognize four different colors

ANS: C Parental expectation is the strongest predictor of school success in children. Parents who are worried about what may happen in school can transmit this anxiety to the child. Children who have a secure family life with daily routines will do better in school. Having older siblings who attend school increases success. Children at this age are expected to know four colors, so this is an indication of school readiness.

An adolescent female has a history of repaired tetralogy of Fallot. Which long-term complication is a concern for this patient? Aortic stenosis Chronic cyanosis Mitral valve prolapse Ventricular failure

ANS: C Patients with repaired TOF, especially adolescent females, are at risk for mitral valve prolapse. Aortic aneurysm is a long-term risk for those with a history of left-sided lesions. Chronic cyanosis is a concern for lesions causing Eisenmenger syndrome or defects causing right ventricular outflow obstruction. Ventricular failure can occur with prolonged aortic or pulmonic valvar stenosis.

A 16-year-old sexually active female has a fever, bilateral lower abdominal pain, and malaise. A speculum and bimanual exam reveals adnexal tenderness. The urinalysis is normal and cervical cultures are pending. What medications will the primary care pediatric nurse practitioner prescribe for this patient? Azithromycin, doxycycline, and penicillin Cefotaxime, azithromycin, and penicillin Ceftriaxone, doxycycline, and metronidazole Doxycycline, penicillin, and metronidazole

ANS: C Patients with suspected PID may be given ceftriaxone 250 mg IM once, doxycycline 100 mg PO bid for 14 days, and metronidazole 500 mg PO bid for 14 days. The other options are not recommended by the CDC.

During a routine well child exam on a 5-year-old child, the primary care pediatric nurse practitioner auscultates a grade II/VI, harsh, late systolic ejection murmur at the upper left sternal border that transmits to both lung fields. The child has normal growth and development. What will the nurse practitioner suspect? Aortic stenosis Patent ductus arteriosus Pulmonic stenosis Tricuspid atresia

ANS: C Pulmonic stenosis may be asymptomatic with a murmur as described above. Aortic stenosis is characterized by a louder, harsh systolic crescendo-decrescendo murmur at the upper right sternal border with radiation to the neck, LLSB, and apex. PDA has a machinery-like murmur. Tricuspid atresia is characterized by cyanosis.

During a well child exam on a 13-year-old female, the primary care pediatric nurse practitioner notes that the child is at Tanner Stage 3. During the exam, when the nurse practitioner initiates a conversation about healthy sexuality education, the parent states that this topic is "off limits." What will the nurse practitioner do? Ask the adolescent whether she wishes to discuss these matters since she is becoming an adult. Separate the parent from the adolescent to discuss the adolescent's concerns in private. Spend private time with the parent to discuss how sexuality education reduces the risk of early sexual intercourse and risky sexual behaviors. Tell the parent that this information is a routine part of adolescent well child examinations and must be included.

ANS: C Research has shown that sexuality education leads to a reduction in early onset of sexual intercourse and risky sexual behaviors. It is important for the PNP to be sensitive to the values of the family but also to advocate for the child. The child should be told, especially when she shows an interest in sexual relationships, that she may seek care independently of her parent and that it will remain confidential. When possible, the parent's wishes should be taken into account and both the adolescent and the parent should be encouraged to begin an open dialogue about these matters.

The parent of an 8-year-old child tells the primary care pediatric nurse practitioner that the child has begun to ask questions about why a schoolmate has "2 daddies" and wonders how to talk to the child about this. What will the nurse practitioner recommend? Beginning a discussion about different types of sexual relationships and same-sex partners Discussing the issue with the child in terms of the parent's religious values and norms Explaining that not all families are the same and what is most important is that they love and care for their children Telling the child that some adult relationships are complicated and will be understood when the child is older

ANS: C School age is a good time for parents to reinforce the notion that there is diversity in families within which parents and adults love and care for their children. It is not necessary to be explicit but to establish a good history of communication and to explain complex issues to children at a level of the child's understanding. In this way, the child will know that parents are accessible and open to discussion of complex and/or puzzling issues.

A 12-year-old child whose weight and BMI are in the 75th percentile has a diastolic blood pressure that is between the 95th and 99th percentiles for age, sex, and height on three separate occasions. Initial tests for this child will include complete blood count. erythrocyte sedimentation rate. renal function and plasma renin tests. urinalysis and electrolytes.

ANS: C Since the majority of children with stage 1 or 2 hypertension have renal or renovascular causes for elevated BP, renal function and plasma renin tests should be performed. Children under 10 years of age with stage 2 hypertension should have more aggressive laboratory evaluation, including CBC, ESR, UA, and electrolytes.

A 6-year-old child has a systolic blood pressure between the 95th and 99th percentile for age, sex, and height and a diastolic blood pressure between the 90th and the 95th percentile on three separate clinic visits. This child's blood pressure is classified as normotensive. pre-hypertensive. stage 1 hypertensive. stage 2 hypertensive.

ANS: C Stage 1 hypertensive pressure ranges from the 95th percentile or from 120/80 mm Hg to 5 mm Hg above the 99th percentile for age, sex, and height for either systolic or diastolic pressure. Normotensive pressure is below the 90th percentile. Pre-hypertensive pressure is between the 90th and 95th percentiles. Stage 2 hypertensive pressure is greater than the 99th percentile.

The primary care pediatric nurse practitioner elicits positive Ortolani and Barlow signs in a 6-month-old infant not previously noted in the medical record. What is the correct treatment? Pavlik harness Spica cast Surgical intervention Triple diapering

ANS: C The 6- to 18-month-old infant with a dislocated hip is likely to require either closed manipulation or open reduction. The other interventions should be used in younger infants.

The parent of a 6-year-old child expresses concern that the child may have ADHD. Which screening tool will the primary care pediatric nurse practitioner use to evaluate this possibility? Behavioral and Emotional Screening System for Children (BESS-2) Behavioral Assessment for Children - 2nd ed. (BASC-2) Conner's 3 Parent and Teacher Rating Scale Pediatric Symptom Checklist (PSC)

ANS: C The Conner's Parent and Teacher Rating Scale is used to assess ADHD symptoms in children aged 6 to 18 years. The BESS-2 is used to evaluate social emotional and mental health in children. The BASC-2 is used to further assess children who have positive findings on the BESS-2. The PSC is used to assess cognitive, emotional, and behavioral problems in children.

The parent of a toddler is concerned that the child may have autism. The primary care pediatric nurse practitioner completes a Modified Checklist for Autism in Toddlers (M-CHAT) tool, which indicates several areas of concern. What will the nurse practitioner do? Administer a Childhood Autism Rating Scale (CARS) in the clinic. Consult a specialist to determine appropriate early intervention strategies. Refer the child to a behavioral specialist for further evaluation. Tell the parent that this result indicates that the child has autism.

ANS: C The M-CHAT is a screening tool and is useful for detecting behaviors that may indicate autism. This instrument has been found to have acceptable sensitivity, specificity, and significant positive predictive value. If these behaviors are detected, the PNP should refer the child to a specialist for further assessment, using more diagnostic tools. The CARS may be used but requires specialty training and proper credentials. Until the diagnosis is determined, strategies for intervention are not discussed. The M-CHAT is a screening tool and is not diagnostic.

A 16-year-old female reports dull, achy cramping pain in her lower abdomen lasting 2 or 3 hours that occurs between her menstrual periods each month. The adolescent is not sexually active. What is the treatment for this condition? Abdominal ultrasound to rule out ovarian cyst Oral contraceptives to suppress ovulation Prostaglandin inhibitor analgesics and a heating pad Referral to a pediatric gynecologist

ANS: C The adolescent is experiencing mittelschmerz pain, which is thought to occur when the follicle ruptures at the time of ovulation. Unless the pain is severe, the adolescent should be reassured and offered strategies to relieve discomfort, such as a heating pad and NSAIDs. The pain is intermittent and occurs between periods; if it were persistent and severe, abdominal US would be indicated. Oral contraceptives are rarely used to suppress ovulation when symptoms are severe. Referral to a pediatric gynecologist is not indicated.

7. During a well child assessment of an 18-month-old child, the primary care pediatric nurse practitioner observes the child becoming irritable and uncooperative. The parent tells the child to stop fussing. What will the nurse practitioner do? a. Allow the parent to put the child in a "timeout." b. Ask the parent about usual discipline practices. c. Offer the child a book or a toy to look at. d. Stop the exam since the child has reached a "meltdown."

ANS: C The child has exhibited early signs of misbehavior. At this stage, distraction and active engagement may be used to stop more problems from occurring. It is not necessary to use a timeout because the child hasn't reached the point where cooperation is impossible. The PNP should model appropriate interventions by offering the child a distraction and may ask the parent about discipline practices later in the visit. The child is not at a "meltdown" state.

During a well child assessment of an 18-month-old child, the primary care pediatric nurse practitioner observes the child becoming irritable and uncooperative. The parent tells the child to stop fussing. What will the nurse practitioner do? Allow the parent to put the child in a "timeout."

ANS: C The child has exhibited early signs of misbehavior. At this stage, distraction and active engagement may be used to stop more problems from occurring. It is not necessary to use a timeout because the child hasn't reached the point where cooperation is impossible. The PNP should model appropriate interventions by offering the child a distraction and may ask the parent about discipline practices later in the visit. The child is not at a "meltdown" state.

A young adolescent reports chest pain associated with coughing and lifting. Physical examination reveals tenderness over several ribs, radiating to the back. Auscultation of the heart, lungs, and abdomen are normal. There is no history of injury. What will the primary care pediatric nurse practitioner do? Obtain a chest radiograph to evaluate possible causes for these symptoms. Order an electrocardiogram to rule out potential cardiovascular disease. Recommend NSAIDs, stretching exercises, and ice packs to the area. Refer the child to a pediatric orthopedist for evaluation and treatment.

ANS: C The child has symptoms and a history consistent with costochondritis. Treatment is symptomatic with analgesics, ice packs, and exercise and rest. Chest radiographs offer no diagnostic value, except to rule out other causes. Unless heart sounds are abnormal, or there are other signs of cardiovascular disease, an ECG is not indicated. Referral is not necessary unless symptoms persist.

The mother of a 12-month-old uncircumcised male infant reports that the child seems to have pain associated with voiding. A physical examination reveals a tight, pinpoint opening of the foreskin, which thickened and inflamed. What will the primary care pediatric nurse practitioner do? Attempt to retract the foreskin to visualize the penis. Order corticosteroid cream 3 times daily for 4 weeks. Refer the child to a pediatric urologist. Teach the mother to gently stretch the foreskin with cleaning.

ANS: C The child has symptoms consistent with pathologic phimosis and should be referred for possible circumcision. The foreskin should never be forcefully retracted. Non-pathologic phimosis can usually be managed by normal cleansing and gentle stretching. Corticosteroid cream is used for persistent, non-pathologic phimosis.

The primary care pediatric nurse practitioner is examining a 6-year-old child who attends first grade. The child reports "hating" school. The parent states that the child pretends to be sick frequently in order to stay home from school. To further assess this situation, the nurse practitioner will first ask the child : about school performance and grades. why school is so distressing. to name one or two friends. whether bullying is taking place.

ANS: C The earliest school-age psychosocial milestone occurs when children learn to separate easily from family, allowing them to go to school. Mastery of these skills enables them to develop and maintain peer friendships. Social interaction skills are necessary in order to develop mastery over school activities. Asking the child to describe why school is distressing may not elicit information, since the child may not be able to articulate this. Bullying is not the only reason for disliking school, but, if it is, will emerge during a discussion about friends and schoolmates.

When meeting with a new family, the primary care pediatric nurse practitioner develops a database that identifies family members and others living in the household, relationships with others outside the household, and significant behavioral and emotional problems. Which tool will the nurse practitioner use to record this information? CRAFFT Ecomap Genogram Pedigree

ANS: C The genogram is an approach to developing a family database to provide a graphic representation of family structure, roles, and problems of recurring significance in a family. The CRAFFT tool is used to assess substance abuse in adolescents. The ecomap is used to identify relationships in the family and community that are supportive or harmful. The pedigree is used to identify potential genetic disorders.

The primary care pediatric nurse practitioner is providing nutritional counseling for a 9-year-old female whose weight is at the 95th percentile for her age. What is the goal for this patient? A loss of 10 to 15 pounds in 6 months An average weight loss of 2 pounds per month Maintenance of her current weight Weight loss of 5% of her current body weight

ANS: C The goal for weight management in obese children is weight maintenance and not weight loss, allowing the child to grow into her weight to achieve a BMI less than the 85th percentile.

A 5-year-old child has enlarged tonsils and a history of four throat infections in the previous year with fever, cervical lymphadenopathy, and positive Group A Streptococcus pyogenes (GABHS) cultures. The parent reports that the child snores at night and expresses concerns about the child's quality of sleep. The next step in managing this child's condition is to : continue to observe the child for two or more GABHS infections. prescribe prophylactic antibiotics to prevent recurrent infection. refer to a pulmonologist for polysomnography evaluation. refer to an otolaryngologist for possible tonsillectomy.

ANS: C The potential for sleep apnea should be evaluated since the parent reports snoring and concerns with sleep in a child with frequent throat infections. This child has not had a high enough number of GABHS throat infections to warrant tonsillectomy and should be watched. Prophylactic antibiotics are not indicated.

The primary care pediatric nurse practitioner obtains a tympanogram on a child that reveals a sharp peak of -180 mm H2O. What does this value indicate? A normal tympanic membrane Middle ear effusion Negative ear pressure Tympanic membrane perforation

ANS: C The type C tympanogram has a sharp peak between -100 and -200 mm H2O and reflects negative ear pressure. A normal tympanogram has a sharp positive peak or a type A tympanogram. Middle ear effusion and a TM perforation both cause a type B tympanogram with either no peak or a flattened wave.

An adolescent is diagnosed with functional abdominal pain (FAP). The child's symptoms worsen during stressful events, especially with school anxiety. What will be an important part of treatment for this child? Informing the parents that the pain is most likely not real Instituting a lactose-free diet along with lactobacillus supplements Teaching about the brain-gut interaction causing symptoms Using histamine2-blockers to help alleviate symptoms

ANS: C This child has symptoms associated with stress, and treatment should be aimed at biobehavioral methods, beginning with teaching about the brain-gut interaction. Even though the pain is functional, it is real. Lactose-free diets and lactobacillus supplements may be used with documented lactose intolerance, although there is a lack of high-quality evidence of their effectiveness. H2-blockers should not be used unless dyspepsia is present.

15. A 14-year-old child has a 2-week history of severe itching and tearing of both eyes. The primary care pediatric nurse practitioner notes redness and swelling of the eyelids along with stringy, mucoid discharge. What will the nurse practitioner prescribe? a. Saline solution or artificial tears b. Topical mast cell stabilizer c. Topical NSAID drops d. Topical vasoconstrictor drops

ANS: C This child has symptoms of allergic conjunctivitis. Topical NSAIDs work for acute symptoms to reduce inflammation and may be used in children over age 12 years. Saline solution or artificial tears are useful for milder symptoms. Topical mast cell stabilizers are useful for chronic symptoms and maintenance therapy. Topical vasoconstrictors should be avoided because of rebound hyperemia.

A 14-year-old child has a 2-week history of severe itching and tearing of both eyes. The primary care pediatric nurse practitioner notes redness and swelling of the eyelids along with stringy, mucoid discharge. What will the nurse practitioner prescribe? Saline solution or artificial tears Topical mast cell stabilizer Topical NSAID drops Topical vasoconstrictor drops

ANS: C This child has symptoms of allergic conjunctivitis. Topical NSAIDs work for acute symptoms to reduce inflammation and may be used in children over age 12 years. Saline solution or artificial tears are useful for milder symptoms. Topical mast cell stabilizers are useful for chronic symptoms and maintenance therapy. Topical vasoconstrictors should be avoided because of rebound hyperemia.

A 13-year-old Native American female has a BMI at the 90th percentile for age. The primary care pediatric nurse practitioner notes the presence of a hyperpigmented velvet-like rash in skin folds. The child denies polydipsia, polyphagia, and polyuria. The nurse practitioner will counsel the child to lose weight to prevent type 2 diabetes. diagnose type 2 diabetes if the child has a random glucose of 180 mg/dL. order a fasting blood sample for a metabolic screen for type 2 diabetes. refer the child to a pediatric endocrinologist.

ANS: C This child has three risk factors for type 2 diabetes: Native American ethnicity, overweight, and acanthosis nigricans. The PNP should perform metabolic screening on a fasting blood sample to diagnose this. Lifestyle changes may be necessary to control the disease if diagnosed, but this child may already have the disease, and management options aren't clear until the diagnosis is made. Diagnosis is based on a random glucose >200 mg/dL. It is not necessary to refer this child until a diagnosis is made.

1. The primary care pediatric nurse practitioner performs a well baby examination on a 7-day-old infant who is nursing well, according to the mother. The nurse practitioner notes that the infant weighed 3250 grams at birth and 2990 grams when discharged on the second day of life. The infant weighs 3080 grams at this visit. Which action is correct? a. Follow up at the 2-month checkup. b. Refer to a lactation consultant. c. Schedule a weight check in 1 week. d. Suggest supplementing with formula.

ANS: C This infant lost about 8% of its birth weight, which is normal and, since discharge home, has gained at least 15 grams per day, which is also normal. The PNP should schedule a weight check in a week to make sure the infant regains its birth weight, since most should regain this in 10 to 14 days and since this loss of birth weight is at the high end of normal. It is not necessary to refer to a lactation consultant or supplement with formula, since the infant is gaining weight adequately.

The primary care pediatric nurse practitioner performs a well baby examination on a 7-day-old infant who is nursing well, according to the mother. The nurse practitioner notes that the infant weighed 3250 grams at birth and 2990 grams when discharged on the second day of life. The infant weighs 3080 grams at this visit. Which action is correct? Follow up at the 2-month checkup. Refer to a lactation consultant. Schedule a weight check in 1 week. Suggest supplementing with formula.

ANS: C This infant lost about 8% of its birth weight, which is normal and, since discharge home, has gained at least 15 grams per day, which is also normal. The PNP should schedule a weight check in a week to make sure the infant regains its birth weight, since most should regain this in 10 to 14 days and since this loss of birth weight is at the high end of normal. It is not necessary to refer to a lactation consultant or supplement with formula, since the infant is gaining weight adequately.

A school-age child has a rash without fever or preceding symptoms. Physical examination reveals a 3-cm ovoid, erythematous lesion on the trunk with a finely scaled elevated border, along with generalized macular, ovoid lesions appearing in a "Christmas tree" pattern on the child's back. What is the initial action? Obtain a KOH preparation of a skin scraping to verify the diagnosis. Prescribe topical steroid creams to shorten the course of the disease. Reassure the child's parents that the rash is benign and self-limited. Recommend topical antihistamines and emollients to control the spread.

ANS: C This rash is typical of pityriasis rosea, a benign, self-limited papulosquamous disease that is not contagious. Patients may be reassured that this is the case. Because the herald lesion is characteristic, it is not necessary to obtain a KOH scraping to look for tinea corporis. Topical steroids do not alter the course of the disease. Topical antihistamines and emollients may be used if itching occurs, but this is not the initial management action.

The primary care pediatric nurse practitioner is conducting a follow-up examination on a child who has recently begun taking a low-dose stimulant medication to treat ADHD. The child's school performance and home behaviors have improved. The child's parent reports noticing a few tics, such a twitching of the eyelids, but the child is unaware of them and isn't bothered by them. What will the nurse practitioner recommend? Adding an alpha-agonist medication Changing to a non-stimulant medication Continuing the medication as prescribed Stopping the medication immediately

ANS: C Tics may occur as a side effect of stimulant medications but do not need to be discontinued if there is a net benefit and the symptoms are not disturbing to the child. It is not necessary to add an alpha-agonist, change to a non-stimulant medication, or stop the medication.

The parent of a 24-month-old child asks the primary care pediatric nurse practitioner when toilet training should begin. How will the pediatric nurse practitioner respond? "Begin by reading to your child about toileting." "Most children are capable by age 2 years." "Tell me about your child's daily habits." "We should assess your child's motor skills."

ANS: C To assess the parent's understanding of toilet readiness, the nurse practitioner will ask the parents about the child's daily habits and routines to see if the child has predictable patterns that can be the basis for toilet training. While providing storybooks about toileting can help children learn, the first step is to assess toilet readiness. Even though many children are capable at this age, evaluating personal readiness is key to beginning toilet training. Assessment of motor skills may be a second step.

An adolescent has acne characterized by papules and pustules mostly on the forehead and chin. What will the primary care pediatric nurse practitioner prescribe? Azelaic acid applied daily at nighttime Benzoyl peroxide applied twice daily Topical erythromycin with benzoyl peroxide Tretinoin applied nightly after washing the face

ANS: C Topical antibiotics combined with BPO are more effective than either drug alone and are especially effective in mild to moderate inflammatory acne or as adjunctive therapy with oral antibiotics. Azelaic acid is useful in persons with sensitive or dark skin and used for non-inflammatory acne. Topical antibiotics are best used in conjunction with BPO. Tretinoin is a keratolytic, useful for non-inflammatory acne.

A 16-year-old female has not had a menstrual period yet and is concerned. She denies sexual activity. An exam reveals an adult sexual maturity rating. Which laboratory test will the primary care pediatric nurse practitioner order initially? Genetic test for Turner syndrome Pituitary hormone tests Pregnancy test Thyroid function tests

ANS: C When amenorrhea occurs, initial laboratory studies should include a pregnancy test regardless of sexual history. Other tests are ordered after pregnancy is ruled out.

4. The primary care pediatric nurse practitioner is examining a newborn infant recently discharged from the neonatal intensive care unit after a premature birth. The parent is upset and expresses worry about whether the infant will be normal. What will the nurse practitioner do in this situation? a. Explain to the parent that developmental delays often do not manifest at first. b. Perform a developmental assessment and tell the parent which delays are evident. c. Point out the tasks that the infant can perform while conducting the assessment. d. Refer the infant to a developmental specialist for a complete evaluation.

ANS: C When discussing developmental delays with parents, it is important to be positive and to initially focus on strengths. Explaining that developmental delays develop over time is true but does not reassure the parent or help the parent cope with feelings. Referrals are not indicated unless delays are present and may take time.

The primary care pediatric nurse practitioner is examining a newborn infant recently discharged from the neonatal intensive care unit after a premature birth. The parent is upset and expresses worry about whether the infant will be normal. What will the nurse practitioner do in this situation? Explain to the parent that developmental delays often do not manifest at first. Perform a developmental assessment and tell the parent which delays are evident. Point out the tasks that the infant can perform while conducting the assessment. Refer the infant to a developmental specialist for a complete evaluation.

ANS: C When discussing developmental delays with parents, it is important to be positive and to initially focus on strengths. Explaining that developmental delays develop over time is true but does not reassure the parent or help the parent cope with feelings. Referrals are not indicated unless delays are present and may take time.

A child has several circular, scaly lesions on the arms and abdomen, some of which have central clearing. The primary care pediatric nurse practitioner notes a smaller, scaly lesion on the child's scalp. How will the nurse practitioner treat this child? Obtain scrapings of the lesions for fungal cultures. Order prescription-strength antifungal creams. Prescribe oral griseofulvin for 2 to 4 weeks. Recommend OTC antifungal creams and shampoos.

ANS: C Whenever tinea lesions occur on the scalp or nails, oral griseofulvin must be given for 2 to 4 weeks. Unless the infection is resistant to treatment, fungal cultures are not necessary. Topical medications alone are not effective for tinea capitus.

A school-age child has an abrupt onset of sore throat, nausea, headache, and a temperature of 102.3°F. An examination reveals petechiae on the soft palate, beefy-red tonsils with yellow exudate, and a scarlatiniform rash. A Rapid Antigen Detection Test (RADT) is negative. What is the next step in management for this child? Consider a sexual abuse diagnosis. Obtain an anti-streptococcal antibody titer. Perform a follow-up throat culture. Prescribe amoxicillin for 10 days.

ANS: C While an RADT has a high specificity, it has variable sensitivity, and a negative test does not mean that streptococcal infection is not present. A culture should be performed to confirm the diagnosis. If the throat culture is negative for GABHS, other causes, such as gonococcal infection, may be considered but are less likely. The RADT does not assess for sexual abuse. An ASO titer is not useful in the diagnosis of acute pharyngitis, since the titers remain elevated for months after an acute infection. Amoxicillin is not indicated unless infection is confirmed.

The parent of a high school basketball player tells the primary care pediatric nurse practitioner that the adolescent becomes short of breath only when exercising. What will the nurse practitioner recommend? Permanent discontinuation of all strenuous and aerobic activities Enrollment in a conditioning program to improve performance Evaluation for underlying cardiac causes of this symptom Treatment for exercise-induced asthma with a bronchodilator

ANS: C While shortness of breath may indicate several more benign causes, athletes who exhibit this symptom should be evaluated for underlying cardiac causes to prevent sudden cardiac death. Once this is ruled out, other causes may be considered, such as EIA or poor conditioning.

During a well child assessment, the primary care pediatric nurse practitioner auscultates a harsh, blowing grade IV/VI murmur in a 6-month-old infant. What will the nurse practitioner do next? Get a complete blood count to rule out severe anemia. Obtain an electrocardiogram to assess for arrhythmia. Order a chest radiograph to evaluate for cardiomegaly. Refer to a pediatric cardiologist for further evaluation.

ANS: D A harsh, blowing murmur is suspicious for pathology, so a cardiology referral is warranted. The cardiologist will determine which tests and procedures should be performed.

A toddler who was born prematurely refuses most solid foods and has poor weight gain. A barium swallow study reveals a normal esophagus. What will the primary care pediatric nurse practitioner consider next to manage this child's nutritional needs? Consultation with a dietician Fiberoptic endoscopy evaluation Magnetic resonance imaging Videofluoroscopy swallowing study

ANS: D A videofluoroscopy swallowing study will evaluate other structural defects that may interfere with swallowing and is relatively non-invasive. A dietician consult may be a part of the overall plan, but the toddler first needs a thorough evaluation of potential problems. Fiberoptic endoscopy is invasive. MRI may be performed if videofluoroscopy is inconclusive, but this is an expensive test.

The mother of a 16-year-old male was recently divorced after several years of an abusive relationship and tells the primary care pediatric nurse practitioner that the adolescent has begun skipping school and hanging out with friends at the local shopping mall. When she confronts her child, he responds by saying that he hates her. What will the nurse practitioner tell this mother? Adolescence is marked by an inability to comprehend complex situations. Adolescence is typically marked by tempestuous and transient episodes. Adolescents normally have extreme, disruptive conflicts with parents. Adolescents often need counseling to help them cope with life events.

ANS: D Adolescent brains respond differently to toxic stress, so counseling is indicated to help them manage serious events, such as family abuse and divorce. Early adolescents have concrete thinking, but the formal operational thinking occurs later. "Storm and stress" are not the norm in adolescence nor are disruptive periods of conflict.

The primary care pediatric nurse practitioner is performing an exam on an adolescent male who asks about sexual identity because of concern that a friend is worried about being gay. Which response will the nurse practitioner make in this situation? Provide the teen with a questionnaire to gain information about his sexuality. Remind the adolescent that mandatory reporting requires disclosure to parents. Suggest that the adolescent discuss sexual concerns with his parents. Tell the adolescent that, unless he is at risk, what he says will be confidential.

ANS: D Adolescents should be encouraged to divulge information about their sexuality to providers by assuring them that confidentiality will be maintained unless the health of the child or others is at risk. The adolescent may be trying to ask questions about himself in a manner that doesn't implicate his own sexuality, so the PNP should attempt to gain his confidence. Questionnaires may be useful when collecting information, but this adolescent has already begun a discussion about the topic. An adolescent who is concerned about being gay may not be ready to come out to his parents.

The parents of a 12-year-old child are concerned that some of the child's older classmates may be a bad influence on their child, who, they say, has been raised to believe in right and wrong. What will the primary care pediatric nurse practitioner tell the parent? Allowing the child to make poor choices and accept consequences is important for learning values Children at this age have a high regard for authority and social norms, so this is not likely to happen Moral values instilled in the early school-age period will persist throughout childhood The pressures from outside influences may supersede parental teachings and should be confronted

ANS: D Although early school-age children learn values from their parents, these may be challenged as children learn that others have different values. Parents must confront and negotiate these issues daily with their children. While children may make poor choices and subsequently learn from the consequences, it is best for parents to actively discuss these issues with their children. Children do have a high regard for authority and social norms but may easily transfer this authority to other, less reliable people, such as peers. Moral values may not persist if other sources of authority become prominent.

The primary care pediatric nurse practitioner is obtaining a medical history about a child. To integrate both nursing and medical aspects of primary care, which will be included in the medical history? Complementary medications, alternative health practices, and chief complaint Developmental delays, nutritional status, and linear growth patterns Medication currently taking, allergy information, and family medical history Speech and language development, beliefs about health, and previous illnesses

ANS: D An assessment model that integrates the nursing and medical aspects of primary care uses three domains: developmental problems (speech and language development), functional health problems (beliefs about health), and diseases (chief complaint). The other examples all use domains associated with the traditional medical model and do not contain nursing aspects associated with functional health problems.

6. During a well child assessment of an African-American infant, the primary care pediatric nurse practitioner notes a dark red-brown light reflex in the left eye and a slightly brighter, red-orange light reflex in the right eye. The nurse practitioner will a. dilate the pupils and reassess the red reflex. b. order auto-refractor screening of the eyes. c. recheck the red reflex in 1 month. d. refer the infant to an ophthalmologist.

ANS: D Any asymmetry, dark or white spots, opacities, or leukokoria should be referred immediately to a pediatric ophthalmologist. The PNP does not dilate pupils or order auto-refractor exams; these are done by an ophthalmologist. Because retinoblastoma is a concern, any unusual finding should be immediately referred.

During a well child assessment of an African-American infant, the primary care pediatric nurse practitioner notes a dark red-brown light reflex in the left eye and a slightly brighter, red-orange light reflex in the right eye. The nurse practitioner will dilate the pupils and reassess the red reflex. order auto-refractor screening of the eyes. recheck the red reflex in 1 month. refer the infant to an ophthalmologist.

ANS: D Any asymmetry, dark or white spots, opacities, or leukokoria should be referred immediately to a pediatric ophthalmologist. The PNP does not dilate pupils or order auto-refractor exams; these are done by an ophthalmologist. Because retinoblastoma is a concern, any unusual finding should be immediately referred.

The primary care pediatric nurse practitioner is assessing a toddler whose weight and body mass index (BMI) are below the 3rd percentile for age. The nurse practitioner learns that the child does not have regular mealtimes and is allowed to carry a bottle of juice around at all times. The nurse practitioner plans to work with this family to develop improved meal patterns. Which diagnosis will the nurse practitioner use for this problem? Failure to thrive Home care resources inadequate Nutrition alteration - less than required Parenting alteration

ANS: D Because the PNP is planning to intervene by helping the parents to provide appropriate food habits, the correct diagnosis should be "Parenting alteration." "Failure to thrive" is a medical diagnosis and requires a medical and social evaluation to rule out organic causes or detect neglect. "Home care resources inadequate" would be used if the PNP suspects that the family lacks adequate funds to purchase food. "Nutrition alteration" is a NANDA diagnosis and would be used if the PNP planned to consult with a dietician or give nutritional information.

An infant is brought to clinic with bright erythema in the neck and flexural folds after recent treatment with antibiotics for otitis media. What is the treatment for this condition? 1% hydrocortisone cream to affected areas for 1 to 2 days Oral fluconazole 6 mg/kg on day 1, then 3 mg/kg/dose for 14 days Topical keratolytics and topical antibiotics for 7 to 10 days Topical nystatin cream applied several times daily

ANS: D Candida skin infections can occur in intertriginous areas in the neck, axilla, and groin, and appear as a bright erythematous rash. Topical nystatin is first-line therapy. Fifteen percent hydrocortisone is used if inflammation is severe but not instead of topical antifungal therapy. Oral fluconazole is used if resistant to treatment. Keratolytics and antibiotics are used to treat superficial folliculitis.

16. During a well child exam on a 4-year-old child, the primary care pediatric nurse practitioner notes that the clinic nurse recorded "20/50" for the child's vision and noted that the child had difficulty cooperating with the exam. What will the nurse practitioner recommend? a. Follow up with a visual acuity screen in 6 months. b. Refer to a pediatric ophthalmologist. c. Re-test the child in 1 year. d. Test the child's vision in 1 month.

ANS: D Children age 4 years and older who have difficulty cooperating with a vision screen should be retested in 1 month; if they continue to have difficulty cooperating, they should be referred for a formal examination. Children who are 3 years old should be re-evaluated in 6 months.

During a well child exam on a 4-year-old child, the primary care pediatric nurse practitioner notes that the clinic nurse recorded "20/50" for the child's vision and noted that the child had difficulty cooperating with the exam. What will the nurse practitioner recommend? Follow up with a visual acuity screen in 6 months. Refer to a pediatric ophthalmologist. Re-test the child in 1 year. Test the child's vision in 1 month.

ANS: D Children age 4 years and older who have difficulty cooperating with a vision screen should be retested in 1 month; if they continue to have difficulty cooperating, they should be referred for a formal examination. Children who are 3 years old should be re-evaluated in 6 months.

A child has a 1-cm laceration on the forehead proximal to the hairline after running into a pole while playing sports. To minimize the risk of infection, the primary care pediatric nurse practitioner will irrigate the wound and a. allow the wound to heal by secondary intention. b. delay closure of the wound for several days. c. refer the child to a plastic surgeon for wound closure. d. suture the wound within 6 hours.

ANS: D Children are less likely than adults to get wound infections, with an infection rate from sutured lacerations at 2%. The PNP should clean and suture the wound. Wounds from animal bites are often left to heal by secondary intention to prevent infection. Referral to a plastic surgeon is necessary for cosmetic reasons. Delaying closure for several days is recommended for heavily contaminated wounds and those caused by high-velocity missile injuries, crush injuries, and explosion injuries.

The parent of a 4-year-old points to a picture and says, "That's your sister." The child responds by saying, "No! It's my baby!" This is an example of which type of thinking in preschool-age children? Animism Artificialism Egocentrism Realism

ANS: D Children at this age are developing their ability to establish causality. Nominal realism occurs when children think that one type of thing can only be called by one name. All dogs are dogs and not various breeds. Animism refers to the belief that objects possess person-like qualities. Artificialism occurs when children think things are caused by a controlling force. Egocentrism is when children see things only as they relate to themselves.

A 12-year-old child whose BMI is greater than the 95th percentile has a blood pressure at the 98th percentile for age, sex, and height. After lifestyle changes that include diet and exercise, the child's BMI drops to the 90th percentile, but the blood pressure remains the same. What is the primary care pediatric nurse practitioner's next step in treating this child? Continued close monitoring of blood pressure Ordering an echocardiogram or MRI Prescribing an ACE inhibitor medication Referral to a nephrologist or cardiologist

ANS: D Children who have persistent BP elevation after lifestyle changes are made should be referred to a nephrologist or cardiologist who has experience using antihypertensive agents in children. The specialist orders necessary tests and medications, not the primary care provider.

A 7-month-old infant has had two prior acute ear infections and is currently on the 10th day of therapy with amoxicillin-clavulanate after a failed course of amoxicillin. The primary care pediatric nurse practitioner notes marked middle ear effusion and erythema of the TM. The child is irritable and has a temperature of 99.8°F. What is the next step in management of this child's ear infection? Order a second course of amoxicillin-clavulanate. Perform tympanocentesis for culture. Prescribe clindamycin twice daily. Refer the child to an otolaryngologist.

ANS: D Children who have persistent infection who have failed appropriate therapy and those who have had three or more episodes of AOM in 6 months should be referred to an otolaryngologist. Ceftriaxone is ordered when Augmentin fails. The PNP does not perform tympanocentesis. Clindamycin is used for ceftriaxone failure but only if the susceptibilities are known.

The primary care pediatric nurse practitioner is performing a well child examination on a 2-year-old child with a history of intrauterine growth retardation (IUGR) whose height remains less than the 3rd percentile on a WHO growth chart. What will the nurse practitioner do? Consider prescribing growth hormone therapy. Reassure the parent that this is normal for this child. Refer the child to a dietician for dietary supplementation. Refer the child to a pediatric endocrinologist.

ANS: D Children with IUGR who fail to catch up by age 2 may be candidates for GH therapy and should be referred to a pediatric endocrinologist. The PNP does not prescribe GH; this must be done by a pediatric endocrinologist. Although this is common for these children, parents should be offered options for treatment. Dietary supplementation may be part of therapy, but the need for GH therapy should be evaluated.

A 5-year-old child who had a repair for transposition of the great arteries shortly after birth is growing normally and has been asymptomatic since the surgery. The primary care nurse practitioner notes mild shortness of breath with exertion and, upon questioning, learns that the child has recently complained of dizziness. What will the nurse practitioner do? Order an echocardiogram and chest radiograph. Perform pulmonary function testing. Reassure the parent that these symptoms are common. Refer the child to the cardiologist immediately.

ANS: D Children with a history of d-TGA who have a history of palpitations, syncope, or shortness of breath should be referred to a cardiologist. Echocardiograms should be performed annually under the supervision of the cardiologist. Pulmonary function testing is not indicated. These symptoms may represent problems in patency with the coronary arteries and are not common.

A school-age child sustained a contusion on the front of one thigh while playing football and reports some difficulty flexing his foot on the affected side. What will the primary care pediatric nurse practitioner do to treat this injury? a. Place the child on crutches and limit weight-bearing until symptoms subside. b. Prescribe acetaminophen with hydrocodone along with NSAIDs. c. Recommend rest, ice packs, compression, and elevation of the extremity. d. Refer the child to an orthopedic specialist for immediate evaluation and treatment.

ANS: D Children with contusions that restrict movement or sensation and those affecting the quadriceps muscle may include compartment syndrome. These children should be referred to orthopedic specialists immediately so that the compartment pressure does not result in irreplaceable damage. The other options may be performed in consultation with a specialist.

A 6-year-old female has had a recent growth spurt and an exam reveals breast and pubic hair development. Her bone age is determined to be 8 years. What will the primary care pediatric nurse practitioner do next? Order LH and FSH levels and a long-acting GnRH agonist. Order thyroid function tests to exclude primary hypothyroidism. Reassure the parent that this is most likely idiopathic. Refer the child to a pediatric endocrinologist for management.

ANS: D Children with early puberty should always be referred to a pediatric endocrinologist for evaluation and management. The PNP may order lab work in consultation with an endocrinologist but should refer when this condition is suspected. Thyroid function tests are performed for isolated menarche. Reassuring the parent without completely diagnosing the condition is not indicated.

A school-age child has frequent nosebleeds. Nasal visualization reveals fresh clots and excoriated nasal mucosa but no visible site of bleeding. Coagulation studies are normal. In spite of symptomatic measures, the child continues to have nosebleeds. What is the next course of action? Cauterize the mucosa with silver nitrate sticks. Order a topical vasoconstrictor medication. Prescribe a barrier agent such as petrolatum jelly. Refer to an otolaryngologist for further evaluation.

ANS: D Children with persistent epistaxis should be referred for evaluation and treatment after usual symptomatic measures are ineffective. Cautery works well for exposed vessels, but the site must be easily accessible, visible, and not bleeding briskly. Topical vasoconstrictors are occasionally used. Petrolatum jelly has not been shown to be effective.

The primary care pediatric nurse practitioner is selecting a medication for a 12-year-old child who is newly diagnosed with ADHD. The child is overweight, has a history of an atrial septal defect at birth, and reports mild shortness of breath during exercise. What will the nurse practitioner prescribe? A low-dose stimulant medication A non-stimulant medication Behavioral therapy only Cardiovascular pre-screening

ANS: D Children with potential heart problems with symptoms such as previously detected cardiac abnormalities and shortness of breath with exercise should have a cardiovascular evaluation by a cardiologist prior to initiating treatment. If the screening and assessment are normal, a stimulant medication may be prescribed.

10. A toddler exhibits exotropia of the right eye during a cover-uncover screen. The primary care pediatric nurse practitioner will refer to a pediatric ophthalmologist to initiate which treatment? a. Botulinum toxin injection b. Corrective lenses c. Occluding the affected eye for 6 hours per day d. Patching of the unaffected eye for 2 hours each day

ANS: D Deviations are initially treated by patching the unaffected eye for 2 hours each day to force the affected eye to move correctly. Botulinum toxin injection may be used with some deviations but is not a first-line therapy. Corrective lenses alone improve amblyopia in 27% of patients. The unaffected eye is patched; 2 hours per day is as effective as 6 hours per day.

A toddler exhibits exotropia of the right eye during a cover-uncover screen. The primary care pediatric nurse practitioner will refer to a pediatric ophthalmologist to initiate which treatment? Botulinum toxin injection Corrective lenses Occluding the affected eye for 6 hours per day Patching of the unaffected eye for 2 hours each day

ANS: D Deviations are initially treated by patching the unaffected eye for 2 hours each day to force the affected eye to move correctly. Botulinum toxin injection may be used with some deviations but is not a first-line therapy. Corrective lenses alone improve amblyopia in 27% of patients. The unaffected eye is patched; 2 hours per day is as effective as 6 hours per day.

A school-age child has a 3-month history of dull, aching epigastric pain that worsens with eating and awakens the child from sleep. A complete blood count shows a hemoglobin of 8 mg/dL. What is the next step in management? Administration of H2RA or PPI medications Empiric therapy for H. pylori (HP) Ordering an upper GI series Referral for esophagogastroduodenoscopy (EGD)

ANS: D EGD is the procedure of choice in children for detecting PUD because it allows direct visualization of mucosa, localization of the source of bleeding, and collection of tissue specimens. Empiric therapy for HP is not recommended due to increased antibiotic resistance. An upper GI series may have false negative findings. Once peptic ulcer disease is diagnosed, H2RA or PPI medications are first-line drugs.

9. The primary care pediatric nurse practitioner performs a vision screen on a 4-month-old infant and notes the presence of convergence and accommodation with mild esotropia of the left eye. What will the nurse practitioner do? a. Patch the right eye to improve coordination of the left eye. b. Reassure the parents that the infant will outgrow this. c. Recheck the infant's eyes in 2 to 4 weeks. d. Refer the infant to a pediatric ophthalmologist.

ANS: D Esotropia that continues or occurs at 3 to 4 months of age is abnormal, so the infant should be referred to a pediatric ophthalmologist. The PNP does not determine whether an eye patch should be used. Because it is abnormal at this age, the PNP will not reassure the parents that the infant will outgrow this. Esotropia after 3 to 4 months of age must be evaluated by a specialist and not reevaluated in 2 to 4 weeks.

The primary care pediatric nurse practitioner performs a vision screen on a 4-month-old infant and notes the presence of convergence and accommodation with mild esotropia of the left eye. What will the nurse practitioner do? Patch the right eye to improve coordination of the left eye. Reassure the parents that the infant will outgrow this. Recheck the infant's eyes in 2 to 4 weeks. Refer the infant to a pediatric ophthalmologist.

ANS: D Esotropia that continues or occurs at 3 to 4 months of age is abnormal, so the infant should be referred to a pediatric ophthalmologist. The PNP does not determine whether an eye patch should be used. Because it is abnormal at this age, the PNP will not reassure the parents that the infant will outgrow this. Esotropia after 3 to 4 months of age must be evaluated by a specialist and not reevaluated in 2 to 4 weeks.

A 4-year-old female who has had two urinary tract infections has persistent dysuria and genital redness. The physical exam reveals a thin, flat membrane from the posterior fourchette almost to the clitoris. Which treatment is indicated? Application of A&D ointment Counseling about hygiene Reassurance and observation Use of estrogen-containing cream

ANS: D Estrogen-containing 1% cream should be used when children are symptomatic, especially when UTI and pain occur. A&D ointment is used for milder symptoms and when the opening allows drainage. Reassurance, observation, and counseling about hygiene are indicated when UTI and obstruction do not occur.

A 15-year-old female basketball player who has secondary amenorrhea is evaluated by the primary care pediatric nurse practitioner who notes a BMI in the 3rd percentile. What will the nurse practitioner counsel this patient? That amenorrhea in female athletes is not concerning That she should begin a program of plyometrics and strength trainin To consider a different sport, such as volleyball To work with a dietician to improve healthy weight gain

ANS: D Female athletes who have amenorrhea have an increased risk of stress fractures. The adolescent should work to attain a healthy weight, which should allow normal periods to return and reduce this risk. Even though amenorrhea in female athletes is common, it is concerning. Plyometrics and volleyball can increase the risk of stress fractures since both involve jumping and thus not be suggested.

A 30-month-old girl who has been toilet trained for 6 months has daytime enuresis and dysuria and a low-grade fever. A dipstick urinalysis is negative for leukocyte esterase and nitrites. What is the next step? Begin empiric treatment with trimethoprim-sulfamethoxazole. Discuss behavioral interventions for toilet training. Reassure the child's parents that the child does not have a urinary tract infection. Send the urine to the lab for culture.

ANS: D Girls over age 24 months have a higher risk than boys for UTI. This child is symptomatic, so her urine should be cultured even though the leukocyte esterase and nitrites are negative; urine in the bladder less than 4 hours may be tested as negative for leukocyte esterase. Empiric treatment may be initiated if the child had signs of sepsis. Behavioral interventions are not indicated - the child has dysuria and fever along with enuresis. Until the culture is found to be negative, it is not certain that the child does not have an UTI, and thus reassurance is not the correct action.

A child is bitten by a snake near a swimming pool in an area where copperhead snakes are known to inhabit, although the parents cannot describe the snake. An examination of the bite reveals a severe local reaction at the site with edema and intense pain. What will the primary care pediatric nurse practitioner do first? a. Administer narcotic analgesics to provide comfort. b. Begin treatment with oral amoxicillin-clavulanate for 5 days. c. Clean the wound and administer tetanus prophylaxis. d. Transport the child by ambulance to a medical center.

ANS: D If a venomous snakebite is suspected, rapid transportation to a medical center with referral to appropriate specialists and antivenin therapy is indicated. Narcotics may impair clinical evaluation. Non-venomous snakebites are treated with oral antibiotics if signs of infection are present after the wound is cleaned and tetanus prophylaxis is given.

The clean catch urine specimen of a child with dysuria, frequency, and fever has a colony count between 50,000 and 100,000 of E. coli. What is the treatment for this child? Obtain a complete blood count and C-reactive protein. Perform sensitivity testing before treating with antibiotics. Repeat the culture if symptoms persist or worsen. Treat with antibiotics for urinary tract infection.

ANS: D If children are symptomatic and have more than 10,000 colonies of a single pathogen, they are considered to have a UTI and are treated. If pyelonephritis symptoms such as flank pain and sepsis are present, CBC and CRP are useful tests. Sensitivity testing is done for patients who appear toxic, have pyelonephritis, or are non-responsive to antibiotics.

A parent is concerned that a 12-month-old child is "bow-legged." A physical examination reveals internal tibial torsion bilaterally. A radiograph reveals asymmetric bowing of the legs with an angle greater than 15 degrees. What is the correct action for the primary care pediatric nurse practitioner? Observe the child's condition over time to assess progression. Order physical therapy to prevent progression of symptoms. Reassure the parent that the child will outgrow this deformity. Refer to a pediatric orthopedic specialist for treatment.

ANS: D In Blount disease, the bowing is asymmetrical; children with this disorder need immediate referral to an orthopedist. Physiologic bowing may self-resolve and may be managed with observation, physical therapy, and reassurance.

The mother of a female infant is concerned that her daughter is developing breasts. The primary care pediatric nurse practitioner notes mild breast development but no pubic or axillary hair. What is the likely diagnosis? Congenital adrenal hyperplasia causing breast development Precocious puberty needing endocrinology management Premature adrenarche which will lead to pubic hair onset Premature thelarche which will resolve over time

ANS: D Infant and toddler girls may have isolated breast development, or premature thelarche, which usually resolves over time and rarely progresses to true precocious puberty. CAD will have other symptoms and is associated with premature adrenarche, which is the early development of pubic and axillary hair. Precocious puberty involves the early onset of multiple features of puberty and not just breast development.

4. The mother of a 6-week-old breastfeeding infant tells the primary care pediatric nurse practitioner that her baby, who previously had bowel movements with each feeding, now has a bowel movement once every third day. What will the nurse practitioner tell her? a. Her baby is probably constipated. b. It may be related to her dietary intake. c. She should consume more water. d. This may be normal for breastfed babies.

ANS: D Infants begin to have fewer bowel movements and may have bowel movements ranging from once or twice daily to once every other day when breastfed. Unless there are other signs, the baby is probably not constipated. The mother does not need to change her intake of foods or water, unless constipation is present.

The mother of a 6-week-old breastfeeding infant tells the primary care pediatric nurse practitioner that her baby, who previously had bowel movements with each feeding, now has a bowel movement once every third day. What will the nurse practitioner tell her? Her baby is probably constipated. It may be related to her dietary intake. She should consume more water. This may be normal for breastfed babies.

ANS: D Infants begin to have fewer bowel movements and may have bowel movements ranging from once or twice daily to once every other day when breastfed. Unless there are other signs, the baby is probably not constipated. The mother does not need to change her intake of foods or water, unless constipation is present.

1. The primary care pediatric nurse practitioner is treating an infant with lacrimal duct obstruction who has developed bacterial conjunctivitis. After 2 weeks of treatment with topical antibiotics along with massage and frequent cleansing of secretions, the infant's symptoms have not improved. Which action is correct? a. Perform massage more frequently. b. Prescribe an oral antibiotic. c. Recommend hot compresses. d. Refer to an ophthalmologist.

ANS: D Infants treated for a secondary bacterial conjunctivitis with lacrimal duct obstruction who do not improve after 1 to 2 weeks of topical antibiotic therapy must be referred to an ophthalmologist for possible lacrimal duct probe. Performing the massage more often or applying hot compresses will not help clear the infections. Oral antibiotics are not indicated.

The primary care pediatric nurse practitioner is treating an infant with lacrimal duct obstruction who has developed bacterial conjunctivitis. After 2 weeks of treatment with topical antibiotics along with massage and frequent cleansing of secretions, the infant's symptoms have not improved. Which action is correct? Perform massage more frequently. Prescribe an oral antibiotic. Recommend hot compresses. Refer to an ophthalmologist.

ANS: D Infants treated for a secondary bacterial conjunctivitis with lacrimal duct obstruction who do not improve after 1 to 2 weeks of topical antibiotic therapy must be referred to an ophthalmologist for possible lacrimal duct probe. Performing the massage more often or applying hot compresses will not help clear the infections. Oral antibiotics are not indicated.

During a well baby examination of a 6-week-old infant, the primary care pediatric nurse practitioner notes poor weight gain, acrocyanosis of the hands and feet, and a respiratory rate of 60 breaths per minute. Oxygen saturation on room air is 93%. The remainder of the exam is unremarkable. Which action is correct? Follow-up in 1 week to assess the infant's weight. Order a chest radiograph and an electrocardiogram. Reassure the parents that the exam is within normal limits. Refer the infant to a pediatric cardiologist.

ANS: D Infants with oxygen saturation less than 95% and those with poor feeding should be referred emergently to a cardiologist. The infant may have CHF and will need to be evaluated.

The primary care pediatric nurse practitioner is performing a well child examination on a 3-year-old. The child's parent reports that the child has recently begun masturbating. What will the nurse practitioner counsel this parent? To allow the behavior whenever it occurs, since it is normal To discuss sexuality with the child To explore whether the child is being abused To teach the child about privacy and hand hygiene

ANS: D Masturbation is normal at this age and children do this because it is pleasurable. Parents should be taught to discuss privacy and hygiene with the child and to encourage the child to limit the activity to a private place. At this age, the behavior is not associated with sexual fantasies, so a discussion of sexuality is not warranted. Masturbation at this age is common and is not usually an indication of abuse.

The primary care pediatric nurse practitioner learns that the mother of a 3-year-old child has been treated for depression for over 5 years. Which aspect of this child's development will be of the most concern to the nurse practitioner? Fine motor Gross motor Social/emotional Speech and language

ANS: D Maternal depression in the first year of life has been associated with poorer language development at 3 years of age.

A healthy 14-year-old female has a dipstick urinalysis that is positive for 5-6 RBCs per hpf but otherwise normal. What is the first question the primary care pediatric nurse practitioner will ask this patient? "Are you sexually active?" "Are you taking any medications?" "Have you had a recent fever?" "When was your last menstrual period (LMP)?"

ANS: D Menstrual blood may appear in urine and is a common cause of urine with RBCs present, so this would be an appropriate first question of an adolescent. Asking about sexual activity or recent fevers may be part of the diagnostic reasoning if common causes are not present. Medications may discolor the urine but do not cause RBCs to be present.

The primary care pediatric nurse practitioner is offering anticipatory guidance to the parents of a 12-month-old child. The parents are bilingual in Spanish and English and have many Spanish-speaking relatives nearby. They are resisting exposing the child to Spanish out of concern that the child will not learn English well. What will the pediatric nurse practitioner tell the parents? Children who learn two languages simultaneously often confuse them in conversation. Children with multi-language proficiency do not understand that others cannot do this. Learning two languages at an early age prevents children from developing a dominant language. Most bilingual children are able to shift from one language to another when appropriate.

ANS: D Most children who are bilingual are able to sort out the languages in conversation but may "code switch" at times for clarity as they speak. They seem to understand that not everyone has this ability. Most children who are bilingual develop a dominant language.

The mother of a 3-month-old male infant tells the primary care pediatric nurse practitioner that she occasionally notices he has a penile erection just after nursing. What will the nurse practitioner tell the mother? Infants should be prevented from masturbating. The infant is conscious of the pleasure associated with nursing. This is a form of infantile priapism. This is a normal, reflexive behavior at this age.

ANS: D Newborn infants are reflexive beings, and sexual reflexes, which are present prenatally, are easily stimulated. A penile erection may occur while nursing. Infants explore with their hands and may touch their own genitalia for pleasure and for the purpose of soothing, and this is normal. A penile erection at this young age is reflexive and not conscious and intentional. It is not a form of priapism.

An adolescent female has grouped vesicles on her oral mucosa. To determine whether these are caused by HSV-1 or HSV-2, the primary care pediatric nurse practitioner will order which test? Direct fluorescent antibody test Enzyme-linked immunosorbent assay Tzanck smear Viral culture

ANS: D Oral lesions are possible with both forms of herpesvirus. Viral culture is the gold standard for distinguishing HSV-1 from HSV-2. DFA and ELISA are usually used only with severe forms of infection. Tzanck smear dose not distinguish HSV-1 from HSV-2.

A 16-year-old adolescent female whose BMI is at the 90th percentile reports irregular periods. The primary care pediatric nurse practitioner notes widespread acne on her face and back and an abnormal distribution of facial hair. The nurse practitioner will evaluate her further based on a suspicion of which diagnosis? Dyslipidemia Hypothyroidism Nonalcoholic steatohepatitis Polycystic ovary syndrome

ANS: D PCOS has symptoms of irregular menses, acne, and hirsutism and is associated with obesity. Dyslipidemia, hypothyroidism, and nonalcoholic steatohepatitis are all possible conditions associated with obesity but do not have the symptoms described in the scenario.

During a well child assessment of an 18-month-old child, the primary care pediatric nurse practitioner observes the child point to a picture of a dog and say, "Want puppy!" The nurse practitioner recognizes this as an example of holophrastic speech. receptive speech. semantic speech. telegraphic speech.

ANS: D Syntax, or the structure of words in sentences or phrases, is developed in stages between the ages of 8 months and 3.5 years. Telegraphic speech begins at about 18 months of age when children speak in phrases with many words omitted, so that the sentence sounds like a telegram. Holophrastic speech is the use of a single word to express a complete idea. Receptive speech refers to the ability to understand a word without necessarily being able to use the word. Semantics is the understanding that words have specific meanings.

A 9-month-old infant with a history of three urinary tract infections is diagnosed with grade II vesicoureteral reflux. Which medication will be prescribed? Amoxicillin 10 mg/kg as a single daily dose Ceftriaxone IM 50 mg/kg as a single daily dose Nitrofurantoin 1-2 mg/kg as a single daily dose TMP-SMX; TMP 2 mg/kg as a single daily dose

ANS: D TMP-SMX is a first-line medication for grade II VUR prophylaxis. TMP-SMX: TMP 2 mg/kg as a single daily dose or 5 mg/kg twice/wk (based on TMP component) if older than 1 month.Amoxicillin is a first-line treatment in infants younger than 2 months. It is not necessary to give IM medications. Nitrofurantoin is expensive and poorly tolerated.

8. The primary care pediatric nurse practitioner performs a physical examination on a 9-month-old infant and notes two central incisors on the lower gums. The parent states that the infant nurses, takes solid foods three times daily, and occasionally takes water from a cup. What will the pediatric nurse practitioner counsel the parent to promote optimum dental health? a. To begin brushing the infant's teeth with toothpaste b. To consider weaning the infant from breastfeeding c. To discontinue giving fluoride supplements d. To make an appointment for an initial dental examination

ANS: D The American Academy of Pediatric Dentistry recommends a first dental examination at the time of eruption of the first tooth and no later than 12 months old. Parents should be counseled to clean the infant's teeth but with water only. Weaning from breastfeeding is not indicated, although mothers should not let the infant nurse while sleeping to prevent milk from bathing the teeth. Fluoride supplements should not be discontinued.

The primary care pediatric nurse practitioner performs a physical examination on a 9-month-old infant and notes two central incisors on the lower gums. The parent states that the infant nurses, takes solid foods three times daily, and occasionally takes water from a cup. What will the pediatric nurse practitioner counsel the parent to promote optimum dental health? To begin brushing the infant's teeth with toothpaste To consider weaning the infant from breastfeeding To discontinue giving fluoride supplements To make an appointment for an initial dental examination

ANS: D The American Academy of Pediatric Dentistry recommends a first dental examination at the time of eruption of the first tooth and no later than 12 months old. Parents should be counseled to clean the infant's teeth but with water only. Weaning from breastfeeding is not indicated, although mothers should not let the infant nurse while sleeping to prevent milk from bathing the teeth. Fluoride supplements should not be discontinued.

A child is in the clinic for evaluation of an asthma action plan. The primary care pediatric nurse practitioner notes that the child's last visit was for a pre-kindergarten physical and observes that the child is extremely anxious. What will the nurse practitioner do initially? Ask the child's parent why the child is so anxious. Perform a physical assessment to rule out shortness of breath. Reassure the child that there is nothing to be afraid of. Review the purpose of this visit and any anticipated procedures.

ANS: D The PNP should remember that young children are learning "scripts" for health care visits and may be stressed when recalling previous visits, especially if those involved immunizations. The PNP should explain the purpose and any anticipated procedures for this visit to help put the child at ease.

After 14 days of treatment with amoxicillin 45 mg/kg/day for acute rhinosinusitis, a child continues to have mucopurulent nasal discharge along with induration, swelling, and erythema of both eyelids. What is the next course of treatment? Amoxicillin 80 mg/kg/day for 14 days Amoxicillin-clavulanate for 10 to 14 days Antibiotic ophthalmic drops for 5 to 7 days Referral to a pediatric otolaryngologist

ANS: D The child has symptoms consistent with periorbital or preseptal inflammation and needs a referral to an otolaryngologist or infectious disease specialist. For uncomplicated persistent rhinosinusitis, amoxicillin-clavulanate should be prescribed. These symptoms are not consistent with conjunctivitis, so antibiotic eyedrops are not indicated. Although increasing the dose of amoxicillin may be part of the treatment, referral to a pediatric otolaryngologist is a priority.

The primary care pediatric nurse practitioner evaluates a child who awoke with a sore throat and high fever after a nap. The child appears anxious and is sitting on the parent's lap with the neck hyperextended. The physical exam reveals stridor, drooling, nasal flaring, and retractions. What will the nurse practitioner do next? Administer a broad-spectrum intravenous antibiotic. Obtain blood and throat cultures and start antibiotic therapy. Send the child to radiology for a lateral neck radiograph. Transport the child to the hospital via emergency medical services.

ANS: D The child has symptoms of epiglottitis and should be transported immediately for emergency treatment via ambulance. All of the other options may be initiated at the hospital once the diagnosis is more certain. If the possibility of epiglottitis is thought to be remote, a lateral neck radiograph may be obtained prior to visualizing the throat. If epiglottitis is suspected, visualizing the throat is contraindicated.

13. The primary care pediatric nurse practitioner observes a tender, swollen red furuncle on the upper lid margin of a child's eye. What treatment will the nurse practitioner recommend? a. Culture of the lesion to determine causative organism b. Referral to ophthalmology for incision and drainage c. Topical steroid medication d. Warm, moist compresses 3 to 4 times daily

ANS: D The child has symptoms of hordeolum, or stye. Although these often rupture spontaneously, warm, moist compresses may hasten this process. It is not necessary to culture the lesion unless symptoms do not resolve. Referral to ophthalmology is made if the hordeolum does not rupture on its own. Steroids are not indicated.

The primary care pediatric nurse practitioner observes a tender, swollen red furuncle on the upper lid margin of a child's eye. What treatment will the nurse practitioner recommend? Culture of the lesion to determine causative organism Referral to ophthalmology for incision and drainage Topical steroid medication Warm, moist compresses 3 to 4 times daily

ANS: D The child has symptoms of hordeolum, or stye. Although these often rupture spontaneously, warm, moist compresses may hasten this process. It is not necessary to culture the lesion unless symptoms do not resolve. Referral to ophthalmology is made if the hordeolum does not rupture on its own. Steroids are not indicated.

2. The primary care pediatric nurse practitioner conducts a well baby exam on an infant and notes mild gross motor delays but no delays in other areas. Which initial course of action will the nurse practitioner recommend? a. Consult a developmental specialist for a more complete evaluation. b. Prepare the parents for a potentially serious developmental disorder. c. Refer the infant to an early intervention program for physical therapy. d. Teach the parents to provide exercises to encourage motor development.

ANS: D The child who has mild delays in only one area may be managed initially by having the parent provide appropriate exercises. If this is not effective, or if delays become more severe, referrals for evaluation or early intervention services are warranted. A mild delay does not necessarily signal a serious disorder, so this action is not indicated.

The primary care pediatric nurse practitioner conducts a well baby exam on an infant and notes mild gross motor delays but no delays in other areas. Which initial course of action will the nurse practitioner recommend? Consult a developmental specialist for a more complete evaluation. Prepare the parents for a potentially serious developmental disorder. Refer the infant to an early intervention program for physical therapy. Teach the parents to provide exercises to encourage motor development.

ANS: D The child who has mild delays in only one area may be managed initially by having the parent provide appropriate exercises. If this is not effective, or if delays become more severe, referrals for evaluation or early intervention services are warranted. A mild delay does not necessarily signal a serious disorder, so this action is not indicated.

The primary care pediatric nurse practitioner is providing anticipatory guidance to the parent of a school-age boy. The parent expresses concerns that the child prefers to play with dolls, is worried that the child will be a homosexual, and asks what can be done to prevent this from happening. What will the nurse practitioner tell this parent? Homosexual identity formation cannot be predicted by early childhood behavior. Masculinizing boys from an early age helps to determine heterosexual orientation. Sexual orientation identification begins late in adolescence and not in childhood. The development of sexual orientation is generally a multifaceted process.

ANS: D The etiology and age of preferred sexual orientation is unknown, and the sequential developmental signs are debated. The development of sexual orientation is most likely multifaceted and cannot be predicted by one phenomenon, such as playing with dolls alone. Early childhood behavior can predict homosexual orientation as girls may feel "unfeminine" and boys may exhibit feminine tendencies. It is clear that psychosocial components and parenting do not cause or prevent homosexuality.

During a well child examination of a 2-year-old child, the primary care pediatric nurse practitioner palpates a unilateral, smooth, firm abdominal mass which does not cross the midline. What is the next course of action that? Order a CT scan of the chest, abdomen, and pelvis. Perform urinalysis, CBC, and renal function tests. Reevaluate the mass in 1 to 2 weeks. Refer the child to an oncologist immediately.

ANS: D The finding is consistent with Wilms tumor, and referral, diagnosis, and treatment are urgent. Palpating a mass too vigorously could lead to the rupture of a large tumor into the peritoneal cavity so care should be taking in conducting the physical examination. The other tests may be ordered by the oncology team. Treatment and diagnosis must occur immediately.

A 5-month-old infant who has a 3-day history of cough and rhinorrhea has developed symptoms of respiratory distress with audible expiratory wheezes and increased coughing. The infant's immunizations are up-to-date. The physical exam reveals a respiratory rate of 50 breaths per minute, coarse expiratory wheezing, and prolonged expiration. An oxygen saturation is 96% on room air. What is the recommended treatment for this infant? Administer a trial of bronchodilators. Obtain a viral culture of nasal washings. Order an oral corticosteroid medication. Recommend increased fluids and close follow-up.

ANS: D The infant has bronchiolitis and is stable. Increasing fluids and following up closely are indicated as long as oxygen saturations and hydration are normal. Bronchodilator trials are not recommended because of the risk of adverse effects and questionable efficacy. Viral cultures are performed if hospitalization is necessary or when symptoms are severe. Corticosteroid medications are not indicated.

An infant has congenital adrenal hyperplasia. At a routine well baby checkup, the primary care pediatric nurse practitioner notes vomiting, poor feeding, lethargy, and mild dehydration. Which action is correct? Administer an intramuscular stress dose of hydrocortisone succinate. Administer intravenous fluids in the clinic and reassess hydration status. Prescribe an oral hydrocortisone in a replacement dose of 8 to 10 mg/M2. Refer the infant to the emergency department for fluids, dextrose, and steroids.

ANS: D The infant is showing signs of adrenal crisis and will need ED management for IV dextrose, normal saline, and stress doses of hydrocortisone succinate. Because of the presence of dehydration, the infant will need IV fluids but must be sent to the ED because of the risk of a worsening crisis. Oral hydrocortisone in replacement doses is given as part of routine management, not as treatment for adrenal crisis.

During a well child assessment of a 13-year-old male, the primary care pediatric nurse practitioner notes small testicles and pubic and axillary hair. To further evaluate these findings, the nurse practitioner will ask the patient about alcohol and tobacco use. changes in voice. increase in height and weight. participation in sports.

ANS: D The initial sign of puberty in males is testicular enlargement. If this does not precede other changes, the PNP should consider whether the boy is taking exogenous anabolic steroids, common among those who wish to improve athletic ability. These findings are not concerning for alcohol or tobacco use. Voice changes and rapid growth may occur with pubic hair development, but the primary concern is anabolic steroid use.

A 3-month-old infant who was previously healthy now has a persistent cough, bilateral lung crackles, and poor appetite. The primary care pediatric nurse practitioner auscultates a grade III/VI, low-pitched, holosystolic murmur over the left lower sternal border and palpates the liver at one centimeter below the ribs. What diagnosis is likely? Atrial septal defect Coarctation of the aorta Patent ductus arteriosis Ventricular septal defect

ANS: D The symptoms above are characteristic of a VSD and may not present at birth but appear later as CHF becomes more pronounced. An ASD typically does not have a murmur until the child is 2 or 3 years old, but the provider can often hear a split S2 sound. Coarctation of the aorta may cause a systolic ejection murmur. A PDA has a characteristic machinery-like murmur.

An adolescent male comes to the clinic reporting unilateral scrotal pain, nausea, and vomiting that began that morning. The primary care pediatric nurse practitioner palpates a painful, swollen testis and elicits increased pain with slight elevation of the testis (a negative Phren's sign). What will the nurse practitioner do? Administer IM ceftriaxone and prescribe doxycycline twice daily for 10 days. Encourage bed rest, scrotal support, and ice packs to the scrotum as tolerated. Prescribe NSAIDs, limited activities, and warm compresses to the scrotum. Refer the adolescent immediately to a pediatric urologist or surgeon.

ANS: D These symptoms are indicative of testicular torsion. Testicular torsion causes a sudden onset of unilateral pain and is distinguished from epididymitis when elevation of the scrotum causes an increase in pain (Phren's sign). It is a surgical emergency and should warrant immediate referral. Epididymitis is caused by infection and requires antibiotics, bed rest, scrotal support, and ice packs. Testicular appendix torsion is self-limited and can be managed with NSAIDs, bed rest or limited activities, and warm compresses.

An adolescent who recently spent time in a hot tub while on vacation has discrete, erythematous 1- to 2-mm papules that are centered around hair follicles on the thighs, upper arms, and buttocks. How will the primary care pediatric nurse practitioner manage this condition? Culture the lesions and treat with appropriate IM antibiotics. Hospitalize for incision and drainage and intravenous antibiotics. Order an antistaphylococcal beta-lactamase-resistant antibiotic. Prescribe topical keratolytics and topical antibiotics.

ANS: D This adolescent has hot-tub folliculitis that is superficial at this point and may be treated with topical keratolytics and topical antibiotics. Culture is indicated if the lesions are resistant to treatment. IV and oral antibiotics and I&D are indicated for more severe episodes.

3. A school-age child is hit in the face with a baseball bat and reports pain in one eye. The primary care pediatric nurse practitioner is able to see a dark red fluid level between the cornea and iris on gross examination, but the child resists any exam with a light. Which action is correct? a. Administer an oral analgesic medication. b. Apply a Fox shield and reevaluate the eye in 24 hours. c. Instill anesthetic eyedrops into the affected eye. d. Refer the child immediately to an ophthalmologist.

ANS: D This child has a traumatic injury with hyphema to the eye, and an ophthalmologist must examine the eye to rule out orbital hematoma or retinal detachment. Any further attempt to examine the child may result in further injury. A Fox shield is used once more serious injury is excluded.

A school-age child is hit in the face with a baseball bat and reports pain in one eye. The primary care pediatric nurse practitioner is able to see a dark red fluid level between the cornea and iris on gross examination, but the child resists any exam with a light. Which action is correct? Administer an oral analgesic medication. Apply a Fox shield and reevaluate the eye in 24 hours. Instill anesthetic eyedrops into the affected eye. Refer the child immediately to an ophthalmologist.

ANS: D This child has a traumatic injury with hyphema to the eye, and an ophthalmologist must examine the eye to rule out orbital hematoma or retinal detachment. Any further attempt to examine the child may result in further injury. A Fox shield is used once more serious injury is excluded.

A 9-year-old girl has a history of frequent vomiting and her mother has frequent migraine headaches. The child has recently begun having more frequent and prolonged episodes accompanied by headaches. An exam reveals abnormal eye movements and mild ataxia. What is the correct action? Begin using an anti-migraine medication to prevent headaches. Prescribe ondansetron and lorazepam to help manage symptoms. Reassure the parent that this is expected with cyclic vomiting syndrome. Refer to a pediatric gastroenterologist for further workup.

ANS: D This child has an abnormal neurologic examination, which is a red flag warranting referral for further workup for children with cyclic vomiting syndrome. Anti-migraine medications are used in children over age 12 years and therefore should not be used for this patient. Ondansetron and lorazepam may be useful for unrelenting nausea and poor sleep, but this child needs to be referred to evaluate neurologic symptoms. These signs are not expected.

A 2-year-old child is brought to the clinic after developing a hoarse, bark-like cough during the night with "trouble catching his breath" according to the parent. The history reveals a 2 day history of low-grade fever and upper respiratory symptoms. On exam, the child has a respiratory rate of 40 breaths per minute, occasional stridor when crying, and a temperature of 101.3°F. What is the next step in treatment for this child? Administer intramuscular dexamethasone. Admit the child for inpatient hospitalization. Give the child a racemic epinephrine treatment in the office. Prescribe oral dexamethasone for 2 days.

ANS: D This child has croup with milder symptoms and may be managed at home with oral steroids. IM steroids are given to children who are vomiting. Inpatient admission is considered for children with respiratory distress (rates between 70 and 90 breaths per minute, severe retractions, and stridor at rest). Racemic epinephrine is useful for more severe symptoms, especially for stridor, but is done in the ED or hospital and should always be combined with a steroid.

A 4-year-old child has clusters of small, clear, tense vesicles with an erythematous base on one side of the mouth along the vermillion border, which are causing discomfort and difficulty eating. What will the primary care pediatric nurse practitioner recommend as treatment? Mupirocin ointment applied to lesions 3 times daily Oral acyclovir 20 to 40 mg/kg/dose for 7 to 10 days Topical acyclovir applied to lesions 4 times daily Topical diphenhydramine and magnesium hydroxide

ANS: D This child has lesions consistent with HSV-1. Topical anesthetics may be used for comfort and may be applied with cotton-tipped swabs. Mupirocin ointment is used for secondary bacterial infection. Oral acyclovir is used in more severe cases and most often with HSV-2. Topical acyclovir is useful for initial genital herpes infections.

A toddler is seen in the clinic after a 2-day history of intermittent vomiting and diarrhea. An assessment reveals an irritable child with dry mucous membranes, 3-second capillary refill, 2- second recoil of skin, mild tachycardia and tachypnea, and cool hands and feet. The child has had two wet diapers in the past 24 hours. What will the primary care pediatric nurse practitioner recommend? Anti-diarrheal medication and clear fluids for 24 hours Bolus of IV normal saline in the clinic until improvement Hospital admission for IV rehydration and oral fluids Oral rehydration solution with follow-up in 24 hours

ANS: D This child has mild to moderate dehydration, according to vital signs and symptoms, and may be managed with oral rehydration solution with good follow-up. Anti-diarrheal medications are generally not useful, but antiemetics may be used. It is not necessary to administer IV fluids or to hospitalize unless more severe dehydration occurs.

A child who had GABHS 2 weeks prior is in the clinic with periorbital edema, dyspnea, and elevated blood pressure. A urinalysis reveals tea-colored urine with hematuria and mild proteinuria. What will the primary care pediatric nurse practitioner do to manage this condition? Prescribe a 10- to 14-day course of high-dose amoxicillin. Prescribe high-dose steroids in consultation with a nephrologist. Reassure the parents that this condition will resolve spontaneously. Refer the child to a pediatric nephrologist for hospitalization.

ANS: D This child has symptoms of post-streptococcal glomerulonephritis and signs indicating a need for hospitalization: elevated BP, edema, and dyspnea. The PNP should refer the child to a nephrologist for hospital admission and care. Amoxicillin is not indicated; this condition is an immunologic response to GABHS and not an infection. Steroids are not effective in treating this disease. Although the condition usually does self-resolve, the child needs hospitalization for close monitoring and follow-up.

The primary care pediatric nurse practitioner is preparing to close a laceration on a child's forehead using topical skin adhesive. What is the correct way to apply this product? a. Apply the adhesive between the wound margins and then hold the edges together. b. Apply the adhesive to the wound and then secure the edges with surgical tape. c. Have the child remain still for 15 to 20 minutes after the adhesive is applied. d. Hold the wound edges together and apply the adhesive on top of the skin.

ANS: D Topical adhesive is applied by holding the wound edges together (approximating the wound edges) and then applying the adhesive on top, often requiring two or three applications of the adhesive but allowing skin cooling between applications. The adhesive should not be applied between the wound margins or in the wound. Surgical tape and bandages are not used with topical adhesive. It is not necessary for the child to remain still after the adhesive is applied, since it dries quickly.

A child has an area of inflammation on the neck that began after wearing a hand-knot woolen sweater. On examination, the skin appears chafed with mild erythematous patches. The lesions are not pruritic. What is an appropriate initial treatment? Application of a lanolin-based emollient Burow solution soaks and cool compresses Oral antihistamines given 4 times daily Topical corticosteroids applied 2 to 3 times daily

ANS: D Topical corticosteroids are useful for contact dermatitis. Lanolin-based emollients are contraindicated when inflammation is present. Burow solution soaks are useful for vesicular rashes. Oral antihistamines are not indicated unless itching and scratching occur.

A school-age child has several annular lesions on the abdomen characterized by central clearing with scaly, red borders. What is the first step in managing this condition? Fluoresce the lesions with a Wood's lamp. Obtain fungal cultures of the lesions. Perform KOH-treated scrapings of the lesion borders. Treat empirically with antifungal cream.

ANS: D Unless the diagnosis is questionable, or if treatment failure occurs, tinea corporis is treated empirically with topical antifungal creams; therefore, it is not necessary to fluoresce the lesions, culture the lesions, or complete KOH testing of scrapings as an initial management step.

A child has an acute infection causing lower airway obstruction. Which initial symptom is expected in this child? Atelectasis Barrel chest Over-inflation Wheezing

ANS: D Wheezing is the principal sound patients make if the obstruction allows enough air to pass through the narrowed lumen when lower airway obstruction occurs. Eventually, over-inflation and atelectasis occur. Barrel chest is the result of chronic over-inflation.

5. The primary care pediatric nurse practitioner applies fluorescein stain to a child's eye. When examining the eye with a cobalt blue filter light, the entire cornea appears cloudy. What does this indicate? a. The cornea has not been damaged. b. There is too little stain on the cornea. c. There is damage to the cornea. d. There is too much stain on the cornea.

ANS: D When fluorescein stain is applied and the entire cornea appears cloudy, it means that there is too much of the stain. Damaged areas of the cornea should appear greenish after staining with fluorescein dye.

The primary care pediatric nurse practitioner applies fluorescein stain to a child's eye. When examining the eye with a cobalt blue filter light, the entire cornea appears cloudy. What does this indicate? The cornea has not been damaged. There is too little stain on the cornea. There is damage to the cornea. There is too much stain on the cornea.

ANS: D When fluorescein stain is applied and the entire cornea appears cloudy, it means that there is too much of the stain. Damaged areas of the cornea should appear greenish after staining with fluorescein dye.

The parent of an infant asks about using a probiotic medication. What will the primary care pediatric nurse practitioner tell this parent? Probiotic medications have demonstrated efficacy in treating colic. Probiotics are not safe to use to treat infants who have colic. There are no studies showing usefulness of probiotics to manage colic. There is no conclusive evidence about using probiotics to treat colic.

ANS: D While small studies have shown promise in treating colic in infants, the research is contradictory and there is no conclusive evidence about effectiveness. There is no evidence that probiotics are not safe.

12. A preschool-age child who attends day care has a 2-day history of matted eyelids in the morning and burning and itching of the eyes. The primary care pediatric nurse practitioner notes yellow-green purulent discharge from both eyes, conjunctival erythema, and mild URI symptoms. Which action is correct? a. Culture the conjunctival discharge. b. Observe the child for several days. c. Order an oral antibiotic medication. d. Prescribe topical antibiotic drops.

ANS: D Young children with bacterial conjunctivitis may be treated with topical antibiotic drops. Culturing the eyes is not necessary unless there is no improvement. While most cases of bacterial conjunctivitis are self-limiting, using a topical antibiotic will hasten the return to day care. Oral antibiotics are not indicated.

A preschool-age child who attends day care has a 2-day history of matted eyelids in the morning and burning and itching of the eyes. The primary care pediatric nurse practitioner notes yellow-green purulent discharge from both eyes, conjunctival erythema, and mild URI symptoms. Which action is correct? Culture the conjunctival discharge. Observe the child for several days. Order an oral antibiotic medication. Prescribe topical antibiotic drops.

ANS: D Young children with bacterial conjunctivitis may be treated with topical antibiotic drops. Culturing the eyes is not necessary unless there is no improvement. While most cases of bacterial conjunctivitis are self-limiting, using a topical antibiotic will hasten the return to day care. Oral antibiotics are not indicated.

During a well child examination of a 6-year-old girl, the primary care pediatric nurse practitioner notes that the child becomes embarrassed and resists taking off her underwear for the exam. What should the nurse practitioner infer from this observation? The child has been sexually molested. The child is feeling violated by the examiner. The parent is exhibiting regressive behavior. This is a normal reaction in a child of this age.

ANS: D Young school-age children can be extremely modest and embarrassed and resist taking off their clothes for an examiner. Since this is normal, it does not indicate a history of sexual abuse unless other signs are present. Older school-age children more commonly feel violated during an exam, not younger children. This response of increased modesty is age-appropriate and not regressive.

The NP is examining a school-age child who complains of frequent stomach pain and headaches. The parent reports that the child misses several days of school each month. The child has a normal exam. Before preceding with further diagnostic tests, what will the NP initially ask the parent?

About the timing of the symptoms each day and during the week.

14. A preschoolage child is seen in the clinic after waking up a temperature of 102.2°F, swelling and erythema of the upper lid of one eye, and moderate pain when looking from side to side. Which course of treatment is correct?

Admit to the hospital for intravenous antibiotics.

The mother of a 16 year old male was recently divorced after several years of an abusive relationship and tells the NP that the adolescent has begun skipping school and hanging out with friends at the local shopping mall. When she confronts her child, he responds by saying that he hates her. What will the NP tell this mother?

Adolescents often need counseling to help them cope with life events

The primary care pediatric nurse practitioner is counseling the parents of a toddler about appropriate discipline. The parents report that the child is very active and curious, and they are worried about the potential for injury. What will the pediatric nurse practitioner recommend?

Allow the child to explore and experiment whale providing appropriate limits.

Which group is at highest risk for type 2 diabetes? Non hispanic whites African americans American Indians Hispanics

American Indians

A pre-school age child has honey-crusted lesions on erythematous, eroded skin around the nose and mouth, with satellite lesions on the arms and legs. The child's parent has several similar lesions and reports that other children in the day care have similar rash. How will his be treated?

Amoxicillin-clavulanate 90 mg/kg/day for 10 days

A 3yearold child has had one episode of acute otitis media 3 weeks prior with a normal tympanogram just after treatment with amoxicillin. In the clinic today, the child has a type B tympanogram, a temperature of 102.5°F, and a bulging tympanic membrane. What will the primary care pediatric nurse practitioner order?

Amoxicillinclavulanate twice daily

An 18monthold child with no previous history of otitis media awoke during the night with right ear pain. The primary care pediatric nurse practitioner notes an axillary temperature of 100.5°F and an erythematous, bulging tympanic membrane. A tympanogram reveals of peak of +150 mm H2O. What is the recommended treatment for this child?

An analgesic medication and watchful waiting

The primary care pediatric nurse practitioner diagnoses acute otitis media in a 2 year old child who has a history of three ear infections in the first 6 months of life. The child's tympanic membrane is intact and the child has a temperature of 101.5°F. What will the nurse practitioner prescribe for this child?

An analgesic medication and watchful waiting

3. On initial exam of a child with newly diagnosed Kawasaki disease, the APRN would expect: 1. Dry, swollen, fissured lips. 2. Non-palpable lymph nodes. 3. Conjunctivitis with exudates. 4. Cyanosis of the hands and feet

Answer: Dry, swollen, fissured lips are symptoms of Kawasaki disease. (Lymph nodes can be palpable, conjunctivitis is present but without exudates, and hands and feet are typically erythematous.)

A child who has psorasis, who has been using a moderate-potency topical steroid on the thick plaques on the extremities and a high-potency topical steroid on more sever plaques on the elbows and knees continues to have worsening of plaques. In consultation with a dermatologist, which treatment will be added?

Anthralin ointment on high strength applied for 10 to 30 minutes daily.

A 12yearold child is brought to the clinic with joint pain, a 3week history of low grade fever, and a facial rash. The primary care pediatric nurse practitioner palpates an enlarged liver 2 cm below the subcostal margin along with diffuse lymphadenopathy. An ANA test is positive. Which test may be ordered to confirm a diagnosis of SLE?

Antidoublestrand DNA antibodies

The mother of a 15 year old adolescent female tells the NP that her daughter has extreme mood swings prior to her periods, which the adolescent vehemently denies. When asked if she notices anything different just before her periods, the adolescent points to her mother and says, "She gets really hard to live with." This demonstrates which characteristic of adolescent thinking?

Apparent hypocrisy

A child will need an occlusive dressing to treat lichen simplex chronicus. What will the NP tell the parents about applying this treatment?

Apply ointment before the dressing.

The NP is preparing to conduct a well child assessment of an 8 year old child. How will the NP begin the exam?

Ask the child about school, friends, home activities, and sports.

During a well child exam of a school-age child, the NP learns that the child has been having angry episodes at school. The NP observes the child to appear withdrawn and sad. Which action is appropriate?

Ask the child and the parent about stressors at home.

The primary care pediatric nurse practitioner is evaluating a 2 year old with a documented speech delay. Screenings to assess motor skills and cognition are normal and the child passed a recent hearing test. What will the pediatric nurse practitioner do next?

Ask the child's parents whether they read to the child.

The primary care pediatric nurse practitioner examines an infant whose weight is below the 3rd percentile and whose mother does not comply with feeding regimen. When attempting to enlist the help of the infant's grandmother, the grandmother says, "My daughter was like this when she was a baby and she turned out all right," Which approach will the nurse practitioner take to improve the outcome for this infant?

Ask the grandmother about her daughter's health during childhood

The pediatric nurse practitioner provides primary care for a special needs infant whose parents takes an active role in the infant's care. The parent has a high school diploma and asks many questions about her infant's treatments. Which approach will the nurse practitioner take to ensure health literacy for the parent?

Ask the parent to read back all the information given.

The primary care pediatric nurse practitioner sees a child for followup care after hospitalization for ARF. The child has polyarthritis but no cardiac involvement. What will the nurse practitioner teach the family about ongoing care for this child?

Aspirin is given for 2 weeks and then tapered to discontinue the medication.

The primary care pediatric nurse practitioner performs a well child examination on a 9 month old infant who has a history of prematurity at 28 weeks' gestation. The infant was treated for retinopathy of prematurity (ROP) and all symptoms have resolved. When will the infant need an ophthalmologic exam?

At 12 months of age

The mother of a 6 month old infant is distressed because the infant can say "dada" but not "mama" and asks the NP why this is when she is the one who spends more time with the infant. How will the NP respond?

At this age, your baby does not understand the meaning of sounds.

Signs of insulin resistance include, circle all that apply: Acanthosis nigricans PCOS- acne/hirsutism HTN Athletes feet Dyslipidemia

Athletes feet

A 10-year-old boy comes to your clinic for evaluation of a suspected urinary tract infection (UTI). Which of the following signs would lead you to include diagnoses other than UTI in the differential? A. Increased frequency B. Penile discharge C. Costovertebral tenderness D. Dysuria

B

A 12-year-old girl presents to your clinic with symptoms of vaginitis, including older, dysuria, frequency, and discomfort. Which of the following causes of vaginitis is most likely due to sexual transmission? A. Candida B. Chlamydia C. Pinworms D. Gardnerella

B

A 17-year-old, sexually active girl with chancres in the genital area is noted to have a positive venereal disease research laboratory test. The next step should be to: A. Treat w/ ceftriaxone B. Perform a specific treponemal anti-body test C. Culture for c. Trachomatis D. Discuss saber sex practices

B

A four-year-old child with a history of myelomeningocele and ventriculoperitonesl shunt presents to the clinic with a headache, nausea, vomiting, and lethargy. The most probable diagnosis is: A: Viral gastroenteritis B: Shunt malfunction C. Meningitis D: Shunt infection

B

A school-age child has begun refusing all cooked veggies. What will you recommend to the parent? a. Allow the child to make food choices since this is usually a phase. b. Ensure that the child has 3 nutritious meals and 2 nutritious snacks each day. c. Prep veggies separately for the child to encourage adequate intake. d. Teach the child how important it is to eat healthy fruits/veggies.

B

A two-year-old girl has lymphedema of the hands and foot, with a lower posterior hairline, cubitus valgus, and a history of intrauterine growth retardation. Which of the following defects is most common among the children with this genetic syndrome? A. Aortic valve stenosis B. Coarctation of the aorta C. Mitral valve prolapse D. Dissecting aortic aneurysm

B

According to the consensus model for APRN regulation, which of the following is most consistent with the definition of advanced practice registered nursing? A. Optional national certification B. A significant component of education and practice focuses on direct care of individuals C. Accountability excludes prescription pharmacological and nonpharmacological interventions D. Level of rolled autonomy is unchanged

B

H.O. is a 5 yo Vietnamese child who has fallen off of his growth curve. The best intervention would be to: a. Suggest high-calorie breakfast drinks as supplements b. incorporate traditional foods into a management plan that will provide increased calories and nutrients c. educate the family on the need for increased calories and nutrients d. refer the family to growth clinic for evaluation

B

In most states, a learning disability is defined based on: A. The child's IQ as determined by a psychological evaluation B. A discrepancy between the child's actual and expected achievement C. A diagnosis of attention deficits D. Achievement test scores

B

Infants with IUGR are prone to hypoglycemia primarily because they: A. Have a decreased metabolic rate B. Have little glucose stores in the form of glycogen and fats C. Become acidotic D. Are prone to sepsis

B

Mr. Harris calls the pediatric clinic to say that Josh, his 2-year-old son, has tripped on the sidewalk and hit his head on concrete.Which of the following symptoms reported by Mr. Harris would require that Josh be seen in the emergency room? A. Uncontrollable crying B. Loss of consciousness C. Scalp laceration D. Hx of febrile sz

B

The PNP is teaching high school students about prevention of iron deficiency anemia. To teach prevention of the most common cause of iron deficiency in this age group, it is important to emphasize: A. Avoidance of all aspirin containing products B. A Diet high in iron rich foods C. Avoidance of ingestion of carbonated beverages D. Avoidance of high fiber foods

B

Treatment of true (central) precocious puberty is best achieved with: A. Synthetic follicular stimulating hormone B. Gonadotropin releasing hormone C. Dexamethasone D. Thyroid hormone

B

Which chromosomal abnormality is associated with short stature and girls? A. Down syndrome B. Turner syndrome C. Klinefelter's syndrome D. Prader-Willi syndrome

B

Which of the following is found more commonly in children with a meningomyelocele? A. Congenital heart disease B. Latex allergy C. Intestinal malformations D. Cleft palate

B

Which of the following is the purpose of certification? A. To review and approve educational degree programs in nursing B. To assure the public that an individual has mastered a body of knowledge and acquired skills in a particular body of knowledge C. All of the above D. None of the above

B

A parent reports that his 7-year-old child is very short compared to his peers, and his teeth came in slowly when he was younger. After searching the Internet the parent is worried about growth hormone deficiency (GHD). Which response by the nurse is the most appropriate? A. "GHD is the only cause of the symptoms you describe." B. "Has your child ever had his thyroid evaluated?" C. "I will refer your child to an endocrinologist right away." D. "What did the dentist say about his teeth being delayed?"

B Although those manifestations do sound like GHD, they could also be indicative of hypothyroidism. The nurse asks if the child's thyroid function has ever been checked. The nurse cannot refer to an endocrinologist without a provider order. Asking about the dentist's opinion may be important, but it is not as specific for the problem as asking about thyroid function testing.

A child has just been diagnosed with juvenile arthritis (JA). The parents want to know what caused this to happen. Which statement by the nurse is the most appropriate? A. "Genetic abnormalities are triggered by infection." B. "It seems to be an autoimmune disease." C. "Latent infections can recur and cause JA." D. "No one really understands how JA occurs."

B JA is an autoimmune, inflammatory process often thought to be triggered by an infection. The etiology is not genetic, caused by latent infections, or completely unknown.

The nurse gave a diabetic child an injection of Humalog insulin at 0700. At what time would it be most important to check on the child? A. 0715 B. 0800 C. 0930 D. 1200

B The peak action of humalog insulin is in 55 minutes, so the most appropriate time to check on the child is at 0800.

An 18-year-old college freshman is seen in the student clinic with a complaint of migraine headaches. In collecting the history, you would expect him to say he experiences pain: A. Throughout his head without any localization B. Restricted to one side of his head during an episode C. Mostly in the occipital area D. Across his head from one temporal side to another

B (After puberty, migraine headache pain is restricted to one side of the head but may switch sides from one headache episode to another)

Dan is a healthy 17-year-old student. He's thinking about having sex, but asks for more information about condoms. You give him all of the following advice except: A. Male condoms are the more effective barrier method B. Latex condoms are the most recommended and are heat tolerant C. Condoms have a theoretical failure rate of 3% D. It is only safe to use water-based lubricant with latex condoms

B (heat can cause latex condoms to break down)

A nine month old child of Jewish parents presents with increasing irritability and noise sensitivity. What is the next best step? A. Encourage mom to decrease environmental stimuli B. Refer to a pediatrician C. Refer for further developmental screening D. Reevaluate at the 12 month examination

B (tay-Sachs disease is seen in families of Jewish descent and is characterized by degenerative CNS signs and hyperreaction to noise. A consultation with the pediatrician is indicated due to the possibility of Tay-Sachs disease)

Which of the following do not meet the criteria for a selective screening for hyperlipidemia? A) A 16-year-old adolescent who is adopted, who smokes, and who has no known family history B) An 8-year-old child with a small restrictive ventricular septal defect C) A 10-year-old child whose father had a balloon angioplasty at age 49 years D) A 12-year-old child with BMI of 28

B) An 8-year-old child with a small restrictive ventricular septal defect

A 15-year-old girl is being evaluated for possible delayed puberty. There is a documented history of Tanner stage 2 breast development and Tanner stage 1 pubic development at age 9.5 years. What current findings would support the diagnosis of delayed puberty? A) Family history of abnormal puberty B) Palpable breast buds with areolar enlargement C) Dark, coarse, curly pubic hair spreading over the mons D) Weight at 5th percentile

B) Palpable breast buds with areolar enlargement

The nurse is teaching parents of an infant diagnosed with hypothyroidism. Which items are appropriate to include in the teaching plan? (Select all that apply.) A. Dissolving the levothyroxine (Synthroid) completely in formula B. Frequent monitoring of the child's height and weight C. Keeping a log or diary of developmental milestones the child meets D. Monitoring the child for behavioral changes as he or she grows E. The need for frequent laboratory testing during the child's life

B, C, D, E The parents of an infant diagnosed with hypothyroidism need to maintain logs of the child's height, weight, and developmental milestones to help ensure his or her growth and development stays within normal ranges. The child will need frequent blood tests, and the parents can also monitor for behavioral changes that might indicate a need to increase the medication as the child grows. For an infant, the medication is crushed and dissolved in a small amount of liquid and administered via a syringe. It is not added to a bottle of formula.

8. The primary care pediatric nurse practitioner is counseling a parent about bicycle helmet use. The parent reports having a helmet used a year previously by an older child and wonders about using it for a younger child since they are so expensive. What will the nurse practitioner tell the parent? A. "As long as the helmet does not have cracks, you may use it." B. "If the helmet is free from marks, you may use it." C. "You may continue to use a helmet up to 10 years." D. "You should always purchase a new helmet for each child."

B. "If the helmet is free from marks, you may use it."

11. The primary care pediatric nurse practitioner is evaluating a heart murmur during a preparticipation examination of a high school athlete. Which finding would be aconcern requiring referral to a cardiologist? A. A murmur that is louder when squatting and softer when standing B. A murmur that is quieter when squatting and louder with a Valsalva maneuver C. A murmur with narrow and variable splitting of S2 D. A systolic murmur that is grade 1 or 2

B. A murmur that is quieter when squatting and louder with a Valsalva maneuver

3. The primary care pediatric nurse practitioner is performing a well child exam on a 24monthold child. The parent tells the nurse practitioner that the child is being toilet trained and expresses frustration that on some days the child uses the toilet every time and on other days not at all. What will the nurse practitioner do? A. Advise the parent to make the child get clean clothes after an accident. B. Ask the parent about the child's toilet habits and understanding of toilet training. C. Recommend using an awards system to encourage toilet use. D. Suggest that the parent place the child on the toilet at predictable intervals.

B. Ask the parent about the child's toilet habits and understanding of toilet training.

2. The parent of a child who has asthma asks the primary care pediatric nurse practitioner about whether the child may engage in strenuous exercise. What will the nurse practitioner tell the parent? A. Children with asthma should be excluded from vigorous exercise and most strenuous sports. B. Children with asthma show improved aerobic and anaerobic fitness with moderate to vigorous/physical activity. C. Physical activity has been shown to improve overall pulmonary function in children with asthma. D. Vigorous exercise helps improve symptoms in children with poorly controlled asthma.

B. Children with asthma show improved aerobic and anaerobic fitness with moderate to vigorous/physical activity.

17. A 10yearold is hit in the head with a baseball during practice and is diagnosed with concussion, even though no loss of consciousness occurred. The primary care pediatric nurse practitioner is evaluating the child 2 weeks after the injury and learns that the child is still experiencing some sleepiness every day. The neurological exam is normal. The child and the parent are adamant that the child be allowed to return to play baseball. What will the nurse practitioner recommend? A. Continuation of cognitive rest only B. Continuation of physical and cognitive rest C. Continuation of physical rest only D. Returning to play

B. Continuation of physical and cognitive rest

1. The primary care pediatric nurse practitioner provides anticipatory guidance for a 6monthold infant who is breastfed who takes 400 IU of vitamin D daily. The parent reports that the infant has begun taking cereals, fruits, and vegetables in addition to nursing. What will the nurse practitioner recommend to promote healthy nutrition? A. Begin supplementing with iron. B. Continue to nurse as long as desired. C. Discontinue the vitamin D supplement. D. Stop breastfeeding at 1 year of age.

B. Continue to nurse as long as desired.

19. The primary care pediatric nurse practitioner is examining a 17yearold male who is on his high school swim team. The adolescent is concerned about "lumps" on his chest. The nurse practitioner notes a marked increase in weight since the last visit along with worsening of the adolescent's acne. Given this set of symptoms, which performance enhancing substance will the nurse practitioner be most concerned about and ask about? A. Creatine B. Dehydroepiandrosterone (DHEA) C. Ephedra D. Growth hormone

B. Dehydroepiandrosterone (DHEA)

6. The parents of a prepubertal female who is on the local swim team tell the primary care pediatric nurse practitioner that their daughter wants to begin a strength training program to help improve her swimming ability. What will the nurse practitioner recommend? A. Avoiding strength training programs until after puberty to minimize the risk for injury B. Enrolling their daughter in a program that uses fixed weight machines or resistance bands C. Having their daughter participate in weight training 4 or 5 times each week for maximum effect D. Making sure that their daughter begins with the greatest weight tolerable using lower repetitions

B. Enrolling their daughter in a program that uses fixed weight machines or resistance bands

8. A schoolage child has begun refusing all cooked vegetables. What will the primary care pediatric nurse practitioner recommend to the parent? A. Allow the child to make food choices since this is usually a phase B. Ensure that the child has three nutritious meals and two nutritious snacks each day C. Prepare vegetables separately for the child to encourage adequate intake D. Teach the child how important it is to eat healthy fruits and vegetables

B. Ensure that the child has three nutritious meals and two nutritious snacks each day

5. The primary care pediatric nurse practitioner is performing a well child exam on a 12yearold female who has achieved early sexual maturation. The mother reports that she spends more time with her older sister's friends instead of her own classmates. What will the nurse practitioner tell this parent? A. Earlymaturing girls need to identify with older adolescents to feel a sense of belonging. B. Girls who join an older group of peers may become sexually active at an earlier age. C. Spending time with older adolescents indicates a healthy adjustment to her maturing body. D. The association with older adolescents will help her daughter to gain social maturity.

B. Girls who join an older group of peers may become sexually active at an earlier age.

6. The mother of a 3monthold child tells the primary care pediatric nurse practitioner that it is "so much fun" now that her infant coos and smiles and wants to play. What is important for the nurse practitioner to teach this mother? A. Appropriate ways to stimulate and entertain the infant B. How to read the infant's cues for over stimulation C. The importance of scheduling "play dates" with other infants D. To provide musical toys to engage the infant

B. How to read the infant's cues for over stimulation

9. The primary care pediatric nurse practitioner is performing a well child examination on a 15yearold girl who consumes a vegan diet. Based on this assessment, which nutrients may this adolescent need to supplement? A. Calcium, vitamin C, and vitamin A B. Iron, folic acid, and B12 C. Magnesium, vitamin E, and zinc D. Vitamin D, vitamin C, and phosphorus

B. Iron, folic acid, and B12

10. The parents of a 3yearold child are concerned that the child has begun refusing usual foods and wants to eat mashed potatoes and chicken strips at every meal and snack. The child's rate of weight has slowed, but the child remains at the same percentile for weight on a growth chart. What will the primary care pediatric nurse practitioner tell the parents to do? A. Allow the child to choose foods for meals to improve caloric intake. B. Place a variety of nutritious foods on the child's plate at each meal. C. Prepare mashed potatoes and chicken strips for the child at mealtimes. D. Suggest cutting out snacks to improve the child's appetite at mealtimes.

B. Place a variety of nutritious foods on the child's plate at each meal.

2. The primary care pediatric nurse practitioner is performing a well child exam on a 12monthold infant. The parent tells the nurse practitioner that the infant has predictable bowel and bladder habits and asks about toilet training. What will the nurse practitioner tell this parent? A. It is too early to begin introducing the child to the toilet, and the parent should wait until the child is at least 2 years old. B. Placing the child on a "potty" chair helps the child associate elimination cues with the toilet. C. Predictability of elimination patterns indicates readiness for toilet training, and the parent can begin this process. D. The parent should wait until other signs of toilet training readiness occur before introducing the child to the toilet.

B. Placing the child on a "potty" chair helps the child associate elimination cues with the toilet.

9. The primary care pediatric nurse practitioner is performing a well child examination on a 9monthold infant whose hearing is normal but who responds to verbal cues with only single syllable vocalizations. What will the nurse practitioner recommend to the parents to improve speech and language skills in this infant? A. Provide educational videos that focus on language. B. Read simple board books to the infant at bedtime. C. Sing to the child and play lullabies in the baby's room. D. Turn the television to Sesame Streetduring the day.

B. Read simple board books to the infant at bedtime.

10. The primary care pediatric nurse practitioner is evaluating a schoolage child who, after removal of a pituitary tumor, has altered hypothalamic control over hunger and satiety. The child is morbidly obese and expresses feeling depressed because of the obesity. What will the nurse practitioner recommend? A. Developing a system to reward compliance with a dietary regimen B. Restricting all access to food in the house and at school C. Suggesting an afterschool exercise program to help with weight loss D. Using a food diary to track all calories and food intake

B. Restricting all access to food in the house and at school

1. The primary care pediatric nurse practitioner is performing a well child assessment on a 13yearold female whose mother asks when her daughter's periods may start. Which information will the nurse practitioner use to help estimate the onset of periods? A. The age of the mother's menarche B. The patient's age at thelarche C. When adrenarche occurred D. Whether linear growth has stopped

B. The patient's age at thelarche

4. Which recommendation will a primary care pediatric nurse practitioner make when parents ask about ways to discipline their 3yearold child who draws on the walls with crayons? A.Give the child washable markers so the drawings can be removed easily. B.Provide a roll of paper for drawing and teach the child to use this. C.Put the child in "timeout" each time the child draws on the walls. D.Take the crayons away from the child to prevent the behavior.

B.Provide a roll of paper for drawing and teach the child to use this.

6. A 16yearold female reports dull, achy cramping pain in her lower abdomen lasting 2 or 3 hours that occurs between her menstrual periods each month. The adolescent is not sexually active. What is the treatment for this condition? a. Abdominal ultrasound to rule out ovarian cyst b. Oral contraceptives to suppress ovulation c. Prostaglandin inhibitor analgesics and a heating pad d. Referral to a pediatric gynecologist

C

A 12 month old, whose parents have a history of noncompliance for routine care, presents to the clinic with fever, irritability, and nuchal rigidity. The most critical diagnostic step in the child with suspected meningitis is: A. The hx B. The physical exam C. Evaluation of the CSF D. blood cx

C

A 15yo girl, who had menarche at 13, complains of monthly menstrual pain on the first day of her periods for 1 yr. she has never been sexually active. Which of the following is not a characteristic of dysmenorrhea? A. Increase production of uterine prostaglandins Be. Pain usually starts within hours of menstrual flow or may proceed followed by up to two days C. Anovulatory cycles D. Crampy, spasmodic pain in the lower abdominal area, which might radiate to inner thighs

C

A PNP may obtain certification from the PNCB and which other association? A. American college of nurse practitioners B. American Academy of nurse practitioners C. American nurses credentialing center D. American nurses Association

C

A child in your clinic is being evaluated for short stature. Pertinent findings include delayed bone age, delayed onset of puberty, and a stature that is normal for the child's bone age. In addition, the mother states that the child's father grew taller in college and wonders if this will happen with their son. The most likely cause of these findings is: A. Familial short stature B. Chromosomal abnormality C. Constitutional delay of growth and puberty D. Endocrine abnormality

C

A child presents with a flattened philtrum, then upper lip, micrognathia, strabismus, and a ventricular septal defect. What is the most likely diagnosis? A. Congenital toxoplasmosis B. Congenital syphilis C. Fetal alcohol syndrome D. Edwards syndrome

C

A six-year-old child has +4 reflexes, toe walking, and a clumsy gate. Which of the following diagnostic testing would be helpful in making a diagnosis of cerebral palsy? A. Skull x-ray B. CT of the head C. MRI of the head D. MRI of the lumbar sacral spine

C

A social worker brings a child in prior to placement with a chief complaint of copious vaginal discharge. A culture is positive for chlamydia. From the social perspective what is the next best step? A. Reassure the social worker that this is a prenatal transmission B. Tell the social worker the results and allow her to decide the next step C. Tell the social worker the result and inform her of the likelihood of sexual abuse D. Call the foster family with the result

C

Amanda is a six month old infant recovering from a viral illness. At the peak of her illness two days ago, she spiked a temperature of 104 and experienced a febrile seizure. Amanda's mother is young and Inexperienced, but is very open to teaching. She wants to know if the seizure will do anything to Amanda. While teaching this new mother about simple febrile seizures , it would be accurate to say that Amanda: A: is at increased risk for epilepsy as an adult B: Would benefit from Phenytyoin prophylaxis C: May experience repeated seizures D. Would benefit from phenobarbital prophylaxis

C

An apparently healthy child, who is enrolled in Headstart, is suspected of having developmental delays based on Denver II results at two separate clinic visits. The most appropriate next step would be to: A: Request developmental Evaluation from the head start program B: Repeat the Denver II in six months C: Refer the child for a more definitive evaluation D. Discussed ways in which parents can foster the child's development

C

Carol is a 16-year-old adolescent who has been sexually active with a male partner for six months. They have use condoms consistently. Cara is interested in a hormonal method as well. Your advice includes all of the following except: A. Combined oral contraceptives prevent ovulation B. Trans dermal contraceptive patches must be correctly applied and the site rotated C. The vaginal contraceptive ring must be changed weekly D. Long-acting progestins might cause weight gain

C

During a well child exam on a 5yo, you assess the child for school readiness. Which finding may be a factor in limiting school readiness for this child? a. Adherence to daily family routines and regular activities. b. Having 2 older siblings who attend the same school. c. Parental concerns about bullying in the school d. The child's ability to recognize four different colors.

C

During the first well baby visit of Joshua, two weeks old, his mother says that she is concerned because his penis looks different from his three-year-old brothers penis. During the physical exam, you notice that the baby scrotum is hyperpigmented. The PNP knows that the most common cause of ambiguous genitalia is: A. Idiopathic B. A chromosomal defect C. Congenital adrenal hyperplasia D. An embryologic disorder

C

For children with diabetes, in addition to home monitoring of blood glucose and urine ketone levels, glycosylated hemoglobin (Hgb A1c) should be measured every: A. 1 wk B. 1 mos C. 3 mos D. 6 mos

C

In documenting quality care, patient satisfaction is considered to be what type of measure? A. Structural B. Process C. Outcome D. Cost containment

C

Michael is a 15-year-old high school student who presents for a school sports physical. He appears to be in good health, but is concerned about a bad headache he had a few weeks ago. He is concerned because his mother's friend died of a brain tumor. You tell Michael that the most common type of headache with onset in adolescence is: A. Sinus headache B. Vascular headache C. Tension headache D. Migraine headache

C

One month after prescribing iron therapy to treat iron deficiency in a child who has no other known health problems, the PNP should: A. Teach the parent or guardian about iron rich foods B. Check the child's stool for occult blood C. Order a hemoglobin measurements D. Order a complete blood count

C

Political activism through lobbying is an example of what type of change strategy? A. Normative reeducative B. Empirical rational C. Power-coercive D. Confrontational

C

Prescriptive writing privileges: A. Require a formal collaborative relationship with a physician B. Are regulated by both boards of nursing and medicine C. Very according to state statutes D. Are granted upon certification

C

The early and periodic screening, diagnostic, and treatment (EPSDT) program describes a comprehensive set of healthcare services as provided by: A. Private insurance B. State child protection teams C. Medicaid D. Most managed care organizations

C

The granting of authority to practice is known as: A. Credentialing B. Accreditation C. Licensure D. Regulation

C

The most common bacterial cause of a sexually transmitted infection is: A. Neisseria gonorrhoeae B. Trichinomas vaginalis C. Chlamydia trachomatis D. Herpes simplex

C

The parent of a 10yo boy tells you that the child doesn't appear to have any interest in girls and spends most of his time with a couple of other boys. The parent is worried about the child's sexual identity. You will tell the parent: a. Children at this age who prefer interactions w/same-gender peers usually have a homosexual orientation. b. Children experiment w/sexuality at this age as a means of deciding later sexual orientation. c. This attachment to other same-gender children is how the child learns to interact w/others. d. To encourage mixed-gender interactions in order to promote development of sexual values.

C

The parent of a 5yo who has just begun kindergarten expresses concern that the child will have difficulty adjusting to the birth of a sibling. What will you recommend? a. Allowing the child opportunities to discuss feelings about the baby. b. Giving the child specific baby care tasks to promote sibling bonding. c. Having snack time w/the child each day to discuss the school day. d. Providing reassurance that the sibling will not replace the child.

C

The parent of a 6yo expresses concern that the child may have ADHD. Which screening tool will you use to eval this possibility? a. Behavioral and Emotional Screening System for Children (BESS-2) b. Behavioral Assessment for Children - 2nd ed. (BASC-2) c. Conner's 3 Parent and Teacher Rating Scale d. Pediatric Symptom Checklist (PSC)

C

Tonya, at five years of age, has sickle cell disease. Tonya's mother has brought her to the clinic because she has had fever of 101 for the past two days and her appetite has been poor. Physical exam reveals no apparent cause for the fever. Appropriate treatment includes: A. Acetaminophen for fever and reevaluating Tonya in 24 hours B. Ibuprofen for fever and reevaluating Tonya and 24 hours C. Inpatient or outpatient antibiotic therapy D. No treatment unless the fever is above 102

C

Which anticipatory guidance would be helpful in decreasing the risk of toxoplasmosis? A. When camping, treat water from streams with iodine B. Do not go barefoot in high-risk areas C. Wash hands after changing cat litter D. Avoid handling contaminated diapers

C

Which method might be used to assess the vision of a 1 mos old child? a. check the vessel pattern of the fundus of the eye b. watch to see if the infant turns his or her head toward you when you speak c. observe the pattern of interaction with the mother d. perform the titmus test on the infant

C

Which of the following physical exam findings is a major criterion for identifying a patient with Marfan syndrome? A. Freshfield spots B. Shield test with widely spaced nipples C. Pectus excavatum requiring surgery D. Large testicles

C

While examining a four-month-old boy you are unable to palpate one of the testes. The next most appropriate step is to: A. Reassure the parents that this is a normal finding B. Refer the child to an endocrinologist C. Reexamine the baby in two months D. Refer the child to a urologist

C

While taking the history of 6-month old E.M. you learn that she is not sleeping through the night and will not fall back to sleep without the parents rocking or feeding her. This is an example of: a. somnambulism b. pavor nocturnus c. learned behavior d. delayed sleep phase

C

You are examining a 6yo who attends first grade. The child reports "hating" school. The parent states that the child pretends to be sick frequently in order to stay home from school. To further assess this situation, you will first ask the child: a. about school performance and grades. b. why school is so distressing. c. to name one or two friends. d. whether bullying is taking place.

C

The parent of a child with hypopituitarism asks why this condition occurs. Which response by the nurse is the most appropriate? A. "Infection in the brain is the most common cause." B. "It's usually a brain tumor near the pituitary gland." C. "There are many possible causes, and often we don't know." D. "Usually this is due to damage to the brain from trauma."

C The cause of hypopituitarism is often unknown, but there are many possibilities. CNS tumor is the most common, accounting for 47%. Trauma accounts for about 3% of the cases of this condition. Infection is an uncommon cause (1%).

Which long-term goal is most appropriate for the adolescent diabetic patient? A. Appropriate food items chosen for meals B. Glucose within target range 90% of the time C. No evidence of long-term complications D. Takes medications correctly and on time

C The most appropriate long-term goal is that the teen is free from complications, some of which can be devastating. Choosing appropriate foods, blood glucose readings, and taking medications are good goals too, but are too specific for an overall long-term goal.

It is important to look for evidence of a preceding streptococcal infection when ruling out acute nephritis. This is best done by: A. Throat culture B. Skim exam C. ASO titer D. ESR

C (ASO a tighter is elevated and 80% of patients with a preceding streptococcal infection)

Which of the following is not true about the use of Imipramine for the treatment of enuresis? A. It is not generally recommended for this nonfatal disorder due to the potentially lethal side effects B. The lasting cure rate is approximately 17% C. Imipramine should only be given to children who have had a baseline EEG D. The most appropriate imipramine treatment group is adolescent boys with both ADHD and persistent nocturnal enuresis

C (Use is controversial due to dangerous side effects and it's relative ineffectiveness, and EEG is not recommended prior to initiating treatment)

You examine CJ in the newborn nursery and know what a deformity in her left foot consisting of a convex lateral border and a forefoot, which can be abducted past and imaginary line extending from the middle of the heel through the second toe. Which of the following management strategies is most appropriate? A: reverse last shoes B: Out flare shoes C: Stretching exercises D: Orthopedic referral

C (metatarsus adductus Is a Flexural deformity of the foot related most commonly to intrauterine positioning; Flexural deformities, that is, movement past the midline, can be managed with stretching exercises)

You receive the results of newborn screening and find that the TSH, done at day two, is 82. What is your best option? A. Have the child come to the clinic next week for a reevaluation B. Rescreen the child in one month C. Begin thyroid supplementation immediately D. Reassure the family that these are normal results

C (tsh >20)

An 11-year-old girl is brought to the office for an annual well-child visit. When discussing the onset of puberty with the preadolescent and the mother, which information would the PNP provide? A) "Pubic hair develops before breast buds." B) "Girls have their greatest linear growth after menses." C) "The average time from breast buds to menarche is 1.5 to 2.5 years." D) "The average age of onset for menarche is 10 to 12 years."

C) "The average time from breast buds to menarche is 1.5 to 2.5 years."

The PNP is performing a well-child examination on an 8-year-old child with a previous diagnosis of precocious puberty. The patient is Tanner stage III for breast development and pubic hair growth, with height and weight in the 95th percentiles. The patient has missed 6 months of her hormone therapy. The mother states that the child often complains of abdominal cramping, and the mother has noticed an increased vaginal discharge. The remainder of the examination is normal. Addressing the abdominal pain should include: A) Evaluation for a UTI B) Guidance regarding the appropriate dietary intake of fruits, fiber, and water to prevent constipation C) Anticipatory guidance regarding the importance of complying with drug therapy and the likelihood of the onset of menses D) A referral to a gastroenterologist for evaluation

C) Anticipatory guidance regarding the importance of complying with drug therapy and the likelihood of the onset of menses

A 12-year-old girl has questions about the potential for her to get much taller. She aspires to be a professional basketball player. On examination she is in Tanner stage IV. Menarche was 1.5 years ago. In considering answers to her questions, it is important to realize which of the following? A) She will not reach her peak height velocity until she is in Tanner stage V B) Menarche occurs an average of 2 years after thelarche C) Girls reach peak height velocity immediately before menarche D) Boys reach peak height velocity at the average age of 15 years

C) Girls reach peak height velocity immediately before menarche

A 7-year-old child is brought to the clinic for a rash that has been present for 24 hours. The child was "a little droopy about 3 days ago" but had no fever or other symptoms. The immunizations are up-to-date. The child now has a "slapped cheek" appearance, with a lacy, erythematous rash on the torso and upper arms. The rash blanches. The PNP discusses with the mother the diagnosis of: A) A coxsackievirus and explains that the rash will disappear within a week and no treatment is necessary B) Measles, which is highly contagious, and explains that the child should be taken home and kept away from siblings C) Human parvovirus B19, or fifth disease, for which there is no treatment D) Roseola and explains that time is the best treatment and the rash will fade in about a week

C) Human parvovirus B19, or fifth disease, for which there is no treatment

The pediatric nurse is caring for a patient with juvenile arthritis. The health-care provider tells the nurse the patient will be started on disease-modifying antirheumatic drugs (DMARDs). Which drugs does the nurse anticipate administering? (Select all that apply.) A. Acetaminophen (Tylenol) B. Indomethacin (Indocin) C. Infliximab (Remicade) D. Leflunomide (Arava) E. Methotrexate (Rheumatrex)

C, E DMARDs include methotrexate (Rheumatrex), cyclophosphamide (Cytoxan), sulfasalazine (Azulfidine), and infliximab (Remicade). Acetaminophen (Tylenol) has no anti-inflammatory effect and is not used to treat juvenile arthritis. Leflunomide is an immunosuppressant

8. The primary care pediatric nurse practitioner is providing anticipatory guidance to the mother of a breastfed 6monthold infant who asks about "babyled weaning." What will the nurse practitioner tell her about this practice? A. "Foods given for this purpose do not meet all the child's nutritional needs." B. "Giving infants control of the feeding process will help prevent obesity." C. "Infants are given soft, mashable table foods when able to selffeed." D. "Infants must be able to grasp and feed themselves from a spoon to do this."

C. "Infants are given soft, mashable table foods when able to selffeed.

8. The parent of a 24monthold child asks the primary care pediatric nurse practitioner when toilet training should begin. How will the pediatric nurse practitioner respond? A. "Begin by reading to your child about toileting." B. "Most children are capable by age 2 years." C. "Tell me about your child's daily habits." D. "We should assess your child's motor skills."

C. "Tell me about your child's daily habits."

You are following a four-year-old girl and your practice with a history of breast development that appeared 12 months ago and appears to be progressing. She is growing rapidly. The PNP considers ordering a bone age because she knows that most cases of premature thelarche in girls are: A. A result of enzymatic defects B. Due to systemic CNS disease C. Idiopathic D. A result of hypothyroidism

C. (Sexual precocity is idiopathic in 80% of girls however the PNP should refer the child to pediatric endocrine to evaluate for advancing bone age and progressive clinical signs since some children require treatment)

14. The primary care pediatric nurse practitioner diagnoses a high school basketball player with mononucleosis. The adolescent asks when she may resume play. What will the nurse practitioner tell her? A. After 3 weeks, she may begin lifting weights but not full sports. B. After 4 weeks, she may return to full play and practice. C. At 4 weeks, she must have an exam to determine fitness for play. D. She may engage in moderate exertion and practice after 3 weeks.

C. At 4 weeks, she must have an exam to determine fitness for play.

12. The parent of a 12yearold child who has sickle cell trait (SCT) asks the primary care pediatric nurse practitioner whether the child may play football. What will the nurse practitioner tell this parent? A. Children with SCT should not play any contact sports. B. Children with SCT may not play for NCAA schools in college. C. Children with SCT should follow heat acclimatization guidelines. D. Children with SCT should not participate in organized sports.

C. Children with SCT should follow heat acclimatization guidelines.

6. The parent of a 6yearold child expresses concern that the child may have ADHD. Which screening tool will the primary care pediatric nurse practitioner use to evaluate this possibility? A. Behavioral and Emotional Screening System for Children (BESS2) B. Behavioral Assessment for Children - 2nd ed. (BASC2) C. Conner's 3 Parent and Teacher Rating Scale D. Pediatric Symptom Checklist (PSC)

C. Conner's 3 Parent and Teacher Rating Scale

13. The parent of a child newly diagnosed with epilepsy asks the primary care pediatric nurse practitioner if the child will ever be able to participate in gym or sports. What will the nurse practitioner recommend? A. Bicycle riding is not safe for children with seizures. B. Contact sports should be avoided. C. Direct supervision of some activities is necessary. D. Underwater sports are not recommended.

C. Direct supervision of some activities is necessary.

8. The parent of a 14yearold child tells the primary care pediatric nurse practitioner that the child skips classes frequently in spite of various disciplinary measures, such as grounding and extra homework and is earning Cs and Ds in most classes. What will the nurse practitioner recommend? A. Counseling for emotional problems B. Development of an Individual Education Plan C. Evaluation for possible learning disorders D. Referral for a behavioral disorder

C. Evaluation for possible learning disorders

16. The parent of a high school basketball player tells the primary care pediatric nurse practitioner that the adolescent becomes short of breath only when exercising. What will the nurse practitioner recommend? A. Permanent discontinuation of all strenuous and aerobic activities B. Enrollment in a conditioning program to improve performance C. Evaluation for underlying cardiac causes of this symptom D. Treatment for exercise induced asthma with a bronchodilator

C. Evaluation for underlying cardiac causes of this symptom

7. The parent of a 5yearold child who has just begun kindergarten expresses concern that the child will have difficulty adjusting to the birth of a sibling. What will the primary care pediatric nurse practitioner recommend? A. Allowing the child opportunities to discuss feelings about the baby B. Giving the child specific baby care tasks to promote sibling bonding C. Having snack time with the child each day to discuss the school day D. Providing reassurance that the sibling will not replace the child

C. Having snack time with the child each day to discuss the school day

1. The parent of a newborn infant asks the primary care pediatric nurse practitioner when to intervene to help the infant's future intellectual growth. What will the nurse practitioner tell the parent? A. Cognitive learning begins during the toddler years. B. Intellectual growth begin when speech develops. C. Language and literacy skills begin at birth. D. Preschool is an optimal time to begin general learning.

C. Language and literacy skills begin at birth.

10. During a well child exam on a 5yearold child, the primary care pediatric nurse practitioner assesses the child for school readiness. Which finding may be a factor in limiting school readiness for this child? A. Adherence to daily family routines and regular activities B. Having two older siblings who attend the same school C. Parental concerns about bullying in the school D. The child's ability to recognize four different colors

C. Parental concerns about bullying in the school

6. The primary care pediatric nurse practitioner is examining a newborn infant recently discharged from the neonatal intensive care unit after a premature birth. The parent is upset and expresses worry about whether the infant will be normal. What will the nurse practitioner do in this situation? A.Explain to the parent that developmental delays often do not manifest at first. B. Perform a developmental assessment and tell the parent which delays are evident. C. Point out the tasks that the infant can perform while conducting the assessment. D. Refer the infant to a developmental specialist for a complete evaluation.

C. Point out the tasks that the infant can perform while conducting the assessment.

7. The single mother of a 4yearold who attends day care tells the primary care pediatric nurse practitioner that she had difficulty giving her child a twicedaily amoxicillin for 10 days to treat otitis media during a previous episode several months earlier because she works two jobs and is too busy. The child has an ear infection in the clinic today. What will the nurse practitioner do? A. Administer an intramuscular antibiotic. B. Order twicedaily amoxicillin for 5 days. C. Prescribe azithromycin once daily for 5 days. D. Reinforce the need to adhere to the plan of care.

C. Prescribe azithromycin once daily for 5 days.

6. The mother of a 6yearold child tells the primary care pediatric nurse practitioner that the child only wants to eat French fries and hamburgers and refuses most vegetables. What will the nurse practitioner recommend? A. Giving the child a multivitamin since this is a phase B. Having the child eat vegetables before getting the hamburger C. Providing a variety of healthy foods at each meal D. Putting extra lettuce and tomatoes on hamburgers

C. Providing a variety of healthy foods at each meal

7. The parent of a 5yearold child tells the primary care pediatric nurse practitioner that the child has been using the toilet to urinate for since age 3 but continues to defecate in "pullups." The nurse practitioner learns that the child has predictable bowel movements and a physical examination is normal. What will the nurse practitioner recommend? A. Providing a reward system to offer incentives when the child uses the toilet B. Put the child back in diapers and resume toilet training in a few months. C. Putting the child on the toilet for 5 to 10 minutes at the usual time of defecation D. Use of polyethylene glycol until the child is able to use the toilet regularly

C. Putting the child on the toilet for 5 to 10 minutes at the usual time of defecation

2. The primary care pediatric nurse practitioner performs a developmental assessment on a 32monthold child. The child's parent reports that about 70% of the child's speech is intelligible. The pediatric nurse practitioner observes that the child has difficulty pronouncing "t," "d," "k," and "g" sounds. Which action is correct? A. Evaluate the child's cognitive abilities. B. Obtain a hearing evaluation. C. Reassure the parent that this is normal. D. Refer the child to a speech therapist.

C. Reassure the parent that this is normal.

5. The primary care pediatric nurse practitioner is evaluating a 5yearold child who has frequent soiling of stool associated with stomach aches and decreased appetite for the past 2 months. The parent states that the child has two or fewer formed bowel movements each week and has been toilet trained for about 2 years. Which initial assessment will the nurse practitioner make? A. History of neurogenic conditions B. Recent adjustments in the family C. Recent illnesses, fluid intake, changes in diet D. Toilet training history

C. Recent illnesses, fluid intake, changes in diet

2. The primary care pediatric nurse practitioner performs a well baby examination on a 7dayold infant who is nursing well, according to the mother. The nurse practitioner notes that the infant weighed 3250 grams at birth and 2990 grams when discharged on the second day of life. The infant weighs 3080 grams at this visit. Which action is correct? A. Follow up at the 2month checkup. B. Refer to a lactation consultant. C. Schedule a weight check in 1 week. D. Suggest supplementing with formula.

C. Schedule a weight check in 1 week A healthy newborn is expected to lose 7% to 10% of the birth weight, but should regain that weight within the first 2 weeks or so after birth. During their first month, most newborns gain weight at a rate of about 1 ounce (30 grams) per day.

9. The parent of a 14yearold child tells the primary care pediatric nurse practitioner that the adolescent has expressed a desire to be a vegetarian, is refusing all meat served at home, and wants the family to eat vegetarian meals. What will the nurse practitioner tell the parent? A. Do not allow a vegetarian diet in order to maintain appropriate limits for the adolescent. B. Provide vegetarian options for the adolescent that preserve adequate nutrition and protein intake. C. Suggest that the adolescent prepare appropriate vegetarian dishes to complement family meals. D. Tell the adolescent that a vegetarian diet may be considered in adulthood but not while living at home.

C. Suggest that the adolescent prepare appropriate vegetarian dishes to complement family meals.

4. The primary care pediatric nurse practitioner is performing a well baby examination on a 2monthold infant who has gained 25 grams per day in the last interval. The mother is nursing and tells the nurse practitioner that her infant seems fussy and wants to nurse more often. What will the nurse practitioner tell her? A. She may not be making as much breastmilk as before. B. She should keep a log of the frequency and duration of each feeding. C. The infant may be going through an expected growth spurt. D. The infant should stay on the previously established nursing schedule.

C. The infant may be going through an expected growth spurt.

4. The primary care pediatric nurse practitioner is offering anticipatory guidance to the parents of a 6yearold child who has Down syndrome. What will the nurse practitioner tell the parents about physical activity and sports in school? A. Children with Down syndrome get frustrated easily when engaging in sports. B. Children with Down syndrome should not participate in strenuous aerobic activity. C. Their child should have a cervical spine evaluation before participation in sports. D. Their child should only participate in sports sanctioned by the Special Olympics.

C. Their child should have a cervical spine evaluation before participation in sports.

6. The mother of a 3yearold child takes the child to a play group once a week. She expresses concern that the child plays with toys but does not interact with the other toddlers. What will the primary care pediatric nurse practitioner counsel the mother? A. The child probably is very shy but will outgrow this tendency with repeated exposure to other children. B. The toddler may have a language delay that interferes with socialization with other children. C. Toddlers may be interested in other children but usually do not engage in interactive play. D. Toddlers need more structured play to encourage interaction and socialization with others.

C. Toddlers may be interested in other children but usually do not engage in interactive play.

2. The primary care pediatric nurse practitioner prescribes a new medication for a child who develops a previously unknown adverse reaction. To report this, the nurse practitioner will A. access the BPCA website. B. call the PREA hotline. C. log onto the FDA Medwatch website. D. use the AAP online PediaLink program.

C. log onto the FDA Medwatch website.

12. The parent of a schoolage child reports that the child is on a glutenfree diet. When questioned about the reason for this diet, the parent states that the child has fewer stomach aches since beginning the diet but has never been diagnosed with celiac disease. The parent reports using glutenfree grain products for all family members. The nurse practitioner will tell this parent that glutenfree diets A. are generally low in sugar and fat. B. are healthy and help prevent obesity. C. may be deficient in essential nutrients. D. provide adequate protein to meet daily needs.

C. may be deficient in essential nutrients.

10. The primary care pediatric nurse practitioner enters an exam room and finds a 2monthold infant in a car seat on the exam table. The infant's mother is playing a game on her smart phone. The nurse practitioner interprets this behavior as A. a sign that the mother has postpartum depression. B. extremely concerning for potential parental neglect. C. of moderate concern for parenting problems. D. within the normal range of behavior in early parenthood.

C. of moderate concern for parenting problems.

6. The primary care pediatric nurse practitioner is treating a toddler who has a lower respiratory tract illness with a lowgrade fever. The child is eating and taking fluids well and has normal oxygen saturations in the clinic. The nurse practitioner suspects that the child has a viral pneumonia and will A. order an antiviral medication and schedule a followup appointment. B. prescribe a broadspectrum antibiotic until the lab results are received. C. teach the parents symptomatic care and order labs to help with the diagnosis. D. write a prescription for an antibiotic to be given if the child's condition worsens.

C. teach the parents symptomatic care and order labs to help with the diagnosis.

2. The parent of a 10yearold boy tells the primary care pediatric nurse practitioner that the child doesn't appear to have any interest in girls and spends most of his time with a couple of other boys. The parent is worried about the child's sexual identity. The nurse practitioner will tell the parent A. children at this age who prefer interactions with samegender peers usually have a homosexual orientation. B. children experiment with sexuality at this age as a means of deciding later sexual orientation. C. this attachment to other same gender children is how the child learns to interact with others. D. to encourage mixedgender interactions in order to promote development of sexual values.

C. this attachment to other same gender children is how the child learns to interact with others.

1. The primary care pediatric nurse practitioner is examining a 6yearold child who attends first grade. The child reports "hating" school. The parent states that the child pretends to be sick frequently in order to stay home from school. To further assess this situation, the nurse practitioner will first ask the child A. about school performance and grades. B. why school is so distressing. C. to name one or two friends. D. whether bullying is taking place.

C. to name one or two friends.

3. During a well child assessment of an 18monthold child, the primary care pediatric nurse practitioner observes the child becoming irritable and uncooperative. The parent tells the child to stop fussing. What will the nurse practitioner do? A.Allow the parent to put the child in a "timeout." B.Ask the parent about usual discipline practices. C.Offer the child a book or a toy to look at. D.Stop the exam since the child has reached a "meltdown."

C.Offer the child a book or a toy to look at.

2. During a well child exam, the primary care pediatric nurse practitioner learns that the parents of a young child fight frequently about finances. The parents state that they do not fight in front of the child and feel that the situation is temporary and related to the father's job layoff. What will the nurse practitioner do? A.Reassure them that the child is too young to understand. B.Recommend that they continue to not argue in front of the child. C.Suggest counseling to learn ways to handle stress. D.Tell them that the conflict will resolve when the situation changes.

C.Suggest counseling to learn ways to handle stress.

49. Oliver's dad is so excited about having a boy after 5 girls and can't wait to know how tall he is going to be. About how tall will Oliver be? Oliver's dad is 5'11" and his mom is 5'5"

Child's predicted growth potential Boys: (mother's height + 5″) + (father's height)/2 Girls: (father's height - 5″) + (mother's height)/2

A schoolage child has a history of chronic otitis media and is seen in the clinic with vertigo. The primary care pediatric nurse practitioner notes profuse purulent otorrhea from both pressureequalizing tubes and a pearlywhite lesion on one tympanic membrane. Which condition is most likely?

Cholesteatoma

Eva, age 13 comes in with her grandmother, they are concerned because Eva has no features of puberty. What is the most common cause? ( use this scenario for questions 39-41) 39. Type 2 diabetes Atopy Constitutional growth delay HCG- secreting tumor

Constitutional growth delay

A child who has been taking antibiotics is brought to the clinic with a rash. the parent reports that the child had a fever associated with what looked like sunburn and now has blisters all over. A physical examination shows coalescent target lesions and widespread bullae and areas of peeled skin revealing moist, red surfaces. What will the NP do?

Consult with a pediatric intensivist for admission to a pediatric intensive care unit.

An adolescent who is overweight expresses a desire to lose weight in order to participate in sports but tells the primary care pediatric nurse practitioner that he doesn't want to give up sweets and soft drinks because he enjoys them too much. Which stage of change does this represent?

Contemplation

During an assessment of a 4 week old infant the NP learns that a breast fed infant nurses every 2 hours during the day but is able to sleep for a 4 hour period during the night. The infant has gained 20 grams per day in the interval since last seen in the clinic. What will the NP recommend?

Continue the nurse the infant using the current pattern

A 10-year-old boy comes to your clinic for evaluation of a suspected urinary tract infection (UTI). The culture indicates sensitivity to trimethoprim/sulfa. The boys treated with this antibiotic for 10 days. The most appropriate follow-up in this case would include: A. Obtaining a urinalysis in two weeks B. Instructing the parents that the child should return to the clinic if symptoms persist C. Teaching parents home monitoring with nitrate sticks D. Referring the child for renal ultrasound

D

A 12-year-old girl presents to your clinic with symptoms of vaginitis, including odor, dysuria, frequency, and discomfort. Upon further history, physical examination, and laboratory screening, the girl is diagnosed with Candida albicans vaginitis. Appropriate treatment would include: A. Avoiding bubble baths B. Topical acyclovir C. Ceftriaxone D. Clotrimazole

D

A labial adhesion extending from the posterior fourchette to the clitoris is noted during the routine assessment of a four-year-old girl. There is no history of difficulty voiding, dysuria, or discomfort. The most appropriate initial management is to: A. Recommend mechanical lysis using petrolatum appointment B. Prescribe a topical application of estrogen cream C. Refer to a GU specialist D. Reassure parents that no specific treatment is needed at this time

D

A two year old child evaluated in the emergency department for a closed head injury following a 10 foot fall from an open window. When interpreting diagnostic imaging results, you are aware that the most common and generally the least serious type of skull fracture is: A. Basilar fracture B. compound fx C. Depressed fx D. Linear fx

D

An 11 month old African-American boy has just started walking and is found to have severely bowed legs. In the history, you learn that he is exclusively breast-fed with very little other food intake. You must consider: A. Trauma B. Developmental variation C. Chromosomal abnormality D. Rickets

D

An adolescent boy presents to the clinic with a painless mass in the left side of his scrotum. The most likely diagnosis is: A. Epididymitis B. Testicular torsion C. Incarcerated hernia D. Hydrocele

D

Families of children with congenital adrenal hyperplasia must be educated about: A. The self limiting aspect of the disorder B. The need for genetic counseling C. Dietary restrictions D. The need for strict replacement therapy

D

Jackson was born at 34 weeks gestation. He is now 10 weeks old and his mother has brought him to your office for a routine exam. He appears alert and will develop. His mother tells you that he takes 2 to 4 ounces of formula every 2 to 4 hours around the clock. The plan for Jackson should include which of the following? A. Initiation of rice cereal at bedtime B. Addition of two bottles of water each day next line C. Encouraging Jackson's mother to limit his feedings to every four to 6 D. Prescribing Ferrous sulfate to be administered three times a day

D

Main goal of continuous quality improvement is to: A. Incorporate norms, criteria, and standards as evaluative measures B. Emphasize outcome measures in addition to structure and process C. Promote evaluation based on peer review, audits, and chart reviews D. Inform that quality assurance is more than a periodic evaluation of performance

D

Managed-care plans may be certified by which of the following? A. Centers for Medicare and Medicaid services B. Department of health and human services C. National Institutes of Health D. National committee for quality assurance

D

Mrs. B has brought 5 yo Julio to the clinic. She reports that he has been lethargic and has been running a low-grade fever for about two weeks. Physical exam reveals no significant findings other than pallor and lymphadenopathy. A CBC reveals a decreased hematocrit, neutropenia, and thrombocytopenia. The practitioners next action should be to: A. Prescribe a broad spectrum antibiotic and ferrous sulfate B. Instruct Mrs. B on the appropriate use of Acetaminophen to treat Julio's fever C. Reassure Mrs. B that Julio's signs and symptoms are indicative of a viral infection D. Refer Julio to a pediatric hematologist/oncologist for further evaluation

D

Mrs. S has brought her toddler to the clinic for an immunization update. While talking to her, you learn that they live in an old building that has been under renovation for the past two months. Based on this information, you should first assess the child for: A. Asbestosis B. Coccidioidomycosis C. Mold allergy D. Lead poisoning

D

Mrs. W has brought her one-year-old baby to the clinic for well baby exam. She is pregnant with her second child and is concerned about possible risks to the fetus because she has gestational class a diabetes. Which of the following conditions is the fetus not at risk for? A. Congenital anomalies B. Hypoglycemia C. Birth trauma D. Congenital hearing loss

D

The parents of a 12yo are concerned that some of the child's older classmates may be a bad influence on their child, who, they say, has been raised to believe in right and wrong. What will you tell the parent? a. Allowing the child to make poor choices and accept consequences is important for learning values. b. Children at this age have a high regard for authority and social norms, so this is not likely to happen. c. Moral values instilled in the early school-age period will persist throughout childhood. d. The pressures from outside influences may supersede parental teachings and should be confronted.

D

After treating a two-year-old for iron deficiency anemia, lab test show that his hemoglobin level has returned to normal. Which of the following actions is appropriate? A. Discontinue iron therapy and recheck his hemoglobin level in one month B. Discontinue iron therapy and tell the child's mother to reinitiate therapy if she notices any pallor C. Continue iron therapy and to all the medicine at home is gone D. Continue iron therapy for 2 to 3 months

D ( to replenish the body's iron stores)

Which of the following screenings is done annually on a four-year-old child with down syndrome? A. Cervical spine B. Urinalysis C. Celiac screening D. Thyroid screening

D (Children with down syndrome or at great risk for thyroid disease and leukemia)

A three year old boy with known G6PD deficiency has acute purulent otitis media. Which of the following drugs should not be used for his treatment? A. Amoxicillin B. Amoxicillin - clavulanate C. Erythromycin D. Trimethoprim/sulfa

D (sulfa drugs precipitate hemolysis) patients with G6PD)

An 11-year-old child is concerned about breast size. About 6 months ago when her breast development began, she noticed that her breasts were different sizes. The PNP responds: A) "This is abnormal and you need to see a specialist." B) "Menarche will appear soon." C) "A mammogram is needed." D) "The breasts will become closer to the same size within a few years."

D) "The breasts will become closer to the same size within a few years."

A 10-year-old child was seen in the clinic 5 days before for a routine sports physical examination. The child's blood cholesterol level was 186 mg/dL. The most appropriate intervention by the PNP would be to: A) Screen other members of the family for hyperlipidemia B) Implement a diet plan of less than 20% of fat in total daily calories C) Counsel the child and family regarding a healthy diet and exercise and perform another blood cholesterol test in 3 to 5 years D) Repeat the total cholesterol test and obtain lipoprotein analysis

D) Repeat the total cholesterol test and obtain lipoprotein analysis

A child is taking methimazole (MTZ). What does the nurse teach parents to report immediately? (Select all that apply.) A. Brittle hair B. Dry skin C. Headache D. Jaundice E. Pink or red urine

D, E Methimazole has some side effects that can be serious. Jaundice can signal hepatic failure and pink/red urine can be a manifestation of glomerulonephritis, and so both should be reported without delay. The other manifestations are not due to side effects of this drug.

4. The mother of a 16yearold male was recently divorced after several years of an abusive relationship and tells the primary care pediatric nurse practitioner that the adolescent has begun skipping school and hanging out with friends at the local shopping mall. When she confronts her child, he responds by saying that he hates her. What will the nurse practitioner tell this mother? A. Adolescence is marked by an inability to comprehend complex situations. B. Adolescence is typically marked by tempestuous and transient episodes. C. Adolescents normally have extreme, disruptive conflicts with parents. D. Adolescents often need counseling to help them cope with life events.

D. Adolescents often need counseling to help them cope with life events.

4. The primary care pediatric nurse practitioner is offering anticipatory guidance to the parents of a 12monthold child. The parents are bilingual in Spanish and English and have many Spanish speaking relatives nearby. They are resisting exposing the child to Spanish out of concern that the child will not learn English well. What will the pediatric nurse practitioner tell the parents? A. Children who learn two languages simultaneously often confuse them in conversation. B. Children with multilanguage proficiency do not understand that others cannot do this. C. Learning two languages at an early age prevents children from developing a dominant language. D. Most bilingual children are able to shift from one language to another when appropriate.

D. Most bilingual children are able to shift from one language to another when appropriate.

7. The parent of a 4yearold points to a picture and says, "That's your sister." The child responds by saying, "No! It's my baby!" This is an example of which type of thinking in preschoolage children? A. Animism B. Artificialism C. Egocentrism D. Realism

D. Realism

7. The primary care pediatric nurse practitioner is performing an exam on an adolescent male who asks about sexual identity because of concern that a friend is worried about being gay. Which response will the nurse practitioner make in this situation? A. Provide the teen with a questionnaire to gain information about his sexuality. B. Remind the adolescent that mandatory reporting requires disclosure to parents. C. Suggest that the adolescent discuss sexual concerns with his parents. D. Tell the adolescent that, unless he is at risk, what he says will be confidential.

D. Tell the adolescent that, unless he is at risk, what he says will be confidential.

11. The primary care pediatric nurse practitioner is counseling the parent of an 8 yearold child who has primary nocturnal enuresis. The nurse practitioner recommends an enuresis alarm, but the parent wishes to use medication. What will the nurse practitioner tell the parent? A. Anticholinergic medications are most commonly used for enuresis. B. Drug therapy is an effective way to achieve longterm control. C. Drug therapy is safest when the nasal spray form is used. D. The combination of alarm therapy and intermittent drug therapy is best.

D. The combination of alarm therapy and intermittent drug therapy is best.

5. The mother of a 6weekold breastfeeding infant tells the primary care pediatric nurse practitioner that her baby, who previously had bowel movements with each feeding, now has a bowel movement once every third day. What will the nurse practitioner tell her? A. Her baby is probably constipated. B. It may be related to her dietary intake. C. She should consume more water. D. This may be normal for breastfed babies.

D. This may be normal for breastfed babies.

18. A 15yearold female basketball player who has secondary amenorrhea is evaluated by the primary care pediatric nurse practitioner who notes a BMI in the 3rd percentile. What will the nurse practitioner counsel this patient? A. That amenorrhea in female athletes is not concerning B. That she should begin a program of plyometrics and strength training C. To consider a different sport, such as volleyball D. To work with a dietician to improve healthy weight gain

D. To work with a dietician to improve healthy weight gain

8. The parent of a schoolage child who has asthma tells the primary care pediatric nurse practitioner that the child often comes home from school with severe wheezing after gym class and needs to use his metereddose inhaler right away. What will the nurse practitioner do? A. Recommend that the child go to the school nurse when symptoms start. B. Review the child's asthma action plan and possibly increase his steroid dose. C. Suggest asking the school to excuse the child from gym class. D. Write the prescription for two metereddose inhalers with spacers.

D. Write the prescription for two metereddose inhalers with spacers.

3. The parent of a 12monthold infant asks the primary care pediatric nurse practitioner why 2% cow's milk is recommended instead of whole milk. What will the nurse practitioner tell this parent? A. Whole milk is usually not fortified with vitamin D. B. 2% milk is higher in essential proteins and minerals. C. Young children don't need the extra calories found in whole milk. D. Younger children need a limited amount of fats.

D. Younger children need a limited amount of fats.

3. During a well child assessment of a 13yearold male, the primary care pediatric nurse practitioner notes small testicles and pubic and axillary hair. To further evaluate these findings, the nurse practitioner will ask the patient about A. alcohol and tobacco use. B. changes in voice. C. increase in height and weight. D. participation in sports.

D. participation in sports.

11. When counseling an adolescent with a family history of hyperinsulinemia and type 2 diabetes, the primary care pediatric nurse practitioner will recommend avoiding A. baked potato chips. B. canned vegetables. C. highfiber cereals. D. processed breads.

D. processed breads.

2. The parent of a toddler tells the primary care pediatric nurse practitioner that the family has adopted a plantbased diet and the child is receiving rice and almond milk instead of cow's milk. The nurse practitioner will counsel the parents about A. calcium deficiency. B. excess caloric intake. C. excess fat intake. D. protein deficiency.

D. protein deficiency.

3. During a well child assessment of an 18monthold child, the primary care pediatric nurse practitioner observes the child point to a picture of a dog and say, "Want puppy!" The nurse practitioner recognizes this as an example of A. holophrastic speech. B. receptive speech. C. semantic speech. D. telegraphic speech.

D. telegraphic speech.

5. The primary care pediatric nurse practitioner conducts a well baby exam on an infant and notes mild gross motor delays but no delays in other areas. Which initial course of action will the nurse practitioner recommend? A.Consult a developmental specialist for a more complete evaluation. B.Prepare the parents for a potentially serious developmental disorder. C.Refer the infant to an early intervention program for physical therapy. D.Teach the parents to provide exercises to encourage motor development.

D.Teach the parents to provide exercises to encourage motor development.

The NP performs a physical examination on a 12 year old child and notes poor hygiene and inappropriate clothes for the weather. The child's mother appears clean and well dressed. The child reports getting 6-7 hours of sleep each night because of texting with friends late each evening. What action by the NP will help promote healthy practices/

Discuss setting clear expectations about self-care with the mother.

The primary care pediatric nurse practitioner is counseling an obese 16 year old client about weight management. The adolescent says, "I know I need to lose weight, but I don't want to give up all my favorite foods." When using motivational interviewing techniques, how will the nurse practitioner respond?

Do you think there are any foods you could limit or do without for a while?

A child with a history of otitis externa asks about ways to prevent this condition. What will the primary care pediatric nurse practitioner recommend?

Drying the ear canal with a hair dryer

Most Growth Hormone activity occurs during: Intense activity After meals During sleep During the day

During sleep

The parent of a 16 year old tells the NP that the teen was recently caught smoking an electronic cigarette. What will the NP tell this parent.

E-cigarette use may be a risk factor for later substance abuse.

The NP is evaluating recurrent stomach pain in a school-age child. The child's exam is normal. The NP learns that the child reports pain most evening after school and refuses to participate in sports but does not have nausea or vomiting. The child's grandmother recently had gallbladder surgery. Which action is correct?

Encourage the child to keep log of pain, stool patterns, and dietary intake.

A school-age child has begun refusing all cooked vegetables. What will the NP recommend to the parent?

Ensure that the child has three nutritious meals and two nutritious snacks each day.

The parent of a 14 year old child tells the NP that the child skips classes frequently in spite of various disciplinary measures, such as grounding and extra homework and is earning Cs and Ds in most classes. What will the NP recommend?

Evaluation for possible learning disorders.

For at risk children, how often should you screen for type 2 diabetes? Every 5 years When they go through puberty Age 13 Every 3 years

Every 3 years

8. Which statement about fever is NOT true? Antipyretics should be given for a temp > 101F Fever with signs of toxicity requires a full workup Fever in infancy without a known source is not worrisome Fever is defined at a rectal temp of 100.4

Fever is defined at a rectal temp of 100.4

The primary care pediatric nurse practitioner provides patient teaching for children newly diagnosed with irritable bowel syndrome (IBS). At which stage of development will children be able to understand the link between stress and the symptoms of the disease?

Formal-operational stage

The NP is teaching a parent of a child with dry skin about hydrating the skin with bathing. What will the NP include in teaching?

Have the child soak in lukewarm water bath..

The parent of a 5 year old child who has just begun kindergarten expresses concern that the child will have difficulty adjusting to the birth of a sibling. What will the NP recommend?

Having snack time with the child each day to discuss the school day.

The primary care pediatric nurse practitioner is counseling an obese adolescent whose parents both have type 2 diabetes mellitus. Which health behavior prediction useful when the nurse practitioner discusses lifestyle changes with this client?

Health belief model

The primary care pediatric nurse practitioner is performing a focused problem assessment on a child who has asthma and learns that one of the child's parents smokes around the child in spite of being advised against this. The nurse practitioner recognizes this as a possible alteration in which functional health pattern?

Health perception

The primary care pediatric nurse practitioner is working with a 12 year old female who has poor diabetes control. The child tells the nurse practitioner that the parent forgets to remind her to check her blood sugars. Which action is correct?

Help the child develop a strategy to remember without parental prompts.

23. Jonny, 5 yo, is in your clinic for a well child check. You not he has not received any immunizations. Which of the following is not necessary at this age? MMR DTaP Hib Varicella

Hib

The mother of a 3 month old child tells the NP that it is "so much fun" now that her infant coos and smiles and wants to play. What is important for the NP to teach this mother?

How to read the infant's cues for over stimulation.

Riley, age 6 has the following symptoms, what do you as the NP suspect she has? Tachycardia, wide pulse pressure, warm/moist skin, exophthalmos, tremor, hyperreflexia Hypothyroidism Hyperthyroidism Chediak-Hegashi Syndrome Danon Disease

Hyperthyroidism

Liam is a 1 year old who had a home birth and has had no labs or immunizations, but his mother is concerned because he has not grown like her 6 other children, has a lump that someone called a goiter in his neck and has been gaining weight, which is odd for her children. What do you suspect as the NP? Cystinosis Hyperthyroidism Hypothyroidism Willi Frank syndrome

Hypothyroidism

Sophia is a 4 day old infant, when you do the newborn assessment as the NP you note macroglossia, large fontanelles, and decreased muscle tone. What do you expect on the Newborn screen to note? Aicardi syndrome Hypothyroidism Normal labs Constitutional growth delay

Hypothyroidism

The most common cause of preventable mental retardation is? Type 1 diabetes Hyperthyroidism Bloom syndrome Hypothyroidism

Hypothyroidism

30. Which is not a reportable to VAERS (Vaccine Adverse Events Reporting System)? Death Encephalitis Anaphylaxis Intussusception

Intussusception

Aiden, age 2 was diagnosed with Hypothyroidism when he was born. What do you suspect he is being treated with by his pediatric endocrinologist ? Levothyroxine Eliquis Iodine in the form of table salt Iodine tablets

Levothyroxine

22. The nurse practitioner is assessing the head of an 10 month-old girl. During the assessment, the NP should do which of the following to make sure the child has adequate skull and brain growth? Check the child's eye tracking with a pen light Measure the head circumference, assess the anterior fontanel Check the circumference of the head Check the suture lines and posterior fontanel for alignment

Measure the head circumference, assess the anterior fontanel

In primary adrenal insufficiency, there is a deficiency of what? Mineralocorticoid Glucocorticoid Mineralocorticoid and glucocorticoid Testosterone

Mineralocorticoid and glucocorticoid

A 4monthold infant has a history of reddened, dry, itchy skin. The primary care pediatric nurse practitioner notes fine papules on the extensor aspect of the infant's arms, anterior thighs, and lateral aspects of the cheeks. What is the initial treatment?

Moisturizers

A child who was treated with amoxicillin and then amoxicillinclavulanate for acute otitis media is seen for followup. The primary care pediatric nurse practitioner notes dullgray tympanic membranes with a visible airfluid level. The child is afebrile and without pain. What is the next course of action?

Monitoring ear fluid levels for 3 months

The primary care pediatric nurse practitioner is offering anticipatory guidance to the parents of a 12 month old child. The parents are bilingual in Spanish and English and have many Spanish-speaking relatives nearby. They are resisting exposing the child to Spanish out of concern that the child will not learn English well. What will the pediatric nurse practitioner tell the parents?

Most bilingual children are able to shift from one language to another when appropriate.

The primary care pediatric nurse practitioner is counseling a school-age child about asthma management strategies. Teh child states that it is "too much trouble" to remember to use an inhaled corticosteroid medication twice daily and reports feeling fine, in spite of exhibiting expiratory wheezes. Which action uses the health belief and self-efficacy model to teach this child about asthma management?

Obtaining pre- and post-treatment spirometry testing

The primary care pediatric nurse practitioner is managing care for a child who has JIA who has a positive ANA. Which specialty referral is critical for this child?

Ophthalmology

The primary care pediatric nurse practitioner is performing a wellbaby checkup on a 6monthold infant and notes a candida diaper rash and oral thrush. The infant has had two ear infections in the past 2 months and is in the 3rd percentile for weight. What will the nurse practitioner do?

Order a CBC with differential and platelets and quantitative immunoglobulins.

A 10yearold child has a 1week history of fever of 104°C that is unresponsive to antipyretics. The primary care pediatric nurse practitioner examines the child and notes bilateral conjunctival injection and a polymorphous exanthema, with no other symptoms. Lab tests show elevated ESR, CRP, and platelets. Cultures are all negative. What will the nurse practitioner do?

Order a baseline echocardiogram today and another in 2 weeks

An adolescent who has exerciseinduced asthma (EIA) is on the high school track ID: 13348419888 team and has recently begun to practice daily during the school week. The adolescent uses 2 puffs of albuterol via a metereddose inhaler 20 minutes before exercise but reports decreased effectiveness since beginning daily practice. What will the primary care pediatric nurse practitioner do?

Order a daily inhaled corticosteroid medication.

The parent of a schoolage child reports that the child usually has allergic rhinitis symptoms beginning each fall and that nonsedating antihistamines are only marginally effective, especially for nasal obstruction symptoms. What will the primary care pediatric nurse practitioner do?

Order an intranasal corticosteroid to begin 1 to 2 weeks prior to pollen season

A previously healthy school-age child develops herpes zoster on the lower back. What will the NP do to manage this condition?

Order burrow solution and warm soothing aths as comfort measures.

A child complains of itching in both ears and is having trouble hearing. The primary care pediatric nurse practitioner notes periauricular edema and marked swelling of the external auditory canal and elicits severe pain when manipulating the external ear structures. Which is an appropriate intervention?

Order ototopical antibiotic/corticosteroid drops

A 3yearold child with pressureequalizing tubes (PET) in both ears has otalgia in one ear. The primary care pediatric nurse practitioner is able to visualize the tube and does not see exudate in the ear canal and obtains a type A tympanogram. What will the nurse practitioner do?

Order ototopical antibiotic/corticosteroid drops.

During a well child exam on a 5 year old child, the NP assesses the child for school readiness. Which finding may be a factor in limiting school readiness for this child?

Parental concerns about bullying in the school

What will the primary care pediatric nurse practitioner teach the parents of a child who has new pressureequalizing tubes (PET) in both ears?

Parents should notice improved hearing in their child.

A toddler exhibits exotropia of the right eye during a coveruncover screen. The primary care pediatric nurse practitioner will refer to a pediatric ophthalmologist to initiate which treatment?

Patching of the unaffected eye for 2 hours each day

If you refer Eva, who do you refer her to? A family practice physician OBGYN Dietician for better nutrition Pediatric endocrinologist

Pediatric endocrinologist

A 15 year old sexually active female has a temperature with lower abdominal pain. The exam reveals pain with adnexal tenderness. Cultures are pending. Which is the mostly likely diagnosis?

Pelvic inflammatory disease

The NP is examining a 12 month old infant who was 6 weeks premature and observes that the infant uses a raking motion to pick up small objects. The PEDS questionnaire completed by the parent did not show significant developmental delays. What will the NP do first?

Perform an in-depth developmental assessment.

The NP is examining a 15 year old female who reports having her first period at age 13. She states that she has had five periods in the last year, with the last one 2 months prior. She participates in basketball at school. Which action is correct?

Perform biometric screening to determine lean body mass.

An AfricanAmerican child has recurrent tinea capitis and has just developed a new area of alopecia after successful treatment several months prior. When prescribing treatment with griseofulvin and selenium shampoo, what else will the primary care pediatric nurse practitioner do?

Perform fungal cultures on family members and pets

The primary care pediatric nurse practitioner notes velvety, brown thickening of skin in the axillae, groin, and neck folds of an adolescent Hispanic female who is overweight. What is the initial step in managing this condition?

Performing metabolic laboratory tests

An 8yearold child is diagnosed with systemic lupus erythematosus (SLE), and the child's parent asks if there is a cure. What will the primary care pediatric nurse practitioner tell the parent?

Periods of remission may occur but there is no permanent cure.

A 3 year old child has head lice. What will the initial treatment recommendation be to treat this child?

Permethrin

A 9 month old infant has vesiculopustular lesion on hte palms and soles, on the face and neck, and in skin folds of the extremities. The NP notes linear and S shaped burrow lesions on the parent's hands and wrist. What is the treatment for this rash for this infant?

Permethrin 5% cream applied to face, neck and body and rinsed off in 8 to 14 hours.

An adolescent who recently spent time in a hot tub while on vacation has discrete erythematous 1- to 2- mm papules that are centered around hair follicles on the thighs, upper arms, and buttocks. How will the NP manage this condition?

Prescribe topical keratolytics and topical antibiotics.

The NP is performing a well child exam on a 17 year old female whose mother is present during the history. The mother expresses concern that her daughter wishes to have an eyebrow piercing and states that she is opposed to the idea. What will the NP do?

Provide information about piercing and encourage continued discussion.

1. 46. Jason, an 8 year old with type 1 DM is being seen for a 3-day history of frequency and nocturia. He denies flank pain and is afebrile. The urinalysis result is negative for blood and nitrites but is positive for a large amount of leukocytes and ketones. He has a trace amount of protein. Which of the following is the best test to order initially? A) Urine for culture and sensitivity B) 24-hour urine for protein and creatinine clearance C) 24-hour urine for microalbumin D) An intravenous pyelogram

Rational: A) Urine for culture and sensitivity An 8-year-old male patient with the diagnosis of diabetes has a high risk of urinary tract infections. A large amount of leukocytes in the urinalysis is abnormal and he has been having symptoms of frequency and nocturia for the past 3 days. The urine culture would be ordered because he has a high risk of infection. The urine culture and sensitivity (C& S) is the best evaluation for diagnosing a urinary tract infection

2. In which of the following groups of patients is tight glycemic control contraindicated? a. Adolescent males. b. Middle-aged females. c. Middle-aged males. d. Children under age 6.

Rationale: Tight glycemic control is contraindicated in infants < 2 years of age and should be instituted with extreme caution in children < 7 years of age to avoid injuring the developing brain.

11. Antipyretics are ineffective in what types of fever? Raising of the hypothalamic set point in the CNS Result of heat production exceeding heat loss Defective heat loss

Rationale: antipyretics are ineffective for 2 and 3

The NP is performing a well child examination on a 9 month old infant whose hearing is normal but who responds to verbal cues with only single syllable vocalizations. What will the NP recommend to the parents to improve speech and language skills in this infant?

Read simple board books to the infant at bedtime.

The parent of a 4 year old points to a picture and says, "That's your sister." The child responds by saying, "No! It's my baby!" This is an example of which type of thinking in preschool-age children?

Realism

A school age child has a rash without fever or preceding symptoms. Physical examination reveals 3 cm ovoid, erythematous lesion on the trunk with a finely scaled elevated border, along with generalized macular, ovoid lesions appearing in a "Christmas tree" pattern on the child's back. What is the initial action?

Reassure the chid's parents that the ras is benign and self-limited.

The primary care pediatric nurse practitioner performs a developmental assessment on a 32 month old child. The child's parent reports and about 70% of the child's speech is intelligible. The pediatric nurse practitioner observes that the child has difficulty pronouncing "t," "d," "k," and "g" sounds. Which action is correct?

Reassure the parent that this is normal

A child has small, firm, flesh-colored papules in both axillae which are mildly puritic. What is an acceptable initial approach to managing this condition?

Reassuring the parents that these are benign and may disappear spontaneously.

A schoolage child is seen in the clinic after a fragment from a glass bottle flew into the eye. What will the primary care pediatric nurse practitioner do?

Refer immediately to an ophthalmologist.

An adolescent female reports poor sleep, fatigue, muscle and joint paint, and anxiety lasting for several months. The primary care pediatric nurse practitioner notes point tenderness at several sites. What will the nurse practitioner do next?

Refer the adolescent to a rheumatologist for further evaluation

A schoolage child is hit in the face with a baseball bat and reports pain in one eye. The primary care pediatric nurse practitioner is able to see a dark red fluid level between the cornea and iris on gross examination, but the child resists any exam with a light. Which action is correct?

Refer the child immediately to an ophthalmologist.

A 7monthold infant has had two prior acute ear infections and is currently on the 10th day of therapy with amoxicillinclavulanate after a failed course of amoxicillin. The primary care pediatric nurse practitioner notes marked middle ear effusion and erythema of the TM. The child is irritable and has a temperature of 99.8°F. What is the next step in management of this child's ear infection?

Refer the child to an otolaryngologist.

The primary care pediatric nurse practitioner performs a vision screen on a 4 monthold infant and notes the presence of convergence and accommodation with mild esotropia of the left eye. What will the nurse practitioner do?

Refer the infant to a pediatric ophthalmologist

An adolescent who has asthma and severe perennial allergies has poor asthma control in spite of appropriate use of a SABA and a daily highdose inhaled corticosteroid. What will the primary care pediatric nurse practitioner do next to manage this child's asthma?

Refer to a pulmonologist for omalizumab therapy.

he primary care pediatric nurse practitioner is treating an infant with lacrimal duct obstruction who has developed bacterial conjunctivitis. After 2 weeks of treatment with topical antibiotics along with massage and frequent cleansing of secretions, the infant's symptoms have not improved. Which action is correct?

Refer to an ophthalmologist.

25. If two live virus parenteral vaccines are given less than 28 days apart, what should happen? Nothing Repeat the second one 4 weeks later Repeat both vaccines Give a third dose in 6 months

Repeat the second one 4 weeks later

Johnny, 2 months, was just diagnosed with intussusception, he had a set of immunizations 5 days ago, which immunization is responsible? Hep 2 second dose MMR Rotavirus DTaP

Rotavirus

The NP performs a well baby examination on a 7 day old infant who is nursing well, according to the mother. The NP notes that the infant weighed 3250 grams at birth and 2990 grams when discharged on the second day of life. The infant weighs 3080 grams at the visit. Which action is correct?

Schedule a weight check in 1 week.

The parent of a 14 year old child tells the NP that the adolescent has expressed a desire to be a vegetarian, is refusing all meat served at home, and wants the family to eat vegetarian meals. What will the NP tell the parent?

Suggest that the adolescent prepare appropriate vegetarian dishes to complement family meals.

21. The parent of a school-age child who is diagnosed with oligoarticular JIA asks the nurse practitioner what exercises the child may do to help reduce symptoms: Weight-lifting Swimming Cross country skiing Yoga

Swimming

The parent of a schoolage child who is diagnosed with oligoarticular JIA asks the primary care pediatric nurse practitioner what exercises the child may do to help reduce symptoms. What will the nurse practitioner recommend?

Swimming

36. Nocturia/bedwetting is a seen in type 2 diabetes in children? T/F

T

95% of girls have signs of puberty at age 12 and achieve menarche by age 14. T/F

T

27. In older adults what is the booster that should be given since we see more Pertussis? Pertussis only TD Tdap DTap

Tdap

A schoolage child with asthma is seen for a well child checkup and, in spite of "feeling fine," has pronounced expiratory wheezes, decreased breath sounds, and an FEV1less than 70% of personal best. The primary care pediatric nurse practitioner learns that the child's parent administers the daily mediumdose ICS but that the child is responsible for using the SABA. A treatment of 4 puffs of a SABA in clinic results in marked improvement in the child's status. What will the nurse practitioner do?

Teach the child and parent how to use home PEF monitoring

The parent of a 24 month old child asks the primary care pediatric nurse practitioner when toilet training should begin. How will the pediatric nurse practitioner respond?

Tell me about your child's daily habits.

The NP is performing an exam on an adolescent male who asks about sexual identity because of concern that a friend is worried about being gay. Which response will the NP make in this situation?

Tell the adolescent that, unless he is at risk, what he says will be confidential.

During a well child exam on a 4yearold child, the primary care pediatric nurse practitioner notes that the clinic nurse recorded "20/50" for the child's vision and noted that the child had difficulty cooperating with the exam. What will the nurse practitioner recommend?

Test the child's vision in 1 month.

The NP is performing a well baby examination on a 2 month old infant who has gained 25 grams per day in the last interval. The mother is nursing and tells the NP that her infant seems fussy and wants to nurse more often. What will the NP tell her?

The infant may be going though an expected growth spurt.

The NP is performing a well child assessment on a 13 year old female whose mother asks when her daughter's periods may start. Which information will the NP use to help estimate the onset of periods?

The patents' age at thelarche

The primary care pediatric nurse practitioner applies fluorescein stain to a child's eye. When examining the eye with a cobalt blue filter light, the entire cornea appears cloudy. What does this indicate?

There is too much stain on the cornea.

Jalisha, a non-Hispanic african american girl, age 8, comes in with her mother and is upset because her daughter has thelarche, what do you tell mom as the NP: This is concerning we need to run tests She will get her period in 6 months This is normal and does not mean she will get her period soon How long has she been this way?

This is normal and does not mean she will get her period soon

The mother of a 6 week old breastfeeding infant tells the NP that her baby, who previously had bowel movements with each feeding, now has a BM once every third day. What will the NP tell her?

This may be normal for breastfed babies.

The primary care pediatric nurse practitioner performs a physical examination on a 9 month old infant and notes two central incisors on the lower gums. The parent states that the infant nurses, takes solid foods three times daily, and occasionally takes water forma a cup. What will the pediatric nurse practitioner counsel the parent to promote optimum dental health?

To make an appointment for an initial dental examination.

The mother of a 3-year old child takes the child to a play group once a week. She expresses concern that the child plays with toys but does not interact with the other toddlers. What will the primary care pediatric nurse practitioner counsel the mother?

Toddlers may be interested in other children but usually do not engate in interactive play.

A 14yearold child has a 2week history of severe itching and tearing of both eyes. The primary care pediatric nurse practitioner notes redness and swelling of the eyelids along with stringy, mucoid discharge. What will the nurse practitioner prescribe?

Topical NSAID drops

A child has an area of inflammation on the neck that began after wearing a hank-know woolen sweater. On examination, the skin appears chafed with mild erythematous patches. the lesions are not pruritic. What is the appropriate treatment?

Topical corticosteroids applied 2-3 times daily.

A 4 year old child has clusters of small, clear tense vesicles with an erythematous base on one side of the mouth along the vermillion border, which are causing discomfort and difficulty eating. What will the NP recommend as treatment?

Topical diphenhydramine and magnesium hydroxide

An adolescent has acne characterized by papules and pustules mostly on the forehead and chin. What will the NP prescribe?

Topical erythromycin with benzoyl peroxide

An infant is brought to the clinic with bright erythema in the neck and flexural folds after recent treatment with antibiotics for otitis media. What is the treatment for this condition?

Topical nystatin cream applied several times daily.

A school-age child has several annular lesions on the abdomen characterized by central clearing with scaly, red borders. What is the first step in managing this condition.

Treat empirically with antifungal cream.

Aubrey, is an 11 year old Hispanic female and comes in with the following signs, what do you suspect as the NP? Nocturia, obesity, velvet-like rash, fatigue Hypertelorism Janson's syndrome Type 2 diabetes Type 1 diabetes

Type 2 diabetes

The primary care pediatric nurse practitioner is reviewing the rheumatology plan of care for a child who is diagnosed with SLE. Besides reinforcing information about prescribed medications, what will the nurse practitioner teach the family to help minimize flaring of episodes?

Use UVA and UVB sunscreen daily.

20. For a child with fever which method should be used for bathing to reduce fever? Using warm water to reduce the fever Using tepid water to sponge bathe Using a mix of alcohol and water to bathe Plunging the child in ice water like an athlete for muscle recovery

Using tepid water to sponge bathe

The primary care pediatric nurse practitioner understands that, to achieve the greatest world-wide reduction in child mortality from pneumonia and diarrhea, which intervention is most effective?

Vaccinations

9. The primary care pediatric nurse practitioner is concerned that a toddler may have vesicoureteral reflux based on a history of dysfunctional voiding patterns and a series of urinary tract infections. Which intervention is appropriate? A. Initiating a bladder retraining program B. Ordering a voiding cystourethrogram C. Referral to a urologist for evaluation D. Treatment with prophylactic antibiotics

Vesicoureteral reflux (VUR) is a condition in which urine flows backward from the bladder to one or both ureters and sometimes to the kidneys C. Referral to a urologist for evaluation

In endocrine pathology what are 3 symptoms? Vomiting Dehydration Exophthalmos Tachycardia

Vomiting

The primary care pediatric nurse practitioner observes a tender, swollen red furuncle on the upper lid margin of a child's eye. What treatment will the nurse practitioner recommend?

Warm, moist compresses 3 to 4 times daily

As the nurse practitioner, you see Carter, who you have diagnosed with a constitutional growth delay, what do you tell his parents? We will watch and see, I will reassure you and give you support I will refer you to a real physician I will refer you to the dietician so Carter get the best extra calories You should give Carter pediasure every morning and evening

We will watch and see, I will reassure you and give you support

10. A child is diagnosed with tinea versicolor. What is the correct management of this disorder? a. Application of selenium sulfide 2.5% lotion twice weekly for 2 to 4 weeks b. Oral antifungal treatment with fluconazole once weekly for 2 to 3 weeks c. Sun exposure for up to an hour every day for 2 to 4 weeks d. Using ketoconazole 2% shampoo on lesions twice daily for 2 to 4 weeks

a

10. A child who plays soccer is in the clinic reporting pain and swelling in both knees. A physical examination reveals swelling and focal tenderness at the tibial tuberosities, with pain worsening when asked to extend the knees against resistance. What is the treatment for this condition? a. Apply ice packs to both knees and avoid activities that cause pain. b. Begin quadricepsstretching exercises now to prevent further injury. c. Obtain radiographic studies to rule out fractures or ligament tears. d. Refer to a pediatric orthopedic specialist to evaluate the need for surgery.

a

11. During a well baby examination of a newborn, the primary care pediatric nurse practitioner notes adduction of the right forefoot, with normal position of the mid and hindfoot, along with a convexshaped lateral border of the foot. What will the nurse practitioner do to evaluate this deformity? a. Grasp the heel with one hand and abduct the forefoot with the other hand. b. Observe both legs for medial and lateral rotations. c. Order anteriorposterior and lateral radiographs of both feet. d. Refer the infant immediately to a pediatric orthopedic specialist.

a

12. An adolescent female has heavy periods that are also irregular. The physical exam is normal. A complete blood count reveals a hemoglobin of 8.9 g/dL. What test will the primary care pediatric nurse practitioner order next? a. Coagulation studies b. Creactive protein c. Thyroid function d. Ultrasound of pelvis

a

13. A 14yearold female has menometrorrhagia with moderate increase in menstrual flow and irregular periods. Her hemoglobin is 13.1 g/dL. How will this be managed? a. Iron supplementation and prostaglandin inhibitors b. One OCP twice daily for 3 to 4 days and then daily c. Progestin every day for 10 to 14 days d. Referral to a pediatric gynecologist for treatment

a

13. A previously healthy schoolage child develops herpes zoster on the lower back. What will the primary care pediatric nurse practitioner do to manage this condition? a. Order Burow solution and warm soothing baths as comfort measures. b. Prescribe oral acyclovir 30 mg/kg/day in 4 doses/day for 5 days. c. Recommend topical antihistamines to control itching. d. Stress the need to remain home from school until the lesions are gone.

a

2. The primary care pediatric nurse practitioner is prescribing contraception for an adolescent who has not used birth control previously. The adolescent has a normal exam and has no family history of cardiovascular and peripheral vascular disease or diabetes. Which preparation is used initially? a. A combination oral contraceptive pills (OCP) with 30 to 35 mcg of estrogen and low progestin b. A combination OCP with low androgenic potency, such as OrthoCyclen c. A progestinonly minipill oral contraceptive d. A subdermal implant contraception, such as Implanon or Nexplanon

a

2. What will the primary care pediatric nurse practitioner elicit when obtaining a positive Barlow maneuver when screening for developmental dysplasia of the hip? a. Dislocation of an unstable hip b. Dropping of the iliac crest with a raised leg c. Reduction of a dislocated hip d. Unequal knee heights in a supine child

a

21. An adolescent who had cradle cap as an infant is in the clinic with thick crusts of yellow, greasy scales on the forehead and behind the ears. What will the primary care pediatric nurse practitioner recommend? a. Daily application of ketoconazole 2% topical cream b. Highpotency topical corticosteroids applied daily c. Mineral oil and shampoo on the affected areas d. Selenium sulfide shampoo twice weekly to the face

a

23. A child who has been taking antibiotics is brought to the clinic with a rash. The parent reports that the child had a fever associated with what looked like sunburn and now has "blisters" all over. A physical examination shows coalescent target lesions and widespread bullae and areas of peeled skin revealing moist, red surfaces. What will the primary care pediatric nurse practitioner do? a. Consult with a pediatric intensivist for admission to a pediatric intensive care unit. b. Order oral acyclovir 20 mg/kg/day in two doses for 6 to 12 months. c. Prescribe systemic antihistamines and antimicrobial medications as prophylaxis. d. Recommend analgesics, cool compresses, and oral antihistamines for comfort.

a

3 mo rachel cannot remain seated upright without support. At what age will she acquire the seated position with support? a. 7mo b. 9mo c. 12mo d. 14mo

a

3. An adolescent female tells the primary care pediatric nurse practitioner that she had unprotected sexual intercourse 4 days prior and is worried she might become pregnant. What will the nurse practitioner do? a. Prescribe ulipristal acetate (Ella). b. Recommend levonorgestrel (Plan B One Step). c. Start a combination OCP at regular doses. d. Suggest using the less expensive After Pill preparation.

a

5. A schoolage female has had vulvovaginitis for 2 months. All cultures and tests are negative, but the symptoms persist after treatment with both topical antibiotics and oral amoxicillin. What is the next course of action to treat this condition? a. Estrogen cream at bedtime for 2 to 3 weeks b. Referral to a pediatric gynecologist for further evaluation c. Trimethoprimsulfamethoxazole daily for 1 to 2 months d. Workup for possible sexual abuse

a

5. A toddler is brought to the clinic after grabbing the hot end of his mother's curling iron. An examination reveals a pale, yellow burned area to the palm of one hand. What is true about this burn? a. It may take up to 3 weeks to heal with scarring likely b. Scarring is unlikely, with healing expected in 3 to 7 days c. Surgical intervention and skin grafting are usually required d. This type of burn usually heals without scarring in 7 to 14 days

a

6. A young adolescent female is observed to have mild unequal scapula prominences on gross examination while standing. In the Adams forward bending position, this inequality disappears. What will the primary care pediatric nurse practitioner do? a. Discuss posture and exercise and ask about backpacks and books. b. Obtain radiographic studies of the entire spine and neck. c. Reassure the child's parent that functional scoliosis will selfresolve. d. Refer to an orthopedic specialist for evaluation and possible bracing.

a

7. A child is bitten on one arm by a neighbor's dog. The dog is immunized against rabies and the child's last tetanus immunization was 4 years prior. The wound edges are gaping and avulsed. What is an important initial intervention when treating this injury? a. Administration of rabies prophylaxis and a tetanus booster b. Debriding and suturing the wound to prevent infection c. Irrigation of the wounds with high-pressure normal saline d. Reporting the animal bite to the local animal control authority

a

8. A 14yearold boy who is overweight develops a unilateral limp with pain in the hip and knee on the affected side. An exam reveals external rotation of the hip when flexed and pain associated with attempts to internally rotate the hip. What is most important initially when managing this child's condition? a. Place the child on crutches or in a wheelchair to prevent weightbearing. b. Provide information about weight loss to minimize further injury. c. Recommend seeing an orthopedic specialist as soon as possible. d. Refer the child to physical therapy to improve range of motion.

a

8. A 15yearold female has a positive pregnancy test and asks the primary care pediatric nurse practitioner not to tell her parents. She is tearful and says she isn't sure she wants to keep the baby. What will the nurse practitioner do first? a. Determine the statemandated reporting laws. b. Encourage the adolescent to talk to her parents. c. Obtain a social work consult to discuss adoption options. d. Refer her to a prenatal care specialist for followup.

a

A 14 yo adolescent has been dx with mononucleosis. The PNP should teach the adolescent and the parents that which of the following should be avoided? a. strenuous exercise if the spleen is palpable b. wt-bearing activities until lab test show resolution of the dz c. unnecessary activity until lymph nodes return to normal size d. stretching and reaching activities during the acute stage of illness

a

A 2-month-old infant has symptoms of a severe cough followed by vomiting. The infant had an upper respiratory tract infection 2 weeks before the onset of coughing. The PNP suspects pertussis and orders a chest x-ray film and culture of the nasopharynx. The PNP refers the infant for hospitalization and begins treatment with: A) Erythromycin B) Trimethoprim/sulfamethoxazole C) Corticosteroids and beta-agonist aerosol D) Amoxicillin

a

A 3 wk old infant has a 1 day hx of irritability pallor and poor feeding. He is afebrile. On physical exam, his HR is 240 bpm while asleep. The most likely dx is: a. supraventricular tachycardia b. premature ventricular contractions c. sinus tachycardia d. cyanotic heart defect

a

A 4-year-old child is brought to the office for a preschool physical examination. The PNP asks the child to sit on the floor in a cross-legged position and then get up without using the hands. The child is unable to do so and rolls onto all four extremities before standing. The PNP is concerned that the child may have a: A) Neuromuscular disorder B) Developmental problem C) Cognitive deficit D) Neurologic deficit

a

A 5-year-old child has sudden onset of nonblanching purpuric lesions scattered over the body and petechiae scattered over the neck and shoulders. The mother reports that the child has been healthy, except for a cold a few weeks ago. The child is not taking any medications. Physical examination reveals a healthy, afebrile child with no other significant findings. The laboratory data show a hemoglobin level of 12.5 g/dL, white blood cell count of 6500/mm3, and platelet count of 20,000/mm3. Based on this information, what should the PNP do next? A) Reassure the parents that these findings are consistent with acute idiopathic thrombocytopenia purpura (ITP), and advise a hematology consultation for confirmation B) Refer the child immediately to the pediatric hematology/oncology department of the nearest tertiary care center C) Report the family to the local protective services department as soon as possible because of the possibility of child abuse D) Order additional laboratory tests, including bleeding studies, an autoimmune panel, and an Epstein-Barr titer; more information is needed before a diagnosis can be made

a

A 7-year-old child with cerebral palsy is brought to the clinic. The child is unable to walk but is able to use a wheelchair. The child uses the arms and hands to eat and perform self-care tasks. The PNP diagnoses this child with: A) Spastic diplegia B) Spastic quadriplegia C) Athetoid cerebral palsy D) Spastic hemiplegia

a

A 9-month-old infant was diagnosed with sickle cell disease shortly after birth. The mother telephones the PNP to report that the infant has a fever of 103.2°F. The best response to the mother is: A) "Take the infant to the emergency room immediately." B) "Administer a dose of ibuprofen, and call back in 6 hours if the fever continues." C) "Give extra fluids and acetaminophen, and call back tomorrow if the fever continues." D) "Give extra fluids and acetaminophen, and bring the infant to the clinic tomorrow morning."

a

A NICU graduate with bronchopulmonary dysplasia (BPD) has been discharged to home. A potential problem area that requires close monitoring is: a. insufficient caloric intake b. atrophy of abdominal muscles due to abdominal breathing patterns c. lack of tactile stimuli due to restrictions on parental handling d. the predisposition to development of nasal polyps

a

A PNP has been asked by the emergency room physician to evaluate a child for costochondritis. The PNP would expect to find: A) Tenderness of the midsternal area B) Atelectasis on chest x-ray film C) Dullness on percussion D) Muffled heart sounds

a

A healthy 12yo female is at the clinic for a well checkup. On physical examination, a marked elevation of the right scapula and right thoracic hump and spinal curve abnormally are noted. Spinal films indicate 20 degree curve. The PNP should: a. refer the patient to an orthopedist b. monitor the patient every 3 mo until menarche c. refer the patient for physical therapy d. recommend bedrest and back brace a

a

A mother calls the office and asks what she can do to decrease the discomfort her 9-year-old child is experiencing with an upper respiratory tract infection. The mother should be instructed to: A) Administer pseudoephedrine and normal saline nose drops B) Administer normal saline nose drops and benzonatate C) Place a cool-mist humidifier in the child's room, and administer benzonatate D) Administer dextromethorphan and normal saline nose drops a

a

A tenderness is detected over the tibial tuberosity of a 10 year old boy during a routine exam at a school based clinic. The PNP knows this may be a sign of: a. osgood-schlatter dz b. blount's dz c. plantar fasciitis d. effusion in the joint space

a

AW, at 3 days old, presents with vomiting, abd distention, and constipation. Which of the following should be included in the diff dx? a. hirschsprung dz b. pyloric stenosis c. celiac dz d. meckel diverticulum

a

Adolescents who engage in risky behavior, such as driving without a seat belt, are displaying: a. a type of egocentrism b. a need for independence c. role experimentation d. low self-esteem

a

An adolescent is being evaluated for childhood depression. Which behavior or sign is least likely to be evident? a. evidence of hallucinations and delusions b. a history of recurrent "accidents" c. a sense of guilt d. the presence of eating disorders

a

An essential test in the evaluation of a 2yo child being managed for Kawasaki dx is : a. an echocardiogram b. electrolytes c. cholesterol d. streptoccoccal antibody titer

a

Brent, a 22 mos old, has been brought to the clinic by his mother who says he has been coughing for 2 days and is now making a funny noise when he breathes. Examination reveals a fussy child with a brassy cough and inspiratory stridor. Lips and nail beds are pink. Axillary temp is 103F and RR is 50 bpm. The most likely diagnosis of Brent's condition is: a. laryngotracheobronchitis b. brochiolitis c. respiratory distress syndrome d. reactive airway disease

a

DJ is a 4 yo AA child with a depigmented macular lesion on his forehead. The lesion has sharp borders. No scales are present. The most appropriate treatment would be: a. 1% hydrocortisone b. alpha hydroxy acid c. ketoconazole d. silver sulfadiazine

a

DM, 7 yo, presents with a beefy red macular-papular rash on the diaper area with satellite lesions on the abdomen. the appropriate treatment would be: a. clotrimazole b. A & D ointment c. gentian violet 1 to 2 % d. cornstarch

a

During a check up of a 15 mos old girl, you note that the child has dropped significantly in percentile for weight over the past few months. In evaluating a child with failure to thrive, the most important part of you assessment involves: a. the history b. the physical exam c. laboratory studies d. observation of family interactions

a

Following tympanostomy tube insertion, it is important that the tubes remain patent. Which of the following methods may by used to determine patency? a. visual inspection b. impedance tympanometry c. valsalva maneuver d. instillation of an ototopical suspension

a

HB is 2 days old. Her mother calls and reports a rash consisting of redness with yellow-white "bumps" all over her body except for the palms and soles. The infant most likely has: a. erythema toxicum b. transient neonatal pustular melanosis c. molluscum contagiosum d. milia

a

In infants, the lesions associated with atopic dermatitis are most likely to be distributed on the: a. cheeks and forehead b. wrists and ankles c. antecubital and popliteal fossae d. flexural surfaces

a

In mild to moderate attacks of acute asthma, albuterol should be given every 3 to 4 hrs and routine medications should be: a. continued as usual b. discontinued until albuterol treatments are deemed unnecessary c. given only if the albuterol is ineffective d. be decreased to the minimum recommended dose

a

Initial management of a 4 yo, overweight boy with a repeat total cholesterol level of 185 md/dl includes all of the following excepts: a. educate the family about decreasing saturated fat intake to less than 7% of calories and cholesterol intake to less that 200mg per day b. blood sent for lipoprotien analysis c. educate the family about decreasing/eliminating daily intake of fruit juices, sugar-sweetened food and beverages, and salt intake d. educate the family about increasing aerobic activities

a

Initial treatment for a child with uncomplicated infectious mono should include: a. home care with bed rest progressing to activity as tolerated b. home care with complete bed rest until afebrile c. hospitalization with complete bed rest until lab values return to normal d. hospitalization with daily, planned physical therapy

a

JV, 7 yo, presents with a 1 wk hx of fever, nausea, and anorexia. His mother reports that his skin "looks funny" as well. Further lab studies confirm a dx of viral hepatitis. which type of hepatitis is the most likely? a. A b. b c. C d. D

a

Management of the infant with suspected heart dz and a reported cyanotic spell should include: a. prompt referral to a cardiolgist b. an apnea monitor for 30 days to document further events c. instructing the parent to keep a diary of these episodes d. continuous administation of oxygen

a

Marie, at 4 years old, has been brought to the clinic bc she " has something wrong with her eye." Marie and her mother report that there has been no injury to the eye and that it has been red since yesterday. Exam reveals conjuntival hyperemia and a copious amount of purulent discharge bilaterally. Vision, pupilary reflexes, and corneal clarity are normal. Which treatment should be ordered? a. sodium sulfacetamide ophthalmic solution b. gentamycin ophth sol c. tobramycin ophth sol d. cromolyn sodium ophth sol

a

Mrs. S has brought 10 day old Jacob to the clinic bc she is concerned about his breathing. She says that while she is feeding the baby, he often stops breathing for periods of about 10 secs. History reveals that Jacob eats well, has never appeared pale or cyanotic, and has never become limp during any of the apnea episodes. Your management plan is based on which of the following? a. this is a normal breathing pattern for an infant b. these episodes likely indicate aspiration of formula and should be evaluated c. a variety of pathologic processes are associated with the episodes described d. neurologic deficits in infants are often manifested by such episodes

a

RD, at 14 years of age, presents with a c/o abd pain that has occurred several times over the past 3 mos. She describes the pain as an intermittent sharp pain, occasionally relieved with a heating pad. Her physical exam is WNLs. You suspect recurrent abd pain (RAP). Which clinical finding is most consistent with RAP? a. periumbilical pain b. constipation c. pain worsens with defecation d. wt loss

a

T.T. is a 9 week old preterm infant whose birth wt was 2.3 kg. Mom was HBs Ag Neg. He is seen today for the first time since discharge from the nursery. He has received no prior immunizations. The appropriate immunizations to give at this time would be: a. Dtap, Hib, IPV, rotavirus, HepB 1, pneumo b. Dtap, Hib, OPV, rotavirus, HepB 1 c. Dtap, Hib, rotavirus, IPV, pneumo d. Dtap, Hib, IPV, pneumo

a

The PNP is following a 15-year-old male adolescent with consistent blood pressure readings of 132 to 138/84 to 86 mm Hg, which is classified as significant hypertension. After performing a workup, the PNP determines that the adolescent has primary hypertension. The most judicious recommendation for therapy is: A) Diet and exercise counseling and referral to a specialist B) A diuretic, low-salt diet, exercise, and counseling by a dietician C) Perform an extensive family history to determine other risk factors D) A vasodilator, restricted activity, and a low-fat diet

a

The mother of 2yo Heather has brought her to the clinic with a "bad cough". Hx reveals onset of illness 4 days ago with clear rhinorrhea and coughing. Her mother says that Heather's fever has been as high 103 axillary. Physical exam reveals a temp of 101F and RR of 56 bpm, w/ slight nasal flaring and intercostal, subcostal, and suprasternal retractions. The pharynx is red without tonsillar exudate. Chest auscultation reveals widespread rales and wheezing. The lips and nail beds are slightly pale but pink, skin turgor is good and mucous membranes are moist. The most likely diagnosis of Heather's condition is: a. viral pneumonia b. pneumococcal pneumo c. strep pneumo d. Hib pneumo

a

The mother of 2yo Shanda has brought her to the clinic bc she thinks the child is having trouble hearing. Your evaluation of the compliant should start with: a. asking detailed questions related to Shanda's medical history b. examination of the ear c. tympanometry and hearing tests d. assuring the mother that transient hearing loss in childhood is common

a

The mother of 4 yo TW states that the infant has been irritable and has not been sleeping well. During the physical exam, you note papular lesions on his feet and erythematous papules over his back. Having confirmed the dx of scabies in TW, the treatment of choice would be: a. permethrin 5% b. lindane 1% c. sulfur oitment 6% d. crotamiton 10%

a

The mother of 7 yo Bob has brought him to the clinic bc he has a rash. Physcial exam reveals vesicles on the hands, feet and in the mouth. Hand, foot and mouth dz is diagnosed. Treatment is based on: a. alleviating symptoms b. eradication of causative bacteria c. prevention of febrile convulsions d. prevention of secondary infections

a

The mother of 8 yo John telephones to tell you that he developed chicken pox 3 days ago. She wants to know if there is anything she can do to make him more comfortable. You should tell his mother to: a. apply topical calamine lotion b. apply a topical antibiotic to the new vesicles c. give aspirin for fever and discomfort d. keep John our of bright light

a

While conducting the Denver II developmental screening test, the mother of an 18 mos old child reports to you that the toddler doesn't imitate activities. You decide to assess the child's development further by giving him tasks from which sector? a. personal-social b. fine motor-adaptive c. language d. gross motor

a

While examining 7yo SRs scalp you note three small patches of hair loss. Broken hair is present, as is erythema and scaling. Based on this information, which of the following dx is most likely? a. tinea capitis b. traction alopecia c. trichotillomania d. alopecia areata

a

While listening to 2.5 yo KL talk, you note that she frequently omits final consonants and her sentences are two and three words in length. The appropriate plan of care would be. a. routine follow up at next well child visit b. referring for hearing screening c. assessing for developmental delays d. referring to a speech pathologist

a

You are performing a physical exam on a 12 yo boy who is planning to go to summer camp in a primitive wooded area. What teaching is appropriate for the prevention of Rocky Mountain spotted fever? a. inspect the body several times a day and remove ticks immediately b. inspect the skin while taking a shower and apply soap to ticks before removing c. ticks should be removed only after they have been killed with alcohol d. tick removal should be performed by a healthcare professional

a

You have been treating 14-month-old JV for torticollis since birth. The condition has not resolved. The appropriate management plan would be to: A: referral for surgical consultation B: Continue with passive range of motion C: provide environmental stimulation opposite the contracture D: apply cervical collar at night

a

Growth in muscle length is related to growth in length of: a. underlying bone b. underlying ligament c. underlying tendon d. opposing muscle group

a (growth in muscles is d/t the ROM muscle has to perform as underlying bone lengthens)

Which of the following factors most affects outcomes of patients with LCPD (legg claves perthe dz)? a. age b. severity of pain and antalgic gait c. family hx of LCPD d. bilateral involvment

a (younger children have more time to remodel)

Which things are screened for in the newborn screening? Galactosemia PKU MCADD Congenital adrenal hyperplasia CAH Congenital Hypothyroidism Maple syrup urine disease All of the above

all of the above

1. A child is brought to the clinic after falling from a swing and scraping both knees and hands. An examination reveals abraded skin with oozing serous fluid and blood, along with dirt and grime from the playground surface. What will the primary care pediatric nurse practitioner do to minimize the risk of infection? a. Apply povidone-iodine to all areas b. Irrigate gently with normal saline c. Rinse with hydrogen peroxide d. Scrub the abraded areas with alcohol

b

1. A schoolage child has a fractured wrist with a SalterHarris Type II fracture, according to the radiologist. What is true about this type of fracture? a. Growth disturbance of the long bones of the arm is likely b. There is a metaphyseal fragment on the compression side of fracture. c. There is usually a compression or crushing injury to the physis. d. This will require anatomic reduction using an open approach.

b

1. The primary care pediatric nurse practitioner is discussing newborn care with parents prior to the delivery of their first child. What will the nurse practitioner tell them about circumcision? a. Circumcision is a relatively painless procedure. b. The benefits of the procedure outweigh the risks of the procedure. c. There is a slight increase in penile cancer in circumcised males. d. There is no good evidence to support the practice.

b

1. The primary care pediatric nurse practitioner is teaching a parent of a child with dry skin about hydrating the skin with bathing. What will the nurse practitioner include in teaching? a. Apply lubricating agents at least 1 hour after the bath. b. Have the child soak in a lukewarm water bath. c. Keep the child in the bath until the skin begins to "prune." d. Soaping should be done at the beginning of the bath.

b

1. The primary care pediatric nurse practitioner needs to assess a potential hymenal tear in a prepubertal female who is apprehensive about the exam. Which approach will the nurse practitioner use? a. Have the child sit froglegged on the parent's lap. b. Place the child in the kneechest position on the exam table. c. Put the child supine on the exam table with her feet in the stirrups. d. Refer the child for a speculum exam under sedation.

b

11. An adolescent female has periods every 30 days that are consistently heavy and last from 5 to 8 days. What is her diagnosis? a. Menometrorrhagia b. Menorrhagia c. Metrorrhagia d. Polymenorrhea

b

14. A sexually active adolescent female tests positive for N. gonorrhoeae and C.trachomatis. She tells the primary care pediatric nurse practitioner that she wants to be treated today since she is moving out of town the next day. What will the nurse practitioner order? a. Azithromycin 1 g PO in a single dose b. Ceftriaxone 250 mg IM and azithromycin 1 g PO one time each c. Doxycycline 100 mg PO bid for 7 days d. Erythromycin base 500 mg PO qid for 7 days

b

16. A 3yearold child has head lice. What will the initial treatment recommendation be to treat this child? a. Lindane b. Permethrin c. Pyrethrin d. Spinosad

b

17. A 9monthold infant has vesiculopustular lesions on the palms and soles, on the face and neck, and in skin folds of the extremities. The primary care pediatric nurse practitioner notes linear and Sshaped burrow lesions on the parent's hands and wrists. What is the treatment for this rash for this infant? a. Ivermectin 200 mcg/kg for 7 to 14 days, along with symptomatic treatment for itching b. Permethrin 5% cream applied to face, neck, and body and rinsed off in 8 to 14 hours c. Treatment of all family members except the infant with permethrin 5% cream and ivermectin d. Treatment with permethrin 5% cream for 7 days in conjunction with ivermectin 200 mcg/kg

b

2. A schoolage child steps on a nail while wearing tennis shoes and develops cellulitis in that foot. The child's immunizations are uptodate. What antibiotic will the pediatric nurse practitioner empirically prescribe? a. Amoxicillin-clavulanate b. Ciprofloxacin c. Clindamycin d. Trimethoprim-sulfamethoxazole

b

27. The primary care pediatric nurse practitioner notes velvety, brown thickening of skin in the axillae, groin, and neck folds of an adolescent Hispanic female who is overweight. What is the initial step in managing this condition? a. Consultation with a pediatric dermatologist b. Performing metabolic laboratory tests c. Prescribing topical retinoic acid cream d. Referral to a pediatric endocrinologist

b

3. A 3yearold child is brought to the clinic by a parent who reports that the child refuses to use the right arm after being swung by both arms while playing. The child is sitting with the right arm held slightly flexed and close to the body. There is no swelling or ecchymosis present. What will the primary care pediatric nurse practitioner do? a. Consider maltreatment as a possible cause of injury. b. Gently attempt a supination and flexion technique. c. Immobilize the arm with a sling and refer to orthopedics. d. Obtain a radiograph of the child's right arm and elbow.

b

3. A wellbaby examination of a 3dayold infant born to a primigravida mother reveals swelling on the right parietal area of the scalp that stops at the suture line. What action will the primary care pediatric nurse practitioner take based on this finding? a. Assess the infant for sacral dimpling. b. Observe the infant for hyperbilirubinemia. c. Obtain a computerized tomography scan. d. Perform serum coagulation studies.

b

4. A preschool age child has honeycrusted lesions on erythematous, eroded skin around the nose and mouth, with satellite lesions on the arms and legs. The child's parent has several similar lesions and reports that other children in the day care have a similar rash. How will this be treated? a. Amoxicillin 40 to 5 mg/kg/day for 7 to 10 days b. Amoxicillinclavulanate 90 mg/kg/day for 10 c. Bacitracin cream applied to lesions for 10 to 14 days d. Mupirocin ointment applied to lesions until clear

b

5. A child is brought to clinic with several bright red lesions on the buttocks. The primary care pediatric nurse practitioner examines the lesions and notes sharp margins and an "orange peel" look and feel. The child is afebrile and does not appear toxic. What is the course of treatment for these lesions? a. Hospitalize the child for intravenous antibiotics and possible I&D of the lesions. b. Initiate empiric antibiotic therapy and follow up in 24 hours to assess response. c. Obtain blood cultures prior to beginning antibiotic treatment. d. Perform gram stain and culture of the lesions before initiating antibiotics.

b

5. The parent of a 4-week-old infant reports that the infant began having forceful vomiting 1 week prior, which has worsened over time. The infant continues to nurse well but is losing weight. A physical examination reveals a 90g weight loss over the past 2 weeks, dry mucous membranes, and a sunken fontanel. What will the primary care pediatric nurse practitioner do? a. Encourage the mother to nurse the infant more frequently for shorter duration. b. Obtain serum electrolytes and hospitalize for surgical intervention. c. Recommend oral rehydration fluids for 24 to 48 hours to correct dehydration. d. Suggest trying a soybased or hydrolyzed protein formula until vomiting resolves.

b

6. A 5dayold infant who was delivered at home has abdominal distension and poor feeding. The mother is worried that the infant is constipated because he didn't have a first stool until yesterday and has only passed a small amount of meconium. What will the primary care pediatric nurse practitioner do? a. Obtain a sweat chloride skin test to evaluate for possible cystic fibrosis. b. Order an abdominal radiograph and refer the infant to a pediatric surgeon. c. Prescribe glycerin suppositories to use as needed until bowel function is normal. d. Suggest that the mother increase her fluid intake to help with constipation.

b

7. The parent of a 4dayold infant tells the primary care pediatric nurse practitioner that the infant was diagnosed with hydronephrosis while in utero and asks what will be done. What will the nurse practitioner tell this parent? a. Renal function will be abnormal and will require lifetime treatment. b. Spontaneous resolution often occurs within 6 months to a year of age. c. The affected kidney will be nonfunctional but the other kidney will compensate. d. The infant will eventually require renal transplantation for that kidney.

b

8. A 2monthold infant has increased head circumference from the 10th percentile at the 2week exam to the 30th percentile today. What will the primary care pediatric nurse practitioner do? a. Order a magnetic resonance imaging exam of the infant's head. b. Refer the infant immediately to a pediatric neurosurgeon. c. Schedule frequent clinic visits to monitor head growth. d. Watch the infant closely over time if the rest of the exam is normal.

b

A 1 wk old infant has been diagnosed with nasolacrimal duct obstruction. A typical initial therapy includes: a. use of prophylactic oral antibiotics b. nasolacrimal sac massage c. surgical opening of obstructed ducts d. referral to an ophthalmologist

b

A 12 yo girl seen at a routine visit has a bp of 145/90. She denies any symptoms. The initial management would include: a. intravenous pyelogram b. return for two repeat blood pressure measurements c. no f/u needed--blood pressure probably related to anxiety d. diuretic therapy

b

A 16 yo boy has been dx with measles (rubeola). He is also c/o ear pain. His tympanic membranes are red and bulging. Appropriate mgmt of the ear problem is to treat the ears with: a. pain medication until the virus has run its course b. the same med used to treat any case of otitis media c. liquid topical antibiotics and topical steroids d. acyclovir given by mouth

b

A 17-year-old adolescent is anticipating marriage after completing high school. The adolescent does not have sickle cell disease and is not a carrier of the trait, but the intended spouse has sickle cell trait. The adolescent asks if it is possible that any of their future children will have sickle cell disease. The adolescent is told that there is a 50% chance that: A) Their children will have sickle cell disease B) Their children will have sickle cell trait C) Only their sons will have sickle cell disease D) Only their sons will have sickle cell trait

b

A 4yo caucasian female presents for a well-child check. Her bp by auscultation is 135/80. There is no significant family hx of HTN or heart dz. Besides wanting to repeat the bp for accuracy, what are potential differential diagnoses for this child? a. severe coronary artery dz b. renal artery stenosis and coarctation of the aorta c. rheumatic fever and ventricular septal defect d. tetralogy of fallot and coarctation of the aorta

b

A 9 yo boy presents with a fever of 102 and complains of leg pains. His mother reports that he had an URI with a sore throat approximately 2 wks ago which subsided without therapy. On physical exam, he has tender swollen knees bilaterally. His heart rate is 120/min and a blowing systolic murmur is heard at the apex that was not noted on previous visits. The most likely diagnosis is: a. kawasaki dz b. rheumatic fever c. sickle cell anemia d. viral infection

b

A child with growing pains is most likely to experience: a. mild limp b. bilateral lower extremity pain c. lower extremity pain primarily during the day d. lower extremity pain associated with decreased ROM

b

A father accompanies his 4-year-old child to the clinic. Significant past history reveals that the child had lipomyelomeningocele. The father states that there are no specific complaints, but during the history he reports that the child has complained of back pain over the last 3 months. Further evaluation reveals that the frequency of bowel movements has decreased from once daily to three times a week and that the child is "wetter" between catheterizations. The most likely cause is: A) Acute muscle strain B) Tethered cord C) Urinary tract infection D) A diet low in fiber, fruits, and vegetables

b

A mother of an 8 month old infant asks you for advice about continued introduction of solids. Which of the following food groups do you recommend be introduce to the baby last? a. egg yolk b. egg white c. fruits d. vegetables

b

A vibratory systolic murmur is heard between the lower left sternal border and the apex in a healthy 4yo girl at her preschool physical. There are no concerns from the parent. The cardiovascular exam is otherwise normal. A likely diagnosis: a. venous hum b. still's murmur c. transposition of the great arteries d. rheumatic heart disease

b

An adolescent is being seen at a community health center because of recurrent respiratory tract infections. The complete blood cell count (CBC) with differential shows a white blood cell count of 20,500 with 35% blast cells. The next step in managing the adolescent's care is to: A) Repeat the CBC with differential in 1 week B) Refer to a specialist in pediatric hematology C) Perform bone marrow aspiration D) Hospitalize the adolescent immediately

b

An injury at which of the following sites will most likely result in a bone length discrepancy? a. diaphysis b. epiphysis c. medullary cavity d. metaphysis

b

At 12 yo, Peter has been dx with constitutional growth delay. Appropriate management would include: a. starting low-dose testosterone therapy now b. counseling regarding delayed onset of puberty c. thyroxine replacement d. nutritional counseling

b

Chest pain in young children is usually: a. a symptom of congenital heart disease b. noncardiac in origin c. an early sign of hypercholesterolemia d. a symptom of CHF

b

Connor, 12 mos of age, has been treated 5 times for AOM. When planning Connor's f/u care, it is most important to evaluate for which of the following? a. otitis externa b. hearing loss c. enlarged tonsils d. shotty lymph nodes

b

During 15 yo NMs routine physical exam, she c/o getting pimples all the time. You note open and closed comedones over her forehead and chin. There are more than 15 papules and pustules, but no cysts. Which of the following statements is not consistent with an appropriate management plan for acne? a. improvement with use of keratolytic agents should occur within 4-6 wks b. facial scrubs are recommended before applying topical antibiotics c. noncomedogenic moisturizers and cosmetics may be used d. sunscreens should always be used in conjunction with retinoic acid

b

EG, at 9 years of age, presents with diffuse abd pain and acute onset of diarrhea described as a frequent urge to defecate. She is passing large amounts of flatus, small amounts of stool, and c/o tenderness during rectal examination. Which of the following lab test would confirm your dx for EG? a. serum albumin b. abd ultrasound c. stool for ova and parasites d. bone age

b

In the emergency room, you encounter a toddler whose injuries are not consistent with the history that is given. Which of the following would be the best step a provider could take in order to foster communication with abusive parents? a. realize that abusive parents have essentially different goals for their children than other caregivers do b. understand that parental hostility and resistance are potent symptoms of fear and inadequacy c. consider referring the parents to substance abuse program d. be cautious when sharing the results of medical findings

b

Kawasaki dz is most common in: a. neonates b. children younger than 5 years of age c. children older than 6 years of age d. females

b

Mrs. Franklin is concerned about a light pink lesion on the back of 2 mos old Aaron's neck that darkens with crying. The description is consistent wtih: a. sturge-weber dz b. salmon patch c. port-wine stain d. hemangioma

b

SL, at 9 years of age, is brought to the clinic for eval of abd pain that wakes her at night. Her parents have recently divorced an she is attending a new school. She has missed 8 days of school in the past 6 wks. She reports occasional emesis. An appropriate management plan would be: a. a band diet with small frequent feedings b. a referral to a gastroenterologist c. to stress importance of school attendance d. to consult with the school psychologist

b

Sam's mother has telephoned the clinic because chicken pox has been "going around" at Sam's school and she has just noticed a few red spots along the hairline on his face. She asks if there is anything that can be given to shorten the duration or severity of the illness. Which of the following is most accurate? a. diphenhydramine elixir has been shown to shorten the duration and severity of the rash b. acyclovir has been shown to shorten the duration and severity of the illness c. aspirin taken 4x a day has been shown to shorten the duration and severity of the illness d. there is no medication known to alter the course of the illness

b

Sam, at 8 years of age, presents to your clinic with his third episode of diarrhea in 3 mos. He also c/o frequent flatulence and a decreased appetite. After going over the hx, you recall that he went on a camping trip with his Cub scout troop several months ago. What is the most likely dx? a. celiac dz b. giardiasis c. rotovirus d. crohn's dz

b

Stephanie, at 1 year old, is infected with HIV, but is asymptomatic at this time. When should her MMr immunization be given? a. according to the usual schedule b. after she has been asymptomatic for 1 year c. only if she attends day care d. MMR should not be given to her

b

TJ, 13 yo, reluctantly shares with you that his "chest hurts." On physical exam, you note unilateral breast enlargement, which is tender to palpation. You suspect physiolgic gynecomastia. Which tanner stage would support that diagnosis? a. tanner stage I b. tanner stage III c. tanner stage IV d. tanner stage V

b

The PNP in the Emergency Departmetnt is evaluating a 3-year-old child with a history of sudden onset of high fever and stridor. The child refuses to move and is sitting upright, leaning forward with the mouth open, and drooling. The safest and most helpful diagnostic test to order now would be: A) Urine culture B) A lateral neck x-ray film C) A chest x-ray film D) A throat culture

b

The PNP is teaching a group of expectant parents about infant care and illness prevention. It is most important for the PNP to stress: a. keeping all animals out of the house b. keeping the infant away form cigarette smoke c. keeping the infant well covered at night d. keeping the infant away from crowds

b

The diagnois of AOM in a 1 yo child is based on: a. abnormal findings when pneumatic otoscopy and hearing test are performed b. changes in the tympanic membranes contour, color, and mobility c. presence of fever and color of the tympanic membrane d. presence of fever, ear pain, and tenderness of the pinna

b

The first step in the process of developing an IEP for a child with disabilities is to: A) Provide government funds to the child and family to supplement income B) Perform a comprehensive assessment of the child by a multidisciplinary team C) Perform a complete history and physical examination to identify organic causes D) Perform a battery of psychologic tests that are valid and reliable

b

The initial attack of acute rheumatic fever is preceded by: a. a viral illness b. a group A streptococcal infxn c. an exposure to mites d. an exposure o chicken pox

b

The mother of 5yo DW is concerned that her son often cheats when playing board games with his older sister. What is the most appropriate response to DWs behavior? a. encourage the parent to use 5 minute time-outs when cheating occurs b. explain that DW is developmentally unable to comprehend rigid rules c. make sure that DW understands the rules before starting to play the game d. explain to DW that cheating is like lying and is not acceptable behavior

b

The mother of 8 yo John telephones to tell you that he developed chicken pox 3 days ago. She wants to know if there is anything she can do to make him more comfortable. You should also tell his mother to: a. avoid getting the lesions wet b. encourage him to take a bath every day c. have John take a bath only if he develops a fever and sweats d. encourage John to take only sponge baths until all lesions are healed

b

The parents of a 3-year-old child have noticed that the child experiences episodes of abrupt, brief loss of consciousness many times during the day. The PNP suspects that the child is having: A) Status epilepticus B) Absence seizures C) Atonic seizures D) Myoclonic seizures

b

The parents of an 8yo child are concerned that their son does not want to attend school. Which of the following historical findings aren't usually associated with the diagnosis of school phobia? a. sporadic school absence b. chronic medical illness c. vague physical symptoms d. depression and anxiety

b

The parents of an adolescent with pectus excavatum and asthma are interested in surgery for their child, in the hopes of improving his asthma. Which would be MOST accurate response? a. the only purpose of surgery is cosmetic b. studies of post-operative cardiac and pulmonary functioning following pectus repair show inconsistent results. c. reduction of lung volume and degree of symptoms correlates well with extent of the deformity d. improvement of asthma after surgery is dependent on the family history

b

The principle that growth and development becomes increasingly integrated is best demonstrated by: a. gaining head control before raising the chest b. bringing the cup to mouth, tipping and swallowing c. rolling over before sitting d. grasping with fist before using fingers

b

Vomitus that is bilious suggests: a. GI obstruction proximal to the pylorus b. GI obstruction below the ampulla of vater c. pyloric stenosis d. peptic ulcer dz

b

When educating parents regarding transmission of RSV, it is important to stress which of the following? a. children with RSV should be totally isolated from other children b. RSV can be spread by airborne droplets or from contact with a hard surface that has been contaminated c. children who attend day care centers should take prophylactic antibiotics early each fall d. wiping hard surfaces with soap and water or disinfectant will not help in the prevention of RSV transmission

b

Which of the following represents appropriate anticipatory guidance for a child diagnosed with slipped capital femoral epiphysis? a. avoid contact sports until pain resolves b. crutches to facilitate mobility during acute phase c. apply ice to affected area d. ROM and strengthening exercises

b

Which of the following secondary skin changes is not associate with atopic dermatitis? a. lichenification (thick, leathery patches of skin) b. striae c. pigment changes d. excoriations

b

Which of the following statements regarding treatment of pediculosis capitis is true? a. carpeting and furniture must be shampooed and sprayed with a pediculicide b. nonwashable items that have come into contact with an infected person should be sealed in plastic bags for 2 - 4 wks c. hair must be trimmed close to the scalp to insure elimination of nits d. frequent shampooing with permethrin 1 % will prevent reinfestation

b

While examining 10 yo RMs teeth you note that the upper incisors slightly overlap the lower incisors. The second and lower first molars are absent. Your assessment is: a. malocclusion b. delayed mandibular dentition c. normal dentition d. hyperdontia

b

8. Which of the following is not a signs or symptoms of congenital hypothyroidism? a-hoarse cry b-frequent stooling c-course features D-lethargy

b-frequent stooling

9. A 9 yo boy presents with a fever of 102 F and complaint of leg pain. His mother reports that he had an upper respiratory infection with a sore throat about 2 weeks ago, which subsided without therapy. On physical examination he has tender swollen knees. Hr is 120 and a blowing systolic murmur is heard at the apex. No murmur was noted at the previous well visit. The most likely diagnosis is: a-kawasaki disease b-rheumatic fever c-sickle cell anemia d-viral illness

b-rheumatic fever

10. A 16yearold female has not had a menstrual period yet and is concerned. She denies sexual activity. An exam reveals an adult sexual maturity rating. Which laboratory test will the primary care pediatric nurse practitioner order initially? a. Genetic test for Turner syndrome b. Pituitary hormone tests c. Pregnancy test d. Thyroid function tests

c

14. A child has small, firm, fleshcolored papules in both axillae which are mildly pruritic. What is an acceptable initial approach to managing this condition? a. Application of trichloroacetic acid 25% to 50% using a dropper b. Applying liquid nitrogen for 2 to 3 seconds to each lesion c. Reassuring the parents that these are benign and may disappear spontaneously d. Referral to a dermatologist for manual removal of lesions with curettage

c

15. A 16yearold sexually active female has a fever, bilateral lower abdominal pain, and malaise. A speculum and bimanual exam reveals adnexal tenderness. The urinalysis is normal and cervical cultures are pending. What medications will the primary care pediatric nurse practitioner prescribe for this patient? a. Azithromycin, doxycycline, and penicillin b. Cefotaxime, azithromycin, and penicillin c. Ceftriaxone, doxycycline, and metronidazole d. Doxycycline, penicillin, and metronidazole

c

15. A schoolage child is brought to clinic after a pediculosis capitis infestation is reported at the child's school. If this child is positive, what will the primary care pediatric nurse practitioner expect to find on physical examination, along with live lice near the scalp? a. Excoriated macules along the child's collar and underwear lines b. Inflammation and pustules on the face and neck c. Itching of the scalp, with skin excoriation on the back of the head d. Linear or Sshaped lesions in webs of fingers and sides of hands

c

18. An adolescent has acne with lesions on the cheeks and under the chin. Which distribution is this? a. Athletic b. Frictional c. Hormonal d. Pomadal

c

19. An adolescent has acne characterized by papules and pustules mostly on the forehead and chin. What will the primary care pediatric nurse practitioner prescribe? a. Azelaic acid applied daily at nighttime b. Benzoyl peroxide applied twice daily c. Topical erythromycin with benzoyl peroxide. d. Tretinoin applied nightly after washing the face

c

22. A child is brought to the clinic with a generalized, annular rash characterized by raised wheals with pale centers. On physical examination, the child's lungs are clear and there is no peripheral edema. A history reveals ingestion of strawberries earlier in the day. What is the initial treatment? a. Aqueous epinephrine 1:1000 subcutaneously b. Cetirizine once in clinic and then once daily for 2 weeks c. Diphenhydramine 0.5 to 1 mg/kg/dose every 4 to 6 hours d. Prednisone 1 to 2 mg/kg/day for 1 week with rapid taper

c

24. A schoolage child has a rash without fever or preceding symptoms. Physical examination reveals a 3cm ovoid, erythematous lesion on the trunk with a finely scaled elevated border, along with generalized macular, ovoid lesions appearing in a "Christmas tree" pattern on the child's back. What is the initial action? a. Obtain a KOH preparation of a skin scraping to verify the diagnosis. b. Prescribe topical steroid creams to shorten the course of the disease. c. Reassure the child's parents that the rash is benign and selflimited. d. Recommend topical antihistamines and emollients to control the spread.

c

26. During a well child examination of an infant, the primary care pediatric nurse practitioner notes 10 café au lait spots on the infant's trunk. What is the potential concern associated with this finding? a. Endocrine disorders b. Malignancy c. Neurofibromatosis d. SturgeWeber syndrome

c

28. An AfricanAmerican child has recurrent tinea capitis and has just developed a new area of alopecia after successful treatment several months prior. When prescribing treatment with griseofulvin and selenium shampoo, what else will the primary care pediatric nurse practitioner do? a. Monitor CBC, LFT, and renal function during therapy. b. Order oral prednisone daily for 5 to 14 days. c. Perform fungal cultures on family members and pets. d. Prescribe oral itraconazole or terbinafine.

c

3. When prescribing topical glucocorticoids to treat inflammatory skin conditions, the primary care pediatric nurse practitioner will a. initiate therapy with a highpotency glucocorticoid. b. order lotions when higher potency is necessary. c. prescribe brandname preparations for consistent effects. d. use fluorinated steroids to minimize adverse effects.

c

4. A schoolage child falls off a swing and suffers a closed fracture of the righ clavicle. How will this be managed? a. Application of a figureeight clavicle brace for 6 to 8 weeks b. Hospitalization for traction of the affected extremity and shoulder c. Immobilization with a sling to support the affected extremity d. Referral to an orthopedic specialist for possible surgical reduction

c

4. The parent of a 2weekold infant reports that the infant was diagnosed with transient tachypnea of the newborn shortly after birth. The primary care pediatric nurse practitioner understands that, in this condition, a. antibiotic therapy is generally necessary to prevent severe infection. b. home oxygen therapy is needed until the infant grows out of the condition. c. recovery is usually complete with minimal intervention and treatments. d. treatment with exogenous surfactant and mechanical ventilation is needed.

c

5. A young adolescent reports chest pain associated with coughing and lifting. Physical examination reveals tenderness over several ribs, radiating to the back. Auscultation of the heart, lungs, and abdomen are normal. There is no history of injury. What will the primary care pediatric nurse practitioner do? a. Obtain a chest radiograph to evaluate possible causes for these symptoms. b. Order an electrocardiogram to rule out potential cardiovascular disease. c. Recommend NSAIDs, stretching exercises, and ice packs to the area. d. Refer the child to a pediatric orthopedist for evaluation and treatment.

c

7. The primary care pediatric nurse practitioner elicits positive Ortolani and Barlow signs in a 6monthold infant not previously noted in the medical record. What is the correct treatment? a. Pavlik harness b. Spica cast c. Surgical intervention d. Triple diapering

c

9. A 16yearold female reports breast tenderness and a "lump." The primary care pediatric nurse practitioner palpates a small fluidfilled mass in her right breast. A pregnancy test is negative. Which action is correct? a. Obtain a CBC to rule out infection. C. Order an ultrasound of the mass. D. Prescribe NSAIDs to treat her discomfort. E. Reassure her that the findings are normal.

c

9. A 3dayold infant has a total serum bilirubin (TSB) level of 15.7 mg/dL after having a TSB of 10.8 mg/dL 24 hours prior. The infant nursed 8 times, had 7 wet diapers, and passed 4 stools in the past 24 hours. What is the indicated treatment for this infant? a. Admit to an inpatient setting for phototherapy and every12hour bilirubin monitoring. b. Have the mother supplement with extra fluids and return to the clinic in 24 hours for a repeat TSB. c. Recommend nursing every 2 hours, order a biliblanket, and recheck TSB in 24 hours. d. Suspend breastfeeding for 24 to 72 hours and have the mother save pumped breast milk.

c

9. A child has several circular, scaly lesions on the arms and abdomen, some of which have central clearing. The primary care pediatric nurse practitioner notes a smaller, scaly lesion on the child's scalp. How will the nurse practitioner treat this child? a. Obtain scrapings of the lesions for fungal cultures. b. Order prescriptionstrength antifungal creams. c. Prescribe oral griseofulvin for 2 to 4 weeks. d. Recommend OTC antifungal creams and shampoos.

c

A 10 mos old infant presents with a 3 day hx of fever, runny nose and coughing. Per the parent, has not been eating well, and only drinking a few cups of milk and juice over the last 2 days. On exam, he feels very warm, his RR is 45 and his HR is 210. He has no rashes. His abd is soft and round. Your initial dx includes: a. SVT based on HR; needs immediate transfer b. CHF based on HR and RR, need immediate transfer c. Sinus tachycardia based on clinical hx and HR; needs antipyretics and possibly fluids d. kawasaki dz based on clinical hx and fever; needs transfer for IV IgG

c

A 10-month-old infant is noted to have cranial and facial asymmetry during a well-child visit. The PNP should: A) Observe the infant, and reevaluate cranial shape at the 1-year well visit B) Refer immediately to a neurologist C) Obtain a cranial radiologic film, CT scan of the skull, or both D) Perform a Denver Developmental Screening Test

c

A 10-year-old child is brought to the office with coughing, abdominal pain, splinting on the right side, fever, headache, and chills. The child appears to be in severe distress. There are decreased breath sounds, crackles, and dullness to percussion in the area of the right middle and right lower lobe. The diagnosis of pneumonia is made. The PNP orders an anteroposterior and lateral view of the chest and: A) Sputum stain B) Blood culture C) Complete blood cell count and blood culture D) Blood gases and pulse oximetry

c

A 12 yo boy is brought into the clinic for an urgent visit after having ingested 10 diazepam tablets. Following the initial emergent care and stabilization of the child, the most important aspect of your management is: a. referring the case to social services b. assessing the family support available to the child c. obtaining a psychiatric consultation d. reviewing the history for signs of depression

c

A 12-year-old child comes to the school-based clinic complaining of unilateral chest pain that radiates to the back and abdomen. On physical examination, tenderness over the costochondral junction is noted. The chest pain is most likely caused by: A) A fractured rib B) Gastric reflux C) Costochondritis D) Pneumonia

c

A 14-year-old adolescent has pain in the knee. The pain increases with activity and is relieved with rest. The PNP diagnoses Osgood-Schlatter disease and orders: A) An x-ray examination, application of hot packs to the knee, and rest B) Application of hot packs to the knee, aspirin, and rest C) A reduction in activity, application of ice to the knee, and ibuprofen D) Application of ice to the knee and continued participation in sports

c

A 14-year-old adolescent, who appears to be in acute distress and is anxious, is brought to the clinic with symptoms of high fever, chills, malaise, pharyngitis, vomiting, peripheral cyanosis, tachypnea, tachycardia, low blood pressure, and erythroderma. The PNP recognizes this as toxic shock syndrome and: A) Orders a CBC and blood culture immediately B) Orders a CBC and blood culture, and sends the adolescent to an emergency room for a lumbar puncture C) Sends the adolescent to the emergency room by ambulance immediately without providing any treatment in the clinic D) Collaborates with the clinic physician to determine appropriate antibiotic use in this patient

c

A 16-year-old adolescent comes to the school-based clinic with chest pain. The most common cause of cardiac pain in children and adolescents is: A) Myocarditis B) Tachycardia C) Mitral valve prolapse D) Preventricular contractions

c

A 16-year-old adolescent comes to the school-based clinic with chest pain. The most common cause of cardiac pain in children and adolescents is: A) Myocarditis B) Tachycardia C) Mitral valve prolapse D) Preventricular contractions c

c

A 4-year-old child is scheduled for a tonsillectomy and adenoidectomy. The preoperative laboratory tests indicate a prolonged active partial thromboplastin time (aPTT). The PNP should suggest that they: A) Continue with the surgery, and monitor the child closely for bleeding complications B) Cancel the surgery, and recheck the aPTT in 1 week C) Cancel the surgery, and refer the child to a hematologist D) Obtain a family history, and determine whether there are other relatives with a bleeding disorder c

c

A 6 mos old boy is brought to the clinic bc he has been coughing since yesterday. His mother states that he has never been sick before. She thinks he has been afebrile but is not sure. Physical exam reveals a well-developed baby with a RR of 50 bpm, mild retraction, wheezes, and a dry cough. Chest radiograph reveals diffuse hyperinflation and patchy areas of infiltration. These findings are most consistent with a dx of: a. laryngotracheobronchitis b. cystic fibrosis c. bronchiolitis d. respiratory distress syndrome

c

A 6-month-old infant is brought to the clinic for a well-child visit. On physical examination, increased tone and lower leg scissoring are noted. The PNP recognizes this as a probable sign of: A) Muscular dystrophy B) Down's syndrome C) Cerebral palsy D) Fragile X syndrome

c

A 6-year-old child with asthma is brought to the clinic because the parents have noticed daily wheezing for the past week, especially during the night. The PNP recognizes that this may indicate the child has: A) Intermittent asthma B) Mild persistent asthma C) Moderate persistent asthma D) Severe persistent asthma

c

A 7 yo AA female presents with several hyperkeratotic raised, periungal lesions on the two middle fingers of her left hand. She has a history of nail biting. The most likely dx is: a. impetigo b. molloscum contagiosum c. verruca vulgaris d. herpetic whitlow

c

A child is brought to the clinic after being hit in the head with a hockey stick. The child now has profuse rhinorrhea but is in no distress. The examination reveals a hemotympanum, which is indicative of: A) A foreign body in the canal B) Perforation of the tympanic membrane C) A basilar skull fracture D) Acute serosanguineous otitis media c

c

An 8 yo boy has been brought to the clinic with a chief c/o ear pain. When you grasp the pinna of he ear, he says "that hurts real bad." These findings are consistent with a diagnosis of: a. serous otitis media b. mastoiditis c. otitis externa d. cholesteatoma

c

An 8yo afebrile child presents with a ST. Upon examination, you note that his tonsils are 4+ without exudate. Differential diagnosis includes: a. normal b. strep throat c. hodgkin disease d. phayngitis

c

An umbilical hernia: a. occurs more frequently in full-term infants b. resolves spontaneously in 3 to 6 months c. is frequently associated with diastasis recti d. responds well to taping

c

Brent, a 22 mos old, has been brought to the clinic by his mother who says he has been coughing for 2 days and is now making a funny noise when he breathes. Examination reveals a fussy child with a brassy cough and inspiratory stridor. Lips and nail beds are pink. Axillary temp is 103F and RR is 50 bpm. Which diagnostic test should be ordered first for Brent? a. pulmonary function tests b. throat culture c. radiograph of the upper airway d. laryngoscopic exam

c

CR, 4 yo, was dx with celiac dz a 18 mos. In addition to closely monitioring her growth, you also monitor for anemia. At this visit her lab results confirm an elevated MCV. An appropriate f/u lab test would be serum: a. protein b. ferritin c. folate d. transferrin

c

Characteristics of a venous hum include: a. a systolic murmur b. radiation over precordium c. marked decrease or disappearance of murmur when child is supine d. heard best at lower left sternal border

c

During 15 yo NMs routine physical exam, she c/o getting pimples all the time. You note open and closed comedones over her forehead and chin. There are more than 15 papules and pustules, but no cysts. NMs clinical presentation is consistent with: a. comedonal acne b. mild acne c. moderate acne d. severe acne

c

During 15 yo NMs routine physical exam, she c/o getting pimples all the time. You note open and closed comedones over her forehead and chin. There are more than 15 papules and pustules, but no cysts. Which of the following medications below is the appropriate choice? a. antiandrgens b. isotretinoin c. minocycline d. corticosteroids

c

During 3 yo JTs physical exam, you observe eight, light brown macules, ranging in size from 0.5 to 0.75 cm on his trunk, arms, and legs. Your management plan would be to: a. educate the family to apply sunscreen frequently b. explain that the lesions will fade with time c. refer to dematologist d. document the findings an reevaluate in 6 mos

c

During a physical exam of 10.5 yo Melissa, you note the appearance of breast buds. You tell her that she can expect which of the following in approximately 2 years? a. growth of pubic hair b. peak height velocity c. onset of menses d. axillary hair

c

During exam of a 2 wk old JP you note irritability when lifted, asymmetrical Moro reflex, and spasm along the right sternocleidmastoid. What does this suggest? a. torticollis b. sprengel deformity c. fractured clavicle d. klippel-feil syndrome

c

During the routine exam of a 12 yo boy, you detect a group of hard fixed nontender lymph nodes, each of which measure about 1 cm, in the posterior cervical chain. You are unable to detect any signs of infection. Your management should include: a. recording the finding and reassessing the nodes in 1 month b. ordering a 10d course of a broad spectrum abx and reevaluating the nodes in 2wks c. ordering a CBC, ESR, and chest radiograh d. referring the child to and allergist

c

In the management of a child with bronchiolitis, the early use of which of the following is likely to be the most benficial? a. antihistamines b. broad spectrum antibiotics. c. fluids and nutritional support d. bronchodilators

c

JD, at 7 mos of age, is seen with a 2 day hx of diarrhea. He has had 3 to 4 wet diapers in the past 24 hrs. The anterior fontanel is slightly depressed. Capillary refill is normal. Based on your assessment of JD, the appropriate management plan for his dehydration would be to: a. begin BRAT diet b. withhold formula for 24 hrs and give electrolyte solution c. begin rehydration in the office and observe for 3 to 4 hrs d. refer immediately for parenteral fluids

c

SBE prophylaxis is recommended for: a. all children with congenital heart disease on a daily basis b. all children with cong. heart dz before dental, GI, and GU procedures c. children with repaired cong. heart dz with a residual defect at the repair site d. children who have undergone surgical repair of a cong. heart condition for 5 yrs after the procedure

c

The mother of 2yo Bridget has brought her to the clinic because "she got bathroom cleaner in her eye." Hx reveals that about 30 min ago, Bridget was sitting on the floor playing with a squeeze bottle of bathroom cleaner, when the bottle accidentally opened and the liquid splashed into her right eye. Physical exam reveals a reddened right eye with an edematous lid. Initial treatment should include: a. allowing the natural tearing process to cleanse the eye b. performing a retinal fundoscopic exam to assess for burns c. irrigating the eye with copious amount of normal saline d. referring Bridget to an opthalmologist

c

The mother of 2yo Heather has brought her to the clinic with a "bad cough". Hx reveals onset of illness 4 days ago with clear rhinorrhea and coughing. Her mother says that Heather's fever has been as high 103 axillary. Physical exam reveals a temp of 101F and RR of 56 bpm, w/ slight nasal flaring and intercostal, subcostal, and suprasternal retractions. The pharynx is red without tonsillar exudate. Chest auscultation reveals widespread rales and wheezing. The lips and nail beds are slightly pale but pink, skin turgor is good and mucous membranes are moist. Initially, Heather should receive which diagnostic test? a. sputum culture b. sputum gram stain c. chest radiograph d. erythrocyte sedimentation rate

c

The mother of 2yo Heather has brought her to the clinic with a "bad cough". Hx reveals onset of illness 4 days ago with clear rhinorrhea and coughing. Her mother says that Heather's fever has been as high 103 axillary. Physical exam reveals a temp of 101F and RR of 56 bpm, w/ slight nasal flaring and intercostal, subcostal, and suprasternal retractions. The pharynx is red without tonsillar exudate. Chest auscultation reveals widespread rales and wheezing. The lips and nail beds are slightly pale but pink, skin turgor is good and mucous membranes are moist. When deciding whether Heather should be treated at home or in the hospital, it is most important to consider Heather's: a. max temp b. frequency of coughing episodes c. hydration status d. total length of illness

c

The mother of 3 yo GW reports that he has begun to stutter. Further probing reveals that the stuttering occurs frequently and lasts 1 to 2 seconds. GW does not seem bothered by the stuttering. The appropirate management would be: a. referral to a speech pathologist b. referral for an evaluation for an anxiety disorder c. reassuring the mother that this is a mild problem d. demonstrating to GW slow, deep breathing before talking

c

The mother of 4 mos old ND reports episodes of vomiting and diarrhea beginning 2 days ago. He has also had several episodes of screaming and drawing up his legs. Prior to this he has been healthy with a normal wt gain. Physical exam of ND reveals a sausage-shaped mass and guaiac-positive stool. This would confirm a dx of: a. incarcerated hernia b. gastroenteritis c. intussusception d. pyloric stenosis

c

The mother of 6 yo Cali brings her to the clinic bc the family is planning a trip to a tropical area, and she wants to know how to avoid illness. Knowing that cases of malaria have been reported in the area, you should teach: a. avoidance of contact with infected people b. hand washing after touching contaminated fomites c. avoidance of mosquito bites d. cooking of all vegetables

c

The mother of a 5yo boy has brought him to the clinic bc she thinks he has pinkeye. Which of the following would lead you to consider a dx other than bacterial conjunctivitis? a. hyperemic conjunctiva b. scratchy sensation in the eye c. decreased corneal clarity d. copious tearing

c

The murmur heard when a child has rheumatic heart disease is the result of: A) Myocarditis B) Pericarditis C) Valvulitis D) Coronary artery involvement

c

The parents of a 1 wk old infant are concerned about the unusual shape of their child's head. In the physical exam of the infant, which of the following signs would not support a diagnosis of craniosynostosis? a. palpation of a ridge along a given suture line b. unusual skull configuration c. a palpable lesion at the occipital region d. abnormal head cirumference

c

Varus between the tibia and femur of up to 15 degrees followed by a progression to a neutral angle, which then progresses to valgus between 7 to 9 degrees, is associated with which of the following? a. blount dz b. internal tibial torsion c. normal developmental growth d. abnormal tibiofermoral growth pattern

c

Which of the following is not a component of Metabolic syndrome, a clustering of factors in children shown to be associated with increased risk of coronary artery disease later in life? a. obesity b. elevated lipid levels c. heart murmur d. elevated fasting glucose levels

c

Which of the following may cause vol. overload leading to CHF if not appropriately followed and managed? a. coarctation of the aorta b. pulmonary stenosis c. ventricular septal defect d. supraventricular tachycardia

c

While examining a 2.5 week old infant, you notice irritability when you lift the infant and an asymmetrical Moro reflex. A spasm of the sternocleidomastoid on the left side is also present. These findings suggest: a. Torticollis b. Genu varum c. Fracture clavicle d. Pierre-Robin syndrome

c

While performing a well child check exam on a 3yo you discover a heart murmur. In your eval, you remember the following about innocent murmurs: a. they are best heard in diastole b. you must radiate to the axilla c. the intensity is no greater than I or II/VI d. There is no variation with changes in position

c

You are performing a newborn physical exam and you notice white specks around the circumference of the iris. You will also have to assess for: a. hypertonicity b. cherry red spot on the macula c. heart murmur d. cleft palate

c

You detect a hear murmur while examining a 3 yo child. In determining whether of not a referral is necessary, you determine it is an "innocent" heart murmur for which of the following reasons? a. It is best heard during diastole b. it radiates to the axilla c. the intensity is no greater than I or II/VI d. there is no variation with change in child's position

c

You would expect a school age child to: a. grow 1.5 inches per year b. grow 0.5 inch per year c. gain about 6 lbs per year d. gain about 3 lbs per year

c

Education and counseling of the parents of a 4 mos old child with gastroesophageal reflux should include all of the following except: a. thicken formula with rice cereal b. change to a hypoallergenic formula c. place infant in a car seat after feeding d. avoid exposure of the infant to tobacco smoke

c (sitting the infant in a car seat or similar device compresses the stomach, making the infant more likely to reflux)

1. The primary care pediatric nurse practitioner is preparing to close a laceration on a child's forehead using topical skin adhesive. What is the correct way to apply this product? a. Apply the adhesive between the wound margins and then hold the edges together. b. Apply the adhesive to the wound and then secure the edges with surgical tape c. Have the child remain still for 15 to 20 minutes after the adhesive is applied d. Hold the wound edges together and apply the adhesive on top of the skin

d

10. The primary care pediatric nurse practitioner is performing a wellbaby examination on a 7dayold infant born to a Chlamydiapositive mother. The infant's eyes are clear without exudate. The infant is free from cough and lungs are clear. What will the primary care pediatric nurse practitioner do to prevent illness in this infant? a. Administer a single dose of intramuscular ceftriaxone. b. Obtain bacterial cultures of both conjunctivae. c. Prescribe prophylactic erythromycin suspension. d. Schedule a followup appointment in 1 week.

d

11. An adolescent female has grouped vesicles on her oral mucosa. To determine whether these are caused by HSV1 or HSV2, the primary care pediatric nurse practitioner will order which test? a. Direct fluorescent antibody test b. Enzymelinked immunosorbent assay c. Tzanck smear d. Viral culture

d

12. A 4yearold child has clusters of small, clear, tense vesicles with an erythematous base on one side of the mouth along the vermillion border, which are causing discomfort and difficulty eating. What will the primary care pediatric nurse practitioner recommend as treatment? a. Mupirocin ointment applied to lesions 3 times daily b. Oral acyclovir 20 to 40 mg/kg/dose for 7 to 10 days c. Topical acyclovir applied to lesions 4 times daily d. Topical diphenhydramine and magnesium hydroxide

d

2. The primary care pediatric nurse practitioner performs a well baby exam on a 2dayold infant and notes clusters of firm, yellowwhite papules with a surrounding erythematous flare. What will the nurse practitioner do? a. Encourage frequent cleansing with mild soap. b. Obtain a Wrightstained smear of the lesions. c. Prescribe a topical antibiotic medication. d. Reassure the parents that no treatment is necessary.

d

20. A child has an area of inflammation on the neck that began after wearing a handknot woolen sweater. On examination, the skin appears chafed with mild erythematous patches. The lesions are not pruritic. What is an appropriate initial treatment? a. Application of a lanolinbased emollient b. Burow solution soaks and cool compresses c. Oral antihistamines given 4 times daily d. Topical corticosteroids applied 2 to 3 times daily

d

3. A child has a 1cm laceration on the forehead proximal to the hairline after running into a pole while playing sports. To minimize the risk of infection, the primary care pediatric nurse practitioner will irrigate the wound and a. Allow the wound to heal by secondary intention b. Delay closure of the wound for several days c. Refer the child to a plastic surgeon for wound closure d. Suture the wound within 6 hours

d

4. A 4yearold female who has had two urinary tract infections has persistent dysuria and genital redness. The physical exam reveals a thin, flat membrane from the posterior fourchette almost to the clitoris. Which treatment is indicated? a. Application of A&D ointment b. Counseling about hygiene c. Reassurance and observation d. Use of estrogencontaining cream

d

6. A schoolage child sustained a contusion on the front of one thigh while playing football and reports some difficulty flexing his foot on the affected side. What will the primary care pediatric nurse practitioner do to treat this injury? a. Place the child on crutches and limit weight-bearing until symptoms subside. b. Prescribe acetaminophen with hydrocodone along with NSAIDs c. Recommend rest, ice packs, compression, and evaluation of the extremity d. Refer the child to an orthopedic specialist for immediate evaluation and treatment

d

6. An adolescent who recently spent time in a hot tub while on vacation has discrete, erythematous 1 to 2mm papules that are centered around hair follicles on the thighs, upper arms, and buttocks. How will the primary care pediatric nurse practitioner manage this condition? a. Culture the lesions and treat with appropriate IM antibiotics. b. Hospitalize for incision and drainage and intravenous antibiotics. c. Order an antistaphylococcal betalactamaseresistant antibiotic. d. Prescribe topical keratolytics and topical antibiotics.

d

7. An infant is brought to clinic with bright erythema in the neck and flexural folds after recent treatment with antibiotics for otitis media. What is the treatment for this condition? a. 1% hydrocortisone cream to affected areas for 1 to 2 days b. Oral fluconazole 6 mg/kg on day 1, then 3 mg/kg/dose for 14 days c. Topical keratolytics and topical antibiotics for 7 to 10 days d. Topical nystatin cream applied several times daily

d

8. A schoolage child has several annular lesions on the abdomen characterized by central clearing with scaly, red borders. What is the first step in managing this condition? a. Fluoresce the lesions with a Wood's lamp. b. Obtain fungal cultures of the lesions. c. Perform KOHtreated scrapings of the lesion borders. d. Treat empirically with antifungal cream.

d

9. A child is bitten by a snake near a swimming pool in an area where copperhead snakes are known to inhabit, although the parents cannot describe the snake. An examination of the bite reveals a severe local reaction at the site with edema and intense pain. What will the primary care pediatric nurse practitioner do first? a. Administer narcotic analgesics to provide comfort b. Begin treatment with oral amoxicillin-clavulanate for 5 days c. Clean the wound and administer tetanus prophylaxis d. Transport the child by ambulance to a medical center

d

9. A parent is concerned that a 12monthold child is "bowlegged." A physical examination reveals internal tibial torsion bilaterally. A radiograph reveals asymmetric bowing of the legs with an angle greater than 15 degrees. What is the correct action for the primary care pediatric nurse practitioner? a. Observe the child's condition over time to assess progression. b. Order physical therapy to prevent progression of symptoms. c. Reassure the parent that the child will outgrow this deformity d. Refer to a pediatric orthopedic specialist for treatment.

d

A 13-year-old Tanner stage III/IV female adolescent who competitively runs cross-country track is scheduled for an annual sports physical examination. The adolescent complains of mild abdominal cramps and backache. When asked about voiding difficulties, the adolescent reports frequency symptoms. The dipstick screening reveals yellow urine, +1 blood, and no protein, leukocytes, and nitrites on the reagent strip. This morning the adolescent started having some dark vaginal discharge. The physical examination is otherwise within normal limits. The blood pressure was 110/78. Which of the following would be appropriate management? A) Obtain a culture and microscopic urinalysis, and start the patient on antibiotics, confirming improvement in 24 to 48 hours; perform repeat urinalysis or culture in 14 days B) Reassure the adolescent and the family that transient hematuria can be related to vigorous exercise, urinary tract infection (UTI), or menarche C) Obtain a urinalysis and complete blood cell count, and order tests to determine electrolyte levels, blood urea nitrogen, creatinine level, and creatinine clearance D) Obtain urinalysis and culture, and explain that these problems may be transient and related to exercise

d

A 13-year-old adolescent complains of having difficulty walking and the arms feeling weak. The adolescent had a cold 2 weeks ago. On examination, the PNP is not able to elicit deep tendon reflexes. The PNP knows that the most appropriate course of action is to: A) Refer the adolescent to an orthopedic surgeon B) Schedule a follow-up visit in 1 week C) Refer the adolescent to a neurologist D) Refer the adolescent to an emergency room d

d

A 15-year-old adolescent who has joint pain related to juvenile rheumatoid arthritis has been taking NSAIDs to relieve the pain. The adolescent came to the school-based clinic with epigastric and abdominal pain. The PNP suspects that the pain is related to the NSAIDs and manages the symptoms by: A) Modifying the adolescent's diet B) Substituting salicylates for NSAIDs C) Treating for H. pylori D) Administering a H2 receptor antagonist

d

A 2 mos old infant at your clinic received a combined Dtap/HepB/IPV vaccine and the parents are in need of teaching about possible side effects. Which of the following is not an adverse effect following administrations of the Dtap vaccination? a. local reaction b. fever c. increased fussiness d. transient morbilliform rash

d

A 4 yo child with cystic fibrosis (CF) comes to the primary care office with c/o runny nose, cough, congestion, and fever. You know that children with CF: a. are more likely to have normal CXR and LFTs findings b. usually are poor eaters with accompanying poor growth c. routinely take an oral mucolytic agent d. warrant more liberal use of antibiotics for respiratory infections

d

A common cause of CHF in the first year of life is: a. mild pulmonary stenosis b. inflammatory heart disease c. rheumatic fever d. complete heart block

d

A dx of croup is substantiated by which radiographic finding? a. ground glass appearance in the upper airway b. sparse areas of atelectasis c. "thumb sign" on lateral view d. "hourglass" narrowing in the subglottic region

d

A lethargic appearing 18 mos old child presents to the clinic with s/s of croup. Physical exam reveals a RR of 20 bpm and mild dehydration. Appropriate management includes: a. instructing the mother to force fluids and use a cool mist humidifier in the child's room b. instructing the mother to liberally give fluids and to encourage intake of solid food c. prescribing an oral broad spectrum antibiotic and prednisone d. referring the child for hospitalization and IV fluids

d

A principal clinical feature that aids in the diagnosis of Kawasaki dz includes: a. low-grade fever for 24 hrs and a pruritic rash b. conjunctivitis with exudate and facial rash c. arthritis and chorea d. High (>39C/102F) fever persisting at least 5 days and acute erythema and/or edema of hands and feet

d

An increase in which of the following behaviors is seen more frequently in late, rather than in early, adolescence? a. value conflict with parents b. focus on physical apperance c. peer group involvement d. understanding inner motivations of others

d

Brian, 13 yo, presents to the clinic with a sore throat. Hx reveals that he had a sore throat a couple of weeks ago and thought he had gotten well. He now has severe pharyngeal pain of 2 days duration and says he has been sweating and thinks he has fever. Physical exam reveals a temp of 102 and erythematous and edematous pharynx and soft palate. The right tonsil is swollen and inflammed without exudate and the uvula is displaced to the left. Right cervical nodes are tender. Lungs are clear to auscultation. Brian's signs and symptoms are suggestive of: a. acute uvulitis b. viral pharyngitis c. epiglottitis d. peritonsillar abscess

d

CW, a 20 mos old, presents in the ER with a greenstick fracture of his left femur. Physical exam also reveals and enlarged anterior fontanel and enlarged costochondral junction. What do these clinical findings suggest. a. child abuse b. osteogenesis imperfecta c. osteoporosis d. rickets

d

During 2.5 yo Jason's physical exam you note large muscular calves and observe his difficulty rising from a sitting position. The Denver screening exam reveals delays in the gross motor area. Which of the following lab test would be most beneficial? a. serum calcium b. serum magnesium c. serum phosphurus d. seurm creatine kinase

d

During 8 mon old LBs physical exam, the father boasts that LB is going to be a left-handed batter since he prefers doing everything with his left hand. The appropriate response would be to: a. ask if others in the family are left handed b. suggest play activities that require using both hands c. present toys more often to the right hand d. perform a careful neurological exam

d

During a prenatal visit, you review the mother's record for routine prenatal screening results. While education the mother, you explain that the screening of maternal serum for alpha-fetoprotein (MSAFP) between the 15th - 21st weeks of pregnancy is done primarily to screen for: a. phenylketonuria b. galactosemia c. cystic fibrosis d. neural tube defects

d

During the newborn exam of KL, you note a generalized lacy reticulated blue discoloration. This clinical presentation describes: a. harlequin color change b. mongolian spots c. blue nevus d. cutis marmorata

d

EG, at 9 years of age, presents with diffuse abd pain and acute onset of diarrhea described as a frequent urge to defecate. She is passing large amounts of flatus, small amounts of stool, and c/o tenderness during rectal examination. This clinical picture is highly suggestive of: a. gastoenteritis b. ulcerative colitis c. giardiasis d. appendicitis

d

Emily, a 4 yo, stays with her great aunt during the day while her mother is at work. Emily's mother has brought her to the clinic bc the great aunt has just been dx with TB. Emily's Mantoux skin test is positive but there is not clinical or radiographic evidence of dz. Appropriate managment includes: a. reassuring Emily's mother that no treatment is needed b. Administering another skin test in 3 mos c. oral penicillin therapy d. oral preventative isoniazid therapy

d

In a child with chronic sinusitis, the most accurate method of identifying abnormalities is: a. dark room transillumination of the sinuses b. percussion of the paranasal sinuses c. AP, lateral, and occipitomental sinus radiographs d. CT scan of the sinuses

d

In a child with chronic sinusitis, the most accurate method of identifying sinus abnormalities is: a. dark room transillumination of the sinuses b. percussion of the paranasal sinuses c. AP, lateral, and occipitomental sinus radiograhps d. CT scan of the sinuses

d

In males, Tanner stage III can be distinguished from tanner stage II by: a. fine downy pubic hair at the base of penis b. adultlike pubic hair not extending to thighs c. penile growth in width d. penile growth in length

d

James, at 4 years old, has been dx with cat scratch dz. His mother asks what should be done about the cat. What should your response be? a. the cat should be isolated until it can be treated with antibiotics b. the cat should be isolated until it can be treated with anthelmintics c. the cat should be evaluated by a medical lab and destroyed d. there are no recommendations for treating and destroying the cat

d

Jeffrey, at 8 years of age, has been diagnosed with ADHD and is receiving stimulant medication. Which of the following interventions would be least helpful? a. monthly ht and wt checks b. small frequent meals and snack c. high-calorie supplemental drinks d. elimination of refined sugar from diet

d

Josh, age 5, presents to the clinic with inspiratory stridor, drooling, and a temp of 105. He insists on sitting up during the clinical examination. Appropriate initial management of Josh includes: a. high dose of an oral broad spectrum antibiotic and antipyretics b. teaching the mother to administer racemic epi by nebulization c. teaching the mother how to administer loading and decreasing doses of prednisione d. immediate hospitalization with intravenous antibiotics

d

Josh, age 5, presents to the clinic with inspiratory stridor, drooling, and a temp of 105. He insists on sitting up during the clinical examination. This clinical picture is most consistent with a dx of: a. aspirated foreign body b. reactive airway dz c. viral croup d. epiglottitis

d

Julie, at 18 mos of age, has been brought to the clinic by her mother who tells you that Julie has had a cold for the past 4 days. There is no hx of cough and the mother is unsure whether Julie has had fever. Physical exam reveals greenish, blood-tinges mucus with a strong, foul odor, draining from the right nostril. The clinical picture is most consistent with a diagnois of: a. allergic rhinitis b. viral rhinits c. acute sinusitis d. nasal foreign body

d

LR, at 6 years of age, presents at clinic with a solitary nonprruritic lesion around his upper lip. Closer inspection reveals some vesicles and honey colored crusts. The most likely dx is: a. herpes simplex b. varicella c. nummular eczema d. impetigo

d

Matthew, at 5 mos old, is brought to the clinic bc he has been coughing and has had clear rhinorrhea for the last 2 days. His mother tells you that he has never been sick before. Family hx is positive for allergies and you hear generalized wheezing. You may conclude that: a. Matthew has familial asthma b. Matthew has asthma exacerbated by a viral infection c. Matthew should be referred for allergy testing d. Asthma should not be diagnosed at this stage

d

Mrs. D reports starting her 6 mos old infant on rice cereal sweetened with one tablespoon of honey. In addition, the infant is consuming 42 oz of forumla. His ht and wt are at 50% on the growth curve. You would recommend: a. adding pureed vegetables b. substituting 4oz juice for a formula feeding c. adding scrambled egg whits d. discontinuing honey

d

Steven, at 10 years of age, as been diagnosed with otitis externa twice this year. Health teaching for Steven and his mother should include: a. emphasis on consistent use of low-dose prophylactic antibiotics b. sleeping with affected ear in the dependent position c. info on the use of decongestants to open the eustachian tube d. info on the use of acetic acid after ear canal contact with water

d

Steven, at 6 years of age, has been dx with erythema infectiosum (fifth dz). His mother asks you how to prevent the spread of the dz to her other children. Your answer should be: a. the dz is not thought to be contagious b. stephen should eat and drink from disposable containers c. other children should not be allowed to touch the erythematous areas d. the patient is no longer contagious by the time the rash appears

d

The mother of 12yo Nathan has brought him to the clinic bc he has had a runny nose for 2 wks. Hx reveals that Nathan has visited the clinic 3 other times this year for upper respiratory complaints. Examination reveals slightly edematous and erythematous eyes, pale nasal mucosa with clear mucus, and pharynx with thin secretions posteriorly. There is no tonsillar swelling or exudate. Lips and nail beds are pink. Lymph node exam is significant for multiple shotty nodes. Lungs are clear to auscultation. Which actions is appropriate at this time? a. discuss symptomatic relief of the common cold with Nathan and his mother b. culture nasal drainage and delay treatment until result are known c. order an antibitotic d. order an antihistamine

d

The mother of 4 day old Susan has brought her to the clinic bc she has not seemed interested in her bottle since yesterday. Physical exam reveals a lethargic infant with an axillary temp of 96F and a pulse rate of 100. Extremities are slightly cool. Appropriate mngmt includes: a. reassuring the mother that infants tend to regulate their own feeding habits b. instructing the mother on proper nipple placement and feeding techniques c. changing the infant to a soy-based formula and reevaluating her in 24 hrs d. considering the possibility of sepsis with appropriate referral

d

The mother of 8 yo John telephones to tell you that he developed chicken pox 3 days ago. She wants to know if there is anything she can do to make him more comfortable. John's mother calls you again 10 days after onset of the initial rash to report that John has had a severe headache since yesterday and that he is very irritable. Which of the following would be advised? a. ask the mother if John has had a bowel mvmt since he has been ill b. tell John's mother that these symptoms are common when chicken pox is resolving c. ask the mother if there is a family hx of severe headaches d. ask John's mother to bring him to the office today for evaluation

d

The mother of a 15 mos old child informs you that she feeds the baby skim milk. You advise the mother to change to whole milk primarily because skim milk: a. is not as easily digested as whole milk b. contains an insufficient amount of calcium c. contains too little protein d. provides an inadequate amount of essential fatty acids

d

The mother of a 4 mos old infant reports that he turned "blue" and seemed to have fast, labored breathing after vigorous crying soon after awakening. He settled down and his color and breathing seemed to improve. On physical exam, the mucous membranes of the lips and mouth appear mildly cyanotic. A systolic murmur is heard best at the left sternal border. Vital signs are normal with normal peripheral pulses. There is no hepatomegaly. A likely diagnosis is: a. congestive heart failure b. acute life-threatening event related to reflux c. coarctation of the aorta d. cyanotic spell related to tetraology of Fallot

d

The mother of a toddler with a typical roseola-type rash and a hx of high fever asks if there is any treatment available for the condition. The PNP tells the mother that: a. topical corticosteroids are helpful to relieve itching b. oral diphenhydramine is helpful to decrease desquamation c. aspirin should be used to treat the typical high fver d. there is not medical treatment for roseola

d

You are preparing a drug prevention program for middle school students. Your educational approach is based on the knowledge that the most common substance abuse in adolescence is: a. marijuana b. cocaine c. heroin d. alcohol

d

You are seeing a 15 mos old boy with leukemia for a check up. If indicated, this child may receive all of the following vaccines except: a. inactivated polio vaccine (IPV) b. H. influenzae type B (Hib) c. DTap d. Varicella (VAR)

d

You have ordered routine blood screening for a 2yo girl who because of dietary habits, is at risk for iron deficiency anemia. Which of the following findings is not associated with iron deficiency anemia? a. hypochromic RBC b. microcytic RBC c. low reticulocyte count d. low free erythrocyte protoporphyrin (FEP) level

d

You not a single, large, oval, pink, patch with central clearing on 16 yo MPs back. Lesions are not present elsewhere. Results of the KOH prep of the lesion are negative. This would confirm a dx of: a. seborrheic dermatitis b. secondary syphilis c. tinea corporis d. pityriasis rosea

d

CA, 2 wks old, is being seen for the first time since discharge from the newborn nursery. She currently weighs 3.6kg, which is 0.3 kg below her birth wt. While interviewing her mother you learn she has been using too little water when preparing the formula. Which of the following is least likely related to the error? a. vomiting b. diarrhea c. dehydration d. flatus

d ( too little H20 will increase GI and renal solute load, may result in vomiting, diarrhea and dehydration)

Considering the patho associated with erythema infectiosum (fifth dz), a child with hx of which of the following diseases must be monitored closely? a. frequent respiratory infections b. multiple skin allergies c. malabsorption syndrome d. hemolytic anemia

d (bc of reticulocytopenia associated with EI may result in dangerous decrease of hemoglobin concentration in the child with hemolytic anemia)

An adolescent who has cradle cap as an infant is in the clinic with thick crusts of yellow, greasy scales on the forehead and behind the ears. What will the NP recommend?

daily application of ketoconazole 2% topical cream

6. The primary care pediatric nurse practitioner is managing a 6yearold child who has chronic constipation and encopresis. The nurse practitioner has ruled out neurogenic etiology. The parents report that the child was difficult to toilet train as a toddler. What is key to managing this child's condition? A. Encouraging use of maintenance medications for at least 2 months after resolution of constipation B. Referral to a mental health consultant to manage problems in the parentchild dyad C. Spending time with the parents to uncover their feelings about their child's condition D. Teaching the parents that the symptom of stool retention is often voluntary for the child

encopresis - soiling of underwear with stool by children who are past the age of toilet training A. Encouraging use of maintenance medications for at least 2 months after resolution of constipation

The primary care pediatric nurse practitioner performs a Hirschberg test to evaluate

ocular alignment.

The primary care pediatric nurse practitioner examines a child who has had stiffness and warmth in the right knee and left ankle for 7 or 8 months but no back pain. The nurse practitioner will refer the child to a rheumatology specialist to evaluate for

oligoarticular JIA.

During a well child assessment of a 13 year old male, the NP notes small testicles and pubic and axillary hair. To further evaluate these findings, the NP will as the patient about.

participation in sports.

A 16 year old adolescent female whose BMI is at the 90 th percentile reports irregular periods. The primary care pediatric nurse practitioner notes widespread acne on her face and back and an abnormal distribution of facial hair. The nurse practitioner will evaluate her further based on a suspicion of which diagnosis?

polycystic ovary disease

When prescribing topical glucocorticoids to treat inflammatory skin conditions, the NP will:

prescribe brand-name preparations for consistent effects.

an adolescent has a TB skin test prior to working as a volunteer in a hospital. The adolescent is healthy and has not traveled or from a TB-endemic area or had close contact with anyone who has TB. The skin test shows 10 mm of induration after 48 hours. What should the prudent nurse practitioner do?

reassure the adolescent that this is a negative screen.

During a well child assessment of an AfricanAmerican infant, the primary care pediatric nurse practitioner notes a dark redbrown light reflex in the left eye and a slightly brighter, redorange light reflex in the right eye. The nurse practitioner will

refer the infant to an ophthalmologist.

The primary care pediatric nurse practitioner sees a 17 year old client who quit smoking almost a year prior but who reports having renewed cravings when around friends who smoke. Using knowledge of the maintenance stage of change, the primary care pediatric nurse practitioner will:

suggest that the teen considered taking up a sport or other physical activity.

During a well child assessment of an 18 month old child, the primary care pediatric nurse practitioner observes the child point to a picture of a dog and say, "Want puppy!" The nurse practitioner recognizes this as an example of:

telegraphic speech

The parents of a 12 year old child are concerned that some of the child's older classmates may be a bad influence on their child, who they say, has been raised to believe in right and wrong. What will the NP tell the parent?

the pressures from outside influences may supersede parental teachings and should be confronted.

The parent of a 10 year old boy tells the NP that the child doesn't appear to have any interest in girls and spends most of his time with a couple of other boys. The parent is worried about the child's sexual identity. The NP will tell the parent:

this attachment to other same-gender children is how the child learns to interact with others

The NP is examining a 6 year old child who attends first grade. The child reports "hating" school. The parent states that the child pretends to be sick frequently in order to stay home from school. To further assess this situation, the NP will first ask the child:

to name one or two friends

14. Neonates who are at risk for neonatal herpes should be considered when? Select all that apply Jaundice Respiratory distress Mothers with hx of recent genital infection Vesicular rash Irritability Seizures

•Neonates at risk for congenital herpes: born to mothers with hx of recent genital infection and high-risk sexual activity; delivery-related risk factors include rupture of membranes for longer than 6 hours and use of a scalp electrode.


Kaugnay na mga set ng pag-aaral

Chapter 9 - Device Management Protocols - NTP

View Set

Strength Training Exam 2: 4,7,17

View Set

Chapter Twelve: Information Systems

View Set

Adult health Urinary/Reproductive EAQs 2

View Set

ISOM 1380 Module 2 Innovation & Society: 2.2 Standards & Dominant Design

View Set

Tener calambre - to have a cramp

View Set

Performance and Discharge of Contracts in Indiana

View Set

ch 14: community and public health and the environment

View Set